Você está na página 1de 163

MAYO CLI NI C

I NTERNAL MEDI CI NE
BOARD REVI EW
QUESTI ONS AND ANSWERS

Mayo Clinic Scientifc Press
Mayo Clinic Atlas of Regional Anesthesia and Ultrasound-Guided Nerve Blockade
Edited by James R. Hebl, MD, and Robert L. Lennon, DO
Mayo Clinic Preventive Medicine and Public Health Board Review
Edited by Prathibha Varkey, MBBS, MPH, MHPE
Mayo Clinic Challenging Images for Pulmonary Board Review
Edited by Edward C. Rosenow III, MD
Mayo Clinic Gastroenterology and Hepatology Board Review , Fourth Edition
Edited by Stephen C. Hauser, MD
Mayo Clinic Infectious Diseases Board Review
Edited by Zelalem Temesgen, MD
Mayo Clinic Antimicrobial Handbook: Quick Guide , Second Edition
Edited by John W. Wilson, MD, and Lynn L. Estes, PharmD
Just Enough Physiology
By James R. Munis, MD, PhD
Mayo Clinic Cardiology: Concise Textbook , Fourth Edition
Edited by Joseph G. Murphy, MD, and Margaret A. Lloyd, MD
Mayo Clinic Internal Medicine Board Review, Tenth Edition
Edited by Robert D. Ficalora, MD
MAYO CLINIC
INTERNAL MEDICINE
BOARD REVIEW
QUESTI ONS AND ANSWERS

E DI TOR - I N- CHI E F
Robert D. Ficalora , MD
C O N S U L T A N T , D I V I S I O N O F G E N E R A L I N T E R N A L M E D I C I N E
M A Y O C L I N I C , R O C H E S T E R , M I N N E S O T A
A S S O C I A T E P R O F E S S O R O F M E D I C I N E
C O L L E G E O F M E D I C I N E , M A Y O C L I N I C
E DI TOR
Paul S. Mueller , MD
Thomas J. Beckman, MD
Margaret Beliveau , MD
Mark C. Lee , MD
Nicole P. Sandhu , MD, PhD
Amy T. Wang , MD
Christopher M. Wittich , MD
MAYO CLINIC SCIENTIFIC PRESS OXFORD UNIVERSITY PRESS
A S S OCI AT E E DI TOR S
Te triple-shield Mayo logo and the words MAYO, MAYO CLINIC, and MAYO CLINIC
SCIENTIFIC PRESS are marks of Mayo Foundation for Medical Education and Research.
3
Oxford University Press is a department of the University of Oxford.
It furthers the Universitys objective of excellence in research, scholarship,
and education by publishing worldwide.
Oxford New York
Auckland Cape Town Dar es Salaam Hong Kong Karachi
Kuala Lumpur Madrid Melbourne Mexico City Nairobi
New Delhi Shanghai Taipei Toronto
With of ces in
Argentina Austria Brazil Chile Czech Republic France Greece
Guatemala Hungary Italy Japan Poland Portugal Singapore
South Korea Switzerland Tailand Turkey Ukraine Vietnam
Oxford is a registered trademark of Oxford University Press in the UK and certain other
countries.
Published in the United States of America by
Oxford University Press
198 Madison Avenue, New York, NY 10016
Mayo Foundation for Medical Education and Research 2013
All rights reserved. No part of this publication may be reproduced, stored in a retrieval system, or transmitted, in any form or by
any means, electronic, mechanical, photocopying, recording, or otherwise, without the prior permission of Mayo Foundation for
Medical Education and Research. Inquiries should be addressed to Scientifc Publications, Plummer 10, Mayo Clinic, 200 First St SW,
Rochester, MN 55905.

Library of Congress Cataloging-in-Publication Data
Mayo Clinic internal medicine board review : questions and answers / editor-in-chief Robert D. Ficalora ; editor, Paul S. Mueller ;
associate editors, Tomas J. Beckman ... [et al.].10th ed.
p. ; cm.(Mayo Clinic scientifc press)
Internal medicine board review : questions and answers
Companion volume to: Mayo Clinic internal medicine board review. 10th ed. c2013.
Includes bibliographical references and index.
ISBN 9780199985876 (alk. paper)ISBN 9780199322039 (alk. paper)ISBN 9780199322046 (alk. paper)
I. Ficalora, Robert D. II. Mayo Clinic. III. Mayo Foundation for Medical Education and Research. IV. Mayo Clinic
internal medicine board review. V. Title: Internal medicine board review : questions and answers. VI. Series: Mayo Clinic
scientifc press (Series)
[DNLM: 1. Internal MedicineExamination Questions. WB 18.2]
616.0076dc23 2013001640

Mayo Foundation does not endorse any particular products or services, and the reference to any products or services in this book is
for informational purposes only and should not be taken as an endorsement by the authors or Mayo Foundation. Care has been taken
to confrm the accuracy of the information presented and to describe generally accepted practices. However, the authors, editors, and
publisher are not responsible for errors or omissions or for any consequences from application of the information in this book and
make no warranty, express or implied, with respect to the contents of the publication. Tis book should not be relied on apart from the
advice of a qualifed health care provider.
Te authors, editors, and publisher have exerted eforts to ensure that drug selection and dosage set forth in this text are in accordance
with current recommendations and practice at the time of publication. However, in view of ongoing research, changes in government
regulations, and the constant fow of information relating to drug therapy and drug reactions, readers are urged to check the package
insert for each drug for any change in indications and dosage and for added wordings and precautions. Tis is particularly important
when the recommended agent is a new or infrequently employed drug.
Some drugs and medical devices presented in this publication have US Food and Drug Administration (FDA) clearance for limited
use in restricted research settings. It is the responsibility of the health care providers to ascertain the FDA status of each drug or device
planned for use in their clinical practice.
9 8 7 6 5 4 3 2 1
Printed in China on acid-free paper
Dedicated to all the patients who help us, as internists,
learn, practice, and master internal medicine.
Robert D. Ficalora, MD
This page intentionally left blank
vii
volume make both volumes more portable and user-friendly.
New questions and answers simulate the types of questions
included on the American Board of Internal Medicine exami-
nation. Te editors and associate editors added their depth of
experience to ensure that this edition is the fnest in the long
history of this book. Te text is not only informational but
also of great assistance in preparing for board certifcation and
recertifcation, and it allows for the practical application of
knowledge to serve our patients.
Morie A. Gertz, MD
Chair, Department of Internal Medicine
Mayo Clinic, Rochester, Minnesota
Professor of Medicine
College of Medicine
Mayo Clinic
One of the Department of Medicines strategic goals is to
provide premier education in the science and art of medicine.
Established goals include leading the nation in the develop-
ment of lifelong learning programs and educating physician
and nonphysician learners at all levels and along all points of
the education continuum. Tese goals are attained by provid-
ing state-of-the-art graduate medical education. Te rapid
pace at which medical knowledge is being discovered neces-
sitates frequent updates. Mayo Clinic Internal Medicine Board
Review: Questions and Answers refects changes in the science
of medicine and contains features that facilitate retention of
the knowledge imparted. Te chapters in the companion text-
book have been completely revised to correspond to American
Board of Internal Medicine objectives and include evidence-
based recommendations. Bulleted points allow easy access to
key points. Te questions and answers placed in this companion
FOREWORD
This page intentionally left blank
ix
without the dedication of the associate editors who labored
in isolation over the chapter drafs. We are all indebted to
stafs of the Department of Medicine; Section of Scientifc
Publications, Joseph G. Murphy, MD, Chair; LeAnn M.
Stee and Randall J. Fritz, DVM (editors), Kenna Atherton
(manager), Jane M. Craig (editorial assistant), and Alissa K.
Baumgartner (proofreader); and Section of Illustration and
Design, Deb Veerkamp and Ryan Ledebuhr, at Mayo Clinic
for their contributions to this edition. I gratefully acknowl-
edge the support and cooperation of the publisher, Oxford
University Press. In particular, I am indebted to my admin-
istrative partner, Michael OBrien, whose unfailing support
helped me through some very dif cult times.
In the spirit of the previous editions, I trust that Mayo
Clinic Internal Medicine Board Review: Questions and Answers
will serve our readers well in preparation for the primary certi-
fcation or maintenance-of-certifcation examination.
Robert D. Ficalora, MD
Editor-in-Chief
Mayo Clinic Internal Medicine Board Review: Questions and
Answers is the result of the combined eforts of Mayo Clinic
physicians who practice in all the various subspecialties of
Internal Medicine. Many have achieved certifcates in medical
education and thus understand how to communicate informa-
tion to our readersphysicians who are in training and prac-
ticing clinicians who are preparing for the American Board of
Internal Medicine (ABIM) certifcation and maintenance-of-
certifcation examinations in internal medicine.
Our annual Mayo Clinic Board Review Course, now in its
27th year, gives the authors and editors the unique opportu-
nity to interact with our readers and tailor our approach to
the way the current generation of learners prepares for a high
stakes examination. In response to learner feedback, we have
placed the questions and answers in this volume separate from
the specialty-area content. More than 300 ABIM-format
multiple-choice questions with a single answer and explana-
tion are keyed to each chapter in the content textbook.
I am grateful to the current and past authors for their
careful attention and hard work. Tis book would not exist
PREFACE
This page intentionally left blank
xi
Contributors xiii
1. Preparing for the ABIM Examination 1
2. Cardiology Questions and Answers 9
3. Gastroenterology and Hepatology Questions and Answers 27
4. Pulmonary Diseases Questions and Answers 35
5. Infectious Diseases Questions and Answers 45
6. Rheumatology Questions and Answers 59
7. Endocrinology Questions and Answers 67
8. Oncology Questions and Answers 79
9. Hematology Questions and Answers 83
10. Nephrology Questions and Answers 93
11. Allergy Questions and Answers 105
12. Psychiatry Questions and Answers 109
13. Neurology Questions and Answers 113
14. Dermatology Questions and Answers 117
15. Cross-Content Area Questions and Answers 123
Index 141
CONTENTS
This page intentionally left blank
xiii
Robert D. Ficalora, MD
Consultant, Division of General Internal Medicine
Mayo Clinic, Rochester, Minnesota; and
Associate Professor of Medicine
College of Medicine, Mayo Clinic
William W. Ginsburg, MD
Consultant, Division of Rheumatology
Mayo Clinic, Jacksonville, Florida; and
Associate Professor of Medicine
College of Medicine, Mayo Clinic
Lyell K. Jones Jr, MD
Consultant, Department of Neurology
Mayo Clinic, Rochester, Minnesota; and
Assistant Professor of Neurology
College of Medicine, Mayo Clinic
Mary J. Kasten, MD
Consultant, Divisions of General Internal Medicine and
Infectious Diseases
Mayo Clinic, Rochester, Minnesota; and
Assistant Professor of Medicine
College of Medicine, Mayo Clinic
Kyle W. Klarich, MD
Consultant, Division of Cardiovascular Diseases
Mayo Clinic, Rochester, Minnesota; and
Associate Professor of Medicine
College of Medicine, Mayo Clinic
Mark C. Lee, MD
Consultant, Division of General Internal Medicine
Mayo Clinic, Rochester, Minnesota; and
Assistant Professor of Medicine
College of Medicine, Mayo Clinic
Scott C. Litin, MD
Consultant, Division of General Internal Medicine
Mayo Clinic, Rochester, Minnesota; and
Professor of Medicine
College of Medicine, Mayo Clinic
Conor G. Lofus, MD
Consultant, Division of Gastroenterology and Hepatology
Mayo Clinic, Rochester, Minnesota; and
Assistant Professor of Medicine
College of Medicine, Mayo Clinic
Charles F. Abboud, MB, ChB
Consultant, Division of Endocrinology, Diabetes,
Metabolism, & Nutrition
Mayo Clinic, Rochester, Minnesota; and
Associate Professor of Medicine
College of Medicine, Mayo Clinic
Tomas J. Beckman, MD
Consultant, Division of General Internal Medicine
Mayo Clinic, Rochester, Minnesota; and
Professor of Medicine and of Medical Education
College of Medicine, Mayo Clinic
Margaret Beliveau, MD
Consultant, Division of General Internal Medicine
Mayo Clinic, Rochester, Minnesota; and
Assistant Professor of Medicine
College of Medicine, Mayo Clinic
Elie F. Berbari, MD
Consultant, Division of Infectious Diseases
Mayo Clinic, Rochester, Minnesota; and
Associate Professor of Medicine
College of Medicine, Mayo Clinic
Peter A. Brady, MB, ChB, MD
Consultant, Division of Cardiovascular Diseases
Mayo Clinic, Rochester, Minnesota; and
Associate Professor of Medicine
College of Medicine, Mayo Clinic
Maria L. Collazo-Clavell, MD
Consultant, Division of Endocrinology, Diabetes,
Metabolism, & Nutrition
Mayo Clinic, Rochester, Minnesota; and
Associate Professor of Medicine
College of Medicine, Mayo Clinic
Lisa A. Drage, MD
Consultant, Department of Dermatology
Mayo Clinic, Rochester, Minnesota; and
Assistant Professor of Dermatology
College of Medicine, Mayo Clinic
J. Christopher Farmer, MD
Consultant, Division of Pulmonary and Critical
Care Medicine
Mayo Clinic, Rochester, Minnesota; and
Professor of Medicine
College of Medicine, Mayo Clinic
CONTRIBUTORS
xiv

CONTRIBUTORS
John J. Poterucha, MD
Consultant, Division of Gastroenterology and Hepatology
Mayo Clinic, Rochester, Minnesota; and
Professor of Medicine
College of Medicine, Mayo Clinic
Abhiram Prasad, MD
Consultant, Division of Cardiovascular Diseases
Mayo Clinic, Rochester, Minnesota; and
Professor of Medicine
College of Medicine, Mayo Clinic
Rajiv K. Pruthi, MBBS
Consultant, Division of Hematology
Mayo Clinic, Rochester, Minnesota; and
Associate Professor of Medicine
College of Medicine, Mayo Clinic
Qi Qian, MD
Consultant, Division of Nephrology & Hypertension
Mayo Clinic, Rochester, Minnesota; and
Associate Professor of Medicine and of Physiology
College of Medicine, Mayo Clinic
Nicole P. Sandhu, MD, PhD
Consultant, Division of General Internal Medicine
Mayo Clinic, Rochester, Minnesota; and
Assistant Professor of Medicine
College of Medicine, Mayo Clinic
M. Rizwan Sohail, MD
Consultant, Division of Infectious Diseases
Mayo Clinic, Rochester, Minnesota; and
Assistant Professor of Medicine
College of Medicine, Mayo Clinic
Marius N. Stan, MD
Consultant, Division of Endocrinology, Diabetes,
Metabolism, & Nutrition
Mayo Clinic, Rochester, Minnesota; and
Assistant Professor of Medicine
College of Medicine, Mayo Clinic
Bruce Sutor, MD
Consultant, Division of Psychiatry and Psychology
Mayo Clinic, Rochester, Minnesota; and
Assistant Professor of Psychiatry
College of Medicine, Mayo Clinic
Karen L. Swanson, DO
Consultant, Division of Pulmonary and Critical Care
Medicine
Mayo Clinic, Rochester, Minnesota; and
Associate Professor of Medicine
College of Medicine, Mayo Clinic
Seth R. Sweetser, MD
Consultant, Division of Gastroenterology and Hepatology
Mayo Clinic, Rochester, Minnesota; and
Assistant Professor of Medicine
College of Medicine, Mayo Clinic
Fabien Maldonado, MD
Consultant, Division of Pulmonary and Critical Care
Medicine
Mayo Clinic, Rochester, Minnesota; and
Assistant Professor of Medicine
College of Medicine, Mayo Clinic
Robert D. McBane, MD
Consultant, Division of Cardiovascular Diseases
Mayo Clinic, Rochester, Minnesota; and
Professor of Medicine
College of Medicine, Mayo Clinic
Kevin G. Moder, MD
Consultant, Division of Rheumatology
Mayo Clinic, Rochester, Minnesota; and
Associate Professor of Medicine
College of Medicine, Mayo Clinic
Timothy J. Moynihan, MD
Consultant, Division of Medical Oncology
Mayo Clinic, Rochester, Minnesota; and
Associate Professor of Oncology
College of Medicine, Mayo Clinic
Paul S. Mueller, MD
Chair, Division of General Internal Medicine
Mayo Clinic, Rochester, Minnesota; and
Professor of Biomedical Ethics and of Medicine
College of Medicine, Mayo Clinic
Suzanne M. Norby, MD
Consultant, Division of Nephrology & Hypertension
Mayo Clinic, Rochester, Minnesota; and
Assistant Professor of Medicine
College of Medicine, Mayo Clinic
Amy S. Oxentenko, MD
Consultant, Division of Gastroenterology and Hepatology
Mayo Clinic, Rochester, Minnesota; and
Associate Professor of Medicine
College of Medicine, Mayo Clinic
John G. Park, MD
Consultant, Division of Pulmonary and Critical Care
Medicine
Mayo Clinic, Rochester, Minnesota; and
Assistant Professor of Medicine
College of Medicine, Mayo Clinic
Naveen L. Pereira, MD
Consultant, Division of Cardiovascular Diseases
Mayo Clinic, Rochester, Minnesota; and
Assistant Professor of Medicine
College of Medicine, Mayo Clinic
Axel Pfueger, MD, PhD
Consultant, Division of Nephrology & Hypertension
Mayo Clinic, Rochester, Minnesota; and
Professor of Medicine
College of Medicine, Mayo Clinic
CONTRIBUTORS

xv
Amy T. Wang, MD
Senior Associate Consultant, Division of General Internal
Medicine
Mayo Clinic, Rochester, Minnesota; and
Assistant Professor of Medicine
College of Medicine, Mayo Clinic
Christopher M. Wittich, MD
Consultant, Division of General Internal Medicine
Mayo Clinic, Rochester, Minnesota; and
Assistant Professor of Medicine
College of Medicine, Mayo Clinic
Alexandra P. Wolanskyj, MD
Consultant, Division of Hematology
Mayo Clinic, Rochester, Minnesota; and
Associate Professor of Medicine
College of Medicine, Mayo Clinic

Keith M. Swetz, MD
Consultant, Division of General Internal Medicine
Mayo Clinic, Rochester, Minnesota; and
Assistant Professor of Medicine
College of Medicine, Mayo Clinic
Carrie A. Tompson, MD
Consultant, Division of Hematology
Mayo Clinic, Rochester, Minnesota; and
Assistant Professor of Medicine
College of Medicine, Mayo Clinic
Pritish K. Tosh, MD
Mayo Clinic Scholar in Infectious Diseases
Mayo Clinic, Rochester, Minnesota; and
Assistant Professor of Medicine
College of Medicine, Mayo Clinic
Gerald W. Volcheck, MD
Consultant, Division of Allergic Diseases
Mayo Clinic, Rochester, Minnesota; and
Associate Professor of Medicine
College of Medicine, Mayo Clinic
This page intentionally left blank
1
EXAMINATI ON FORMAT
Almost all of the questions are clinical and based on cor-
rect diagnosis and management. Because there is no penalty
for guessing, candidates should answer every question. Most
questions are based on clinical cases. Among these, 75% are
related to the outpatient and emergency department settings,
and the remainder are related to the inpatient setting, includ-
ing the critical care unit and nursing home. Increasing empha-
sis is placed on patient safety and evidence-based quality of
care. Selecting the correct answer to these questions requires
integration of information provided from several sources (eg,
history, physical examination, laboratory test results, and
consultations), prioritization of alternatives, or use of clinical
judgment. Up to one-third of questions are experimental and
included to test question quality. Tey are not scored and can-
not be identifed during the examination. Patient management
with a cost-efective, evidence-based approach is stressed. Very
few questions require simple recall of medical facts. Tere are
no intentional trick questions.
Te ABIM examination has a uniform question approach
that stresses clinical reasoning over simple recall.
EXAMINATI ON CONTENT
Te questions in the examination cover a broad area of inter-
nal medicine. Tey are divided into primary and cross-content
groups (http://www.abim.org/pdf/blueprint/im_cert.pdf ).
Each session (4 for initial certifcation and 3 for mainte-
nance of certifcation) contains 60 multiple-choice questions.
Te question may include a case history, a brief statement, a
radiograph, a graph, or a photograph (such as a blood smear
or Gram stain). Each question has 5 possible answers, and the
candidates should identify the single-best answer. More than
1 answer may appear correct or partially correct for a question.
Sample questions are included in the ABIM tutorial, http://
www.abim.org/exam/prepare.aspx.
COMPUTER-BASED TESTI NG
Candidates currently take a computer-based certifcation
examination that has been designed to provide a fexible,
quiet, and professional environment for examination. Te
OVERVIEW
Since 2006, more than 7,000 individuals per year have taken
the American Board of Internal Medicine (ABIM) initial
certifcation examination, and between 3,000 and 5,000 indi-
viduals per year have taken the Maintenance of Certifcation
(MOC) examination. Pass rates have ranged from 79% to
94%. Pass rates for frst-time takers on both examinations
exceed those of repeat takers. Tere is no doubt that careful
and serious preparation for the examination is valuable and
necessary. Although some individuals can take and pass the
examination with minimal preparation, most takers need
rigorous preparation. In recent years, board certifcation has
assumed greater importance in the minds of patients. In a
2003 Gallup poll of 1,001 US adults aged 18 years or older,
98% wanted their physicians to be board-certifed, 79%
thought that the recertifcation process was very important,
and 54% would choose a new internist if their physicians
board certifcation had expired.
EXAMINATI ON: BASI C INFORMATI ON
Te ABIM website (www.abim.org) has a wealth of informa-
tion for test takers. No one should approach the examina-
tion without reading the ABIM Information and Statistics
(http://www.abim.org/exam/prepare.aspx) and the ABIM
Certifcation and Recertifcation Exam Guide (http://www.
abim.org/exam/default.aspx). Te Exam Day: What to
Expect section, http://www.abim.org/exam/exam-day.aspx,
has up-to-date information about changes to and navigation
of the ABIM approach to computer-based testing, such as the
following:
How do I answer questions?
How do I change answers?
How do I make notes?
How do I mark questions for review?
Review the ABIM website materials to understand the
testing approach.
1.
PREPARING FOR THE ABIM EXAMINATION
a

Robert D. Ficalora , MD

a
Reprinted from Ficalora RD, editor. Mayo Clinic internal medicine board review.
10th ed. Rochester (MN): Mayo Clinic Scientifc Press and New York (NY):
Oxford University Press; c2013.
2

MAYO CLINI C INTERNAL MEDI CINE BOARD REVIEW: QUESTI ONS AND ANSWERS
Te self-evaluation modules evaluate performance in clini-
cal skills, preventive services, practice performance, fund of
medical knowledge, and feedback from patients and colleagues.
Successfully completed self-evaluation modules are valid for
10 years. Candidates may apply to begin the MOC process
any time afer initial certifcation. Te ABIM recommends
that completion of the self-evaluation modules be spread out
over the certifcation period. A candidate should complete
1 self-assessment module every 1 to 2 years. Te ABIM encour-
ages candidates to enroll within 4 years of certifcation in order
to have adequate time to complete the program.
Candidates who passed the ABIM certifcation
examination in internal medicine in 1990 and thereafer
have a certifcate that is valid for 10 years.
Te MOC process is called continuous professional
development and consists of a 3-step process.
10 TI PS FOR EFFECTI VE EXAMI NATI ON
PREPARATI ON
1. HAVE A STUDY PLAN
We all have busy lives. Successful candidates stress that the most
valuable preparation strategies must include scheduling a time
to study . Preparing in small, discrete pieces improves recall, facil-
itates review, and makes the overall task less onerous. Spending
3 or 4 hours a week, using various approaches such as directed
reading, practice questions, and group review, is enough to stay
focused. Simply reading by itself is usually a bad strategy. You
may not be able to retain much of the ABIM material by read-
ing without focus. Start with a question, a problem to solve, or
a patient scenario in mind. Tis approach to a study session will
help you and your group understand what you are studying, the
clinical context, pathophysiology, and management and the rea-
sons for it. Keep asking yourself why? and why not?
How can I study such a large mass of material?
Plan a pace of no more than 3 major topics per hour.
Survey the material.
Consider the major subsections as potential questions.
Review the material in each subsection carefully to answer
the question.
Recite in your own words.
Revise in your notes.
Take notes! Even if you never look at them again, the act of
synthesizing the information in writing will help you retain it.
To study, use active learning approaches to maximize
ef ciency.
Simply reading, no matter how much, is generally an
inefective preparation strategy.
computer-based test is administered by about 200 centers in
the United States. Candidates schedule their examination
date according to the updated instructions on the ABIM
website, http://www.abim.org/exam/. Candidates are well
advised to access the online tutorial at http://www.abim.
org/exam/prepare.aspx. Tis tutorial allows the candidate
to become familiar with answering questions, changing
answers, making notes electronically, accessing the table
of normal laboratory values, and marking questions for
review.
Candidates are advised to familiarize themselves with the
computer-based testing format by accessing the online
tutorial.
MAI NTENANCE OF CERTI FI CATI ON
Te diplomate certifcates issued to candidates who have
passed the ABIM examination in internal medicine since
1990 are valid for 10 years. Te total number of candidates
who took the ABIM MOC examination for the frst time in
2007 was 3,837. Of these, 83% passed.
ENHANCEMENTS TO MOC PROGRAM
In January 2006, the ABIM enhanced the MOC pro-
gram to increase fexibility and assess performance in
clinical practice. Te 3 retained general components (creden-
tialing, self-evaluation, and secure examination) and the added
self-evaluation module each have a point value.
Every candidate must complete a total of 100 points in
self-evaluation modules. Unlike the previous system, renewal
of more than 1 certifcate does not necessitate taking addi-
tional self-evaluation modules (ie, the same number of points,
100, satisfes the requirement to sit for these examinations).
Candidates must complete at least 20 points in medical
knowledge and at least 20 points in practice performance.
Te remaining 60 points may be obtained from comple-
tion of modules developed by ABIM or other organizations
that meet the ABIM standards. Tus, one could combine an
ABIM knowledge module (20 points) and an ABIM practice
improvement module (20 points) with the American College
of Physicians Medical Knowledge Self-assessment Program
(MKSAP) (3 modules, 60 points), or one could combine
an ABIM practice improvement module (20 points) with
6 annual-update ABIM knowledge modules (60 points) and
the ABIM peer and patient feedback module (20 points). All
points are valid for 10 years. Further refnements to this pro-
cess are likely. Tus, candidates should check for updates on
the ABIM website.
MOC is a multistep process in addition to the
examination.
Always check the ABIM website for information and
updates.
1. PREPARING FOR THE ABIM EXAMINATI ON

3
MOC candidates should use practice questions to guide
your study choices. Do as many questions as you can, and
monitor your performance by the ABIM blueprint section,
http://www.abim.org/pdf/blueprint/im_cert.pdf.
Serious preparation for the examination actually starts
at the beginning of residency training. In addition to daily
reading and achieving subspecialty-based profciency, most
candidates require a minimum of 6 to 8 months of intense
preparation for the examination. Cramming before the exami-
nation, whether by yourself or at a review course, is unlikely to
be successful.
Use a standard textbook of internal medicine. Ideally,
you should use one good textbook and not jump from one to
another. Although online, just-in-time resources may be use-
ful for fact checking, they rarely give an inclusive, case-based
review. Te most efective way to use the textbook is with
patient-centered reading. Read the descriptions of the symp-
toms and signs carefully because ofen they are part of the ques-
tions in the examination. Table 1.1 provides several examples
of the common descriptions of symptoms and signs that could
be part of the examination. Rather than reading chapters at
random, read the literature in a structured manner to assist in
future recall of facts. Tis book and similar books are excellent
tools for brushing up on important board-relevant informa-
tion several weeks to months before the examination. Tey,
however, cannot take the place of comprehensive textbooks
of internal medicine. Tis book is designed as a study guide
rather than a comprehensive textbook of medicine. Terefore,
it should not be used as the sole source of medical information
for the examination.
Study frst with a standard textbook of internal medicine.
Tis book is designed as a study guide and should not be
used as the sole source of information for preparation for
the examination.
Pay attention to the descriptions of signs and symptoms.
4. CREATE AND DEVOTE TI ME
Board preparation should be part of your daily routine, like
exercising, showering, or brushing your teeth. If you dont
regularly do some preparation, it will fall of your routine, and
your preparation just wont happen. You can spend as little or
as much time as you want on a particular activity. Ofen you
can review familiar topics in small discrete time periods (eg,
before or afer lunch). Less familiar topics may require an hour
in a quiet room, and this time may be best reserved for early
in the morning or on a weekend. Keep in mind that the more
time and energy you spend actively learning a topic, the bet-
ter your command, and the less dependent you will be on rote
memorization.
Some people can study efectively while on a treadmill,
on a train, or in a car. If you can do this, you can incorpo-
rate this into your studying routine. How much time it takes
to perform these tasks varies from person to person and will
improve as you solidify your study habits. Regardless, every
2. FORM A STUDY GROUP
If possible, form a study group. You will be more likely to
make and stay on a schedule if individuals feel a responsibility
to the groups progress. A group will boost everyones morale
and give a common sense of purpose. A group size of only 2
to 5 candidates permits study of diferent textbooks, board
review materials, and review articles in journals. Make sure
that you have a committed, available group of study partners.
Individuals who push ahead on their own and those who dont
keep their commitments can sabotage an otherwise produc-
tive group. Schedule regular meetings and assign individuals
specifc topics. Tis approach saves time, covers more top-
ics per session, and allows everyone to retain more from the
discussion. Take turns acting as group moderator, to keep to
the topic and schedule. Te moderator should be responsible,
congratulating productive members and ofering a friendly
word to someone who might be slacking of. If everyone has
a turn, no one person has to be the bad guy. Selected review
articles on common and important topics, such as represented
by the ABIM objectives, should be included in every session.
Avoid indiscriminate reading of articles from many journals.
Remember that questions are tested for several examina-
tion cycles before they are included in the examination. It is
unlikely that new information or current controversies will be
represented on your examination. Notes and other materials
the candidates have gathered during residency training can
be good sources of information. Finding the justifcation for
these pearls can cement ones command of a particular topic.
Tese clinical pearls gathered from mentors will be of help
in remembering certain important points. Always save some
time each session to review questions and discuss the answers
and their rationales. Dont forget to discuss each of the options
in detail. Tis will develop your thought process and sharpen
your test-taking skills.
Keep your study group small and stay focused.
Make a schedule and read ahead of the discussion.
Discuss study material.
Do multiple-choice questions in groups.
Indiscriminate reading of articles from many journals
should be avoided.
Information in recent journals is unlikely to be included in
the examination.
3. DETERMI NE WHAT YOU NEED TO STUDY
For recent graduates attempting primary certifcation, let
your in-training examination subsection score results guide
your study choices. In general, if your score in a given area
was below the ffh decile, or ffieth percentile, you should
consider that an area for intensive review and preparation.
Use the section called educational objectives, which gives
your performance by content area, to guide your choice of
preparation topics.
Table 1.1. COMMON DESCRIPTIONS OF SIGNS AND SYMPTOMS IN EXAMINATION QUESTIONS
HISTORY (SYMPTOMS) PHYSICAL FINDINGS (SIGNS) LIKELY DIAGNOSIS
Cardiology
Shortness of breath or asymptomatic
Asymptomatic, headache
Late peaking systolic murmur, intensity decreases with
handgrip & increases with squatting
Hypertension, diminished or absent lower extremity
pulses, systolic murmur, bruit over chest wall
Hypertrophic obstructive cardiomyopathy
Coarctation of aorta
Neurology
Gait impairment, falls, dysphagia, dysarthria
Diplopia, oscillating images, reading fatigue, loss of depth perception
Fluctuating memory, confusion, visual hallucinations
Inappropriate behavior, dementia, poor social skills
Inability to look up & side to side
Impaired adduction on lateral gaze, with nystagmus in
the contralateral abducting eye
Mild parkinsonism, dementia
Dementia
Progressive supranuclear palsy
Internuclear ophthalmoplegia (consider multiple sclerosis,
cerebrovascular disease)
Lewy body dementia
Frontotemporal dementia
Paroxysmal pain afecting the side of the face
Muscle stifness, clumsiness, occasional emotional lability
Altered mental status, fever, headache
Usually normal
Brisk refexes, spasticity (upper motor neuron signs),
atrophy, fasciculation (lower motor neuron signs)
Flaccid paralysis, neck rigidity , altered mental status
Trigeminal neuralgia afecting 1 of the branches of cranial nerve V
Amyotrophic lateral sclerosis
West Nile virus encephalitis
Infectious disease
Recurrent sinusitis, skin, or pulmonary infections due to
Staphylococcus aureus
Recurrent Neisseria infections
Recurrent episodes of bacterial pneumonia, sinusitis, diarrhea due to
Streptococcus pneumoniae
Sinus tenderness, abnormal lung sounds
Neck rigidity , altered mental status
Malnourished, abnormal lung sounds
Chronic granulomatous disorder
Inherited defciencies of complement (C5, 6, 7, 8, 9), factor D, or
properdin
Common variable immunodefciency
Gastroenterology
Cirrhosis of liver, ingestion of raw oysters
Diarrhea

Hepatitis C, photosensitivity
Fever, hypotension, hemorrhagic bullae, signs of
cirrhosis of liver
Pruritus, grouped vesicles over the elbow, knee, scalp,
or back of neck
Skin fragility, erosions, blisters on dorsum of hand,
hyperpigmentation
Vibrio vulnifcus

Dermatitis herpetiformis due to celiac sprue

Porphyria cutanea tarda
Dermatology
Facial rash, photosensitivity
Rash
Cough with sore throat
Ulcerative colitis
Flushing, diarrhea, rapid heart rate
Papules & pustules on bridge of nose & face,
telangiectasia
Sharply demarcated erythematous papules, silvery
white scales over scalp, extensor surfaces of
extremities, & nails
Tender, erythematous pretibial nodules
Irregular, undermined ulcer with violaceous border or
scarring in lower extremities
Brown-red macules, urticaria on stroking skin
Rosacea
Psoriasis
Erythema nodosum
Pyoderma gangrenosum
Systemic mastocytosis
1. PREPARING FOR THE ABIM EXAMINATI ON

5
range from 5 to 10 with some estimated to have 0 to 2. Pace
your preparation by subject:
Subjects with a large estimated number of questions are
very likely to be there. Master them.
Subjects with a medium to low estimated number of
questions will be there in some form. Review them.
Subjects with a very low estimated number of questions
may be tangential or favorite board zebras . Depending on
your available study time, it may be worthwhile for you to
consider them only for last-minute review.
Plot out your objectives review on a calendar, mixing more
and less complex objectives. Leave time for discussion, lit-
erature and online searches, and follow-up for problem items
from previous study sessions. Always plan to cover new mate-
rial and to periodically cycle back to previously reviewed dif -
cult or detailed information. Imagine cases that might go with
the material at hand.
Te most efective way to manage your study time is to
periodically assess your progress through practice tests and
test questions. It is impossible to overemphasize the impor-
tance of this point. Boardsmanship is a real skill, and there
is no substitute for familiarity with the form and content of
the ABIM-type questions. Terefore, you are strongly encour-
aged to take at least a few mock examinations and simulate
the actual testing environment (ie, no breaks, snacks, music,
phones, pagers). Taking mock questions is an efective use of
a board review or board questions book. Practice material at
intervals during your long-term test preparation schedule.
Your schedule should be the most intense in the 3 months
before the test. Afer that, focus on review, consolidating key
points, and resolving previously dif cult problems.
Once you or your group has a schedule, stick to it. Add ses-
sions, but never delete any. Stick to your start and stop times.
Schedule your progress and build in assessment sessions.
Make changes, but no deletions.
Plan to review material that you have chosen several times
(minimum twice).
Stay focused throughout the months before the
examination.
Board review preparation must be at its peak by 3 months
before the examination. If you have not yet formed a study
group, now is your last chance.
6. ANSWER QUESTI ONS SI MI LAR TO
EXAMI NATI ON CONTENT
Te purpose of standardized testing is to measure a candi-
dates command of the material so that scores from diferent
test dates can be reliably compared with one another. Te
results must correlate statistically with the results of all the
test-takers who have answered the same questions. Persons
learning task takes time, and you must budget for that time.
Only you can decide how much time you want to spend in
solitary study, in groups, or in summary objective review. You
can make great plans, but life and work arent predictable,
so you should build in some catch-up time for unexpected
distractions.
You also have to consider how much time it takes to
organize your studying. Review of cardiology may go
quickly, whereas glomerulopathies may take a pad and pen-
cil to fgure out. You may have to travel to study sessions
or spend time looking for information to ensure your com-
mand of a given objective. Many candidates try to set aside
large blocks of time. With our busy lives, that may be laud-
able but impractical. Many shorter sessions not only allow
for study and catch-up but also can be worked in around
standing commitments more efectively than large blocks,
and thus a missed session wont be a major setback for that
week. Te time you spend will come back to you when you
pass, and a failure only means you have to devote the time
all over again.
Certifying or recertifying board examinations can be stress-
ful. Te sheer mass of information can be overwhelming to
some. Te press of occupational and personal responsibilities
makes fnding the time to study very dif cult, so many opt to
take a review course. A review course should be the fnal inte-
grating activity once you have completed your own primary
preparation. To be successful you should go to the review
course prepared and ready to fll in objectives you may have
missed or to learn from experts objectives that you couldnt
understand on your own. Dont expect a course to substitute
for primary preparation. Attending a highly focused, no-fuf
course that delivers the information in a concentrated,
high-yield manner right before the examination may seem like
an easy way out, but it is unlikely to be the diference between
success and failure.
Residents: Prepare for the boards during residency.
You will not fnd that kind of quality time afer your
residency.
Once residency is over and you start fellowship or a job,
you will not fnd time to study .
MOC: Schedule the time.
Schedule multiple short preparation sessions rather than
fewer long ones.
Do not rely solely on a review course; they are not a
substitute for primary preparation.
5. PREPARE A PLAN AND SCHEDULE IT
All ABIM objectives are not equal. Review the relative per-
centage of the contents of the examination and the number
of questions per objective (http://www.abim.org/pdf/blue-
print/im_cert.pdf ). Note that some areas may have an esti-
mated number of questions of 11 to 15, whereas many will
6

MAYO CLINI C INTERNAL MEDI CINE BOARD REVIEW: QUESTI ONS AND ANSWERS
Familiarize yourself with the teaching principle and the
testing objective, which may give you insight into the
questions and the possible responses.
7. DON T FORGET ABOUT I MAGES
Every image-based question will also include text or a clinical
case or both. Dont simply focus on the image without reading
the text. Familiarize yourself with the image and its details afer
you have read the case, and then read it again. Photographs
of skin disorders, radiographs, electrocardiograms, and other
images given in board questions are generally easy ways to
score points. Reading the ABIM question stem last helps
put the pieces of the puzzle in place. Methodically review a
radiograph as you would in a patient encounter. Immediately
focusing on an obvious abnormality can distract you from a
more subtle fnding that may alert you to the correct answer.
You may miss the pneumothorax as the cause of the dyspnea if
you focus only on the heart size and the small pleural efusion.
Likewise for skin fndings; use your clinical skills to interpret
the fnding. Is it fat or raised, erythematous or pigmented?
Approach an image question as you would a patient.
Methodically examine the image.
Use the text and the stem to focus your inquiry.
8. SPEND SOME TI ME BECOMI NG EXPERT I N
BOARDSMANSHI P
Some candidates fail the examination despite intense prepara-
tion and the clinical competence necessary to pass the exami-
nation. Failing usually happens because they dont understand
or interpret the questions properly. Te ability to understand
the nuances of the question format is sometimes referred to as
boardsmanship. Intelligent interpretation of the questions
is very important for candidates who are not well versed in
the format of multiple-choice questions. Answer the ques-
tions whose answers you know frst, making sure you under-
stand what is being asked to ensure that they are answered
correctly. It is easy to become overconfdent with such ques-
tions, and thus you may fail to read the questions or the
answer options carefully. Make sure you never make mistakes
on easy questions. Read the fnal sentence (that appears just
before the multiple answers) several times to understand how
an answer should be selected. Recheck the question format
before selecting the correct answer. Read each answer option
completely.
Occasionally, a response may be only partially correct. At
times, the traditionally correct answer is not listed. In these
situations, select the best alternative listed. Watch for qualifers
such as next, immediately, or initially. Avoid answers that con-
tain absolute or very restrictive words such as always, never, or
must. Answer options that contain absolutes are likely incor-
rect. Try to think of the correct answer to the question before
looking at the list of potential answers. Assume you have
who construct board review materials and questions go to
great pains to build them for the same content and content
level as on the actual board examination. Te good ones are
validated and have been tested to make sure that they perform
in a reliable, predictable manner and that they adequately test
the content they purport to test. Any questions prepared for
other courses, local residency, fellowship rotations, or other
venues that cover similar material likely wont test the mate-
rial in a way that predicts your performance on the board
examination. Read a board review book, go to a board review
course, and always practice answering questions. When you
practice answering questions, do it as you would during the
examination; just reading the book and reading the answers
likely will not prepare you for the actual test. For every ques-
tion, identify in your own mind the concept being tested.
Make sure you read all the wrong answers and make sure you
understand why they werent the best answer for the question
asked.
Sometimes candidates try to prepare by studying mate-
rials that are harder than the real test, such as subspecialty
boardlevel review courses and practice tests. Te idea is that
becoming familiar with something harder will make the real
thing easier by comparison. Preparing with something more
challenging can be a good idea in some types of athletic or
endurance preparation, but it is a bad idea for the ABIM
examination. Because the objectives are specifc and pub-
lic, preparing by using objectives for another examination
may cause you to misinterpret or overinterpret what is being
asked. Reliably fnding the easiest approach to a test question
requires being aware that the test cant require you to use a cer-
tain higher-level data set or decision tree. Because the harder
material is testing a diferent skill set, reviewing this may lead
to incorrect answers.
As silly as this piece of advice may seem, read the ques-
tions carefully! Doing so can make a big diference in your
score. If you read questions hastily, there is a high likelihood
that you will misinterpret them. Some questions ofer incor-
rect choices that are designed to answer a common misinter-
pretation of the actual question. Be particularly careful with
answers that have more than one part. Only one part may be
correct. Other distraction techniques include 2 responses that
are similar except for a word or phrase. Watch for responses
that contradict others; usually, both of these can be ruled out.
What if you read a question and the traditional correct answer
isnt an option? What if more than one answer could be cor-
rect? Ten select the best option available. Be very careful
of responses that are the longest or the unique answer. Tey
are no more likely or unlikely to be correct despite prevailing
wisdom.
Dont try to read the board review material from cover to
cover.
Te best way to prepare is to review and always practice
answering questions.
To improve your understanding, read the explanation,
and look up additional information related to each of the
choicesboth correct and incorrect.
1. PREPARING FOR THE ABIM EXAMINATI ON

7
been given all the necessary information to answer the ques-
tion. If the answer you had formulated is not among the list
of answers provided, you may have interpreted the question
incorrectly. When a patients case is presented, think of the
diagnosis before looking at the list of answers. If you do not
know the answer to a question, very ofen you are able to rule
out one or several answer options. Determine whether your
diagnosis is supported by any of the answers. If you can elimi-
nate any answers as clearly wrong, you will improve your odds
at guessing. Occasionally, you can use information presented
in one question to help you answer other, dif cult questions.
Many questions are on the test for trial or validation purposes
and are not scored. If a question seems to you to be a bad or
confusing question, it may be in this category. It is best not to
spend an inordinate amount of time trying to second guess
this type of question. Come back to it afer you have fnished,
if you still have time.
When reading long multiple-choice cases:
First read the actual lead line of the question
Once you understand what the question is asking:
Stay focused and look for clues in the long stem of the
question.
As you read through the questions:
Note the key facts and abnormal fndings
Skip questions about which you have no idea, and come
back afer a complete frst pass
9. USE YOUR REFLEXES
Associations, causes, complications, and other relationships
between a phenomenon or disease and clinical features are
important to remember and recognize. Each subspecialty has
many common connections, and candidates for the ABIM
and other examinations may want to prepare lists like this
for diferent areas. For example, a case that presents a patient
with health careassociated pneumonia should immediately
bring to mind antipseudomonal antibiotics, not antibiot-
ics traditionally used for community-acquired pneumonia.
Combined knee and hip pain should have you consider-
ing a gait abnormality rather than abnormality in 2 joints
simultaneously.
Use the basic fund of knowledge accumulated from
clinical experience and reading to solve the questions.
Approaching the questions as real-life encounters with
patients is far better than trying to second-guess the exam-
iners or trying to analyze whether the question is tricky. As
indicated above, the questions are never tricky, and there is
no reason for the ABIM to trick the candidates into choos-
ing wrong answers.
Use examination techniques to your advantage. Look for
target populations in questions. Start with a basic premise in
mind, then modify it as the information warrants. Examples
are as follows:
For young patients, aim for aggressive management.
For elderly patients, aim for less aggressive alternatives,
especially in those with multisystem disease.
Beware of adverse medication efects and polypharmacy.
For asymptomatic healthy patients, do nothing and
observe.
Use your existing fund of knowledge of internal medicine
and your previous clinical experience.
Approach each question as a real-life patient encounter.
Tere are no trick questions.
10. PLAN FOR THE DAY OF THE EXAMI NATI ON
You should have adequate time to read and answer all the ques-
tions; therefore, there is no need to rush or become anxious.
Watch the time to ensure that you are at least halfway through
the examination when half of the time has elapsed. Start by
answering the frst question and continue sequentially. Almost
all of the questions follow a case-presentation format. At times,
subsequent questions will give you information that may help
you answer a previous question. Do not be alarmed by lengthy
questions; look for the questions salient points. When faced
with a confusing question, do not become distracted by that
question. Mark it so you can fnd it later, then go to the next
question and come back to the unanswered ones at the end.
Extremely lengthy stem statements or case presentations are
intended to test the candidates ability to separate the essen-
tial from the unnecessary or unimportant information. You
may want to highlight important information presented in
the question in order to review this information afer reading
the entire question and the answer options. Tere is no pen-
alty for guessing, so you should never leave an answer blank.
Every time you can eliminate just one choice you increase your
chance of choosing a correct answer by 20%, so its best to
guess among the remaining choices. If you truly have no idea
about any of the choices, the B answer has been statistically
more likely to be correct. It is better to choose B if you truly
dont know the answer.
Look for the salient points in each question.
If a question is confusing, mark it to fnd it and come back
to the unanswered questions at the end.
If you must guess, choose B; statistically, it is more likely
to be correct.
Its really not productive to discuss the questions or answers
afer the examination with other candidates. Such discussions
usually cause more consternation, although some candidates
may derive a false sense of having performed well on the exam-
ination. In any case, the candidates are bound by their oath to
the ABIM not to discuss or disseminate the questions. Do not
study between examination sessions. To minimize stress, stick
8

MAYO CLINI C INTERNAL MEDI CINE BOARD REVIEW: QUESTI ONS AND ANSWERS
SUMMARY
Preparation for the ABIM examination requires a serious and
organized approach. Devote adequate time. Familiarize your-
self with the examination format and objectives. Use com-
monsense test-taking strategies, including practice tests and
question analysis. Treat the examination day as you would for
any competitive event by preparing physically.

to your daily routine; dont start or stop exercising or using
cafeine, and dont skip meals or load up on carbohydrates. Be
as rested and refreshed as you can be. Forget about your elec-
tronic devices such as pagers and cell phones.
Dont study the day before the examination or between the
examination sessions.
Discussing the examination questions with others raises
anxiety and can adversely afect your performance in the
next session.
Maintain your normal routine.
9
2. A 78-year-old woman comes to your of ce with irregu-
lar palpitations, shortness of breath, and fatigue. She
has type 2 diabetes mellitus and hypertension, both of
which she has kept under excellent control since she had
a minor stroke at age 69. She has no other known medi-
cal problems of note. Examination fndings are normal
except that she has a sof, barely audible right carotid
bruit. Te electrocardiogram (ECG) from a week ago is
shown (Figure 2.Q2). A transthoracic echocardiogram
shows normal lef ventricular function and moderate
lef atrial enlargement. You order another ECG today
since her regular heart rate confrms normal sinus
rhythm. What anticoagulation therapy should you rec-
ommend for this patient at this time?
a. Warfarin (goal international normalized ratio, 2.03.0)
b. Aspirin 325 mg
c. Catheter ablation of this arrhythmia to eliminate stroke
risk
d. Aspirin 325 mg and clopidogrel 75 mg
e. No anticoagulation since she is now in normal sinus rhythm
and at low risk
QUESTI ONS
Multiple Choice (choose the best answer)
ARRHYTHMI AS AND CLI NI CAL SYNDROMES
1. You are asked to see a 22-year-old man who under-
went a routine electrocardiographic (ECG) examina-
tion as part of a preemployment physical examination
for a large manufacturing company. Other than having
a fairly sedentary lifestyle, he is healthy and reports
no symptoms. Cardiac examination fndings are nor-
mal. You view the 12-lead ECG shown in Figure 2.Q1.
Which of the following would be the best management
strategy for this patient at this time?
a. Perform an electrophysiology study and catheter ablation.
b. Observe.
c. Start therapy with metoprolol 25 mg twice daily.
d. Start therapy with procainamide 150 mg twice daily.
e. Start therapy with digoxin 0.125 mg daily.
2.
CARDIOLOGY QUESTIONS AND ANSWERS
I
II
III
II
aVR
aVL
aVF
V
1
V
2
V
3
V
4
V
5
V
6
Figure 2.Q1
10

MAYO CLINI C INTERNAL MEDI CINE BOARD REVIEW: QUESTI ONS AND ANSWERS
On examination, the patient was fully orientated
and appeared younger than his age. His heart rate and
blood pressure were satisfactory, but his systolic pres-
sure decreased by 10 mm Hg on standing. Bilateral
carotid bruits and a harsh 3/6 ejection murmur were
present. Te murmur was loudest at the lef sternal
border, with some radiation into the neck. Reversed
splitting of the second heart sound (separation during
expiration rather than inspiration) was also present, but
no diastolic murmur was heard. Tere was no evidence
of jugular venous distention, and his lungs were clear,
3. An 85-year-old man has had 2 separate syncopal epi-
sodes that occurred suddenly and without warning. On
the frst occasion, he was talking to his daughter on the
phonethen he was lying on the foor. Recovery was
immediate, but he injured his lef hand. On the second
occasion, he was in the kitchen making a sandwich. His
daughter was present and was able to break his fall. She
noted that both arms made jerking movements, which
resolved when he was placed into a recovery position
on his lef side. On both occasions, a full recovery
occurred within a few minutes without sequelae.
I
II
III
II
aVR
aVL
aVF
V
1
V
2
V
1
V
5
V
3
V
4
V
5
V
6
Figure 2.Q2
I
II
III
II
aVR
aVL
aVF
V
1
V
2
V
1
V
5
V
3
V
4
V
5
V
6
Figure 2.Q3
2. CARDI OLOGY QUESTI ONS AND ANSWERS

11
a. Add sotalol.
b. Add dronedarone.
c. Refer for catheter ablation.
d. Continue the current regimen.
CARDI AC PHYSI CAL EXAMI NATI ON
6. A 30-year-old woman is referred for a pre-insurance
examination. She has no complaints. She is active
and runs with her large dog about 1 hour daily (45
miles). She has no exercise limitations and was hiking
at an elevation of 3,030 m in the Rockies last summer.
On examination, her pulse rate is 50 beats per minute,
blood pressure is 120/80 mm Hg, and she is thin. On
cardiac examination, there is a normal frst heart sound
and normal splitting; an early low-pitched diastolic
sound is consistent with a third heart sound (S
3
). Tere
is a grade 1 to 2 midsystolic murmur at the lef upper
sternal border. Te murmur disappears with the Valsalva
maneuver. Carotid upstrokes are normal, and the jugu-
lar venous pressure is normal. No systolic clicks can be
appreciated at rest or with the squat-to-stand maneuver.
Te electrocardiogram shows sinus bradycardia (heart
rate 48 beats per minute) but is otherwise normal. What
further cardiac testing needs to be done?
a. Cardiac catheterization to measure a gradient across the
mitral valve
b. Stress echocardiography
c. Treadmill ECG
d. No additional testing
e. Holter monitoring
7. Which of the following statements about shunts in the
atrial septum is true?
a. Te auscultatory fndings associated with atrial septal defect
(ASD) are paradoxical splitting of the second heart sound
(S
2
) and a diastolic fow rumble heard midsternum.
b. Patent foramen ovale (PFO) is uncommon in healthy adults
and is found in approximately 2.5% of adults.
c. A fxed splitting of the S
2
and a right ventricular (RV) lif are
found in ASD with signifcant shunting.
d. Secundum ASD and PFO require subacute bacterial endo-
carditis prophylaxis.
8. A 30-year-old woman comes to your of ce for evalua-
tion. She appears healthy and is not limited by physi-
cal activity. She says that she has occasional chest pains
that occur at rest, ofen with a change in position. Her
blood pressure is 110/70 mm Hg, and her heart rate is
76 beats per minute and regular. Auscultatory fndings
are a midsystolic click followed by a 2/6 apical systolic
murmur that radiates to her axilla. Which of the fol-
lowing dynamic physical examination fndings would
confrm your suspicion of mitral valve prolapse?
a. Tere is no change in the click with squatting or standing.
b. A passive leg raise makes the click-murmur occur earlier.
c. A squat maneuver makes the click-murmur occur later.
d. Te mitral regurgitant murmur does not change with posi-
tional maneuvers.
but a small amount of peripheral edema was present.
Gait was normal and there were no neurologic signs of
note. Te electrocardiogram is shown in Figure 2.Q3.
Tis patients presentation is most consistent with
which of the following?
a. Cardiogenic syncope
b. A possible seizure disorder
c. Stroke or transient ischemic attack
d. Neurocardiogenic syncope
e. Orthostatic response
4. A 55-year-old woman with nonischemic dilated car-
diomyopathy and exertional dyspnea (New York Heart
Association [NYHA] functional class III) and 2-pillow
orthopnea underwent placement of an implantable
cardioverter-defbrillator 4 years ago for sudden cardiac
arrest prophylaxis. Now she returns and asks whether
cardiac resynchronization therapy may be appropri-
ate. Her current medications are lisinopril 5 mg daily,
carvedilol 6.25 mg twice daily, spironolactone 25 mg
daily, and furosemide 40 mg daily.
Physical examination and diagnostic study fndings
were as follows: heart rate 97 beats per minute, blood
pressure 142/70 mm Hg, jugular venous pressure 12 cm
water, lungs clear, lateral displacement of the cardiac
impulse, audible third heart sound, and no murmurs;
electrocardiography showed lef bundle branch block
(LBBB) with QRS duration of 130 ms, and transtho-
racic echocardiography showed lef ventricular ejection
fraction (LVEF) of 20%. What is the most appropriate
management for this patient at this point?
a. Tere is no indication for cardiac resynchronization therapy
(CRT)the symptoms are not severe enough.
b. Tere is no indication for CRTthe QRS complex is not
wide enough.
c. Upgrade to CRT now.
d. Up-titrate the dosages of lisinopril and carvedilol.
5. A 32-year-old woman who received a diagnosis of peri-
partum cardiomyopathy (New York Heart Association
class II) afer the birth of her second child presents
with symptomatic paroxysmal atrial fbrillation (AF).
Typically, she experiences approximately 4 episodes of
AF per year, each lasting 2 to 4 hours. During episodes,
she notes palpitations, shortness of breath, and fatigue,
with heart rates around 90 beats per minute, but she
is able to continue working in her home. Although
she has generally done well from a heart failure (HF)
standpoint, she was hospitalized 3 weeks ago with
acute decompensated HF. Current medications include
carvedilol 25 mg twice daily, lisinopril 20 mg daily,
furosemide 40 mg daily, and digoxin 0.125 mg daily.
Physical examination and diagnostic study fndings
were as follows: heart rate 60 beats per minute (regular
rhythm), blood pressure 100/70 mm Hg, jugular pres-
sure not elevated, and lungs clear; electrocardiography
showed normal sinus rhythm and lef bundle branch
block. Which of the following would you recommend
at this time?
12

MAYO CLINI C INTERNAL MEDI CINE BOARD REVIEW: QUESTI ONS AND ANSWERS
CONGESTI VE HEART FAI LURE
13. A 72-year-old man presents with progressively increas-
ing dyspnea and edema of the lower extremities. He
denies having chest discomfort or palpitations. He
experiences dyspnea on minimal exertion, orthopnea,
and occasional episodes of paroxysmal nocturnal dysp-
nea. He had a myocardial infarction 10 years ago and
underwent primary percutaneous coronary interven-
tion then. His residual lef ventricular ejection fraction
(LVEF) afer that event was 30% by echocardiography.
He has adhered to his medical regimen. He has been
hospitalized for decompensated heart failure repeat-
edly over the past 3 years. He received an implantable
cardioverter-defbrillator with cardiac resynchroniza-
tion therapy 2 years before the current presentation.
He has a history of hypertension and hyperlipidemia.
His medical regimen consists of enalapril 10 mg twice
daily, carvedilol 6.25 mg twice daily, simvastatin 20 mg
daily, aspirin 81 mg daily, spironolactone 25 mg daily,
and furosemide 80 mg daily.
Examination fndings were as follows: afebrile, heart
rate 62 beats per minute, blood pressure 90/50 mm Hg,
signifcantly elevated jugular venous pressures with
prominent v waves, lungs clear, a 3/6 pansystolic murmur
at the apex, a third heart sound (S
3
), and pitting edema
(3+). Laboratory test results were as follows: hemoglo-
bin 10 g/dL, sodium 126 mEq/L, potassium 4.5 mEq/L,
serum urea nitrogen 42 mg/dL, and creatinine 1.6 mg/
dL. Te electrocardiogram showed a paced rhythm. A
recent exercise stress test done with sestamibi imaging
showed a large fxed defect in the anteroseptal wall with
no evidence of reversible ischemia and an LVEF of 20%.
Te patient was admitted to the hospital for intravenous
diuretic therapy. Te edema improved signifcantly, but
he continued to experience dyspnea on minimal exertion.
His creatinine increased to 2.0 mg/dL. What should be
the next step in this patients management?
a. Increase the carvedilol dosage to 12.5 mg twice daily.
b. Optimize cardiac resynchronization therapy.
c. Add metolazone 5 mg daily.
d. Perform a positron emission tomographic viability scan.
e. Refer for implantation of a lef ventricular assist device
(LVAD).
14. Which statement is false regarding serum brain natri-
uretic peptide (BNP) levels?
a. Serum BNP levels may be in the reference range in patients
with advanced heart failure.
b. A persistently elevated serum BNP level despite optimal
medical therapy is a marker of poor prognosis in heart
failure.
c. Optimization of medical therapy based on serum BNP lev-
els decreases hospitalizations in heart failure.
d. Serum N -terminal pro-BNP has a longer half-life than
serum BNP.
e. Te serum BNP level is frequently elevated in idiopathic
constrictive pericarditis.
9. A 25-year-old man, who is slender and athletic, comes
to your of ce because of an episode of syncope imme-
diately afer playing basketball. He exercises diligently
but says that he has experienced mild dyspnea on exer-
tion over the past 2 months. On physical examination,
his blood pressure is 175/90 mm Hg, his carotid pulse
is bounding, and there is an early systolic click imme-
diately followed by a late peaking 3/6 systolic murmur
heard best at the right second intercostal space and
radiating to the carotids. Tere is no dynamic increase
in murmur intensity with the Valsalva maneuver and
no change in the timing of the click with squatting.
Te apical impulse is sustained. A radial femoral delay
is suspected on examination. What should be the next
step in the evaluation or management of this patient?
a. Coronary angiogram
b. Use of a 24-hour Holter monitor
c. Computed tomographic (CT) scan of the chest
d. Reassurance with recommendations for warm-up and
cooldown routines with exertion
e. Use of a 24-hour blood pressure monitor
10. A 65-year-old patient with a history of coronary artery
bypass graf surgery 1 year ago presents with fatigue,
dyspnea, and progressive lower extremity edema.
Examination reveals a blood pressure of 120/70 mm
Hg and a pulse of 77 beats per minute. Te lungs are
clear. Te heart is quiet, with normal frst and second
heart sounds and no murmurs. Te jugular venous pres-
sure ( JVP) is elevated at midneck (approximately 20 cm
water). Tere is an increase in the JVP with inspiration
with a rapid descent. What does this combination of
fndings strongly suggest?
a. Heart failure due to biventricular dysfunction
b. Superior vena cava syndrome
c. Constrictive pericarditis
d. Failure of the bypass grafs and recurrent coronary artery
disease
11. A patient comes to you for evaluation of peripheral
edema and palpitations. On examination, a loud grade
3/6 holosystolic murmur is heard at the lef lower ster-
nal border, and an intermittent third heart sound (S
3
)
is heard that seems to be present only with inspiration.
A large v wave is noted in the jugular venous pressure
( JVP). What is your diagnosis?
a. Superior vena cava syndrome
b. Atrial fbrillation
c. Constrictive pericarditis
d. Signifcant pulmonary stenosis
e. Signifcant tricuspid regurgitation
12. Which of the following is true about the ausculta-
tory fnding of a fourth heart sound (S
4
) on physical
examination?
a. Ofen associated with hypertension
b. Normal in young children
c. Normal in athletes
d. A hallmark of the onset of atrial fbrillation
2. CARDI OLOGY QUESTI ONS AND ANSWERS

13
hypokinesis, ejection fraction 11%, and severe mitral
regurgitation. Coronary angiography is normal. Which
of the following treatment options would not prolong
this patients survival?
a. -Blockers
b. Biventricular pacing
d. Isosorbide dinitrate in combination with hydralazine
e. Spironolactone
e. Digoxin
18. A 28-year-old man presents with syncope. He has a
family history of sudden cardiac death. On physical
examination, his blood pressure is 120/70 mm Hg and
his heart rate is 68 beats per minute. Jugular venous
pressures show a prominent a wave. Carotid upstroke
is rapid; lef ventricular impulse is sustained and dis-
placed laterally. He has a prominent fourth heart sound
and a 3/6 systolic ejection murmur at the lef sternal
border radiating to the right second intercostal space
and apex. He has a 2/6 pansystolic murmur at the apex.
Te murmur at the lef sternal border is accentuated
with the squat-to-stand maneuver. His lungs are clear,
he has no hepatomegaly, and his extremities are not
edematous. Which of the following statements regard-
ing this patients disease state is false ?
a. A majority of patients are asymptomatic.
b. Te pathophysiology of the disease is characterized by
abnormalities in myocardial energetics.
c. Unexplained syncope is an indication for an implantable
cardioverter-defbrillator (ICD).
d. Patients with heart failure symptoms should be ofered
surgery.
e. All frst-degree relatives should undergo screening for this
disorder.
19. A 74-year-old woman presents with progressive dysp-
nea on exertion. She denies having chest pain. She has a
long-standing history of hypertension and chronic atrial
fbrillation. Her medications include warfarin 3 mg
daily, enalapril 5 mg daily, and digoxin 0.125 mg daily.
On physical examination, her blood pressure is 140/70
mm Hg and her heart rate is 82 beats per minute and
irregular. Jugular venous distention is present. A sof
2/6 pansystolic murmur was heard at the lef sternal
border. Her lungs were clear. Her extremities had pit-
ting edema (2+). Te electrocardiogram showed atrial
fbrillation with lef ventricular hypertrophy. Chest
radiography showed cardiomegaly with mild pulmo-
nary venous hypertension. Echocardiography showed
severe biatrial enlargement and lef ventricular hyper-
trophy with normal lef ventricular cavity dimensions
and an ejection fraction of 65%. Which of the following
would not be included in the diferential diagnosis for
this patients symptoms?
a. Constrictive pericarditis
b. Arteriovenous fstula
c. Amyloidosis
d. Coronary artery disease
e. Hypertensive heart disease
15. A 63-year-old woman presents with dyspnea on exer-
tion and tingling numbness in both lower extremities.
On physical examination, she is afebrile, her heart rate
is 92 beats per minute, and her blood pressure is 110/70
mm Hg in a sitting position and 90/60 mm Hg while
standing. She has jugular venous distention accentu-
ated with inspiration and prominent x and y descents.
Bibasilar crackles are present posteriorly. A fourth heart
sound is present. She has mild bilateral pitting edema
of the lower extremities. Sensation is diminished in
both feet, but refexes and motor power are preserved.
Laboratory test results were as follows: hemoglobin 10
g/dL, sodium 135 mEq/L, potassium 4.5 mEq/L, serum
urea nitrogen 16 mg/dL, and creatinine 1.0 mg/dL. Te
urinanalysis is remarkable for mild proteinuria. Te
electrocardiogram shows sinus rhythm, low voltage,
and an old anteroseptal infarct pattern. An echocardio-
gram shows a lef ventricular ejection fraction of 60%,
severely thickened lef ventricular walls with a speckled
appearance, and mild mitral regurgitation. What is the
best treatment option for this patient?
a. Stem cell transplant
b. Angiotensin-converting enzyme inhibitors
c. Combined heart and liver transplant
d. Diuretic therapy
e. Hospice care
16. A 57-year-old man with a history of diabetes mellitus and
hypertension presents to the hospital with new orthop-
nea and edema. His blood pressure is 95/60 mm Hg,
and his heart rate is 96 beats per minute. He has clear
lungs, a displaced cardiac apex with a third heart sound
but no cardiac murmurs, and no lower extremity edema.
Laboratory test results were as follows: hemoglobin 14
g/dL, sodium 137 mEq/L, potassium 4.4 mEq/L, and
serum creatinine 1.7 mg/dL. Chest radiography shows
pulmonary congestion with cardiomegaly. Te elec-
trocardiogram shows sinus rhythm, nonspecifc ST-T
changes, and frequent premature ventricular complexes
(PVCs). Echocardiography shows a lef ventricular ejec-
tion fraction of 20% and a dilated lef ventricle. What
should be the next step in evaluating this patient?
a. Endomyocardial biopsy
b. Stress testing
c. Coronary angiography
d. Holter monitoring
e. Sleep study
17. A 46-year-old African American woman with pulmo-
nary edema is transferred to your hospital. Her heart
rate is 105 beats per minute and regular; her blood pres-
sure is 130/70 mm Hg. Her jugular venous pressure is
normal. Her lungs are clear. She has a lef ventricular
lif and a 3/6 pansystolic murmur at the apex. A sum-
mation gallop is present. She has no hepatosplenomeg-
aly, ascites, or edema. Chest radiography is consistent
with pulmonary congestion and cardiomegaly. Te
electrocardiogram shows sinus rhythm with a lef bun-
dle branch block. Te echocardiogram shows global
14

MAYO CLINI C INTERNAL MEDI CINE BOARD REVIEW: QUESTI ONS AND ANSWERS
22. A 69-year-old woman presents with chest pain on exer-
tion. Past medical history is remarkable for hyper-
lipidemia, hypertension, abdominal aortic aneurysm
repair, and a stroke from which she has made a good
neurologic recovery. Current treatment includes aspi-
rin 81 mg daily, lisinopril 20 mg daily, and atorvastatin
40 mg daily. On physical examination, she weighs 59
kg, her pulse is regular at 68 beats per minute, and her
blood pressure is 140/84 mm Hg. Jugular venous pres-
sure is normal, and all peripheral pulses are normal. On
auscultation, heart sounds are normal and the lungs are
clear. Tere is no peripheral edema. A stress test is per-
formed: she exercises for 6 minutes in a Bruce protocol,
and typical symptoms develop with 2-mm ST-segment
depression in multiple leads. A coronary angiogram
shows a 90% stenosis in the midsection of the lef ante-
rior descending coronary artery. A drug-eluting stent
is deployed without complications, and the patient is
discharged with clopidogrel 75 mg daily in addition to
her usual medications. Five days later, she calls and says
that an intense pruritic rash has developed on her trunk
and limbs. She believes that it is due to the clopidogrel.
Which of the following is an absolute contraindication
for the use of prasugrel as an alternative to clopidogrel
in this patient?
a. Female sex
b. Age older than 65 years
c. Weight less than 60 kg
d. Percutaneous coronary intervention
e. History of stroke
23. A 68-year-old woman wants your advice about a recent
cardiology evaluation. Having had angina for 5 years
with increasingly limiting symptoms, she was advised
to have coronary angiography, which showed severe
3-vessel disease. An echocardiogram showed that the
lef ventricular ejection fraction was 40%. Current med-
ications include aspirin 81 mg daily, simvastatin 40 mg
daily, atenolol 50 mg daily, and long-acting isosorbide
dinitrate 60 mg daily. Physical examination fndings
include a pulse of 68 beats per minute, blood pressure
of 138/76 mm Hg, and normal jugular venous pressure
and heart sounds. Tere are no murmurs, and the lungs
are clear on auscultation. Which of the following is the
most appropriate recommendation for this patient to
treat her symptoms and improve the prognosis?
a. Increase the dosage of atenolol to 75 mg daily.
b. Add ranolazine to her treatment and reassess in 1 month.
c. Coronary artery bypass graf (CABG) surgery is the optimal
strategy.
d. Percutaneous coronary intervention (PCI) is the optimal
strategy.
e. PCI and CABG are equivalent strategies.
24. A 52-year-old man returns for a clinic visit afer an ante-
rior myocardial infarction 2 months ago. He was treated
successfully with primary percutanteous coronary
intervention. He has made an uncomplicated recov-
ery, completed cardiac rehabilitation, and returned to
CORONARY ARTERY DI SEASE AND
MYOCARDI AL I NFARCTI ON
20. A 64-year-old woman presents to a community hospi-
tal with a 5-day history of intermittent chest pressure
at rest. Te pain is persistent on the day of admission.
She has a history of chronic stable angina treated with
aspirin 325 mg daily, metoprolol 25 mg twice daily, and
isosorbide mononitrate 60 mg daily. Her past medi-
cal history is signifcant for hypertension and diabetes
mellitus. Additional medications include hydrochlo-
rothiazide 25 mg daily. On physical examination, her
pulse is regular at 50 beats per minute, and her blood
pressure is 158/88 mm Hg. Her jugular venous pressure
and all peripheral pulses are normal. On auscultation,
the heart sounds are normal and the lungs are clear.
Tere is trace peripheral edema. Te electrocardiogram
(ECG) shows a 1- to 2-mm ST-segment depression in
the inferior leads and in leads III, aVF, and V
6
. Te tro-
ponin T level is elevated (0.23 ng/mL). Afer treatment
with intravenous heparin and nitroglycerin, the patient
becomes asymptomatic with complete resolution of the
ECG abnormalities. Later that day, recurrent, transient
chest pressure and ST-segment depression develop. In
addition to arranging transfer to a hospital with facili-
ties for coronary angiography, which of the following is
the most appropriate next step?
a. Increase the dosage of metoprolol to 50 mg twice daily.
b. Perform exercise sestamibi testing.
c. Administer morphine intravenously.
d. Administer reteplase 10 units intravenously over 2 minutes.
e. Initiate an infusion of eptifbatide, a glycoprotein IIb/IIIa
inhibitor.
21. A 72-year-old woman presents to the emergency depart-
ment with dyspnea and retrosternal chest pain that
developed 3 hours ago, soon afer she had an intense
argument with her daughter. Her past medical history
is remarkable for hypertension and diabetes mellitus.
Te chest discomfort is not relieved by sublingual nitro-
glycerin. Her heart rate is 100 beats per minute, and her
blood pressure is 156/92 mm Hg. Her jugular venous
pressure, carotid pulse, and peripheral pulses are nor-
mal. On auscultation, heart sounds are normal and the
lungs are clear. Te cardiac troponin T level is 0.12 ng/
mL (reference range 0.01 ng/mL). Te 12-lead electro-
cardiogram shows a 0.5- to 1-mm ST-segment elevation
in precordial leads V
2
through V
4
. An emergency coro-
nary angiogram shows mild coronary atherosclerosis. A
lef ventriculogram shows severe hypokinesis of the api-
cal and midsegments of the heart with normal function
at the base. Which of the following is the most likely
diagnosis?
a. Myocarditis
b. Apical ballooning syndrome (takotsubo cardiomyopathy)
c. Acute coronary syndrome
d. Dilated cardiomyopathy
e. Pericarditis
2. CARDI OLOGY QUESTI ONS AND ANSWERS

15
protocol; toward the end of the test, typical symptoms
and 1-mm ST-segment depression develop. A coronary
angiogram shows a solitary 70% stenosis in the mid-
section of the right coronary artery. Te patient elects
medical therapy and returns to your clinic to discuss
whether he should have percutaneous coronary inter-
vention (PCI). What is the most appropriate advice on
the merits of medical therapy compared with PCI for
this patient?
a. PCI will reduce the need for optimal risk factor
management.
b. PCI will reduce the risk of myocardial infarction.
c. PCI will result in much less angina compared with medical
therapy.
d. Both medical therapy and PCI will result in similar
survival.
e. Te patient will eventually require PCI if medical therapy is
the initial treatment strategy.
VASCULAR MEDI CI NE
27. A 60-year-old man is evaluated for a 1-year history of
progressive right leg pain. He has a history of hyperten-
sion but not diabetes mellitus. His medications include
hydrochlorothiazide and aspirin. On examination,
his blood pressure is 150/70 mm Hg and his pulse is
70 beats per minute and regular. Cardiac examination
fndings are normal. Vascular examination identifes
palpable but diminished pulses in the popliteal and
posterior tibial arteries bilaterally with an absent dor-
salis pedis pulse on the lef. Which of the following fea-
tures would be most useful to distinguish intermittent
claudication from pseudoclaudication as the primary
cause of his symptoms?
a. History of nicotine addiction
b. Symptoms brought on by exertion
c. Sof bruit over the right common femoral artery
d. Ankle-brachial index of 0.80 on the right and 0.85 on the
lef
e. Symptom relief with sitting only
28. A 70-year-old man presents to the emergency depart-
ment with a 1-hour history of back pain radiating to
the anterior chest and neck. He describes the pain as
tearing and severe. His pain came on suddenly while
watching a football game on the television. His past
medical history includes hypertension and ongoing
tobacco use. On examination, he is uncomfortable.
His blood pressure is 190/100 mm Hg in the right arm
and 170/90 mm Hg in the lef arm with a regular pulse
of 110 beats per minute. His chest is clear on ausculta-
tion. On cardiac examination, there is a 2/6 holosys-
tolic murmur at the apex and a sof fourth heart sound.
Vascular examination identifes a slight delay in the lef
radial and ulnar pulses with a bruit over the lef sub-
clavian artery. Femoral, popliteal, and posterior tibial
pulses are symmetrical but diminished. On computed
tomographic (CT) imaging, a type B aortic dissection
work as a salesman. His risk factors for vascular disease
include smoking and hypertension. His current medi-
cations are aspirin 81 mg daily, metoprolol 50 mg twice
daily, clopidogrel 75 mg daily, simvastatin 40 mg daily,
and lisinopril 20 mg daily. On examination, his body
mass index is 29. His pulse rate is 64 beats per minute,
his blood pressure is 126/78 mm Hg, no murmurs or
gallops are noted on heart examination, and the lungs
are clear. Complete blood cell count and serum chem-
istry results are all within the reference ranges. Results
of a fasting lipid panel are as follows: low-density lipo-
protein cholesterol (LDL-C) 68 mg/dL, high-density
lipoprotein cholesterol (HDL-C) 34 mg/dL, and tri-
glycerides 186 mg/dL. Fasting blood glucose is 114 mg/
dL. For secondary prevention, in addition to smoking
cessation, what is the most important next step?
a. Intensify weight management and physical activity.
b. Increase the dosage of simvastatin to 80 mg daily.
c. Measure the plasma high-sensitivity C-reactive protein
(hs-CRP) level.
d. Continue the cardiovascular rehabilitation program
long-term.
e. Measure the plasma lipoprotein (a) level.
25. A 66-year-old woman presents with progressive
angina. Her past medical history is signifcant for
hyperlipidemia, hypertension, and diabetes mellitus.
On physical examination, her heart rate is 72 beats
per minute and her blood pressure is 144/88 mm
Hg. Te heart sounds are normal, and a fourth heart
sound is present. Te lungs are clear on auscultation,
and the peripheral pulses are normal. Current daily
medications include aspirin 325 mg, atenolol 25 mg,
lisinopril 10 mg, atorvastatin 20 mg, insulin, and met-
formin. Laboratory test results are a normal complete
blood cell count and creatinine 1.6 mg/dL. A stress
test is markedly positive for ischemia. Te cardiologist
has recommended proceeding with coronary angiog-
raphy. In addition to discontinuing use of metformin,
which of the following steps would be most important
before angiography?
a. Increase the dosage of atenolol to 50 mg daily.
b. Ensure that the patient is not dehydrated before the
procedure.
c. Add slow-release isosorbide mononitrate 60 mg daily.
d. Decrease the dosage of aspirin to 81 mg daily.
e. Start clopidogrel therapy with 75 mg daily.
26. A 62-year-old man presents with chest pain on exertion.
He can walk approximately one-half mile before symp-
toms develop. He has a history of hypertension and
hyperlipidemia. Current treatment includes lisinopril
40 mg daily and atorvastatin 20 mg daily. On physical
examination, his pulse is regular at 66 beats per minute
and his blood pressure is 134/86 mm Hg; his jugular
venous pressure is normal, and all peripheral pulses
are normal. On auscultation, heart sounds are normal
and the lungs are clear. Tere is no peripheral edema.
During a stress test, he exercises for 8 minutes in a Bruce
16

MAYO CLINI C INTERNAL MEDI CINE BOARD REVIEW: QUESTI ONS AND ANSWERS
c. Right popliteal deep vein thrombosis, heterozygous factor V
Leiden, and right fbular fracture 5 days prior
d. Symptomatic PE 10 days afer a 5-hour fight
e. Lef ovarian vein thrombosis afer vaginal hysterectomy
32. A 75-year-old right-handed woman is being evaluated
for a transient ischemic attack (TIA) that she experi-
enced earlier this morning. While clearing the break-
fast dishes, she noted a 2-minute episode of slurred
speech and right hand and facial numbness. She has a
history of hypertension but has been healthy otherwise
with no additional diagnoses. On examination, her
blood pressure is 150/80 mm Hg with a regular pulse of
72 beats per minute. Her chest is clear on auscultation.
Cardiac examination identifes a normal jugular venous
pulse and apical impulse. On auscultation, there is no
murmur or gallop. A bruit is heard over both carotid
arteries. Carotid upstrokes are normal. Te electrocar-
diogram shows normal sinus rhythm with normal inter-
vals and axes. Which of the following statements is true
for symptomatic carotid disease?
a. Afer a TIA in a patient with an ipsilateral internal carotid
artery stenosis greater than 70%, the combined outcomes of
stroke or death are improved with carotid endarterectomy
surgery compared with medical management.
b. Most TIAs result from a symptomatic ipsilateral carotid
lesion.
c. For an ipsilateral symptomatic carotid stenosis greater than
70%, surgery and medical therapy result in similar 2-year
stroke-free survival rates.
d. For the patient presented, carotid artery stenting with a distal
protection device will provide superior short- and long-term
stroke-free survival compared with carotid endarterectomy.
e. At 2 years, the rate of recurrent stenosis (>70%) is equal for
carotid stenting and endarterectomy.
33. A 46-year-old man with diabetes mellitus, ongo-
ing tobacco use, hypertension, and hyperlipidemia
complains of 2-block claudication in both legs. His
symptoms improve with standing and are consistent
from day to day. He has no rest pain or ulceration. His
symptoms have been stable for the past 2 years. His
blood pressure is 150/70 mm Hg with a regular pulse
of 80 beats per minute. His chest is clear on ausculta-
tion. His carotid upstrokes are normal without bruit.
Cardiac examination identifes a normal jugular venous
pressure and apical impulse. Findings on auscultation
are normal. He has normal pulses in the femoral and
popliteal arteries bilaterally without bruit. Te pedal
pulses are not palpable. His extremities do not have
ulcers, ischemic fssures, dependent rubor, or elevation
pallor. Te ankle-brachial index is 0.55 on the right and
0.52 on the lef. An angiogram performed at his local
medical facility 1 year ago showed severe infrapopliteal
arterial occlusive disease with diseased but patent prox-
imal arteries bilaterally. What is the most appropriate
treatment regimen for this patient with intermittent
claudication?
extends to the aortic bifurcation. Which of the follow-
ing is an indication for surgical intervention?
a. Type B aortic dissection
b. Nonperfused lef kidney identifed on the CT scan
c. Severe hypertension
d. Discordant brachial blood pressures, indicating compromise
of the lef subclavian artery by the dissection
e. Presence of the murmur, indicating disruption of the aortic
valve annulus by the dissection
29. A 75-year-old woman is evaluated for an abdominal
aortic aneurysm (AAA) found at a vascular screening
fair at her local supermarket. Her risk factors for ath-
erosclerosis include hypertension and hyperlipidemia.
She has not smoked in the past and has no history of
diabetes mellitus. On examination, her blood pressure
is 140/70 mm Hg in both arms with a regular pulse of
72 beats per minute. Her lungs are clear. Te carotid
upstrokes are normal. On abdominal examination, an
enlarged pulsatile mass is superior to the umbilicus.
Femoral, popliteal, and posterior tibial pulses are nor-
mal. Which of the following fndings should prompt
surgical referral for intervention?
a. A 5.1-cm infrarenal AAA
b. Annual growth rate of 0.3 cm
c. Accompanying lef iliac artery aneurysm measuring 2.8 cm
d. Father who died of a ruptured AAA
e. Tender aneurysm on examination
30. A 45-year-old man presents for evaluation in the emer-
gency department with acute onset of dyspnea and
severe cough. He has had no recent travel, trauma, or
surgery. He notes that his lef leg has been painful for
2 days, and this morning he noted some swelling. He
is dyspneic. His blood pressure is 86/60 mm Hg with a
regular pulse of 117 beats per minute. His chest is clear.
Cardiac examination identifes mild jugular venous dis-
tention with a subtle right ventricular lif. His lef leg is
slightly edematous. Computed tomographic angiogra-
phy of the chest identifes bilateral pulmonary emboli.
Duplex ultrasonography identifes an extensive lef
femoral-popliteal deep vein thrombosis (DVT). Which
of the following fndings should prompt inferior vena
cava (IVC) flter placement?
a. Hemodynamic instability with coexistent DVT and a large
pulmonary embolism (PE)
b. Mobile thrombus in the lef femoral vein identifed on ultra-
sonographic imaging
c. Family history of PE
d. History of PE 3 years ago
e. Retroperitoneal hemorrhage afer initiation of heparin
therapy
31. Which of the following conditions should prompt pro-
longed secondary prophylaxis with warfarin?
a. Portal vein thrombosis 3 weeks afer colonic resection for
ulcerative colitis
b. Bilateral pulmonary embolism (PE) in a 23-year-old woman
who uses oral contraception
2. CARDI OLOGY QUESTI ONS AND ANSWERS

17
37. A 54-year-old man has an elevated blood pressure (BP)
(150/94 mm Hg) that has been confrmed on several
of ce visits. He is a smoker with stable, mild claudica-
tion. He has hyperlipidemia, which is controlled with
diet and statin therapy. Lifestyle modifcation and drug
therapy are initiated for BP control. What is the recom-
mended BP goal for this patient?
a. Less than 140/90 mm Hg measured in the of ce
b. Less than 135/85 mm Hg measured at home
c. Less than 130/80 mm Hg measured in the of ce and at
home
d. Less than 125/75 mm Hg measured in the of ce
e. Less than 125/75 mm Hg measured at home
38. A 72-year-old woman has long-standing hypertension.
Two days afer total hip arthroplasty, her blood pressure
is 220/110 mm Hg, which is confrmed on a subsequent
measurement. She reports having substernal chest pres-
sure and mild dyspnea. An electrocardiogram shows
ST-segment depression in the inferior leads. What is
the most appropriate parenteral antihypertensive drug
to consider for this patient?
a. Sodium nitroprusside
b. Hydralazine
c. Labetalol
d. Nitroglycerin
e. Nicardipine
39. A 34-year-old woman has had episodes of headache
associated with diaphoresis and nausea. Tese episodes
begin suddenly and vary in duration from 15 to 30 min-
utes. Recently, a blood pressure of 210/140 mm Hg
was measured during an episode. Her family history is
signifcant for pheochromocytoma in her mother. On
examination, her blood pressure is 140/94 mm Hg.
Results of routine laboratory tests were normal. What
is the most appropriate next step in her evaluation?
a. Measure plasma free metanephrines.
b. Begin drug treatment with metoprolol.
c. Obtain a computed tomogram of the abdomen.
d. Obtain a duplex ultrasonogram of the renal arteries.
e. Measure plasma and urine catecholamines.
40. A 42-year-old man with a history of metabolically
active calcium oxalate nephrolithiasis has hypertension
that was recently diagnosed and is not controlled with
lifestyle modifcations. Which of the following drugs
would be the most appropriate initial choice for treat-
ing his hypertension?
a. Lisinopril
b. Furosemide
c. Losartan
d. Atenolol
e. Chlorthalidone
a. Computed tomographic angiography (CTA) of the legs
b. Magnetic resonance angiography (MRA) of the legs
c. Risk factor modifcation and the Canadian walking
program
d. Angioplasty and stenting
e. Bypass surgery
HYPERTENSI ON
34. A 45-year-old man with no complaints comes to you for
a general medical examination. His body mass index is
26, his blood pressure is 145/95 mm Hg, and his pulse
is 65 beats per minute. Te rest of the examination fnd-
ings are normal. He takes acetaminophen for occasional
arthritis pain. He has no other signifcant past medical
or surgical history. What would be your next step in
management?
a. Recheck blood pressure in 2 years.
b. Recheck blood pressure in 1 year.
c. Recheck blood pressure in 6 months.
d. Recheck blood pressure in 2 months.
e. Evaluate and treat blood pressure now.
35. A 34-year-old sexually active woman has a blood pres-
sure of 150/94 mm Hg that has been confrmed on
several of ce visits and with self-monitoring despite a
6-month trial of lifestyle modifcations. Which of the
following drugs would be most appropriate for this
patient?
a. Lisinopril 10 mg once daily
b. Doxazosin 2 mg once daily
c. Losartan 50 mg once daily
d. Hydrochlorothiazide 12.5 mg once daily
e. Aliskiren 150 mg once daily
36. A 40-year-old woman is referred for further evaluation
of an elevated of ce blood pressure (BP) of 150/96 mm
Hg. Her friend, who is a nurse, has measured her BP
several times at home, where it ranged from 130/80 to
136/88 mm Hg. Te patient eats a high-salt diet and
does not exercise. Her mother has hypertension. On
examination, the patients BP is 146/94 mm Hg; exam-
ination fndings are otherwise normal. Routine labora-
tory test results are within the reference ranges. What is
the most appropriate next step in evaluating or treating
this patient?
a. Discuss lifestyle modifcations and begin therapy with
atenolol.
b. Begin therapy with a low dose of hydrochlorothiazide.
c. Obtain a duplex ultrasonogram of the renal arteries.
d. Begin therapy with a low dose of hydrochlorothiazide in
combination with lisinopril.
e. Obtain a 24-hour ambulatory BP recording.
18
be permanent pacemaker implantation. Te other causes of
syncope are most unlikely in this patient.
4. Answer d.
CRT is indicated for patients who have LVEF less than 35%,
LBBB with QRS duration of more than 120 ms, and conges-
tive heart failure in NYHA class III or IV despite receiving
the maximally tolerated medical therapy. Te most appro-
priate management is to up-titrate the dosages of lisinopril
and carvedilol now and reevaluate the patient over the next
3 to 6 months to determine whether CRT is indicated.
5. Answer d.
Te patient is well compensated from an HF standpoint.
Her episodes of AF are infrequent and not particularly long
in duration. Terefore, use of a daily antiarrhythmic drug
(eg, sotalol) is not warranted (toxicity could occur over the
long term with limited beneft). Dronedarone is contrain-
dicated in patients with HF or recent decompensation of
HF. (See K ber et al in the Suggested Reading list.)
6. Answer d.
Tis patient is asymptomatic and presents with bradycardia
that may be readily explained by her athletic conditioning.
Her murmur is systolic, sof (less than grade 2), and heard
1. Answer b.
Te ECG is typical for Wolf-Parkinson-White syndrome.
Since the patient is asymptomatic, there is no indication
for drug therapy or an electrophysiology study unless he
is in a high-risk occupation (eg, pilot, military member).
With the onset of symptoms (palpitations, class I indica-
tion), the preferred approach would be an electrophysiol-
ogy study and catheter ablation (>95% success rate with a
low risk of complications).
2. Answer a.
Te clinical decision on how best to treat the patients
atrial fbrillation (rate control or rhythm control) should
be based on symptom burden and adequacy of rate control,
independently of stroke risk, as determined by calculating
her CHADS2 (congestive heart failure, hypertension,
age >75 years, diabetes mellitus, and previous stroke) risk
score. Her CHADS2 score is greater than 2. According to
current guidelines, warfarin is indicated.
3. Answer a.
Te symptoms and presentation are consistent with car-
diogenic syncope and conduction system disease (there is
electrocardiographic evidence of high-grade atrioventricu-
lar block and lef bundle branch block). Treatment would
Presence of cardiac murmur
Diastolic or continuous murmur Systolic murmur
Grade 1 or 2 &
midsystolic
Asymptomatic
& no associated
fndings
No further workup
Other signs or
symptoms of
cardiac disease
Echocardiography
Catheterization &
angiography if required
Grade 3 or higher
holosystolic or late
systolic
Figure 2.A6. (Adapted from Bonow RO, Carabello B, De Leon AC Jr, Edmunds LH Jr, Fedderly BJ, Freed MD, et al. ACC/AHA guidelines for the
management of patients with valvular heart disease: a report of the American College of Cardiology/American Heart Association. Task Force on Practice
Guidelines [Committee on Management of Patients with Valvular Heart Disease]. J Am Coll Cardiol. 1998 Nov;32[5]:1486588. Used with permission.)
ANSWERS
2. CARDI OLOGY QUESTI ONS AND ANSWERS

19
is not indicated in ASD or PFO. Chest radiography may
be helpful. Classically, with large shunts radiography shows
enlargement of the right atrium (bulging of the right heart
border), RV (decreased retrosternal clear space), and pul-
monary artery and increased pulmonary vascular markings.
Common anatomical types of ASD are ostium secundum
ASD, ostium primum ASD, and sinus venosus ASD. Sinus
venosus ASD is commonly accompanied by anomalous
pulmonary venous return. Ostium primum ASD is com-
monly associated with clef mitral valve and other complex
congenital heart diseases. (See Levin et al, OToole et al,
and Wilson et al in the Suggested Reading list.)
8. Answer c.
Tis 30-year-old patient presents with atypical chest pain,
which may be associated with mitral valve prolapse. Mitral
valve prolapse is essentially a mismatch of the lef ven-
tricular cavity size and the mitral valve leafets, which are
redundant. Te classic bedside maneuver is the squat-to-
stand maneuver; however, any maneuver that increases the
lef ventricular cavity size will delay midsystolic clicks and
the mitral regurgitant murmur (Figure 2.A8). Te Valsalva
maneuver decreases venous return, which would result in
a smaller lef ventricle and thus an earlier mitral valve pro-
lapse. A postextrasystolic beat (due to the compensatory
pause), the passive leg raise, and squat (all would improve
lef ventricular flling) should actually delay midsystolic
clicks and murmur.
in the pulmonary position, and it disappears with the
Valsalva maneuver. If you were worried about hypertrophic
cardiomyopathy, the response to this maneuver fairly well
argues against it. Tere are no associated systolic clicks of
mitral valve prolapse or bicuspid aortic valve, no diastolic
snaps to suggest mitral stenosis, and the electrocardiogram
is normal with the exception of the slow heart rate. Tus,
the murmur is most likely a benign fow murmur due to a
thin chest wall, and you are likely hearing pulmonary out-
fow, anatomically located just below the lef upper sternal
border. An S
3
is a normal fnding in a young athlete. Tere
is no need for further testing (Figure 2.A6).
7. Answer c.
Several clinical fndings in ASD provide clues to the condi-
tion: 1) RV lif or sternal lif; 2) palpable pulmonary artery
pulse consistent with pulmonary artery volume and pres-
sure overload of the RV (associated with signifcant shunt-
ing); 3) fxed splitting of the S
2
(the hallmark of ASD); 4)
accentuation of the tricuspid valve closure leading to split-
ting of the frst heart sound; 5) midsystolic pulmonary ejec-
tion murmur due to increased volume of fow through the
RV outfow tract; and 6) if a large shunt is present, a mid- to
late-diastolic rumble may also be appreciated. PFO is usu-
ally not associated with any signifcant physical examination
fndings, and it is a relatively common fnding in autopsy
studies; on average, PFO is seen in 25% of otherwise
healthy people. Subacute bacterial endocarditis prophylaxis
Supine
Standing
Squatting
S
1
C
S
2
S
1
C
S
2
S
1
C
S
2
Figure 2.A8. C indicates click; S
1
, frst heart sound; S
2
, second heart sound. (Adapted from Shaver JA, Leonard JJ, Leon DF. Examination of the heart.
Part 4: Auscultation of the heart. Dallas [TX]: American Heart Association; c1990. p. 13. Used with permission.)
20

MAYO CLINI C INTERNAL MEDI CINE BOARD REVIEW: QUESTI ONS AND ANSWERS
13. Answer e.
Te patient presents with stage D congestive heart fail-
ure. He has several poor prognostic indicators, including
recurrent hospitalizations for heart failure exacerbations
despite optimal medical therapy, low blood pressure, ane-
mia, renal insuf ciency, and a decreasing LVEF. His blood
pressure would not allow an increase in -blocker dose.
Optimization of cardiac resynchronization therapy is
unlikely to help patients with stage D heart failure and a
low output state. Adding a thiazide diuretic when edema
has improved and renal insuf ciency has worsened would
not be an appropriate next step. Te patient had a remote
history of myocardial infarction and had no active angina
pectoris; hence, determining myocardial viability with a
view toward revascularization is unlikely to provide much
beneft toward improving the patients symptoms of heart
failure or survival. Patients with dilated cardiomyopathy,
an LVEF of 25% or less, and New York Heart Association
(NYHA) class IV symptoms should be considered for
LVAD implantation afer optimization of medical and
electrical therapy. LVADs have been shown to prolong sur-
vival, improve NYHA functional class, and improve qual-
ity of life.
14. Answer e.
Although serum BNP levels correlate with clinical sever-
ity, some patients with advanced heart failure have normal
BNP levels. Several large clinical trials have shown that
a persistently elevated BNP level despite ongoing medi-
cal therapy is a marker of poor prognosis. A BNP-guided
approach to titrate medical therapy has been shown to
decrease hospitalizations compared with standard therapy,
but the beneft may result from more intense medical sur-
veillance and subsequent up-titration of medical therapy;
therefore, its routine use for this purpose cannot be rec-
ommended. Although the serum BNP level is frequently
elevated in constrictive pericarditis due to cardiac surgery
or radiotherapy, it is frequently within the normal range in
patients with idiopathic constrictive pericarditis.
15. Answer c.
Te patient has severe heart failure with preserved ejection
fraction. Te cause of heart failure in this patient is cardiac
amyloidosis, which is consistent with the low voltage on
the electrocardiogram and the thickened lef ventricular
walls with a classic speckled pattern on the echocardio-
gram. Te concomitant presence of peripheral neuropathy
is consistent with a diagnosis of familial amyloidosis. In
familial amyloidosis, the source of the amyloidogenic vari-
ant of the transthyretin protein is the liver. Hence, the best
treatment option for this patient is combined heart and
liver transplant.
16. Answer c.
Te patient has dilated cardiomyopathy. Te most com-
mon cause of dilated cardiomyopathy, especially in a person
with atherosclerotic risk factors, is coronary artery disease.
9. Answer c.
Tis patient, with a systolic click murmur in the absence
of dynamic change, likely has bicuspid aortic valve (AV)
stenosis. Tis diagnosis is more commonly associated with
a coarctation of the aorta whose fow murmur is best heard
over the posterior aspect between the shoulder blades,
slightly to the lef of the midline. In the presence of hyper-
tension and radial-femoral delay, there is enough evidence
on examination to support imaging of the aorta to rule out
coarctation. A CT scan (or magnetic resonance imaging)
of the chest would show the coarctation. Another appro-
priate test would be transthoracic echocardiography to
confrm the diagnosis of a bicuspid AV and to assess the
degree of aortic stenosis. Echocardiography is ofen used to
view the descending thoracic aorta to determine the degree
of coarctation; however, if the aorta is not well visualized
by echocardiography, other imaging is indicated to rule
out the coarctation. A coronary angiogram is unlikely to
be helpful unless it is coupled with an aortogram, which
might provide the diagnosis, but there may be an increased
danger due to the potential for coarctation, which could be
dif cult to cross with the catheter. Tis patient most likely
does not have coronary artery disease. Te murmur and
clinical presentation are not benign, and a thorough evalu-
ation is warranted. Tis patients syncopal event is related
to aortic stenosis or coarctation with signifcant stenosis
and the inability to adequately increase cardiac output,
especially with postexercise vasodilatation.
10. Answer c.
Constrictive pericarditis is a relatively infrequent compli-
cation of open heart surgery, yet cardiac surgery is now the
number one cause of constrictive pericarditis causing heart
failure. Te heart failure ofen begins slowly, with symptoms
of dyspnea, peripheral edema, and signs of elevated neck veins
and the Kussmaul signelevation of JVP with inspiration.
Another fnding ofen reported on the JVP is rapid descent.
11. Answer e.
Te presence of a prominent v wave in the JVP profle is
consistent with signifcant tricuspid regurgitation. Te
right-sided S
3
may be intermittent and more prominent
with the inspiratory phase of respiration. Mitral regurgita-
tion will produce a v wave into the pulmonary circulation,
which can be seen with Swan-Ganz catheter tracings, but the
v wave is generally not transmitted to the JVP. Constrictive
pericarditis has very rapid y descents. Pulmonary stenosis is
associated with a giant a wave due to increased atrial con-
traction caused by right ventricular pressure overload. In
superior vena cava (SVC) syndrome, the pulsatile waves are
ofen lost because of extra-cardiac obstruction of the SVC.
12. Answer a.
An S
4
indicates increased lef ventricular stifness and is
commonly heard in adults with hypertension. It is caused
by the increased flling in late diastole during atrial contrac-
tion. An S
4
is never normal.
2. CARDI OLOGY QUESTI ONS AND ANSWERS

21
hypertensive heart disease. However, she should be evalu-
ated for possible constrictive pericarditis and amyloidosis.
Rarely, arteriovenous fstulae can also cause high-output
heart failure. Coronary artery disease can be present in
patients with heart failure and normal ejection fraction and
frequently needs to be excluded, but it would not account
for the patients symptoms and signs of heart failure.
20. Answer e.
Te patient presents with a nonST-elevation myocardial
infarction. Her age, ECG changes, and elevated levels of
biomarkers give her a high-risk profle. An initially conser-
vative medical management strategy is reasonable because
she presented to a community hospital. However, recur-
rence of chest pain is a clear indication for transferring her
to a facility with a cardiac catheterization laboratory. Several
studies have shown that glycoprotein IIb/IIIa inhibitors
are efective in the management of nonST-segment eleva-
tion acute coronary syndromes, particularly in high-risk
patients who require percutaneous coronary intervention.
Initiating therapy with a glycoprotein IIb/IIIa inhibi-
tor such as eptifbatide is efective in reducing recurrent
ischemia and recurrent myocardial infarction before the
coronary intervention and improves outcomes afer percu-
taneous coronary intervention. Increasing the dose of the
-blocker would not be advisable in view of the bradycar-
dia. Stress testing is absolutely contraindicated for patients
with acute ischemic symptoms. Morphine may be adminis-
tered to relieve symptoms but is not a defnitive therapy for
severe ischemia. Trombolytics such as reteplase are not
indicated in nonST-segment elevation acute coronary
syndromes.
21. Answer b.
Te patient presents with symptoms consistent with an
acute coronary syndrome with ST-segment elevation on the
electrocardiogram. Terefore, it was reasonable to perform
emergency angiography, which demonstrated normal cor-
onary arteries. Te diferential diagnosis includes coronary
spasm, coronary embolism, pericarditis, and myocarditis.
However, this patient had a very characteristic regional
wall motion abnormality on the lef ventriculogram, which
involved the mid and apical segments of the lef ventricle
with sparing of the basal segments. Tis entity has recently
been recognized as the apical ballooning syndrome (tako-
tsubo cardiomyopathy). Apical ballooning syndrome
occurs predominantly in postmenopausal women and is
frequently preceded by mental or physical stress. Acute
coronary syndrome was ruled out by coronary angiography.
Myocarditis is a diferential diagnosis, although it typically
produces global lef ventricular dysfunction. Pericarditis is
not associated with systolic dysfunction.
22. Answer e.
Prasugrel is a new antiplatelet agent that acts by irrevers-
ibly blocking adenosine diphosphate receptors on plate-
lets, preventing their activation and aggregation. It has
been approved for use in patients with unstable angina,
Te most appropriate next step for this patient would be
to perform coronary angiography, the best diagnostic test
in this circumstance. Stress testing may have false-positive
or false-negative results in patients with dilated cardio-
myopathy. Endomyocardial biopsy should not be routinely
performed in the evaluation of patients with heart failure.
Holter monitoring is useful in detecting the PVC burden
over a 24-hour period, and if the PVC burden were espe-
cially high, it could be implicated as a rare cause of heart
failure. However, Holter minitoring would not be the next
step in evaluation and should not be performed without
initiating medical therapy because indicated medical treat-
ment (eg, -blockers) can attenuate PVCs. Sleep apnea
could be a frequent accompaniment of decompensated
heart failure, but a sleep study should be performed only
afer patients are optimally treated.
17. Answer e.
-Blockers, spironolactone, isosorbide dinitrate in com-
bination with hydralazine, and biventricular pacing have
improved survival among patients with severe heart fail-
ure and idiopathic dilated cardiomyopathy. However, the
Digitalis Investigation Group trial did not show a survival
beneft with the use of digoxin in heart failure. Te lack
of beneft may have been related to a higher serum digi-
talis level resulting in a higher mortality presumably due to
a proarrhythmic efect. Digitalis retains a role in treating
patients who have persistently symptomatic heart failure
and patients who have atrial fbrillation with rapid ventric-
ular response and heart failure.
18. Answer d.
Te diagnosis is hypertrophic cardiomyopathy (HCM) and
is made primarily by the physical fndings. Although hos-
pitalized patients present with syncope, chest pain, heart
failure, or sudden cardiac death, most patients with HCM
are asymptomatic. Te pathophysiology of this autosomal
dominant disease has been linked to genetic mutations,
predominantly in the sarcomere and mitochondria, that
result in either increased energy use or decreased energy
production, which ultimately promotes myocyte growth.
Hence, all frst-degree relatives should be evaluated with
electrocardiography and echocardiography. Patients with a
personal or family history of sudden cardiac death, unex-
plained syncope, nonsustained ventricular tachycardia on
Holter monitoring, severe lef ventricular hypertrophy, or
an abnormal blood pressure response to exercise are at the
highest risk for sudden cardiac death and should be consid-
ered for ICD implantation. Treatment of the patient with
HCM and heart failure is initially medical, with -blockers
or calcium channel blockers (or both), but if symptoms are
progressive despite medical therapy, myomectomy or alco-
hol septal ablation should be considered.
19. Answer d.
Tis patient presents with evidence of heart failure with
a normal lef ventricular ejection fraction. She has the
classic presentation of diastolic heart failure likely due to
22

MAYO CLINI C INTERNAL MEDI CINE BOARD REVIEW: QUESTI ONS AND ANSWERS
prophylactic measures include administering intravenous
fuid and N -acetylcysteine before the procedure. Initiating
the use of clopidogrel before angiography is not routine
practice. Te most appropriate time for adjusting the medi-
cal therapy would be afer coronary angiography as part of
the overall management strategy. In the presence of abnor-
mal renal function, it is important to discontinue the use
of metformin before coronary angiography. Tere is an
increased risk of lactic acidosis among diabetic patients
who have renal impairment and contrast nephropathy and
are receiving metformin. Terefore, it is best practice to
discontinue the use of metformin before elective proce-
dures and to resume it 48 hours afer the administration of
radiologic contrast material if the patient has an uncompli-
cated recovery.
26. Answer d.
Tis patient has moderate symptoms with single-vessel coro-
nary artery disease. PCI is an established, efective therapy
for coronary heart disease; however, it has not been shown
to reduce the risk of myocardial infarction or improve sur-
vival among patients with stable angina. Previous studies
have suggested that PCI reduces the amount of medica-
tions required and provides better symptomatic relief.
Te Clinical Outcomes Utilizing Revascularization and
Aggressive Drug Evaluation (COURAGE) trial confrmed
that with optimal medical therapy, PCI does not ofer signif-
icant advantage. (See Boden et al in the Suggested Reading
list.) However, PCI was associated with slightly lower use of
antianginal medications and slightly better symptom con-
trol. Tus, for this particular patient both treatment strate-
gies are equal for survival and risk of myocardial infarction.
PCI is likely to lead to less antianginal medication use and
better control of angina in the short term.
27. Answer e.
Distinguishing true claudication from pseudoclaudica-
tion can be accomplished with a good history and physi-
cal examination. Patients with true claudication typically
describe cramping pain with muscle fatigue occurring in
the calf and progressing to the thigh and buttock with con-
tinued exercise. Te classic description includes exertional
pain involving the calf that impedes walking, resolves
within 10 minutes of rest, and neither begins at rest nor
resolves on walking. In contrast, pseudoclaudication
occurs both with walking and with prolonged standing
or any activity associated with spinal extension. Patients
with pseudoclaudication must sit (with spinal fexion) to
relieve the pinching of the involved nerve roots. Sorting
out the cause of patients symptoms can be dif cult since
many have both true claudication and pseudoclaudication.
Te patient described in this question has peripheral artery
disease (PAD), but from the data provided, his symptoms
cannot be defnitely attributed to PAD. Only 33% of
patients with PAD have classic symptoms of claudication.
Indeed, more than half of patients with documented PAD
have no symptoms of claudication. (See McDermott et al
in Suggested Reading list.)
nonST-segment elevation myocardial infarction, or
ST-segment elevation myocardial infarction managed with
percutaneous coronary intervention. Its use is absolutely
contraindicated in patients with active pathologic bleeding
or a history of transient ischemia attack or stroke. It is gen-
erally not recommended (but may be used) in patients who
are 75 years or older or who weigh less than 60 kg. It is not
contraindicated in women, but women have an increased
risk of bleeding.
23. Answer c.
Te patient has severe 3-vessel disease with reduced ejec-
tion fraction and signifcant angina despite medical ther-
apy. CABG surgery is the treatment of choice to relieve
symptoms and to improve long-term prognosis. PCI is
increasingly being performed for multivessel disease but is
not the standard of care for revascularization in patients
with severe 3-vessel disease. Medical therapy alone in
patients with 3-vessel disease and decreased ejection frac-
tion is associated with worse outcomes compared with sur-
gery followed by optimal medical therapy.
24. Answer a.
Tis patient has premature coronary artery disease.
Aggressive risk factor management is required. Smoking
cessation and adequate treatment of hypertension are
essential. Te patients blood pressure appears to be below
the recommended goal of 130/80 mm Hg. Te Adult
Treatment Panel III (ATP III) guidelines for managing
hyperlipidemia recommend an optimal LDL-C goal of less
than 70 mg/dL. Tis patient has achieved that goal. Te
ATP III guidelines also recommend screening and treat-
ing for the metabolic syndrome as a secondary therapeu-
tic goal for managing hyperlipidemia. Tis patient meets
criteria for metabolic syndrome: the level of triglycerides
is high ( 150 mg/dL), HDL-C is low ( 40 mg/dL), and
fasting blood glucose is high ( 110 mg/dL). Te guidelines
state that the initial step in managing metabolic syndrome
is to treat underlying causes, such as overweight or obesity
and physical inactivity. If these measures are inadequate,
pharmacologic therapy for low HDL-C or elevated tri-
glycerides (or both) is indicated. Increasing the dose of
simvastatin from 40 to 80 mg is not indicated since the
patient has achieved the LDL-C goal and an increase in
the dose is unlikely to substantially increase the HDL-C
level. Elevated levels of hs-CRP are associated with an
increased risk of future events, but the precise clinical
utility of measuring hs-CRP has not been established.
Long-term cardiovascular rehabilitation therapy has not
been shown to improve outcome. Elevated plasma lipopro-
tein (a) levels are associated with an increased risk of car-
diovascular events; however, no therapeutic intervention
has been shown to improve outcomes among patients who
have elevated lipoprotein (a) levels.
25. Answer b.
A patient with diabetes mellitus and chronic renal failure
has an increased risk of contrast nephropathy. Reasonable
2. CARDI OLOGY QUESTI ONS AND ANSWERS

23
warfarin therapy. Te family history of PE is not relevant
to this decision. For cancer patients with recurrent throm-
bosis despite adequate warfarin therapy, the proper treat-
ment is conversion to low-molecular-weight heparin rather
than IVC flter placement. (See Baglin et al in Suggested
Reading list.)
31. Answer d.
When defning appropriate venous thromboembolism
treatment (ie, the frst 3 months of therapy) and dura-
tion of secondary prophylaxis (ie, the time beyond the
initial 3 months of therapy), it is important to distinguish
provoked from unprovoked or spontaneous thrombotic
events. In general, events provoked by a transient risk
factor (as in answer choices a - c and e ) are treated with 3
months of warfarin therapy. While heterozygous factor V
Leiden is a congenital risk factor, it carries a rather weak
propensity for venous thrombosis, with a hazard ratio of
approximately 3. Terefore, neither heterozygous factor V
Leiden nor heterozygous prothrombin G20210A muta-
tions require prolonged secondary prophylaxis in and of
themselves. One might argue that a 5-hour fight 10 days
before the event was a provocation, but the hazard asso-
ciated with such an exposure is low. A stronger argument
is that the PE was unrelated to the air travel and was an
unprovoked event. Idiopathic or unprovoked events carry
a higher rate of recurrence. Furthermore, if this patient has
a recurrent event, it is twice as likely to be another PE. PE,
and especially recurrent PE, carries a high mortality rate.
Severe symptomatic PE in the period immediately afer
travel is extremely rare afer fights of less than 8 hours. In
fights longer than 12 hours, the rate is 5 per million. (See
Kearon et al and see Watson and Baglin in the Suggested
Reading list.)
32. Answer a.
Nearly 800,000 strokes occur each year in the United
States, and stroke is the third leading cause of death and
the leading cause of long-term disability. Most strokes are
ischemic and thromboembolic in nature. Te prevalence of
moderate to severe carotid disease in the United States is
5% to 9% in persons older than 65, accounting for 1.3 to 2.4
million Americans. Te annual prevalence of TIA is nearly
5 million. Of these, one-third occur in patients with mod-
erate to severe carotid stenosis. Both the Asymptomatic
Carotid Atherosclerosis Study (ACAS) and the North
American Symptomatic Carotid Endarterectomy Trial
(NASCET) showed that surgery improved stroke-free
survival at 2 years compared with medical management
in symptomatic patients. Te Endarterectomy Versus
Angioplasty in Patients With Symptomatic Severe Carotid
Stenosis (EVA-3S) trial and the International Carotid
Stenting Study showed that carotid endarterectomy was
the procedure of choice for symptomatic carotid stenosis
compared with carotid stenting (with embolic protection
devices). Te Stent-Protected Angioplasty Versus Carotid
Endarterectomy (SPACE) study found that carotid stent-
ing resulted in a signifcantly greater rate of restenosis
28. Answer b.
Tis patient has a descending aortic dissection. By defni-
tion, the proximal tear occurs distal to the lef subclavian
artery orifce and adjacent to the ligamentum arteriosum. In
contrast to type A (ascending) thoracic aortic dissections,
which are managed surgically (mortality is 1% per hour
without surgery), type B dissections are typically managed
medically. Indications for surgical intervention for type B
dissections include dissection progression despite medical
management, persistent or recurrent pain despite medical
management, and end-organ ischemia due to branch lumi-
nal compromise from the dissection fap. In this patient,
the dissection does not involve the lef subclavian artery
by defnition. Te discordant brachial blood pressures are
a result of prolonged tobacco exposure and associated sub-
clavian arterial atherosclerosis. Te murmur described is
from mitral insuf ciency, not from aortic valve disruption.
(See Hiratzka et al in the Suggested Reading list.)
29. Answer e.
Timing of AAA repair (either surgically or by endograf)
is important and requires balancing the risk of rupture
with the risk associated with the intervention. Currently,
AAA repair is recommended if the diameter is 5.5 cm. Tis
measurement should be anteroposterior and perpendicu-
lar to the axis, which can be dif cult with tortuous aorta.
Other indications for repair include annual growth rates
exceeding 0.5 cm or any symptoms suggesting instability,
such as tenderness of the aorta on examination. Te iliac
artery aneurysm is small and would not require surgery.
Te typical cutof for iliac artery aneurysms is 3.5 cm. Te
family history of ruptured AAA is not a criterion for sur-
gery. Te guidelines support a one-time screening for AAA
in men aged 65 to 75 who have never smoked but who have
a frst-degree relative who required repair of an AAA or
died of a ruptured AAA (grade 2C). A one-time screening
for AAA with abdominal ultrasonography is also recom-
mended for men aged 65 to 75 who have smoked (grade
1A). Screening for asymptomatic women is not recom-
mended; however, Medicare allows for a one-time screen-
ing for AAA in women with a family history of AAA. (See
Hirsch et al in the Suggested Reading list.)
30. Answer e.
More IVC flters are placed in medical institutions in the
United States than in any other country. Current indica-
tions for IVC flter placement include inability to provide
anticoagulation because of active bleeding or circumstances
in which the risk of initiating major bleeding is high (eg,
recent surgery). Te proper treatment of hemodynamic
instability with DVT and a PE would not be placement
of an IVC flter but rather consideration of thrombolytic
therapy. Te fnding of mobile thrombi (rather than immo-
bile thrombi) on cross-sectional imaging does not increase
the risk of embolism. A prior personal history of PE would
not justify IVC flter placement; however, it would sug-
gest the need for prolonged secondary prophylaxis with
24

MAYO CLINI C INTERNAL MEDI CINE BOARD REVIEW: QUESTI ONS AND ANSWERS
hypertension may evolve into sustained hypertension
over time. Tus, when hypertension is diagnosed, patients
should be advised on lifestyle modifcations associated
with lowering BP and be followed. Self-measurement of
BP should be encouraged.
37. Answer c.
In general, the of ce BP goal for persons with hyperten-
sion is less than 140/90 mm Hg and the home BP goal
is less than 135/85 mm Hg. Current guidelines recom-
mend a more aggressive BP goal of less than 130/80 mm
Hg for persons who have diabetes mellitus, chronic kid-
ney disease, known vascular disease, or a high risk of vas-
cular disease. Tis same goal applies for both of ce and
self-measured readings. A goal of less than 125/75 mm Hg
is appropriate for patients who have renal disease associ-
ated with proteinuria.
38. Answer d.
Tis case should be considered a hypertensive emergency
because the patient has evidence of organ injury (coronary
ischemia and infarction). Terefore, an immediate reduc-
tion in blood pressure with a parenteral agent is indicated.
Of the available drugs, nitroglycerin is preferred with myo-
cardial ischemia. It is a balanced arterial and venous dilator
and lessens myocardial oxygen demand by reducing both
preload and aferload. Hydralazine is a direct arterial vaso-
dilator and may worsen myocardial ischemia. Te other
medications listed would be acceptable second-line agents
for this patient. Of these, sodium nitroprusside is the best
studied.
39. Answer a.
Te presentation is highly suggestive of familial pheochro-
mocytoma. Te frst step is to establish the diagnosis by
measurement of free metanephrines in the plasma, which
is the screening test of choice, especially if a familial dis-
order is suspected. Plasma and urine catecholamines lack
diagnostic accuracy. Computed tomography or magnetic
resonance imaging of the abdomen is appropriate as the
initial test to locate the tumor since 90% are located in 1
or both adrenal glands and 98% are located in the abdo-
men. A search for the tumor should follow biochemical
confrmation of the diagnosis. Ten an -blocker should
be used as initial therapy. -Blocker monotherapy can be
associated with a paradoxical increase in blood pressure,
but a -blocker may be used to treat tachycardia that may
occur with adequate -blocker therapy.
40. Answer e.
Tiazide diuretics are associated with decreased urinary
excretion of calcium and are ofen used to treat calcium
nephrolithiasis to reduce the risk of recurrent stone for-
mation. Tus, for this patient, chlorthalidone would be an
appropriate frst agent to treat hypertension. Furosemide
is associated with an increase in urinary calcium excretion
and should not be considered if a patient has concomitant
calcium stone disease.
(>70%) at 2 years (11.1% vs 4.6%) compared with endar-
terectomy. (See Ederle et al, European Carotid Surgery
Trialists Collaborative Group, Mas et al, North American
Symptomatic Carotid Endarterectomy Trial Collaborators,
and Ringleb et al in the Suggested Reading list.)
33. Answer c.
Tis patient has a classic presentation of infrapopliteal arte-
rial occlusive disease with long-standing diabetes mellitus.
Both the latest physical examination and the previous con-
ventional angiogram support this impression. It would not
be helpful to repeat either MRA or CTA now. Angioplasty
and stenting of these distal arteries would have poor dura-
bility. Tere is no indication for surgical bypass since
the patient has neither rest pain nor ulceration. Te best
option for this patient is risk factor modifcation (smoking
cessation, statin therapy, hypertension control, and diabe-
tes therapy). (See Hirsch et al in the Suggested Reading
list.)
34. Answer d.
Te Seventh Report of the Joint National Committee on
Prevention, Detection, Evaluation and Treatment of High
Blood Pressure ( JNC 7) provides follow-up recommenda-
tions based on the classifcation of blood pressure. Patients
with normal blood pressure should be rechecked in 2 years.
Prehypertensive patients should be rechecked in 1 year.
Tose with stage 1 hypertension should be rechecked in 2
months. Tose with stage 2 hypertension should have more
urgent follow-up. Te patient in this question has stage 1
hypertension and should therefore be rechecked within 2
months.
35. Answer d.
Angiotensin-converting enzyme inhibitors, angiotensin
receptor blockers, and direct renin inhibitors are con-
traindicated in pregnancy because their use has been
associated with serious fetal abnormalities (limb defects,
lung hypoplasia, craniofacial deformities, and renal dys-
plasia). Traditionally, fetal abnormalities were thought
to develop afer the frst trimester; however, the latest
information suggests that they can occur at any time
during pregnancy. Tus, it is recommended that these
drugs be avoided in hypertensive women who are sexu-
ally active and can become pregnant. Doxazosin is not
recommended as an initial drug for the treatment of
hypertension.
36. Answer e.
Approximately 10% to 20% of patients with elevated of ce
BP have normal readings outside the clinic environment.
Tis is referred to as of ce or white coat hypertension.
Tese persons are at low risk and do not require drug ther-
apy. Self-measured BP consistently less than 130/80 mm
Hg confrms the diagnosis. If the self-measured systolic BP
is 130 to 140 mm Hg and the self-measured diastolic BP
is 80 to 90 mm Hg, the diagnosis is best confrmed with
noninvasive 24-hour ambulatory BP monitoring. Of ce
2. CARDI OLOGY QUESTI ONS AND ANSWERS

25
Institute; Society for Vascular Nursing; TransAtlantic Inter-Society
Consensus; and Vascular Disease Foundation. Circulation. 2006 Mar
21;113(11):e463654.
Kearon C, Kahn SR, Agnelli G, Goldhaber S, Raskob GE, Comerota
AJ; American College of Chest Physicians. Antithrombotic therapy
for venous thromboembolic disease: American College of Chest
Physicians Evidence-Based Clinical Practice Guidelines (8th Edition).
Chest. 2008 Jun;133(6 Suppl):454S-545S. Erratum in: Chest. 2008
Oct;134(4):892.
K ber L, Torp-Pedersen C, McMurray JJ, G tzsche O, Levy S, Crijns
H, et al; Dronedarone Study Group. Increased mortality afer
dronedarone therapy for severe heart failure. N Engl J Med. 2008
Jun 19;358(25):267887. Erratum in: N Engl J Med. 2010 Sep
30;363(14):1384.
Levin AR, Spach MS, Boineau JP, Canent RV Jr, Capp MP, Jewett PH.
Atrial pressure-fow dynamics in atrial septal defects (secundum type).
Circulation. 1968 Apr;37(4):47688.
Mas JL, Chatellier G, Beyssen B, Branchereau A, Moulin T, Becquemin
JP, et al; EVA-3S Investigators. Endarterectomy versus stenting in
patients with symptomatic severe carotid stenosis. N Engl J Med. 2006
Oct 19;355(16):166071.
McDermott MM, Greenland P, Liu K, Guralnik JM, Criqui MH, Dolan
NC, et al. Leg symptoms in peripheral arterial disease: associated
clinical characteristics and functional impairment. JAMA. 2001 Oct
3;286(13):1599606.
North American Symptomatic Carotid Endarterectomy Trial
Collaborators. Benefcial efect of carotid endarterectomy in symp-
tomatic patients with high-grade carotid stenosis. N Engl J Med. 1991
Aug 15;325(7):44553.
OToole JD, Reddy PS, Curtiss EI, Shaver JA. Te mechanism of split-
ting of the second heart sound in atrial septal defect. Circulation. 1977
Dec;56(6):104753.
Ringleb PA, Allenberg J, Bruckmann H, Eckstein HH, Fraedrich G,
Hartmann M, et al; SPACE Collaborative Group. 30 day results from
the SPACE trial of stent-protected angioplasty versus carotid endart-
erectomy in symptomatic patients: a randomised non-inferiority trial.
Lancet. 2006 Oct 7;368(9543):123947. Erratum in: Lancet. 2006
Oct 7;368(9543):1238.
Watson HG, Baglin TP. Guidelines on travel-related venous thrombosis.
Br J Haematol. 2011 Jan;152(1):314. Epub 2010 Nov 18.
Wilson W, Taubert KA, Gewitz M, Lockhart PB, Baddour LM, Levison
M, et al; American Heart Association Rheumatic Fever, Endocarditis,
and Kawasaki Disease Committee; American Heart Association
Council on Cardiovascular Disease in the Young; American Heart
Association Council on Clinical Cardiology; American Heart
Association Council on Cardiovascular Surgery and Anesthesia;
Quality of Care and Outcomes Research Interdisciplinary Working
Group. Prevention of infective endocarditis: guidelines from the
American Heart Association: a guideline from the American Heart
Association Rheumatic Fever, Endocarditis, and Kawasaki Disease
Committee, Council on Cardiovascular Disease in the Young, and
the Council on Clinical Cardiology, Council on Cardiovascular
Surgery and Anesthesia, and the Quality of Care and Outcomes
Research Interdisciplinary Working Group. Circulation. 2007 Oct
9;116(15):173654. Epub 2007 Apr 19. Erratum in: Circulation.
2007 Oct 9;116(15):e3767.

SUGGESTED READING
Baglin TP, Brush J, Streif M; British Committee for Standards in
Haematology Writing Group. Guidelines on use of vena cava flters. Br
J Haematol. 2006 Sep;134(6):5905. Epub 2006 Jul 26.
Boden WE, ORourke RA, Teo KK, Hartigan PM, Maron DJ, Kostuk
WJ, et al; COURAGE Trial Research Group. Optimal medical ther-
apy with or without PCI for stable coronary disease. N Engl J Med.
2007 Apr 12;356(15):150316. Epub 2007 Mar 26.
Ederle J, Dobson J, Featherstone RL, Bonati LH, van der Worp HB, de
Borst GJ, et al; International Carotid Stenting Study investigators.
Carotid artery stenting compared with endarterectomy in patients
with symptomatic carotid stenosis (International Carotid Stenting
Study): an interim analysis of a randomised controlled trial. Lancet.
2010 Mar 20;375(9719):98597. Epub 2010 Feb 25. Erratum in:
Lancet. 2010 Jul 10;376(9735):90. Nasser, H-C [corrected to Nahser,
H-C].
European Carotid Surgery Trialists Collaborative Group. MRC European
Carotid Surgery Trial: interim results for symptomatic patients with
severe (7099%) or with mild (029%) carotid stenosis. Lancet. 1991
May 25;337(8752):123543.
Hiratzka LF, Bakris GL, Beckman JA, Bersin RM, Carr VF, Casey
DE Jr, et al; American College of Cardiology Foundation/
American Heart Association Task Force on Practice Guidelines;
American Association for Toracic Surgery; American College of
Radiology; American Stroke Association; Society of Cardiovascular
Anesthesiologists; Society for Cardiovascular Angiography and
Interventions; et al. 2010 ACCF/AHA/AATS/ACR/ASA/SCA/
SCAI/SIR/STS/SVM guidelines for the diagnosis and management
of patients with Toracic Aortic Disease: a report of the American
College of Cardiology Foundation/American Heart Association
Task Force on Practice Guidelines, American Association for
Toracic Surgery, American College of Radiology, American Stroke
Association, Society of Cardiovascular Anesthesiologists, Society
for Cardiovascular Angiography and Interventions, Society of
Interventional Radiology, Society of Toracic Surgeons, and Society
for Vascular Medicine. Circulation. 2010 Apr 6;121(13):e266
369. Epub 2010 Mar 16. Erratum in: Circulation. 2010 Jul
27;122(4):e410.
Hirsch AT, Haskal ZJ, Hertzer NR, Bakal CW, Creager MA, Halperin JL,
et al; American Association for Vascular Surgery; Society for Vascular
Surgery; Society for Cardiovascular Angiography and Interventions;
Society for Vascular Medicine and Biology; Society of Interventional
Radiology; ACC/AHA Task Force on Practice Guidelines Writing
Committee to Develop Guidelines for the Management of Patients
With Peripheral Arterial Disease; et al. ACC/AHA 2005 Practice
Guidelines for the management of patients with peripheral arterial
disease (lower extremity, renal, mesenteric, and abdominal aortic):
a collaborative report from the American Association for Vascular
Surgery/Society for Vascular Surgery, Society for Cardiovascular
Angiography and Interventions, Society for Vascular Medicine and
Biology, Society of Interventional Radiology, and the ACC/AHA
Task Force on Practice Guidelines (Writing Committee to Develop
Guidelines for the Management of Patients With Peripheral Arterial
Disease): endorsed by the American Association of Cardiovascular
and Pulmonary Rehabilitation; National Heart, Lung, and Blood
This page intentionally left blank
27
a. Oral prednisone
b. Bismuth subsalicylate
c. Budesonide
d. Ciprofoxacin
e. Metronidazole
4. A 55-year-old woman presented to the emergency
department with a 6-hour history of epigastric pain,
nausea, and vomiting. She has a history of hyperten-
sion and hyperlipidemia. Her medications are aspirin
81 mg daily, lisinopril 10 mg daily, and simvastatin
20 mg daily. Her heart rate is 105 beats per minute,
her blood pressure is 100/60 mm Hg, and her tem-
perature is 36.4 C. On clinical examination, the patient
has moderate epigastric tenderness and reduced bowel
sounds. Laboratory test results include the following:
hemoglobin 13 g/dL, white blood cell count 18 10
9
/L,
amylase 2,563 U/L, lipase 5,637 U/L, aspartate amin-
otransferase (AST) 350 U/L, alanine aminotransferase
(ALT) 250 U/L, and bilirubin 1.1 mg/dL. What should
you recommend next?
a. Computed tomographic (CT) scan of the abdomen with
intravenous (IV) contrast medium
b. Ultrasonography of the abdomen
c. Emergent endoscopic retrograde cholangiopancreatography
(ERCP)
d. Plain abdominal radiography
e. IV fuid, bowel rest, and observation
5. A 50-year-old man presents to the emergency depart-
ment with lef lower abdominal pain. He has not had
fever or a change in bowel habit. He is eating with-
out dif culty. He has never had similar symptoms in
the past. He has not undergone colon cancer screen-
ing. He has no comorbid conditions. On examina-
tion, he has mild tenderness in the lef lower abdomen
without peritoneal signs. Te white blood cell count
is 12.5 10
9
/L. Computed tomography (CT) shows
changes consistent with diverticulitis without abscess.
What should you recommend as the next step for this
patient?
a. Hospital admission, bowel rest, and intravenous antibiotics
b. Outpatient antibiotics
c. Colonoscopy
d. Surgical consultation
e. CT colonography
QUESTI ONS
Multiple Choice (choose the best answer)
COLON AND PANCREAS
1. A 52-year-old man presents for colon cancer screening.
He is asymptomatic, and he states that he is not aware of
any family member with a prior history of either colon
cancer or colon polyps. Te patient undergoes colonos-
copy and is found to have the following polyps and histo-
logic diagnoses: 2-mm cecal polyp and 4-mm transverse
polyp (tubular adenomas, low-grade dysplasia), 6-mm
sigmoid polyp (tubulovillous adenoma, low-grade dys-
plasia), and 3-mm rectal polyp (hyperplasia). If those 4
lesions were completely removed, what should you rec-
ommend for ongoing colon surveillance?
a. Colonoscopy in 1 year
b. Colonoscopy in 3 years
c. Colonoscopy in 5 years
d. Yearly fecal occult blood test and colonoscopy in 5 years
e. Computed tomographic colonography in 5 years
2. A 35-year-old woman with ulcerative colitis of 5 years
duration presents with a red eye. She has no pain or
headache, and her vision is normal. Her stools have
been somewhat looser over recent weeks. She has been
taking mesalamine at her usual dosage of 2.4 g daily,
but she has missed doses occasionally. She recently quit
smoking. What is the most likely cause of her red eye?
a. Uveitis
b. Giant cell arteritis
c. Episcleritis
d. Nicotine withdrawal and insomnia
e. Viral conjunctivitis
3. A 60-year-old woman presents with a 6-week history of
diarrhea. She has had 6 to 8 large-volume watery stools
daily. Tere has been no blood in the stool. She has not
had fever or other systemic symptoms. Recently, she has
not taken antibiotics or changed medication. Clinical
examination fndings were normal. Laboratory test
results were unremarkable. Stool examination for fecal
leukocytes was negative. On colonoscopy, the colon
appeared normal. On biopsy, a thickened subepithe-
lial collagen band was apparent. What initial therapy
should you recommend?
3.
GASTROENTEROLOGY AND HEPATOLOGY
QUESTIONS AND ANSWERS
28

MAYO CLINI C INTERNAL MEDI CINE BOARD REVIEW: QUESTI ONS AND ANSWERS
swallow, and aperistalsis. Which of the following is the
best next step?
a. Perform another EGD now.
b. Inject botulinum toxin into the lower esophageal sphincter.
c. Refer the patient to a surgeon for myotomy.
d. Perform computed tomography of the chest.
e. Test for anticentromere antibodies.
SMALL BOWEL AND I NTESTI NE
9. A 49-year-old man presents with a 1-month history of
diarrhea. He has approximately 10 watery bowel move-
ments daily, and he has lost 4.5 kg while he has had diar-
rhea. Physical examination, complete blood cell count,
and chemistry panel results were normal. A 72-hour
stool collection showed 2,000 g of stool with 10 g of fat
per 24 hours. Stool electrolyte concentrations were as
follows: sodium 80 mEq/L and potassium 60 mEq/L.
From these fndings, what is the most likely cause of this
patients diarrhea?
a. Whipple disease
b. Vasoactive intestinal peptide tumor
c. Celiac sprue
d. Chronic pancreatitis
e. Lactase defciency
10. A 40-year-old woman who has iron defciency anemia
began receiving oral iron therapy without response. She
reports no gastrointestinal tract complaints or heavy
menses. She has a normal appetite and reports no weight
loss. Tere is no family history of colon cancer or infam-
matory bowel disease. Fecal occult blood testing of the
stool is negative. Which test should be performed next?
a. Measurement of serum IgA and IgG tissue transglutaminase
antibodies
b. Upper endoscopy with small bowel biopsies
c. Capsule endoscopy
d. Small bowel follow-through
e. Stool evaluation for ova and parasites
11. A 55-year-old white man presents with a 6-month
history of weight loss and arthralgias. He reports no
anorexia but has had diarrhea with up to 4 loose stools
daily. He has migratory pain involving the shoulders,
elbows, and knees. On physical examination, he has
skin hyperpigmentation and oculomasticatory myo-
rhythmia. A 72-hour stool collection shows 32 g of fat
per 24 hours. Which test is most likely to be positive or
diagnostic for this patient?
a. Measurement of serum IgA and IgG tissue transglutaminase
antibodies
b. Measurement of serum IgA and IgG deamidated gliadin
antibodies
c. Small bowel biopsy showing enlarged villi with periodic
acid-Schif (PAS)-positive macrophages
d. Small bowel biopsy showing villous atrophy and crypt
hyperplasia
e. Duodenal aspirates showing more than 100,000
colony-forming units (CFU)/mL
ESOPHAGUS AND STOMACH
6. A 63-year-old man undergoes esophagogastroduo-
denoscopy (EGD) and colonoscopy as part of an evalu-
ation for mild iron defciency anemia. He denies having
melena or hematochezia, and he has not lost weight.
Te only medication he takes is ibuprofen for intermit-
tent joint aches, and he is otherwise healthy. His fam-
ily history is unremarkable. Te EGD showed multiple
linear antral erosions and a 1.2-cm polyp in the body
of the stomach. Biopsies from throughout the stomach
showed a chemical gastritis with no Helicobacter pylori ,
and biopsies of the polyp identifed a tubular adenoma
with low-grade dysplasia. Small bowel biopsy fndings
were normal. Te colonoscopy showed only scattered
sigmoid diverticula. Which of the following is the best
next step?
a. No further testing
b. Helicobacter pylori stool antigen test
c. Endoscopic ultrasonography
d. EGD now with full removal of the polyp
e. EGD in 1 year
7. A 42-year-old man presents with hoarseness and inter-
mittent heartburn symptoms. He notes that in the
past he had heartburn caused by triggering foods sev-
eral times per month; symptoms now occur several
times per week without obvious precipitants. He also
notes that his voice is hoarse, which has afected his
singing in the church choir. He reports no dysphagia,
odynophagia, nausea, vomiting, early satiety, or weight
loss. Proton pump inhibitor (PPI) therapy twice daily
before meals has provided only partial relief of symp-
toms. Esophagogastroduodenoscopy (EGD) showed
several cystic fundic gland polyps but was otherwise
normal. Which of the following is the best next step?
a. An H
2
receptor blocker at bedtime
b. Gastric scintigraphy with a solid meal
c. A 24-hour ambulatory pH probe
d. Video fuoroscopic swallowing test
e. Esophageal manometry
8. A 78-year-old man presents for evaluation of new swal-
lowing problems. He states that for the past 3 months,
food gets stuck when he swallows, although the food
eventually passes spontaneously. He also notes dif-
fculty swallowing liquids and senses fullness in his
chest for a prolonged period afer drinking any bever-
age. He reports regurgitation of fuid into the back of
his throat. He has a 40pack-year smoking history, and
a long-standing history of refux, well-controlled with
PPI therapy as needed. He has lost 6.8 kg over the past
few months. He had esophagogastroduodenoscopy
(EGD) 1 year ago to screen for Barrett esophagus;
results were negative. With his new symptoms, he now
undergoes a barium esophagram, which shows a birds
beak narrowing at the distal esophagus; esophageal
manometry shows an elevated pressure in the lower
esophageal sphincter, which does not relax afer a
3. GASTROENTEROLOGY AND HEPATOLOGY QUESTI ONS AND ANSWERS

29
a. Bacterial overgrowth frequently develops in patients with
ileal resections.
b. Bile acidbinding resins cause diarrhea.
c. Te bile acid binder has further depleted the bile acid pool.
d. Bile acid binders may cause vitamin defciencies.
e. Te patient most likely increased her consumption of poorly
digestible carbohydrates.
LI VER
16. A 35-year-old man who comes for a preoperative assess-
ment before hernia repair is mildly jaundiced. Other
than a symptomatic inguinal hernia, he feels well. Ten
years ago, the patient had a brief period of illicit drug
use. He currently drinks 3 alcoholic beverages daily. He
takes niacin for hyperlipidemia. Physical examination is
notable only for mild jaundice. Laboratory test results
include the following: hemoglobin 15 g/dL, platelet
count 213 10
9
/L, aspartate aminotransferase (AST)
23 U/L, alanine aminotransferase (ALT) 22 U/L, alka-
line phosphatase 83 U/L, total bilirubin 2.4 mg/dL,
and direct bilirubin 0.2 mg/dL. Which of the following
is the most likely diagnosis?
a. Hepatitis C
b. Hepatitis B
c. Niacin-induced liver disease
d. Alcoholic hepatitis
e. Gilbert syndrome
17. A 60-year-old woman presents with increasing values
on liver tests. Tree months ago, at a general assess-
ment, the level of her alanine aminotransferase (ALT)
was 63 U/L and she had hyperlipidemia. Evaluation
of the elevated liver test results was negative except for
steatosis noted on ultrasonography. She began tak-
ing simvastatin. One month afer starting simvastatin,
her ALT is 85 U/L. She continues to feel well and her
examination is unremarkable except for obesity. Her
only other medication is metformin. She drinks 1 glass
of wine every 6 months. Which of the following would
you advise at this time?
a. Stop all alcohol intake.
b. Undergo liver biopsy.
c. Stop use of simvastatin.
d. Stop use of metformin.
e. Follow with serial monitoring of ALT.
18. A 49-year-old woman is referred because of an elevated
alkaline phosphatase level discovered during evaluation
for fatigue. She has had no previous blood tests. Physical
examination is notable for xanthelasmas. Laboratory
study results include the following: alkaline phosphatase
500 U/L, alanine aminotransferase 73 U/L, and total
cholesterol 300 mg/dL; bilirubin, international nor-
malized ratio, and albumin are all normal. Which of the
following tests is most likely to establish a diagnosis?
a. Antimitochondrial antibody testing
b. Magnetic resonance cholangiopancreatography
12. A 45-year-old woman presents with abdominal discom-
fort and diarrhea. Almost every day, she has variable
abdominal discomfort with up to 3 or 4 watery stools.
She has associated abdominal bloating and fatulence.
Eating and stress aggravate symptoms, and abdomi-
nal discomfort is relieved by defecation. She reports
no anorexia, weight loss, or blood in the stool. You
suspect irritable bowel syndrome (IBS). Laboratory
study results were normal for complete blood cell
count, erythrocyte sedimentation rate, and C-reactive
protein. Fecal leukocytes are present, but culture for
enteric pathogens and testing for Clostridium dif cile
are negative. Which intervention is appropriate at this
time?
a. Loperamide
b. Rifaximin
c. Colonoscopy with biopsies
d. Reassuring and counseling the patient
e. Stool-bulking agents
13. A 70-year-old woman with a history of mitral valve
disease and atrial fbrillation awakens from sleep with
a sudden onset of sharp central abdominal pain and
the sudden urge to have a bowel movement. An explo-
sive stool is passed without blood; however, the pain
persists. On examination, her abdomen is sof, with
minimal tenderness and no rebound or guarding. Te
physician notes that her pain is out of proportion to
the fndings on physical examination. Laboratory test
data are normal, including results for a complete blood
cell count, blood chemistry panel, amylase, lipase, and
serum lactate. An electrocardiogram shows normal
sinus rhythm. A computed tomographic (CT) scan of
the abdomen shows a nonspecifc gas pattern. What
should be the next step in management?
a. Observation
b. A vascular imaging study
c. Laparotomy
d. Laparoscopy
e. Stool studies
14. A 55-year-old woman who had extensive abdominal
radiotherapy 5 years earlier for endometrial cancer has
chronic symptoms of postprandial bloating and peri-
umbilical discomfort. Over the past several weeks she
has had persistent diarrhea. Results of routine stool
studies for infectious agents are negative. Which of the
following treatments would most likely result in sus-
tained improvement in her diarrhea?
a. Oral mesalamine
b. Bile acidbinding resin
c. Ciprofoxacin
d. A proton pump inhibitor
e. Low-fat diet
15. A 35-year-old woman with Crohn disease and a prior
ileal resection receives a bile acidbinding agent for
treatment of chronic diarrhea. However, the diarrhea
worsens. What is the most likely explanation?
30

MAYO CLINI C INTERNAL MEDI CINE BOARD REVIEW: QUESTI ONS AND ANSWERS
21. A 53-year-old man is admitted from the emergency
department with upper gastrointestinal tract bleeding.
He has a history of chronic hepatitis C and cirrhosis but
felt well until early this morning when he had hematem-
esis. Esophagogastroduodenoscopy in the emergency
department showed large esophageal varices, which were
ligated. He currently feels weak but has had no other
complaints. Physical examination fndings include the
following: blood pressure 120/60 mm Hg, heart rate
82 beats per minute, alert and oriented, splenomegaly,
and no ascites or edema. Laboratory study results
are as follows: hemoglobin 9.2 g/L, platelet count
65 10
9
/L, aspartate aminotransferase 84 U/L, alanine
aminotransferase 75 U/L, total bilirubin 1.2 mg/dL,
albumin 3.0 g/dL, and international normalized ratio
(INR) 1.2. Abdominal ultrasonography shows no liver
mass, a patent portal vein, splenomegaly, and no ascites.
Which of the following would you advise now?
a. Transjugular intrahepatic portosystemic shunt
b. Norfoxacin
c. Transfusion of 2 units of packed red blood cells
d. Transfusion of 2 units of fresh frozen plasma
e. Pegylated interferon and ribavirin
22. A 33-year-old Asian woman receives a diagnosis of
non-Hodgkin lymphoma, and chemotherapy is advised.
She has a history of hepatitis B without complications.
Her mother also had hepatitis B. On examination,
the patient has cervical adenopathy consistent with
lymphoma and no stigmata of chronic liver disease.
Laboratory test results are as follows: platelet count
348 10
9
/L, alanine aminotransferase 17 U/L, total bil-
irubin 0.6 mg/dL, hepatitis B surface antigen positive,
hepatitis B e antigen (HBeAg) negative, antibody to
HBeAg positive, IgG antibody to hepatitis B core anti-
gen positive, and hepatitis B virus (HBV) DNA unde-
tectable. Which of the following should you advise at
this time?
a. Hepatitis B vaccination
b. Surveillance for hepatocellular carcinoma
c. Lamivudine
d. Pegylated interferon
e. Nothing further at this time except chemotherapy
c. Antinuclear antibody testing
d. Antismooth muscle antibody testing
e. Angiotensin-converting enzyme level measurement
19. A 43-year-old alcoholic man is referred with a 3-month
history of abdominal distention, leg edema, and dysp-
nea. Physical examination is notable for mild jugular
venous distention, distant heart sounds, marked ascites,
and leg edema. Chest radiography shows mild cardio-
megaly. Laboratory test results are as follows: total bili-
rubin 1.5 mg/dL, albumin 3.5 g/dL, and international
normalized ratio 1.2. Liver Doppler ultrasonography
shows ascites, a coarse echotexture of the liver, and pat-
ent hepatic and portal veins. In the abdominal fuid, the
protein level is 3.9 g/dL and the albumin level is 2.1 g/dL.
Which of the following would you advise next?
a. Hepatic venography
b. Transjugular intrahepatic portosystemic shunt
c. Liver biopsy
d. Echocardiography
e. Laparoscopy
20. A 48-year-old woman is referred with a 3-week history
of fatigue and vague right upper quadrant pain. She
drinks 2 glasses of wine daily and occasionally more
on weekends. Her medical history is notable for hypo-
thyroidism. She reports no prior blood transfusions,
illegal drug use, or a family history of liver disease. Her
only medications are ibuprofen 400 mg up to 3 times
daily, acetaminophen 500 mg up to 4 times daily, and
thyroxine. Physical examination fndings are normal.
Laboratory study results are as follows: aspartate ami-
notransferase (AST) 740 U/L, alanine aminotrans-
ferase (ALT) 900 U/L, alkaline phosphatase 115 U/L,
and -globulin 5.4 g/dL. Bilirubin and albumin levels
and the international normalized ratio (INR) are nor-
mal. Serologic studies are negative for hepatitis A, B,
and C. Te patient is observed, and 2 weeks later her
laboratory test results are as follows: AST 756 U/L,
ALT 945 U/L, total bilirubin 1.4 mg/dL, and INR
1.2. Ultrasonography shows a coarse echotexture to the
liver, borderline splenomegaly, and gallbladder stones
without bile duct dilatation. Two weeks later the ALT
is unchanged, but the bilirubin is 2.0 mg/dL. Which of
the following is the most likely diagnosis?
a. Autoimmune hepatitis
b. Ibuprofen hepatotoxicity
c. Acetaminophen hepatotoxicity
d. Alcoholic hepatitis
e. Cholelithiasis
31
without dif culty, outpatient management may be pur-
sued. If there were evidence of a complication (eg, abscess),
or if the patient could not tolerate oral intake, hospital-
ization would be necessary. Since this is the patients frst
episode of diverticulitis, and it is uncomplicated, surgery is
not indicated. Colonoscopy is contraindicated with acute
diverticulitis, but since this patient has not undergone
colon cancer screening, it would be reasonable to perform
colonoscopy 2 to 4 weeks afer the acute symptoms have
resolved.
6. Answer d.
Tis patient has iron defciency anemia that is likely due to
the antral erosions that appear to have been induced by the
nonsteroidal anti-infammatory drug. However, he also has
a gastric polyp that is a gastric adenoma; similar to colonic
adenomas, gastric adenomas are deemed premalignant and
require full endoscopic removal. Te patient should have
another EGD now with polypectomy since the polyp was
simply biopsied and not fully removed during his previ-
ous EGD. To do no further testing would be inadequate
because this polyp could continue to grow and progress
to gastric cancer. While gastric erosions can be caused by
H pylori , this patients histologic examination was negative
for H pylori , and he was not taking any medications that
could lead to false-negative testing for H pylori (proton
pump inhibitor, antibiotics, etc); therefore, further test-
ing for H pylori would not be needed. If the gastric polyp
had been malignant, endoscopic ultrasonography would
be needed to assess the depth of invasion and to complete
locoregional staging, but that is not needed for an ade-
noma of this size. To wait to repeat the EGD for 1 year is
not recommended since the polyp could continue to grow,
progress to cancer, or cause bleeding, all of which could be
prevented by removal now.
7. Answer c.
Tis patient has features suggestive of gastroesophageal
disease. He has not only progressive heartburn but also the
extra-esophageal feature of hoarseness. Since he has not
fully improved with PPI therapy and the EGD was negative
for esophagitis, a 24-hour ambulatory pH probe would be
helpful to document whether he truly has acid refux and
whether there is symptom correlation. When this test incor-
porates impedance testing, symptoms can be correlated with
acid-mediated refux or with nonacid-mediated refux (eg,
seen with someone receiving PPI therapy). At this point,
1. Answer b.
Hyperplastic polyps do not infer an increased risk of colon
cancer. Terefore, the rectal lesion is of no clinical signifcance.
Te patient had 3 clinically signifcant polyps. Although each
lesion was smaller than 1 cm, the interval to the next colonos-
copy would be 3 years because there were 3 or more polyps
and because 1 of the lesions had a villous component.
2. Answer c.
A patient with infammatory bowel disease who has a red
eye most likely has either episcleritis or uveitis. Uveitis is
associated with pain in the eye. Episcleritis is typically pain-
less. Giant cell arteritis typically does not manifest with red
eye and is usually associated with headache or visual loss.
3. Answer b.
Te clinical presentation and the colonoscopic and his-
tologic fndings are typical of microscopic colitis (in this
case, collagenous colitis). Initial therapy for mild to mod-
erate disease (36 bowel movements daily) is usually with
either an antidiarrheal, such as loperamide hydrochloride,
or bismuth subsalicylate. In more severe cases (>6 bowel
movements daily), budesonide may be considered for ini-
tial therapy. Prednisone would be used only in cases of
microscopic colitis that did not respond to the aforemen-
tioned therapies.
4. Answer e.
Te patient has acute pancreatitis as evidenced by the
clinical presentation and the elevated levels of amylase and
lipase. Te cause is most likely gallstone disease (AST and
ALT were moderately elevated). Te most important initial
step in the management of patients with acute pancreatitis
is to ensure excellent hydration to optimize pancreatic per-
fusion and thereby decrease the risk of pancreatic necrosis.
Te patient does not have evidence of cholangitis (absence
of fever, pain localizing to the right upper quadrant, and
jaundice), so there is no indication for emergent ERCP.
CT scan of the abdomen with IV contrast medium may be
indicated later in the clinical course to evaluate for pancre-
atic necrosis, but it is not indicated at initial presentation.
Ultrasonography of the abdomen is a reasonable step but
only afer IV fuid resuscitation has been initiated.
5. Answer b.
Tis patient is presenting with a frst episode of uncom-
plicated diverticulitis. Since he is tolerating oral intake
ANSWERS
32

MAYO CLINI C INTERNAL MEDI CINE BOARD REVIEW: QUESTI ONS AND ANSWERS
chronic pancreatitis, and lactase defciency) are causes of
osmotic diarrhea and are therefore incorrect.
10. Answer b.
Tis woman has iron defciency without evidence of gas-
trointestinal tract or menstrual blood loss, which suggests
malabsorption of iron. Te most common manifestation
of celiac disease is iron defciency anemia. Iron is mainly
absorbed in the duodenum. Celiac disease preferentially
afects the proximal small bowel, interfering with iron
uptake. Terefore, upper endoscopy with small bowel
biopsies should be performed to evaluate for celiac disease.
A small bowel series or capsule endoscopy may suggest the
diagnosis of celiac disease but does not provide tissue for
diagnosis. Positive serologic testing (tissue transglutami-
nase antibodies) supports the diagnosis of celiac disease
but, if negative, does not exclude the diagnosis in this
patient with a high pretest probability of celiac disease. In
a patient with iron defciency and no gastrointestinal tract
symptoms, stool evaluation for ova and parasites would be
low yield. Furthermore, a parasitic infection (eg, strongy-
loidiasis) would likely be detected on small bowel biopsy.
11. Answer c.
Te diagnosis of Whipple disease should be suspected with
the combination of steatorrhea, weight loss, and migratory
arthralgias in a middle-aged white man. Whipple disease
may cause central nervous system involvement manifesting
with the fnding of oculomasticatory myorhythmia in 20%
of patients. Oculomasticatory myorhythmia is pathog-
nomonic for Whipple disease and consists of continuous
rhythmic jaw contractions that are synchronous with dis-
sociated pendular vergence oscillations. Whipple disease
occurs predominantly in middle-aged white men and is
caused by chronic infection with Tropheryma whipplei . In
most patients with Whipple disease, the intestinal tract is
involved regardless of the presence or absence of gastro-
intestinal tract symptoms. Tus, the primary diagnostic
approach to a patient with clinically suspected Whipple
disease is upper endoscopy with mucosal biopsy. Intestinal
biopsies show the characteristic fndings of macrophages
with PAS-staining particles, which indicate the presence
of T whipplei bacilli. Polymerase chain reaction assays may
assist in the detection of T whipplei DNA in the intestinal
mucosa. Tissue transglutaminase antibodies, deamidated
gliadin antibodies, and small bowel biopsy with villous
atrophy and crypt hyperplasia are characteristic of celiac
disease and do not ft this clinical scenario. Duodenal aspi-
rates showing more than 100,000 CFU/mL are diagnostic
of small intestinal bacterial overgrowth, which can cause
steatorrhea and arthralgias but would not manifest with
oculomasticatory myorhythmia.
12. Answer c.
Tis patient has signs and symptoms consistent with
IBS. Te only test required for patients who have typical
diarrhea-predominant IBS symptoms and no alarm fea-
tures is serologic testing for celiac disease. However, this
establishing a diagnosis would be helpful to direct future
therapy rather than adding additional acid suppression with
an H
2
receptor blocker at bedtime. While patients with
delayed gastric emptying can present with worsening acid
refux, this patient does not report having any of the other
symptoms that commonly occur with gastroparesis (nausea,
vomiting, early satiety), making it less likely at this point.
Although hoarseness may result from neuromuscular weak-
ness, which can cause oropharyngeal dysphagia, this patient
does not report swallowing dif culties that would suggest
that a video swallowing test would be of value. Esophageal
manometry can be useful for evaluating motility disorders
of the esophagus, such as achalasia, but would not have a role
here in the evaluation of persistent gastroesophageal refux
diseaserelated symptoms in the absence of dysphagia.
8. Answer a.
Tis patient has clinical, radiographic, and manometric fea-
tures consistent with achalasia; however, given his age, the
rapid onset of his symptoms, and the weight loss, pseudo-
achalasia due to malignancy needs to be considered and
ruled out. An EGD should be performed now to rule out
esophageal or gastric cardia malignancy because his most
recent EGD was 1 year ago (before the onset of his current
symptoms), and an early lesion could have been missed.
To refer this patient for any therapy targeted at achalasia,
such as botulinum toxin injection into the lower esophageal
sphincter or myotomy, would be premature until EGD has
been performed to rule out cancer. If a patient with clinical
features of pseudoachalasia has negative fndings on EGD,
imaging of the chest may then be considered, especially
with a smoking history. A pulmonary or mediastinal malig-
nancy can infltrate the lower esophageal sphincter complex
and cause pseudoachalasia symptoms; however, this testing
should not take place before another EGD is performed,
allowing direct mucosal inspection. Anticentromere anti-
bodies can be seen in CREST syndrome associated with
scleroderma; similar to patients with achalasia, these
patients may also have dysphagia to solids and liquids and
are at increased risk of esophageal cancer. However, patients
with esophageal involvement with scleroderma typically
have a decreased lower esophageal sphincter tone, which is
the opposite of what is seen in this case.
9. Answer b.
Tis patient has a stool osmotic gap (290 2 [80 + 60])
of less than 50 mOsm/kg, suggesting a secretory cause of
diarrhea. Causes of secretory diarrhea include toxins from
cholera and enterotoxigenic Escherichia coli and peptides
produced from endocrine tumors (vasoactive intestinal
peptide). Te distinction between secretory and osmotic
diarrhea helps in the diferential diagnosis and evaluation
of patients with chronic diarrhea. Te 2 main methods to
help distinguish between secretory and osmotic diarrhea
are by calculating the stool osmotic gap and assessing the
response to fasting. Secretory diarrhea will not decrease
substantially during a fast, whereas osmotic diarrhea will.
Te other answer choices (Whipple disease, celiac sprue,
3. GASTROENTEROLOGY AND HEPATOLOGY QUESTI ONS AND ANSWERS

33
17. Answer e.
Te patient presents with a mildly increasing ALT 1 month
afer she began taking simvastatin. She almost certainly has
nonalcoholic fatty liver disease. It is now well documented
that patients with nonalcoholic fatty liver disease do not
have an increased risk of liver toxicity with statins. Mild
increases in aminotransferases afer starting a statin are
common but are nearly always transient; consequently,
serial monitoring of ALT is the correct answer. Patients
with nonalcoholic fatty liver disease have an increased
risk of death, although much of that increase in mortality
is due to cardiovascular disease rather than liver disease.
Terefore, control of cardiovascular risk factors is an impor-
tant management issue for these patients. Te amount of
alcohol consumed by the patient is irrelevant. Unless there
would be other changes in the patients clinical condition,
liver biopsy is not required but could be considered if there
are further increases in ALT. Metformin is a rare cause of
elevated liver test results; therefore, metformin may be
continued. Simvastatin should be discontinued only if the
ALT level increases to 3 to 5 times the upper limit of the
reference range. (See Torres and Harrison, and Chalasani
in the Suggested Reading list.)
18. Answer a.
Te presence of xanthelasmas and a cholestatic liver pro-
fle in a woman of this age is highly suggestive of primary
biliary cirrhosis (PBC), and antimitochondrial antibody
testing should be done. Magnetic resonance cholang-
iopancreatography would be useful if biliary obstruction
were likely. Te absence of pain, history of ulcerative coli-
tis (associated with primary sclerosing cholangitis), or an
elevation in the bilirubin level makes biliary obstruction
less likely than PBC. Antinuclear and antismooth muscle
antibodies are used to diagnose autoimmune hepatitis,
which elevates predominantly aminotransferases rather
than alkaline phosphatase. An angiotensin-converting
enzyme level is not necessary in the absence of other clini-
cal features of sarcoidosis. (See Kaplan and Gershwin in
the Suggested Reading list.)
19. Answer d.
Te serum-ascites albumin gradient is 1.4, which suggests
portal hypertension. Te fuid protein level greater than
2.5 g/dL is suggestive of hepatic venous outfow obstruc-
tion. Te hepatic veins are patent, and the most likely cause
for the ascites and the other symptoms is heart failure, per-
haps due to alcoholic cardiomyopathy. Echocardiography
should be the next test. Transjugular intrahepatic porto-
systemic shunt is used to treat refractory ascites associated
with cirrhosis. Liver biopsy might be useful later, but it is
invasive and should not be used unless noninvasive tests
do not lead to a diagnosis. Laparoscopy should be reserved
for a suspicion of peritoneal carcinomatosis or infection,
both of which would have a serum-ascites albumin gra-
dient less than 1.1. (See Runyon et al in the Suggested
Reading list.)
patient has fecal leukocytes. Tis fnding suggests colonic
infammation and warrants further investigation with
colonoscopy and biopsy to evaluate for infammatory bowel
disease or microscopic colitis. Reassurance, antidiarrheals,
and stool-bulking agents are therapies to consider for the
patient with IBS. Rifaximin is a nonabsorbable antibiotic
used to treat travelers diarrhea, recurrent hepatic encephal-
opathy, and small intestinal bacterial overgrowth. Although
rifaximin was recently found to alleviate IBS symptoms, it
is not approved by the US Food and Drug Administration
for this indication, and this patient with fecal leukocytes
requires further evaluation with colonoscopy.
13. Answer b.
Tis patient has the classic clinical history for superior
mesenteric artery embolus. Since the CT scan of the abdo-
men was negative, selective mesenteric angiography should
be the next step in management. Mesenteric ischemia
should be diagnosed promptly. Te mortality rate exceeds
80%, even if the embolus is removed and all infarcted
bowel is resected, because decreased splanchnic blood
fow may initiate persistent and irreversible mesenteric
vasoconstriction.
14. Answer c.
Long-term radiotherapy injury to the small bowel may man-
ifest as impaired motility or stricture, as suggested by the
patients chronic stable symptoms. Patients with impaired
motility or stricture are predisposed to small bowel bacte-
rial overgrowth due to stasis. Treatment with a course of
antibiotics, such as ciprofoxacin, would be the therapy most
likely to result in sustained improvement of the diarrhea.
15. Answer c.
Diarrhea afer an ileal resection is usually caused either
by bile acid malabsorption or bile saltinduced colonic
secretion (resection 100 cm) or by bile salt malabsorp-
tion with bile salt pool depletion and fatty acidinduced
colonic secretion (resection >100 cm). Te clinical fnding
of worsening diarrhea with treatment with binding resins
suggests further bile salt depletion.
16. Answer e.
Tis patient has an indirect hyperbilirubinemia with
normal levels of liver enzymes and without evidence
of hemolysis. Te most likely diagnosis is Gilbert syn-
drome, which occurs in about 5% of the general popula-
tion. Further diagnostic workup is unnecessary. Te prior
history of illicit drug use puts the patient at risk for hepa-
titis C and hepatitis B. He should be tested, although
it would be unusual for him to have viral hepatitis with
normal liver enzyme levels, and Gilbert syndrome is
much more common. Patients with alcoholic hepatitis
can have hyperbilirubinemia, but they generally have
abnormal liver tests, with the AST being higher than
the ALT. Niacin-induced liver disease is accompanied by
elevated liver enzymes. (See Kamath in the Suggested
Reading list.)
34

MAYO CLINI C INTERNAL MEDI CINE BOARD REVIEW: QUESTI ONS AND ANSWERS
22. Answer c.
Patients with hepatitis B who need immunosuppressive
therapy are at risk for reactivation of disease and should
receive hepatitis B treatment. An oral nucleoside or nucle-
otide analogue, such as lamivudine, is preferred because
of the reliable antiviral efect and lack of toxicity. Ideally,
hepatitis B treatment is started 2 weeks before initiation
of chemotherapy and continued for several months afer
completion of the lymphoma treatment. Hepatitis B vac-
cination is not useful if the patient already has hepatitis B.
Surveillance for hepatocellular carcinoma (HCC) is
advised in the following hepatitis B patients: patients who
have cirrhosis, Asian women older than 50 years, Asian
men older than 40 years, Africans older than 20 years,
patients with a family history of HCC, and patients with
persistently elevated liver test results and high HBV DNA
levels. Tis patient does not meet any of those criteria. (See
Lok et al in the Suggested Reading list.)
SUGGESTED READING
Chalasani N. Statins and hepatotoxicity: focus on patients with fatty liver.
Hepatology. 2005 Apr;41(4):6905.
Czaja AJ, Freese DK; American Association for the Study of Liver Disease.
Diagnosis and treatment of autoimmune hepatitis. Hepatology. 2002
Aug;36(2):47997.
Kamath PS. Clinical approach to the patient with abnormal liver test
results. Mayo Clin Proc. 1996 Nov;71(11):108994; quiz 10945.
Kaplan MM, Gershwin ME. Primary biliary cirrhosis. N Engl J Med.
2005 Sep 22;353(12):126173. Erratum in: N Engl J Med. 2006 Jan
19;354(3):313.
Lok AS, McMahon BJ; Practice Guidelines Committee, American
Association for the Study of Liver Diseases (AASLD). Chronic
hepatitis B: update of recommendations. Hepatology. 2004
Mar;39(3):85761.
Runyon BA; Practice Guidelines Committee, American Association
for the Study of Liver Diseases (AASLD). Management of
adult patients with ascites due to cirrhosis. Hepatology. 2004
Mar;39(3):84156.
Sanyal AJ, Bosch J, Blei A, Arroyo V. Portal hypertension and its complica-
tions. Gastroenterology. 2008 May;134(6):171528.
Torres DM, Harrison SA. Diagnosis and therapy of nonalcoholic steato-
hepatitis. Gastroenterology. 2008 May;134(6):168298.

20. Answer a.
Te combination of ALT that is persistently abnormal to
this degree and hypergammaglobulinemia in a woman
with a history of thyroid disease and no other identifable
cause for liver disease is most suggestive of autoimmune
hepatitis. Determination of antinuclear and antismooth
muscle antibodies and a liver biopsy would be the next
steps in management. Toxicity from ibuprofen is rare, and
acetaminophen causes an acute, usually marked, increase
in aminotransferases. Alcoholic hepatitis almost never
increases aminotransferases to more than 400 U/mL.
Gallbladder stones can cause abnormalities in liver ducts
either through severe infammation or through passage
of a common bile duct stone. Te patient does not have
clinical features of infammation, and a common bile duct
stone causes transient aminotransferase elevations and
severe abdominal pain, neither of which are present in this
patient. (See Czaja et al in the Suggested Reading list.)
21. Answer b.
Patients with cirrhosis who are admitted with gastro-
intestinal tract bleeding should receive prophylactic
antibiotics even if there is no ascites. Oral norfoxacin is
probably suf cient, although a recent trial suggested that
a third-generation cephalosporin may be more efective if
there is a high prevalence of quinolone resistance. Much of
the recent reduction in mortality related to variceal bleed-
ing is probably due to the increasing use of prophylactic
antibiotics in patients with cirrhosis and gastrointestinal
tract bleeding. Transjugular intrahepatic portal systemic
shunt is not necessary unless bleeding cannot be controlled
with endoscopy or medical therapy. Although the hemo-
globin level should be carefully monitored, the patient is
hemodynamically stable and does not need transfusion
now. Overtransfusion, when hemoglobin is greater than 8
to 9 g/dL, can precipitate recurrence of bleeding and fuid
overload. Fresh frozen plasma is also not necessary since
the INR is relatively normal. A decision about pegylated
interferon and ribavirin treatment of hepatitis C should be
deferred until the patient has recovered from this bleeding
episode. (See Sanyal et al in the Suggested Reading list.)
35
a. Haloperidol
b. Lorazepam
c. Hydromorphone
d. Midazolam
e. Fentanyl
3. A 58-year-old man comes to the emergency depart-
ment with clumsiness of his right (dominant) hand that
began abruptly 3 hours earlier. He has hypertension,
for which he takes a -blocker. His temperature is 37 C,
his heart rate is 90 beats per minute, his respirations are
16 per minute, and his blood pressure is 145/90 mm Hg.
Oxygen saturation as measured by pulse oximetry is
97% with room air. Physical examination reveals an
intact sensorium, no obvious cranial nerve defcits, 3/5
motor strength in the right upper arm with inability
to perform repetitive hand movements (ataxia), and
4/5 motor strength in the right lower leg. No sensory
defcits are noted. Emergent computed tomography of
the head does not show a stroke. All screening labora-
tory study results are normal except for a serum choles-
terol level of 250 mg/dL. It is now 4 hours afer onset
of symptoms. Which of the following is the defnitive
treatment?
a. Alteplase
b. Aspirin
c. Clopidogrel
d. Continuous-infusion eptifbatide
e. Continuous-infusion heparin
4. A 52-year-old man is admitted to the hospital because
of acute dyspnea and a syncopal episode during which
he fell. He is hypotensive and requires intubation and
norepinephrine to maintain adequate blood pressure
and oxygenation. A computed tomographic (CT) scan
of the chest shows a saddle pulmonary embolism, and
the patient is administered tissue plasminogen activa-
tor (tPA) in the emergency department. An echocar-
diogram shows right ventricular dilatation and failure.
Ten hours later, the norepinephrine dosage has been
incrementally increased from 25 to 55 mcg per min-
ute. Oxygen saturation as measured by pulse oximetry
is 98% with a fraction of inspired oxygen of 0.80. His
heart rate is 125 beats per minute, his blood pressure is
100/50 mm Hg, and his central venous pressure is 1 to
2 mm Hg. Te hemoglobin level has decreased from
13 g/dL on admission to 5 g/dL. Activated partial
thromboplastin time is 65 seconds. A formal reading
of the admission CT scan shows lef rib fractures in
QUESTI ONS
Multiple Choice (choose the best answer)
CRITI CAL CARE MEDI CI NE
1. A 59-year-old man underwent a radical prostatectomy
for prostate cancer 24 hours ago. Since hospital admis-
sion, he has received a total of 3.5 L of intravenous iso-
tonic crystalloid and has had a total urine output of 2.5 L.
Te hospital physician on duty examined him earlier
this evening for evaluation of substernal chest pain.
Te patients electrocardiogram (ECG) was normal. He
received intravenous furosemide and oral nitroglycerin
and had 1.0 L of urine output. He became symptomati-
cally worse, so he was transferred to the intensive care
unit for further evaluation. When you initially examine
the patient, his blood pressure is 150/100 mm Hg, his
heart rate is 115 beats per minute, his respiration rate
is 24 breaths per minute, and his temperature is 37 C.
He reports chest pain and shortness of breath. Tere is
no jugular vein distention. You hear bibasilar crackles
posteriorly and a harsh systolic murmur across the pre-
cordium. His extremities are well perfused with good
pulses. An ECG shows sinus tachycardia and delayed
precordial transition, but no other changes are noted.
While you wait for results of additional diagnostic
studies, which of the following should you administer
now?
a. Furosemide
b. Metoprolol
c. Nitroglycerin
d. Aspirin
e. Heparin
2. A 32-year-old man underwent surgical repair and fxa-
tion of multiple traumatic orthopedic injuries 3 days
ago. He also has bilateral lung contusions and has
required mechanical ventilation. He did not have any
head injuries. His condition is improving, and physi-
ologically he is ready to begin being weaned from the
ventilator. His oxygenation is good. However, when the
propofol infusion is decreased, he manifests a severe,
agitated delirium with ventilator dyssynchrony and an
inability to follow commands. Neurologic examination
fndings are otherwise normal. Laboratory test results
are normal, computed tomography of the head is nor-
mal, and the electrocardiogram is normal. Which of the
following should you order now?
4.
PULMONARY DISEASES QUESTIONS AND ANSWERS
36

MAYO CLINI C INTERNAL MEDI CINE BOARD REVIEW: QUESTI ONS AND ANSWERS
DI FFUSE LUNG DI SEASE AND OCCUPATI ONAL
LUNG DI SEASE
8. A 25-year-old female nonsmoker presents with a 1-week
history of mild cough and dyspnea afer a fulike illness
with fever, arthralgias, and tender erythematous lesions
on the anterior aspects of the legs. She has no history
of asthma or signifcant medical illnesses. No environ-
mental or occupational high-risk exposures are noted.
Examination reveals clear lung felds and no other
abnormalities. A chest radiograph shows prominent
bilateral hilar lymphadenopathy without parenchymal
infltrates. What should you do next?
a. Set up blood cultures.
b. Perform human immunodefciency virus serology testing.
c. Determine the erythrocyte sedimentation rate.
d. Perform Lyme serology testing.
e. Observe and repeat the chest radiograph in 12 weeks.
9. A 62-year-old man, a former smoker, presents with a
2-year history of progressive dry cough and dyspnea.
He has no extrapulmonary symptoms. No occupational
or environmental exposures are noted. Findings on
examination include bibasilar coarse rales and digital
clubbing. A chest radiograph shows prominent inter-
stitial infltrates in the middle and lower lung felds.
Te antinuclear antibody titer is borderline elevated
at 1:40. Serum protein electrophoresis shows a poly-
clonal gammopathy. Te rheumatoid factor titer is also
borderline elevated at 1:40. A high-resolution com-
puted tomographic (CT) scan of the chest shows sub-
pleural honeycombing with thickened alveolar septa
in both lower lobes with bilateral mediastinal 1.5-cm
lymph nodes. No ground-glass opacities are present.
Which treatment is most likely to result in clinical
improvement?
a. Azathioprine
b. Systemic corticosteroids
c. Cyclophosphamide
d. Systemic corticosteroids with azathioprine
e. No treatment
10. A 52-year-old man, a current smoker (75 pack-years),
is examined for acute dyspnea and right-sided chest
pain. He denies having fever, chills, sweats, cough,
sputum production, or hemoptysis. On auscultation
of the lungs, diminished breath sounds are heard
throughout, with more on the right than the lef. Te
chest radiograph and computed tomographic scan
of the chest show scattered interstitial changes with
cystic and nodular abnormalities, which are more
prominent in the mid and upper lung zones, and a
right-sided pneumothorax. What is the most likely
diagnosis?
a. Lymphangioleiomyomatosis
b. Pulmonary Langerhans cell histiocytosis (histiocytosis X)
c. Cystic fbrosis
d. Aspiration pneumonia
e. Idiopathic pulmonary fbrosis
addition to pulmonary embolism. A subsequent chest
radiograph is clear. A subsequent echocardiogram
shows a poorly contracting right ventricle. Gastric aspi-
rate is clear, and the stool is negative for occult blood. In
addition to transfusion, which of the following should
you recommend?
a. Perform a CT scan of the abdomen.
b. Repeat the dose of tPA.
c. Stop the heparin and place an inferior vena cava (IVC)
flter.
d. Continue heparin and place an IVC flter.
e. Perform a surgical embolectomy.
5. A 25-year-old man is admitted to the intensive care unit
(ICU) with decelerating injury afer a motor vehicle col-
lision. Te patient required on-scene mechanical extrica-
tion from the vehicle. He has bilateral lower extremity
fractures that required surgical intervention before he
arrived in the ICU. Tus far, he has received 10 L of crys-
talloid and 2 units of packed red blood cells. Over the
frst 4 hours, his blood pressure and urine output decrease
and partially respond to an additional 4 L of crystal-
loid. Currently, his blood pressure is 80/50 mm Hg, his
heart rate is 110 beats per minute, his respiratory rate is
18 breaths per minute, and he is normothermic. Other
than his lower extremity injuries, no abnormal fndings
are noted on his examination. His hemoglobin is 9 g/dL
and his coagulation values are normal. His total creatine
kinase (CK) is 800 U/L. A computed tomographic scan
of the abdomen from the emergency department is nor-
mal. What should you do next?
a. Administer a colloid fuid bolus.
b. Obtain an echocardiogram.
c. Administer methylprednisolone.
d. Perform a focused assessment with sonography for trauma
(FAST).
e. Begin a bicarbonate infusion.
6. Te incidence of transfusion-related acute lung injury
(TRALI) is greatest with transfusion of which of the
following?
a. Fresh frozen plasma
b. Packed red blood cells
c. Pooled platelets
d. Single-donor platelets
e. Salt-poor albumin
7. Which of the following best describes the efects of
critical illness on physiologic sleep?
a. Opiate-benzodiazepine combinations promote physiologic
sleep in patients receiving mechanical ventilatory support.
b. In critical illness, the proportion of rapid eye movement
(REM) sleep is decreased.
c. In critical illness, sleep has predominant waveforms consis-
tent with deep sleep.
d. Te total duration of sleep during a 24-hour period is
increased.
e. Te total duration of sleep during a 24-hour period is mark-
edly decreased.
4. PULMONARY DISEASES QUESTI ONS AND ANSWERS

37
a. Continue cefriaxone and levofoxacin for severe
community-acquired pneumonia.
b. Add vancomycin or linezolid for community-acquired
methicillin-resistant Staphylococcus aureus infection.
c. Check stool samples for ova and parasites.
d. Repeat bronchoscopy with transbronchial lung biopsies.
e. Start treatment with corticosteroids.
13. A 52-year-old man who has never smoked and who has
a past medical history of gastroesophageal refux dis-
ease and hypertension presented for recurrent pneu-
monias. Approximately 6 months before presentation,
he received a diagnosis of community-acquired pneu-
monia and was treated with macrolide antibiotics with
good resolution of his symptoms. He has since had
recurrent similar episodes of fulike illness character-
ized by the acute onset of fever, cough, and occasional
myalgias and arthralgias. Tese episodes last 1 to 2
weeks and generally seem to respond to short courses of
antibiotics. Results of human immunodefciency virus
testing and toxicology screening were negative. Chest
radiographs during those episodes showed multilobar
alveolar opacities, mostly peripheral, without evidence
of pleural efusion. Te precise location of these infl-
trates seems to vary over time without predilection for
particular lobes. Findings from all microbiological
studies have been repeatedly negative, including mul-
tiple blood cultures, urinary antigens for Streptococcus
pneumoniae and Legionella pneumophila , and sputum
fungal and mycobacterial cultures. A computed tomo-
graphic scan of the chest shows triangular-shaped,
pleural-based infltrates without evidence of an endo-
bronchial lesion, an abscess, or pleural efusion. Some
of these infltrates are characterized by ground-glass
attenuation surrounded by more consolidated opaci-
ties. Bronchoscopy with biopsies and bronchoalveolar
lavage show lymphocytic predominance without evi-
dence of infection, and biopsies show the presence of
plugs of granulation tissue within the alveolar spaces.
Which of the following is true?
a. Te location of the infltrates suggests septic emboli and the
need for an echocardiogram.
b. Te presentation is typical of tuberculosis, and the patient
should be treated with antituberculous medications.
c. A trial of corticosteroids without histologic confrmation is
appropriate.
d. Aspiration is the likely cause, and a proton pump inhibitor
should be prescribed.
e. A bronchogenic carcinoma should be excluded with surgical
lung biopsy.
14. An 82-year-old woman who has never smoked is admit-
ted for the subacute onset of shortness of breath that
limits her daily activities. Her past medical history is
signifcant for gastroesophageal refux disease, osteopo-
rosis, hypertension, and recurrent urinary tract infec-
tions. Her usual treatment includes lisinopril, aspirin,
omeprazole, alendronate, and daily nitrofurantoin.
11. A 68-year-old man, a former smoker, presents with a
6- to 12-month history of dry cough and dyspnea. He
denies fever, chest pain, or hemoptysis. No refux or
dysphagia is reported. His past history is signifcant for
diabetes mellitus, hyperlipidemia, and benign prostatic
hypertrophy. No known signifcant hobby, travel, or
environmental exposures are noted. Past occupational
history includes ship refurbishing while in the armed
services. Medications include pravastatin, glyburide,
and prazosin. His vital signs are stable, and he is afe-
brile. No adenopathy is noted. Bibasilar crackles are
present. Te heart rhythm is regular. His abdomen is
normal on palpation and auscultation, and no edema is
noted. Clubbing is present. Blood test results are nor-
mal. Pulmonary function testing indicates mild restric-
tion and a low difusing capacity. A chest radiograph
and high-resolution computed tomographic scan of the
chest show bilateral lower lobe infltrates along with
some honeycombing, pleural thickening, diaphrag-
matic calcifcation, and an area of consolidation in the
right lower lobe consistent with rounded atelectasis.
What should you do next?
a. Begin systemic corticosteroids.
b. Begin azathioprine.
c. Begin methotrexate.
d. Begin cyclophosphamide.
e. Observe only.
12. A 24-year-old man who was previously healthy pres-
ents to the emergency department with an acute onset
of shortness of breath. He denies having chest pain but
reports a signifcant nonproductive cough and general-
ized malaise with myalgias that have developed over the
past few days. He has been febrile, with temperatures up
to 38.9 C, but without chills. He takes no medication
(including over-the-counter or herbal medications)
and has never had lung or heart problems before. A day
before the onset of symptoms, he attended a gradua-
tion party and smoked several cigars with his friends,
none of whom got ill aferward. He has severe hypox-
emia with increased work of breathing and is admit-
ted to the intensive care unit. Shortly afer admission,
he requires endotracheal intubation, and mechanical
ventilation is initiated. A chest radiograph showed dif-
fuse alveolar infltrates. Tere was no pleural efusion,
and the heart size was normal. Results of the following
blood tests were normal: a complete blood cell count
and diferential count, electrolyte levels, and a coagula-
tion profle. Results of human immunodefciency virus
testing and toxicology screening were negative. He has
no relevant travel history. Arterial blood gas results
indicated profound hypoxemia with normal pH and
Pa co
2
. Immediately afer intubation (and before initia-
tion of broad-spectrum antibiotics), bronchoalveolar
lavage was performed. Results with Gram, fungal, and
microbiological stains were negative. Te diferential
leukocyte count showed 55% eosinophils. What is the
most appropriate next step?
38

MAYO CLINI C INTERNAL MEDI CINE BOARD REVIEW: QUESTI ONS AND ANSWERS
and discolored with onycholysis. Laboratory fndings
include the following: hematocrit 35%, leukocyte count
10 10
9
/L, serum creatinine 1.2 mg/dL, and serum cal-
cium 9.4 mg/dL. Chest radiography shows right pleural
efusion and a normal heart size. On diagnostic right
thoracentesis, there is serosanguineous fuid with a lac-
tate dehydrogenase (LDH) level of 500 U/L. What is
the most likely diagnosis?
a. Congestive heart failure
b. Malignant pleural efusion
c. Connective tissue disease
d. Yellow nail syndrome
e. Asbestos-related pleural efusion
17. A 45-year-old man with alcoholic cirrhosis and por-
tal hypertension presents with progressive dyspnea
over the past year. He denies having chest pain, lower
extremity edema, ascites, or fever. On physical exami-
nation, his pulse rate is 104 beats per minute, his blood
pressure is 88/44 mm Hg, and his temperature is
36.3C. Auscultation of the heart and lungs is normal,
and there is no edema. Oxygen saturation in the seated
position is 87% with room air. Laboratory fndings
include the following: hemoglobin 17 g/dL, leukocyte
count 4.1 10
9
/L, serum sodium 129 mEq/L, serum
potassium 4.2 mEq/L, and serum creatinine 1.6 mg/
dL. Computed tomography of the chest is negative for
pulmonary embolism, and pulmonary function testing
is normal with the exception of the difusion capacity,
which is 54% of the predicted value. Which test should
be performed next to help determine the underlying
diagnosis?
a. Pulmonary angiography
b. Transthoracic contrast echocardiography
c. Cardiopulmonary exercise testing
d. Cardiac catheterization
e. Overnight oximetry
18. A 50-year-old man with nonalcoholic steatohepatitis
and portal hypertension presents with progressive dys-
pnea on exertion, chest tightness, and lower extremity
edema. On physical examination, his pulse rate is 110
beats per minute, his blood pressure is 94/52 mm Hg,
and his temperature is 35.8 C. Auscultation of the lungs
is normal. Jugular venous pressure is elevated at 8 cm
water, and cardiac auscultation identifes an accentuated
pulmonic valve component (P
2
) and a right ventricular
heave. Te liver feels pulsatile with positive hepatojugu-
lar refux. Laboratory fndings include the following:
hemoglobin concentration 11.0 g/dL, leukocyte count
3.2 10
9
/L, serum sodium 131 mEq/L, serum potassium
4 mEq/L, serum creatinine 1.4 mg/dL, and N -terminal
B-type natriuretic peptide precursor (NT-pro-BNP)
1,200 pg/mL. Transthoracic echocardiography shows a
dilated inferior vena cava with no inspiratory collapse
and an estimated right ventricular systolic pressure of
84 mm Hg. His right ventricle is moderately enlarged
with reduced systolic function and a D -shaped lef ven-
tricle. On right heart catheterization, pulmonary artery
She denies having cough or sputum production but has
noticed tingling and numbness in the lower extremities.
She denies having a rash or photosensitivity, arthritis,
and Raynaud phenomenon. Her chest radiograph
shows reticular opacities preferentially located in the
bases of the lungs bilaterally, and a computed tomo-
graphic scan of her chest confrms the presence of dense
fbrotic-appearing changes in both bases without pleu-
ral efusions. Results of the following blood tests were
unremarkable: complete blood cell count and diferen-
tial count, electrolytes, and coagulation profle. Tere is
nothing to suggest aspiration. At this point, what is the
best next step?
a. Proceed with bronchoscopy with bronchoalveolar lavage
and transbronchial biopsies.
b. Initiate treatment with broad-spectrum antibiotics.
c. Discontinue use of nitrofurantoin.
d. Refer the patient for lung transplant since no treatment has
proved efective for this disease.
e. Start treatment with corticosteroids.
VASCULAR DI SEASE, EMBOLI SM, AND
HYPERTENSI ON
15. A 36-year-old woman who had a stroke 3 months ago
presents for evaluation. Her initial symptoms included
right upper extremity weakness that has completely
resolved. Magnetic resonance imaging of the brain
shows no underlying pathology other than the infarct.
As part of her evaluation, she underwent carotid ultra-
sonography, which was normal, and transthoracic
echocardiography with a bubble study, which was sug-
gestive of a right-to-lef shunt. Physical examination
fndings were as follows: temperature 37.2 C, pulse rate
90 beats per minute, respirations 20 per minute, blood
pressure 122/60 mm Hg, and oxygen saturation with
room air 91%. Lip and fnger telangiectases were noted.
What test should be performed next?
a. Cerebral angiography
b. Positron emission tomography or computed tomographic
(CT) scan
c. CT of the chest
d. Transesophageal echocardiography
e. Electroencephalography
16. A 56-year-old woman presents with dyspnea on exer-
tion and right pleural efusion over the past 8 years.
Cardiac catheterization fndings were normal. Te
patient uses diuretics; however, her lower extremities
have continued to swell. She has had 5 thoracenteses
over the years for symptomatic relief. Physical examina-
tion fndings are as follows: temperature 38.0 C, pulse
rate 90 beats per minute, respirations 20 per minute,
and blood pressure 168/92 mm Hg. Dullness to percus-
sion and diminished breath sounds are apparent in the
right lung base. Heart examination fndings and jugu-
lar venous pressure are normal. She has pretibial edema
(2+). Tere is no clubbing; however, her nails are short
4. PULMONARY DISEASES QUESTI ONS AND ANSWERS

39
21. A 47-year-old man who had been healthy without
signifcant past medical history is admitted to your
intensive care unit with severe shock. Endotracheal
intubation was performed before he arrived. Te ini-
tial chest radiograph is shown in Figure 4.Q21A. Even
afer receiving several liters of fuid and vasopressors,
he remains hypotensive. You decide to place a pulmo-
nary artery (PA) catheter. Te procedure goes smoothly
and the vessel is cannulated at frst pass. You request
another chest radiograph to confrm placement (Figure
4.Q21B). Which of the following is the cause for the
fnding on the second radiograph?
a. Vessel injury resulting in hemothorax
b. Lobar collapse due to mucous plug
c. Pneumothorax
d. Hemopneumothorax
e. Pulmonary infarct due to overwedging of the PA catheter
pressure is 88/32 mm Hg, pulmonary artery occlusion
pressure is 14 mm Hg, and cardiac output is 5.6 L/min.
What is the most likely diagnosis?
a. Idiopathic pulmonary artery hypertension
b. Diastolic dysfunction
c. Valvular heart disease
d. Portopulmonary hypertension
e. Pulmonary embolism
PULMONARY SI GNS AND SYMPTOMS AND
CHEST RADI OGRAPHY
19. An 80-year-old man with moderately severe chronic
obstructive pulmonary disease (COPD) presents to the
emergency department again for progressively worsen-
ing dyspnea and an increasingly productive cough. He
lives alone. Most recently, he was dismissed from the
hospital 3 weeks ago afer treatment of COPD exac-
erbations. His initial vital signs include oxygen satura-
tion of 87% with room air, respiratory rate 32 breaths
per minute, blood pressure 138/82 mm Hg, and pulse
102 beats per minute. Afer nebulizer treatment with
albuterol and ipratropium, his respiratory rate improves
to 28 breaths per minute and his oxygen saturation is
89% on room air. Which of the following is true about
further management of COPD for this patient?
a. He should be given oral corticosteroids and dismissed to home
with follow-up with his primary care physician in 2 days.
b. He should begin receiving tiotropium and dismissed to
home with follow-up with his primary care physician in 2
days.
c. He should be hospitalized for further management of his
COPD exacerbation.
d. He should be intubated and admitted to the intensive care
unit.
e. Afer another treatment with bronchodilators, he should be
dismissed and receive a home visit by a nurse tomorrow.
20. A 54-year-old woman presents with progressive dyspnea
at rest. Chest radiography shows signifcant lef-sided
efusion. Results of the thoracentesis and blood tests
are shown in Table 4.Q20.
Table 4.Q20
COMPONENT SERUM THORACENTESIS FLUID
Protein, g/dL 6.5 2.5
Lactate dehydrogenase, U/L 155 125
pH 7.1
Which of the following is not a possible cause for the
pleural fuid?
a. Pulmonary embolism
b. Empyema
c. Rheumatoid efusion
d. Tuberculosis
e. Malignancy
Figure 4.Q21A
Figure 4.Q21B
40

MAYO CLINI C INTERNAL MEDI CINE BOARD REVIEW: QUESTI ONS AND ANSWERS
a. Tiotropium
b. Salmeterol
c. Salmeterol with futicasone
d. Long-term oxygen therapy
e. N -acetylcysteine
24. A 19-year-old woman comes to your of ce with her
infant because she is concerned about her recurrent
sinusitis. Although she has never been hospitalized,
her younger brother was hospitalized for recurrent
pancreatitis. She describes a chronic productive cough
with dyspnea. Examination reveals wheezing and digi-
tal clubbing. What is the recommended initial test to
diagnose this disorder?
a. No further testing
b. Sweat chloride testing
c. Testing for a CFTR genetic mutation
d. Computed tomographic (CT) scan of the chest with an
intravenous contrast agent
e. CT scan of the sinuses followed by magnetic resonance
imaging of the brain if abnormalities are detected
25. A 57-year-old man presents for excessive sleepiness and
dif culty functioning at his work. His wife has com-
plained of very disruptive snoring and has witnessed
frequent apneic episodes. His body mass index is 42.
His blood pressure is 155/85 mm Hg. His neck size is
48 cm. Which of the following tests should be per-
formed next to confrm your suspicion?
a. Overnight pulse oximetry
b. Use of a 24-hour ambulatory blood pressure monitor
c. Overnight polysomnography
d. Carotid duplex ultrasonography
e. Adrenal imaging with computed tomography
22. A 57-year-old man presents with dyspnea. Pulmonary
function test (PFT) results are shown in Table 4.Q22.
Te results may be suggestive of which of the
following?
a. Asthma
b. Obesity
c. Chronic obstructive pulmonary disease
d. Severe pulmonary hypertension
e. Idiopathic pulmonary fbrosis
23. A 67-year-old man with severe chronic obstructive pul-
monary disease (COPD) presents for follow-up afer
hospitalization. He was admitted with respiratory fail-
ure and COPD exacerbation for the second time in the
past 7 months. He feels that his dyspnea is at baseline.
Which of the following has not been shown to reduce
exacerbation or rehospitalization rates?
Table 4.Q22
COMPONENT VALUE
PERCENTAGE OF
PREDICTED VALUE
Total lung capacity, L 3.51 52
Residual volume, L 2.57 102
Forced vital capacity (FVC), L 4.29 95
Forced expiratory volume in the frst
second of expiration (FEV
1
), L
3.51 91
FEV
1
/FVC 81.8
Difusing capacity of lung for carbon
monoxide, mLmin


1
mm Hg


1

15.5 60
41
mechanism of accident injury and his elevated CK. A bed-
side echocardiogram can quickly confrm this diagnosis.
Te other answer choices are less plausible.
6. Answer c.
Te risk of TRALI is related to the potential number of
exposures to plasma from diferent sources. Te greatest
risk occurs when multiple pooled sources are used, as in
platelet transfusions. Te transfusion of pooled products
from women, in particular fresh frozen plasma, has also
been associated with an increased risk of TRALI. (See
Gajic et al and Toy et al in the Suggested Reading list.)
7. Answer b.
In critical illness, disrupted and fragmented sleep leads to
impaired cognitive function. Opiates and benzodiazepines
alter normal sleep architecture, but the total duration of
physiologic sleep during a 24-hour period has been found
to be normal. Sleep patterns in critical illness have a pre-
dominance of superfcial stages N1 and N2 and a low pro-
portion of slow-wave deep sleep (stage N3). REM sleep is
severely decreased or absent. (See Cooper et al and Friese et
al in the Suggested Reading list.)
8. Answer e.
Sarcoidosis is a granulomatous disease most ofen afecting
the lungs and lymph nodes. It can occur afer a fulike illness
and may be diagnosed by a specifc constellation of symp-
toms and signs when presenting with L fgren syndrome
(erythematous nodosum, bilateral hilar lymphadenopathy,
fever, and polyarthritis). In most other instances, a diagno-
sis of sarcoidosis requires a compatible history, fndings of
noncaseating granulomas by biopsy, and exclusion of other
possible causes of granulomatous infammation. If there is
systemic involvement, blood work may show abnormali-
ties, including hypercalcemia, anemia, and elevated liver
enzymes. Serum angiotensin-converting enzyme levels are
neither specifc nor sensitive to use as a diagnostic tool but,
when elevated, may be helpful for following disease activ-
ity. Bronchoscopy can confrm granulomatous disease in
over 90% of patients with hilar adenopathy and parenchy-
mal lung involvement. Rales are uncommon in sarcoidosis
even when parenchymal interstitial changes are present.
Incidence, clinical course, and prognosis of sarcoidosis are
infuenced by ethnic and genetic factors. Computed tomo-
graphic scan may show nodular opacities with bronchovas-
cular and subpleural distribution, thickened intralobular
1. Answer b.
Tis patient has hypertrophic cardiomyopathy with outfow
obstruction that is exacerbated by intravascular volume con-
traction. His diagnosis is suggested by 1) the murmur, 2)
tachycardia (shortened lef ventricular diastolic flling time),
3) lung crackles, 4) absence of jugular vein distention, and
5) worsening with furosemide-induced diuresis. His fuid
output and input are matched since admission, but his surgi-
cal procedure is associated with increased third-space fuid
losses. Terefore, his intravascular volume is likely depleted.
Of the answer choices given and the various diferential
diagnostic possibilities (congestive heart failure, myocardial
infarction, pulmonary embolism, and methemoglobinemia),
only metoprolol will improve his outfow obstruction.
2. Answer a.
Tis patient has an acute, agitated delirium. Benzodia-
zepines are not indicated; they may worsen delirium and
merely suppress its manifestations. Similarly, this is not a
pain syndrome, and opioids will suppress but not treat the
underlying pathophysiology.
3. Answer a.
Emerging data suggest that the interval of time from symp-
tom onset to administration of thrombolytic therapy can
be prolonged in selected patients with acute ischemic
stroke, including patients not at increased risk of bleed-
ing. Tese patients had improved survival if they received
thrombolytic therapy up to 4.5 hours afer onset of symp-
toms. (See Hacke et al in the Suggested Reading list.)
4. Answer a.
Tis patient presents with a massive pulmonary embolism
and shock. Trombolysis or embolectomy is indicated. Te
lack of improvement refects either failure of primary throm-
bolysis or development of a new complication. Te central
venous pressure is much lower than would be expected in a
patient with pulmonary embolism and acute right heart fail-
ure. Te rib fractures and decreased hemoglobin suggest an
occult site of bleeding, potentially in the abdomen or retro-
peritoneum, which needs to be identifed before a decision
is made on possible rescue therapy for the pulmonary embo-
lism. (See Meneveau et al in the Suggested Reading list.)
5. Answer b.
Tis patient has myocardial contusion afer trauma, with
possible lef ventricular failure. Tis is suggested by the
ANSWERS
42

MAYO CLINI C INTERNAL MEDI CINE BOARD REVIEW: QUESTI ONS AND ANSWERS
fbers typically are dormant for decades before pulmonary
fbrosis develops. A clinical response to corticosteroids
or other medication is not expected in the treatment of
asbestos-related pulmonary fbrosis. Smoking in the pres-
ence of asbestos-related pulmonary fbrosis increases the
rate of progression of fbrosis and the risk of bronchogenic
carcinoma. Tuberculosis is not a common complication of
asbestosis. No therapy has been efective in preventing pro-
gressive pulmonary fbrosis due to asbestos exposure.
12. Answer e.
Acute eosinophilic pneumonia is a rare but increasingly rec-
ognized cause of acute respiratory distress syndrome. Te
pathophysiology is unclear, but a recent onset of smoking
and an increase in smoking patterns have been recognized
as common precipitating factors. Clinically, acute eosino-
philic pneumonia is characterized by the acute onset of dif-
fuse alveolar infltrates with pulmonary eosinophilia; unlike
in chronic eosinophilic pneumonia, peripheral eosinophilia
is rare. Radiologically, the infltrates are difuse but nonspe-
cifc, and treatment with corticosteroids generally results
in dramatic improvement. Bronchoscopic lung biopsies are
not generally needed for the diagnosis. Other causes of pul-
monary eosinophilia include allergic bronchopulmonary
aspergillosis, Churg-Strauss syndrome, parasitic infections,
some fungal infections, drug-induced lung diseases, chronic
eosinophilic pneumonia, and, rarely, connective tissue dis-
easerelated interstitial lung disease and malignancies.
13. Answer c.
Te likely diagnosis is cryptogenic organizing pneumo-
nia (formerly known as idiopathic bronchiolitis obliter-
ans with organizing pneumonia, or BOOP). Organizing
pneumonia is characterized pathologically by the presence
of plugs of fbroblasts and myofbroblasts occupying the
distal airspaces without disruption of the underlying archi-
tecture of the lung. Cryptogenic organizing pneumonia
typically manifests with recurrent or persistent episodes of
pneumonia-like illness that occasionally responds to anti-
biotics, particularly macrolides. Te treatment consists of a
prolonged course of corticosteroids ( 3 months), and the
absence of response to treatment should suggest an under-
lying cause such as hematologic malignancy, drug-induced
lung disease, or connective tissue disease (ie, secondary
organizing pneumonia). While usually very efective,
rebound afer discontinuation of treatment is frequent.
14. Answer c.
Te possibility of drug-induced lung disease should always
be considered when evaluating a case of difuse parenchy-
mal lung disease. Since nitrofurantoin is a common ofender
(with bleomycin, methotrexate, and amiodarone), its use
should be carefully investigated because patients may not
always volunteer that information. Nitrofurantoin lung
toxicity can manifest either as an acute form of lung toxicity
(when it is usually associated with peripheral eosinophilia
and elevated infammatory markers) or, less frequently
(as in this case), as a chronic form of fbrotic lung disease.
septa, architectural distortion, or conglomerate masses
(late stage). Tobacco use has not been associated with
development of sarcoidosis. Extrapulmonary involvement
from sarcoidosis may involve the heart, liver, spleen, eyes,
bone, skin, bone marrow, parotid glands, pituitary gland,
and reproductive organs. Tis patients presentation is
most consistent with L fgren syndrome, which carries
a very good prognosis; symptoms resolve without treat-
ment. Tus, observation with follow-up chest radiography
is appropriate. If symptoms are more bothersome, symp-
tomatic treatment such as nonsteroidal anti-infammatory
agents may be considered. For progressive pulmonary and
extrapulmonary disease, corticosteroids or immunosup-
pressive therapy should be considered.
9. Answer e.
Te combination of interstitial lung infltrates predomi-
nantly involving the lower lung zones, lack of occupational
exposure, duration of symptoms, and peripheral honey-
combing make the diagnosis of idiopathic pulmonary
fbrosis (IPF) most likely. Favorable prognostic factors in
IPFusual interstitial pneumonia include age younger than
50, female sex, shorter duration of symptoms before pre-
sentation, presence of ground-glass opacities on CT scan
of the chest, and lymphocytosis on examination of bron-
choalveolar lavage fuid. Pulmonary function tests in IPF
usually indicate restrictive impairment. Patients with IPF
generally do not respond to corticosteroids or other immu-
nosuppressive therapies. No clearly efective treatment
options are currently available. Oxygen extends survival
among patients with chronic obstructive pulmonary dis-
ease, but this beneft has not been shown for IPF patients.
Familial clusters of IPF patients suggest a potential genetic
predisposition in some cases of IPF.
10. Answer b.
Tis patients presentation is most consistent with adult
pulmonary Langerhans cell histiocytosis, which, in most
cases, is a form of smoking-related interstitial lung dis-
ease. Smoking cessation is the primary form of treatment.
Stabilization or improvement occurs in up to two-thirds
of patients with smoking cessation alone. Other therapies
including systemic corticosteroids and immunosuppres-
sives have been used with limited success. No role of plas-
mapheresis has been described.
11. Answer e.
Asbestosis typically has a basilar predominance, and
rounded atelectasis is suggestive of asbestos exposure.
Known or remembered histories of occupational asbestos
exposure may not always be present, but they can be help-
ful. Common fndings include pleural plaques or diaphrag-
matic calcifcations (or both). Malignant mesothelioma is
strongly associated with asbestos exposure but not smok-
ing. Pleural surfaces are generally abnormal and involved
with asbestos-related parenchymal lung disease. However,
pleural and parenchymal abnormalities may, in some
instances, occur independently of each other. Asbestos
4. PULMONARY DISEASES QUESTI ONS AND ANSWERS

43
19. Answer c.
Tis patient has multiple risk factors requiring in-patient
management of his COPD exacerbation. As noted by the
Global Initiative for Chronic Obstructive Lung Disease
(GOLD), indications for hospital admission for COPD
exacerbation include the following: marked increase in
intensity of symptoms, severe underlying disease, onset
of new physical signs, failure of exacerbation to respond
to initial medical management, signifcant comorbidi-
ties, newly occurring arrhythmias, diagnostic uncertainty,
frequent exacerbations, older age, and insuf cient home
support. Although the patients symptoms improved some-
what initially, with the persistence of abnormal vital signs
and other risk factors noted above, he should be admitted
for further management. (See Rabe et al in the Suggested
Reading list.)
20. Answer a.
Pleural fluid analysis and the ratio of pleural fluid lactate
dehydrogenase (LDH) to serum LDH (125:155) show
that the fluid is an exudate. For fluid to be considered
an exudate, the fluid needs to meet only 1 of the Light
criteria (ratio of pleural fluid protein to serum protein
>0.5, ratio of pleural fluid LDH to serum LDH >0.6,
pleural fluid LDH greater than two-thirds of the upper
limit of the reference range for serum LDH). While the
protein ratio is less than 0.5, the LDH ratio is greater
than 0.6; thus, this fluid is an exudate. Furthermore,
the low pH suggests a certain diagnosis. When thora-
centesis fluid pH is less than 7.3, diagnostic possibili-
ties include empyema, esophageal rupture, rheumatoid
arthritis, trauma, tuberculosis, and malignancy. Pleural
fluid in pulmonary embolism may be either a transudate
or an exudate, but pH of the pleural fluid should not be
severely acidic.
21. Answer b.
While all the answer choices are potential complications
of PA catheter placement, careful inspection of the radio-
graph shows that lobar collapse is the best answer. Note
the elevation of the minor fssure and the elevation of the
right hemidiaphragm. Tese are characteristic fndings of
a lobar collapse. Hemothorax should not be limited to the
upper lobes only; rather, fuid would accumulate in the
lower portions of the chest, thereby blunting the costo-
phrenic angle. Pneumothorax should result in collapse of
the lower lobes as well in this previously healthy patient.
Furthermore, pneumothorax should create an air interface
and thus appear black, not white, on the radiograph. A
hemopneumothorax should create an air-liquid interface
along the lateral margin of the lung, which is not appar-
ent in this radiograph. Te region of involvement would be
too large for pulmonary infarct due to overwedging. Tis
patient underwent bronchoscopic clearing of the airways;
the chest radiograph 30 minutes later is shown in Figure
4.A21.
Discontinuation of the drug is warranted in both cases and
should take precedence over any other intervention. Te
role of corticosteroids, sometimes recommended for the
acute form of the disease, is less clear in chronic cases, which
may progress and even be fatal in up to 10% of patients.
15. Answer c.
Tis young patient has lip and fnger telangiectases, an
inappropriately low oxygen saturation, and evidence of
a right-to-lef shunt on bubble echocardiography. Her
stroke was likely caused by paradoxical embolism, and her
low oxygen saturation suggests a pulmonary arteriovenous
malformation, so that CT of the chest is the best choice.
With the telangiectases, the shunt, and the suggestion of
a pulmonary arteriovenous malformation, the most likely
diagnosis is hereditary hemorrhagic telangiectasia. Te
clinical criteria for this diagnosis include telangiectases
(lips, mouth, nose, and fngertips), visceral arteriovenous
malformation, epistaxis, and family history of any of the
preceding criteria. Stroke in a young person should prompt
one to think of pulmonary arteriovenous malformation.
16. Answer d.
Tis patient presents with dyspnea, recurrent right pleu-
ral efusion, lower extremity edema, and discolored, dys-
trophic nails. Her thoracentesis fuid is exudative, with an
LDH of 500 U/L. Te most likely diagnosis is yellow nail
syndrome, which consists of the triad of yellow nails, lym-
phedema, and respiratory tract illness (pleural efusions,
bronchiectasis, and recurrent pneumonias). Te nails usu-
ally do not grow, and patients will wonder why they do not
have to cut their nails.
17. Answer b.
Tis man has alcoholic cirrhosis, portal hypertension, and
hypoxemia. Te most likely diagnosis is hepatopulmonary
syndrome. Te clinical triad in the diagnosis is 1) pres-
ence of liver disease with portal hypertension, 2) intrapul-
monary shunting, and 3) hypoxemia. Te transthoracic
contrast echocardiogram will show the presence of intra-
pulmonary shunting by the passage of bubbles into the
lef heart afer 3 to 5 cardiac cycles. Te polycythemia is
due to the chronic hypoxemia, which is present at rest and
worsens with exertion. Liver transplant is the treatment of
choice if the patient is a surgical candidate otherwise.
18. Answer d.
Tis patient has severe pulmonary artery hypertension with
liver disease and portal hypertension. Te most likely diag-
nosis is portopulmonary hypertension because of the follow-
ing criteria: 1) liver disease or portal hypertension, 2) mean
pulmonary artery pressure greater than 25 mm Hg, 3) pul-
monary vascular resistance greater than 240 dynesscm


5
,
and 4) pulmonary artery occlusion pressure less than
15 mm Hg. Liver transplant for this patient is contraindi-
cated, and he should be considered for vasodilator therapy
to try to control the pulmonary artery hypertension.
44

MAYO CLINI C INTERNAL MEDI CINE BOARD REVIEW: QUESTI ONS AND ANSWERS
Figure 4.A21
is recommended. (See Farrell et al and McMullen et al in
the Suggested Reading list.)
25. Answer c.
Tis patient has all the risk factors for at least moderately
severe obstructive sleep apnea (OSA). Terefore, overnight
polysomnography should be performed next to confrm
the diagnosis. If OSA is confrmed, treatment should
begin with a continuous positive airway pressure device.
Overnight oximetry is occasionally used as a screening tool,
but the results are not suf cient to establish the diagnosis
even though they may be suggestive of OSA. Other tests
have no role in establishing the diagnosis of OSA.
SUGGESTED READING
Calverley PM, Anderson JA, Celli B, Ferguson GT, Jenkins C, Jones PW,
et al; TORCH investigators. Salmeterol and futicasone propionate
and survival in chronic obstructive pulmonary disease. N Engl J Med.
2007 Feb 22;356(8):77589.
Cooper AB, Tornley KS, Young GB, Slutsky AS, Stewart TE, Hanly
PJ. Sleep in critically ill patients requiring mechanical ventila-
tion. Chest. 2000 Mar;117(3):80918. Erratum in: Chest 2001
Mar;119(3):993.
Farrell PM, Rosenstein BJ, White TB, Accurso FJ, Castellani C, Cutting
GR, et al; Cystic Fibrosis Foundation. Guidelines for diagnosis of cystic
fbrosis in newborns through older adults: Cystic Fibrosis Foundation
consensus report. J Pediatr. 2008 Aug;153(2):S4-S14.
Friese RS, Diaz-Arrastia R, McBride D, Frankel H, Gentilello LM.
Quantity and quality of sleep in the surgical intensive care unit: are our
patients sleeping? J Trauma. 2007 Dec;63(6):12104.
Gajic O, Rana R, Winters JL, Yilmaz M, Mendez JL, Rickman OB,
et al. Transfusion-related acute lung injury in the critically ill: prospec-
tive nested case-control study. Am J Respir Crit Care Med. 2007 Nov
1;176(9):88691. Epub 2007 Jul 12.
Gerrits CM, Herings RM, Leufens HG, Lammers JW. N-acetylcysteine
reduces the risk of re-hospitalisation among patients with
chronic obstructive pulmonary disease. Eur Respir J. 2003
May;21(5):7958.
Gonzalez C, Servera E, Marin J. Importance of noninvasively measured
respiratory muscle overload among the causes of hospital readmis-
sion of COPD patients. Chest. 2008 Apr;133(4):9417. Epub 2008
Feb 8.
Hacke W, Kaste M, Bluhmki E, Brozman M, Davalos A, Guidetti D, et al;
ECASS Investigators. Trombolysis with alteplase 3 to 4.5 hours afer
acute ischemic stroke. N Engl J Med. 2008 Sep 25;359(13):131729.
McMullen AH, Pasta DJ, Frederick PD, Konstan MW, Morgan WJ,
Schechter MS, et al. Impact of pregnancy on women with cystic fbro-
sis. Chest. 2006 Mar;129(3):70611.
Meneveau N, Seronde MF, Blonde MC, Legalery P, Didier-Petit K, Briand F,
et al. Management of unsuccessful thrombolysis in acute massive pul-
monary embolism. Chest. 2006 Apr;129(4):104350.
Rabe KF, Hurd S, Anzueto A, Barnes PJ, Buist SA, Calverley P, et al;
Global Initiative for Chronic Obstructive Lung Disease. Global strat-
egy for the diagnosis, management, and prevention of chronic obstruc-
tive pulmonary disease: GOLD executive summary. Am J Respir Crit
Care Med. 2007 Sep 15;176(6):53255. Epub 2007 May 16.
Tashkin DP, Celli B, Senn S, Burkhart D, Kesten S, Menjoge S,
et al; UPLIFT Study Investigators. A 4-year trial of tiotropium in
chronic obstructive pulmonary disease. N Engl J Med. 2008 Oct
9;359(15):154354. Epub 2008 Oct 5.
Toy P, Popovsky MA, Abraham E, Ambruso DR, Holness LG, Kopko PM,
et al; National Heart, Lung and Blood Institute Working Group on
TRALI. Transfusion-related acute lung injury: defnition and review.
Crit Care Med. 2005 Apr;33(4):7216.

22. Answer e.
PFT results show a moderately severe restrictive defect with
a markedly reduced difusing capacity of lung for carbon
monoxide ( Dlco ). Of all the options, only idiopathic pul-
monary fbrosis fts the PFT fndings. A restrictive defect
would not be expected in asthma or chronic obstructive
pulmonary disease. Obesity may result in a restrictive lung
defect, but the D lco is ofen normal to elevated. While
the D lco may be reduced in severe pulmonary hyperten-
sion, lung volumes should not be reduced.
23. Answer d.
Several studies have examined readmission rates and mea-
sures that mitigate them. Tiotropium reduced exacerba-
tions, among other outcomes, during a 4-year trial. Another
trial compared the efect of salmeterol alone, futicasone
alone, or the 2 drugs in combination. Exacerbation rates
were signifcantly reduced in all groups compared with
placebo, and combination therapy reduced hospitaliza-
tion rates. Furthermore, use of N -acetylcysteine reduced
the risk of readmissions in a study of 1,219 patients. While
long-term oxygen therapy reduces overall mortality, its
use was associated with increased risk of readmissions in
those with moderate-to-severe COPD. (See Calverley
et al, Gerrits et al, Gonzalez et al, and Tashkin et al in the
Suggested Reading list.)
24. Answer b.
Tis patient has recurrent sinusitis, wheezing, digital club-
bing, and a family member with recurrent pancreatitis.
Cystic fbrosis must be considered as the underlying disor-
der. Women with cystic fbrosis can be fertile if they have
adequate nutritional and pulmonary reserve. In contrast,
men ofen present with azoospermia. Te Cystic Fibrosis
Foundation recommends sweat chloride testing as the ini-
tial diagnostic test. If the sweat chloride concentration is
more than 60 mEq/L, the diagnosis of cystic fbrosis is con-
frmed; if it is 30 to 59 mEq/L, testing for CFTR mutations
45
no clubbing or cyanosis. Te total leukocyte count is
14.4 10
9
/L. Results of serum chemistry tests are nor-
mal. Te chest radiograph shows a new dense consolida-
tion in the lef upper lung feld. Which of the following
antimicrobial regimens would you initiate?
a. Azithromycin orally
b. Cefriaxone intravenously and azithromycin orally
c. Ciprofoxacin intravenously
d. Ampicillin intravenously and clindamycin orally
e. Meropenem intravenously
3. A 44-year-old male business executive is admitted to
the intensive care unit for fevers, diarrhea, cough, and
shortness of breath progressing over the past 48 hours.
His past medical history includes cigarette smok-
ing and weekly binge drinking but no other chronic
medical problems. He is from St Louis, Missouri, but
was attending a conference in Chicago, Illinois, 2 days
before admission. Abdominal pain, diarrhea, cough,
and shortness of breath developed 2 days before admis-
sion. He had mentioned that other people from the
convention had similar illnesses. He has had no other
recent travel and has not had any exposure to animals.
Antimicrobial therapy has been started and blood
and sputum cultures are pending. Examination fnd-
ings include the following: temperature 39.2 C, blood
pressure 85/60 mm Hg, heart rate 72 beats per min-
ute with regular rate and rhythm, and respiratory rate
40 breaths per minute. Oxygen saturation is 90% with
10 L of oxygen per minute by face mask. He is drowsy and
does not reliably follow commands, and he is oriented
to person only. Difuse bilateral crackles are audible on
lung auscultation, his abdomen has active bowel sounds
and is not tender, and his extremities have no club-
bing or cyanosis. Remarkable diagnostic test results
were leukocytosis (16.2 10
9
/L) and hyponatremia
(126 mEq/L). A chest radiograph showed bilateral patchy
infltrates. Gram staining of a sputum showed scant poly-
morphonuclear cells and no bacteria. What is the most
likely cause of the patients respiratory infection?
a. Legionella pneumophila
b. Respiratory syncytial virus (RSV)
c. Enterovirus
d. Chlamydophila psittaci
e. Coccidioides immitis
4. Which of the following is a recommended intervention
to reduce the risk of ventilator-associated pneumonia
among patients receiving mechanical ventilation?
QUESTI ONS
Multiple Choice (choose the best answer)
PNEUMONI A, ZOONOSES, TRAVEL, AND
BI OTERRORI SM
1. A 67-year-old man with uncomplicated type 2 diabe-
tes mellitus and hypertension presents to his primary
care physician with a 3-day history of fever, productive
cough, and shortness of breath. He has been in good
health otherwise, has never been hospitalized, and
has good social support at home. Examination fnd-
ings include the following: temperature 39.1 C, blood
pressure 110/75 mm Hg, heart rate 78 beats per min-
ute with regular rate and rhythm, and respiratory rate
26 breaths per minute. Oxygen saturation is 94% with
room air. He is awake and alert and oriented to person,
place, and time; he responds to questions appropriately.
Inspiratory crackles are audible in the right lower lung
feld, his abdomen has active bowel sounds and is not
tender, and his extremities have no clubbing or cyano-
sis. Which of the following should be done next?
a. Outpatient observation only with follow-up in 2 days
b. Outpatient consultation with an infectious diseases
specialist
c. Hospital admission and treatment with levofoxacin
d. Intensive care unit admission and treatment with
levofoxacin
e. Outpatient treatment with levofoxacin
2. A 57-year-old woman is admitted to an internal medi-
cine hospital ward with a 3-day history of fever, cough,
progressively purulent sputum, and shortness of breath.
She has been well otherwise, has never been hospital-
ized, and has a past medical history of well-controlled
type 2 diabetes mellitus and hypertension. She has not
had any recent antimicrobial exposure, foreign travel,
or animal exposure. She has no known drug allergies.
Examination fndings include the following: tempera-
ture 38.7 C, blood pressure 125/75 mm Hg, heart rate
78 beats per minute with regular rate and rhythm, and
respiratory rate 26 breaths per minute. Oxygen satu-
ration is 95% with 2 L of oxygen per minute by nasal
cannula. She is awake and alert and oriented to person,
place, and time. Inspiratory crackles are audible in the
lef upper lung feld, her abdomen has active bowel
sounds and is not tender, and her extremities have
5.
INFECTIOUS DISEASES QUESTIONS AND ANSWERS
46

MAYO CLINI C INTERNAL MEDI CINE BOARD REVIEW: QUESTI ONS AND ANSWERS
are growing a gram-positive rod in 2 out of 2 bottles.
Terapy with cefriaxone and levofoxacin was started
upon admission to the ICU. What is the most likely
etiologic agent?
a. Streptococcus pneumoniae
b. Avian infuenza virus H5N1
c. Francisella tularensis
d. Bacillus anthracis
e. Listeria monocytogenes
SKI N AND SOFT TI SSUE I NFECTI ONS,
BONE AND J OI NT I NFECTI ONS, AND
MYCOBACTERI AL I NFECTI ONS
8. A 53-year-old diabetic man underwent abdominal
hernia repair 4 days ago afer receiving preoperative
cefazolin. A nurse calls you to evaluate him for fever,
hypotension, and tachycardia. He has a toxic appear-
ance; his incision is tender and erythematous with some
sero-sanguinous drainage. Which group of bacteria
should your therapy target?
a. Gram-positive cocci
b. Gram-positive cocci and gram-positive bacilli
c. Gram-positive cocci and gram-positive and gram-negative
bacilli
d. Gram-negative bacilli
e. Gram-positive bacilli
9. A 31-year-old man underwent septoplasty for a deviated
nasal septum and recurrent sinusitis. He presents to an
emergency department 48 hours later with headache,
fever, chills, myalgia, nausea, vomiting, and abdomi-
nal cramping. His temperature is 39.9 C, his pulse is
142 beats per minute, his respiratory rate is 28 breaths
per minute, and his blood pressure is 74/30 mm Hg. He
has generalized erythroderma. On head and neck exam-
ination, the nasal passages are hyperemic but not puru-
lent. Laboratory test results included the following:
leukocyte count 14.7 10
9
/L, hemoglobin 15.0 g/dL,
and platelet count 84 10
9
/L. Te results of liver func-
tion tests, serum creatinine, and amylase were normal.
Which pathogen is most likely to cause this syndrome?
a. Pseudomonas aeruginosa
b. Haemophilus infuenzae
c. Moraxella catarrhalis
d. Streptococcus pneumoniae
e. Staphylococcus aureus
10. Which of the following patients is best suited for out-
patient parenteral antibiotic therapy (OPAT) for the
infection?
a. A 23-year-old injection drug user with Staphylococcus aureus
tibial osteomyelitis
b. A 76-year-old man with enterococcal prosthetic valve endo-
carditis with a new Wenckebach heart block
c. A 46-year-old woman with a diabetic foot ulcer and
Pseudomonas metatarsal osteomyelitis that requires
imipenem-cilastatin every 6 hours
a. Preference for invasive ventilation over noninvasive
ventilation
b. Preference for nasotracheal intubation over orotracheal
intubation
c. Keeping patients supine during enteral feeding
d. Preference for parenteral nutrition over enteral nutrition
e. Maintaining endotracheal tube cuf pressure greater than
20 cm water
5. For which pair of pathogens should hospital anti-
microbial resistance rates be considered when an
empirical antimicrobial regimen is chosen to treat
hospital-acquired pneumonia?
a. Acinetobacter baumannii and Candida albicans
b. Streptococcus pneumoniae and Haemophilus infuenzae
c. Pseudomonas aeruginosa and Staphylococcus aureus
d. Klebsiella pneumoniae and infuenza virus
e. Serratia marcescens and Enterococcus faecium
6. A previously healthy 32-year-old man presents to his pri-
mary care physician afer his cat bit his arm earlier in the
morning. Te cats vaccinations are current, and the cat
has not been attacked by other animals. Te patient says
that the bite was deep enough to draw blood, although
the bleeding has stopped. You cleanse the wound, apply
a bandage, and update his tetanus-diphtheria vaccina-
tion. Examination fndings include the following: tem-
perature 37.2 C, blood pressure 125/80 mm Hg, heart
rate 72 beats per minute, and respiratory rate 16 breaths
per minute. A 1.5-cm laceration is evident on the dor-
sal aspect of the lef forearm without any erythema or
purulence. Which of the following antimicrobial regi-
mens should be prescribed?
a. Amoxicillin-clavulanic acid
b. Dicloxacillin
c. Cephalexin
d. Clindamycin
e. No antimicrobials
7. A previously healthy 25-year-old man is admitted to an
intensive care unit (ICU) with a 4-hour history of rap-
idly progressive fever, shortness of breath, and cough.
He had spent most of the previous 2 days at the county
fair. Upon arrival at the hospital, the patient underwent
endotracheal intubation, was given fuid resuscitation,
and received vasopressor medications and mechanical
ventilation. Examination fndings include the follow-
ing: temperature 40.2 C, blood pressure 90/60 mm Hg
with vasopressors, heart rate 102 beats per minute
(tachycardic), and respiratory rate 16 breaths per
minute. Oxygen saturation is 95% with assist/control
ventilation and 50% fraction of inspired oxygen. He
is sedated and does not follow commands. Difuse
bilateral crackles are audible on lung auscultation, his
abdomen has active bowel sounds and is not tender,
and his extremities have no clubbing or cyanosis. A
chest radiograph shows difuse pulmonary infltrates
and a widened mediastinum. Blood samples drawn in
the emergency department 2 hours ago for cultures
5. I NFECTI OUS DI SEASES QUESTI ONS AND ANSWERS

47
rate and C-reactive protein level have been normal.
What is the most likely cause of the change in her blood
glucose level and blood pressure?
a. Ongoing infection and need for resection arthroplasty
b. Poor control of diet
c. Medication interaction
d. Inadequate pain control
e. Poor adherence to drug therapy
15. Each of the following patients underwent a tuberculin
skin test (TST) for appropriate indications. Which
result would be considered a positive TST reaction
(ie, a positive purifed protein derivative skin test
[PPD])?
a. A 36-year-old man who is positive for human immunodef-
ciency virus (HIV)an 8-mm induration
b. A 26-year-old Mexican native migrant workera 3-mm
induration last year and a 20-mm induration this year
c. A 42-year-old man whose chest radiograph suggests old
tuberculosisa 6-mm induration
d. A 56-year-old diabetic womana 15-mm induration
e. All of the above
16. Which of the following tuberculosis-suspect patients
should be considered infectious?
a. A 44-year-old man had a smear positive for acid-fast bacilli
(AFB) 2 weeks ago; no further specimens were obtained. He
has received self-administered antitubercular therapy for the
past 7 days and continues to cough.
b. A 22-year-old school teacher with pulmonary tubercu-
losis has been receiving directly observed therapy for 6
weeks and is asymptomatic. Tree sputum smears have
been evaluated; the frst was positive and the rest were
negative.
c. Cough and malaise developed in a 56-year-old Asian
man visiting his grandchildren. A tuberculin skin test and
QuantiFERON-TB Gold test were both positive. His chest
radiograph was normal, and 3 sputum samples were negative
for AFB.
d. A 74-year-old woman with a cough has lost weight. Te
QuantiFERON-TB Gold test was positive, and her chest
radiograph was normal.
e. A 54-year-old health care worker had a newly positive
purifed protein derivative skin test and a normal chest
radiograph.
17. A 24-year-old male landscaper with human immuno-
defciency virus (HIV), sickle cell anemia, and chronic
renal failure has a hot, painful, swollen right knee.
Results of an aspiration of the knee joint are as fol-
lows: the leukocyte count is 65 10
9
/L with 65% poly-
morphonuclear cells, and Gram staining is negative for
microorganisms. Which of the following is the least
likely cause of his symptoms?
a. Acute sickle cell crisis
b. Acute gout
c. Acute gonococcal arthritis
d. Acute Salmonella arthritis
e. Acute Staphylococcus aureus arthritis
d. A 26-year-old woman with pneumococcal meningitis that is
improving on day 7 of a 14-day cefriaxone regimen
e. An 89-year-old man with early Alzheimer disease treated
with cefriaxone for community-acquired pneumonia
11. A 45-year-old man with diabetes mellitus and severe
peripheral vascular disease presents with a 6-week his-
tory of erythema and induration surrounding a 3-cm
plantar ulcer. Today he is nauseated, febrile, and tachy-
cardic. You can insert a metallic probe through the
open wound to the bone surface. Tere is surround-
ing redness and drainage of foul-smelling pus. Which
of the following would be the next appropriate step in
management?
a. Swabbing the patients nose for methicillin-resistant
Staphylococcus aureus (MRSA)
b. Magnetic resonance imaging (MRI) of the foot
c. Parenteral vancomycin and piperacillin-tazobactam
d. Bone scintigraphy
e. Plain radiography of the foot
12. For which of the following would use of the
QuantiFERON-TB Gold test for Mycobacterium tuber-
culosis be preferable to a purifed protein derivative
(PPD) skin test?
a. A 23-year-old resident physician from India who received
BCG vaccine as a child
b. A 45-year-old health care worker with recent exposure to
someone with tuberculosis in the past 3 months
c. A 62-year-old man who recently returned from 1 year of vol-
unteer work in Rwanda
d. A 35-year-old Somali woman with a new diagnosis of
human immunodefciency virus infection (CD4 count 478
cells/ L)
e. A 29-year-old nurse undergoing annual required tuberculo-
sis screening
13. A 34-year-old Mexican immigrant with human immu-
nodifciency virus infection (CD4 count 300 cells/ L)
has been treated with isoniazid, rifampin, pyrazin-
amide, ethambutol, and pyridoxine for pulmonary
tuberculosis for the past 2 months. He is not currently
taking antiretroviral medications. He now presents
with severe pain, swelling, and redness of the lef great
toe. Which of the following medications is most likely
associated with this condition?
a. Isoniazid
b. Rifampin
c. Pyridoxine
d. Ethambutol
e. Pyrazinamide
14. A 76-year-old woman with hypertension and type 2 dia-
betes mellitus is seen for worsening hypertension and
poor glycemic control. She is taking verapamil and gly-
buride. Recently, a methicillin-resistant Staphylococcus
aureus infection developed in her lef prosthetic hip
joint, for which she underwent d bridement and com-
ponent retention and is at a nursing home receiving van-
comycin and rifampin. Her erythrocyte sedimentation
48

MAYO CLINI C INTERNAL MEDI CINE BOARD REVIEW: QUESTI ONS AND ANSWERS
pressure is 120/75 mm Hg, her heart rate is 80 beats
per minute, and her respiratory rate is 18 breaths per
minute. She has cervical lymphadenopathy and mild
difuse abdominal tenderness. Findings on the rest of
the examination, including mouth, lungs, heart, and
pelvis, are unremarkable. Laboratory evaluation shows
the following: hemoglobin 12.5 g/dL, leukocyte count
8.0 10
9
/L, and platelet count 200 10
9
/L; atypical
lymphocytes are noted on the peripheral smear, the
serum creatinine is normal (1.0 mg/dL), and liver func-
tion test values (aspartate aminotransferase, alanine
aminotransferase, bilirubin, and alkaline phosphatase)
are all within the reference range. Which statement is
false ?
a. Results of HIV antibody testing will likely be negative.
b. If the HIV test results are positive, genotyping should be
performed.
c. Testing for acute HIV, Epstein-Barr virus, and cytomega-
lovirus should be done if her symptoms persist for several
more weeks, but it is not currently indicated.
d. If she has acute HIV, her HIV viral load by polymerase chain
reaction should be more than 100,000 copies/mL.
e. Patients with undiagnosed HIV infection ofen deny being
at risk.
21. A 43-year-old woman with long-standing human immu-
nodefciency virus (HIV) infection and hepatitis C
began a salvage program of stavudine/tenofovir/lami-
vudine/darunavir with boosted ritonavir in combination
with raltegravir about 4 months ago. She is also taking
pravastatin and fsh oil for hyperlipidemia. In a routine
appointment 6 weeks ago, she reported mild nausea and
fatigue. A pregnancy test was negative. Her viral load
was suppressed for the frst time in 4 years. Her CD4
count had increased from 205 to 240 cells/ L. Her ala-
nine aminotransferase (ALT) increased slightly from 50
to 75 U/L. She reports feeling increasingly unwell, and
her appetite has been poor. She believes that she may
have lost weight. In the past 3 days she has had new dif-
fuse abdominal discomfort. Laboratory values are as
follows: ALT 200 U/L, glucose 115 mg/dL, leukocyte
count 8.0 10
9
/L, hemoglobin 12.5 g/dL, CD4 count
245 cells/ L, HIV viral load nondetectable, and lactate
12 mmol/L. Which, if any, of her medications is likely
responsible for her symptoms and elevated lactate?
a. Te protease inhibitor darunavir
b. Nucleoside reverse transcriptase inhibitors (NRTIs), espe-
cially stavudine
c. None of her HIV medications, but instead an HIV-associated
infection
d. Pravastatin combined with the ritonavir
e. Raltegravir
22. A 36-year-old surgical nurse was stuck with a suture
needle while assisting with emergent surgery of a patient
who is human immunodefciency virus (HIV) positive.
She saw blood in her glove and called the employee
health of ce for recommendations. Which of the fol-
lowing is false ?
HI V I NFECTI ON
18. A 34-year-old man who is positive for human immu-
nodefciency virus (HIV) presents with progressive
headache, irritability, and low-grade fever over the
past month. A friend who brought him to the emer-
gency department today is worried about his persis-
tent complaints. Te patient has not been seen in the
clinic for 3 years and has not taken antiretrovirals
for more than 2 years. At the initial diagnosis 8 years
ago, cytomegalovirus (CMV) serology was negative,
serum Toxoplasma IgG antibody test results were
positive, hepatitis serology tests were negative, and
the tuberculin skin test was negative. His CD4 count
was 145 cells/ L but increased to 340 cells/ L with
treatment. He reports that his headache has become
progressively disabling. He has not worked as a waiter
for the past week. His temperature is 37C, his blood
pressure is 120/75 mm Hg, and his heart rate is 88
beats per minute. He has questionable nuchal rigid-
ity, but no other neurologic fndings are noted. Te
ophthalmoscopic examination is normal, without
papilledema. His CD4 cell count is 55 cells/ L, and
HIV-1 RNA is 100,000 copies/mL. A computed
tomographic (CT) scan is normal. Which of the fol-
lowing is most appropriate?
a. Empirical fuconazole therapy
b. Empirical treatment with pyrimethamine and sulfadiazine
c. Lumbar puncture and cerebrospinal fuid (CSF) studies
d. CMV serology
e. Magnetic resonance imaging (MRI) of the head
19. A 36-year-old man has a severe cough and is found to
have lobar pneumonia. He is treated with levofoxacin
and improves over the next 2 weeks. He denies having
risk factors for sexually transmitted and blood-borne
diseases. A human immunodefciency virus (HIV)
test is done, and results of both the enzyme-linked
immunosorbent assay (ELISA) and the Western blot
are positive. Which of the following is the most likely
explanation?
a. Levofoxacin can interfere with HIV testing, and the test
should be repeated in 1 month.
b. Acute infections can cause a false-positive HIV antibody test
result, and he can be reassured that it is likely a false-positive
result. Te test should be repeated in 3 months.
c. Te result is a true-positive, and he has not been forthcom-
ing about his risk factors.
d. Te result is likely a false-positive from laboratory error.
e. He should be asked whether he is taking any over-the-counter
herbal supplements since products containing ginseng can
interfere with the test.
20. A 30-year-old married woman presents with a his-
tory of low-grade fever, malaise, sore throat, anorexia,
abdominal discomfort, and diarrhea of 10 days dura-
tion. She denies having risk factors for human immu-
nodefciency virus (HIV) infection. On physical
examination, her temperature is 38.0 C, her blood
5. I NFECTI OUS DI SEASES QUESTI ONS AND ANSWERS

49
enlarged prostate but normal urodynamics. Urinalysis
results are the following: 10 to 20 leukocytes per
high-power feld, a positive leukocyte esterase test, a
negative nitrite test, and numerous gram-negative rods.
Which of the following is the best choice for treatment
of his symptoms?
a. Nitrofurantoin 100 mg daily for 7 days
b. Amoxicillin 500 mg 3 times daily for 7 days
c. Ciprofoxacin 500 mg twice daily for 28 days
d. Trimethoprim-sulfamethoxazole 1 tablet twice daily for
14 days
e. Levofoxacin 500 mg once daily for 14 days
26. A 51-year-old diabetic woman from Hyderabad,
India, has had unrelenting fever and night sweats
for 2 weeks. Blood cultures have been negative and
an abdominal computed tomographic scan shows a
3 6-cm hypodense lesion in the lef lobe of her liver.
Which of the following should be done next for this
patient?
a. Ultrasound-guided needle aspiration
b. Serology test on blood
c. Stool test for ova and parasites
d. Piperacillin-tazobactam 3.375 g intravenously every 6 hours
e. Purifed protein derivative (PPD) skin test
27. A 19-year-old woman reports that her urine has been
cloudy for the past 3 days. She is asymptomatic but has
missed her period for 3 months. A urine pregnancy test
is positive and a urine culture shows more than 100,000
colonies of a gram-negative rod. Which of the follow-
ing should be recommended for management of her
bacteriuria?
a. Ciprofoxacin 500 mg twice daily for 3 days
b. Trimethoprim-sulfamethoxazole 1 tablet twice daily for
14 days
c. Nitrofurantoin 100 mg daily for 3 days
d. Cefriaxone 1 g intravenously daily for 14 days
e. No treatment
28. A 63-year-old woman with acute myelogenous leuke-
mia (AML) is treated with daunorubicin and cytara-
bine. She has been neutropenic for 3 weeks and febrile
for 2 weeks despite taking meropenem, vancomycin,
and acyclovir. A computed tomographic scan of her
chest shows a large, wedge-shaped peripheral infltrate
with some central clearing in the lef lower lobe. Which
of the following antifungal agents should be most efec-
tive for treatment?
a. Caspofungin
b. Itraconazole
c. Voriconazole
d. Amphotericin B
e. Fluconazole
29. Which of the following is true about Clostridium dif-
fcile infection?
a. Vancomycin is less efective than metronidazole for treat-
ment of severe disease.
b. Most relapses are due to antibiotic resistance.
a. Initiation of postexposure prophylaxis with zidovudine in
combination with lamivudine will signifcantly decrease her
risk of acquiring HIV.
b. Postexposure prophylaxis should be continued if tolerated
for 12 weeks.
c. Use of 3 drugs should be considered if the surgical patient
has known resistance to lamivudine.
d. Te risk of acquiring HIV from a needlestick injury is
approximately 3 in 1,000 events.
e. Postexposure prophylaxis is unlikely to be helpful if not
started within 72 hours.
23. A 45-year-old man with a history of AIDS, including
a past history of pneumocystis pneumonia (PCP) and
cryptococcal meningitis, did well while incarcerated
and treated with efavirenz, tenofovir, and emtricitabine.
His CD4 count gradually improved to 420 cells/ L,
and his viral load was suppressed for over 2 years. When
released, he was told to continue the 3 drugs and to follow
up with a community human immunodefciency virus
clinic, but he never sought care. His sister convinced
him to seek care 3 years afer he was incarcerated, and he
seems interested in restarting therapy. He reports feeling
well except for mild fatigue, diarrhea (3 sof stools daily
accompanied by urgency) and a facial rash suggestive
of seborrheic dermatitis. On examination, he has mild
cervical, axillary, and inguinal adenopathy and thrush.
Which of the following is not currently indicated?
a. Restarting trimethoprim-sulfamethoxazole (TMP-SMX)
prophylaxis against PCP
b. Checking the viral load with genotyping
c. Purifed protein derivative skin test or QuantiFERON-TB
Gold blood test for tuberculosis
d. Azithromycin prophylaxis against Mycobacterium avium-
intracellulare (MAI) complex
e. Ketoconazole cream and shampoo
24. A 28-year-old man who is regularly sexually active with
multiple partners, both male and female, asks what he
can do (other than decreasing his sexual activity) to
lower his risk of acquiring human immunodefciency
virus (HIV). Which of the following will not decrease
his risk of acquiring HIV?
a. Use of latex condoms
b. Addition of nonoxynol spermicide in addition to condoms
c. Taking tenofovir in combination with emtricitabine on a
preventive basis
d. Circumcision
e. Engaging in only insertive sex
I NFECTI OUS DI SEASE SYNDROMES: URI NARY
TRACT I NFECTI ON, SEXUALLY TRANSMITTED
I NFECTI ON, AND GASTROI NTESTI NAL TRACT
I NFECTI ON
25. An 87-year-old man presents with pelvic pain and dys-
uria. He has had 3 urinary tract infections in the past
6 months. A urologic evaluation last month showed an
50

MAYO CLINI C INTERNAL MEDI CINE BOARD REVIEW: QUESTI ONS AND ANSWERS
33. A 60-year-old man with a recently implanted perma-
nent pacemaker (3 months ago) is admitted to the hos-
pital because he has had increasing pain, swelling, and
erythema at the site of his pacemaker pocket. He has
a history of type 2 diabetes mellitus, coronary artery
disease, and third-degree heart block. His symptoms
started 3 days ago and are rapidly progressing. He is
febrile on admission, and blood cultures are grow-
ing gram-positive cocci resembling staphylococci at
24 hours. A transesophageal echocardiogram is nega-
tive for any evidence of endocarditis. What is the most
appropriate management for this patient?
a. Start intravenous vancomycin and delay device explantation
until subsequent blood cultures are negative.
b. Start intravenous vancomycin and immediately proceed to
device explantation.
c. Device removal is not necessary since a pacemaker pocket
infection can be cured with combination therapy with van-
comycin and rifampin.
d. Start intravenous cefazolin and proceed with device removal
as soon as possible.
e. Start intravenous daptomycin and oral rifampin and reassess
in 72 hours.
34. Which of the following has not been shown to reduce
the incidence of central venous catheterrelated blood-
stream infections (CRBSIs)?
a. Chlorhexidine patch applied to the exit site
b. Routine catheter exchange over a guidewire at 72 hours
c. Minocycline-rifampincoated central venous catheters
d. Silver sulfadiazinecoated central venous catheters
e. A standardized catheter insertion bundle
35. A 40-year-old woman with recurrent episodes of
sinusitis presents to the emergency department with
a 1-week history of fever and constant headache. A
computed tomographic scan of the head shows opaci-
fcation of the frontal and sphenoid sinuses and an
abscess in the lef frontal lobe. Which of the follow-
ing would be the best initial antimicrobial regimen for
this patient?
a. Vancomycin
b. Piperacillin-tazobactam
c. Cefazolin and metronidazole
d. Vancomycin, cefriaxone, and a lipid formulation of
amphotericin
e. Vancomycin, cefriaxone, and metronidazole
36. Which of the following patients should receive antibi-
otic prophylaxis for infective endocarditis?
a. A 72-year-old man who has a dual chamber permanent pace-
maker and is undergoing dental extraction
b. A 55-year-old woman who has a history of aortic valve endo-
carditis and is scheduled for placement of an orthodontic
appliance
c. A 65-year-old man who has severe mitral valve regurgitation
and is undergoing endoscopy for evaluation of epigastric
pain
c. Vancomycin-resistant enterococcal (VRE) colonization
occurs more frequently with oral vancomycin treatment
than with metronidazole.
d. A single recurrence of infection increases the likelihood of
further relapses.
e. A stool sample for Clostridium dif cile toxin should be
obtained at the end of treatment to document cure.
30. A 47-year-old man comes to your of ce with a 4-day
history of malaise, a difuse rash involving his trunk
and extremities, swollen glands, and headache. His
symptoms began 3 days afer taking ciprofoxacin for
gonorrhea. At that time, his syphilis IgM and IgG were
positive (titer 1:16). His rapid plasma reagin (RPR)
titer was 1:128, and his human immunodefciency virus
(HIV) quantitative viral load and HIV antibody test
were both negative. Which of the following should you
also recommend?
a. Lopinavir-ritonavir plus zidovudine-lamivudine
b. Doxycycline 100 mg twice daily for 3 days
c. Benzathine penicillin 2.4 million units intramuscularly
d. Lumbar puncture
e. Azithromycin 2 g orally
31. A 43-year-old morbidly obese diabetic woman, recently
treated for abdominal wall cellulitis, now presents with
a 3-day history of fever, right fank pain, and dysuria.
A urinalysis is esterase positive and a Gram stain shows
gram-negative bacilli too numerous to count. Which
of the following treatments should you recommend for
management of her urinary tract infection?
a. Cefriaxone 1 g intravenously daily for 10 days
b. Ampicillin 2 g intravenously every 6 hours for 10 days
c. Ciprofoxacin 500 mg orally twice daily for 7 days
d. Trimethoprim-sulfamethoxazole (TMP-SMX) 160480 mg
orally twice daily for 7 days
e. Nitrofurantoin 100 mg orally twice daily for 10 days
I NFECTI OUS DI SEASE SYNDROMES:
CARDI OVASCULAR, BLOODSTREAM, AND
CENTRAL NERVOUS SYSTEM I NFECTI ONS
32. A 70-year-old woman is undergoing her second cycle of
chemotherapy for breast cancer. On her third hospital
day, she has a fever (38.9 C) and you notice a red track
along her Hickman catheter tunnel. Cultures from
catheter-drawn blood and from peripheral vein blood
are negative at 48 hours. Which of the following should
you recommend for management of her febrile illness?
a. Intravenous vancomycin and cefepime for 14 days
b. Intravenous vancomycin and vancomycin lock therapy for
14 days
c. Removal of the Hickman catheter and intravenous vanco-
mycin for 14 days
d. Transesophageal echocardiography to determine the dura-
tion of antibiotic therapy
e. Topical neomycin, polymixin B, and bacitracin to the exit
site and intravenous vancomycin for 7 days
5. I NFECTI OUS DI SEASES QUESTI ONS AND ANSWERS

51
38. A 57-year-old diabetic man is seen in the emergency
department for fever, productive cough, headache,
and altered mental status for the past 6 hours. On
examination, he is febrile (39C) and lethargic,
and he has prominent neck stiffness. A chest radio-
graph shows a dense right lobar consolidative infil-
trate. His leukocyte count is 22.0 10
9
/L, and his
creatinine level is 1.0 mg/dL. A lumbar puncture
shows an opening pressure of 28 mm water, leuko-
cyte count 30.0 10
9
/L (88% neutrophils), protein
168 mg/dL, and glucose 22 mg/dL. Gram staining
shows gram-positive diplococci. Which of the fol-
lowing is the most appropriate regimen for empirical
treatment?
a. Ampicillin, cefriaxone, and vancomycin
b. Cefriaxone and dexamethasone
c. Cefriaxone and vancomycin
d. Cefriaxone, vancomycin, and acyclovir
e. Cefriaxone, vancomycin, and dexamethasone
d. A 60-year-old heart transplant recipient who has cardiac
valvulopathy and is undergoing a dental extraction
e. A 32-year-old woman who has a history of surgically repaired
congenital heart disease in childhood and is undergoing a
root canal
37. A 55-year-old woman who has a history of mitral valve
replacement with a mechanical prosthesis is admitted
with a 1-week history of fever, malaise, and shortness
of breath. Admission blood cultures are positive for
methicillin-resistant Staphylococcus aureus (MRSA),
and a transesophageal echocardiogram is consistent
with a 5-mm vegetation on the anterior leafet of the
mitral valve. Which of the following is the most appro-
priate antibiotic regimen for this patient?
a. Vancomycin for 6 weeks
b. Vancomycin and gentamicin for 6 weeks
c. Daptomycin and gentamicin for 6 weeks, with rifampin for
the frst 2 weeks of therapy
d. Vancomycin and rifampin for 6 weeks, with gentamicin for
the frst 2 weeks of therapy
e. Vancomycin and rifampin for 4 weeks
52
Legionella infections can progress rapidly and are ofen asso-
ciated with diarrhea and other gastrointestinal tract symp-
toms. Alcohol abuse is a known epidemiologic risk factor,
and Legionella ofen causes outbreaks among persons with
common-source exposure. Relative bradycardia and mental
status changes frequently occur in patients with Legionella
infection. Leukocytosis, hyponatremia, and patchy bilat-
eral infltrates occur frequently in Legionella infections,
but those features may not be as helpful for discriminating
between other microbiologic causes of community-acquired
pneumonia. RSV is unlikely to cause respiratory illness of
this severity in an adult without immunocompromising
conditions. Enterovirus is a cause of encephalitis but is not
a cause of community-acquired pneumonia. When C psit-
taci causes atypical community-acquired pneumonia, it is
generally not rapidly progressive or associated with relative
bradycardia or gastrointestinal tract symptoms. Te patient
has not been to a region where C immitis is endemic.
4. Answer e.
Tis question relates to knowledge of modifable risk fac-
tors to reduce the risk of ventilator-associated pneumo-
nia among patients receiving mechanical ventilation. Te
correct answer and incorrect answers (modifed to be
negative) were taken directly from the guideline recom-
mendations on health careassociated pneumonia. (See
American Toracic Society and Infectious Diseases Society
of America in the Suggested Reading list.)
5. Answer c.
Tis question is based on an understanding of the epidemi-
ology of hospital-acquired pneumonia and how it relates
to the choice of empirical antimicrobials for its treatment.
Te hospital prevalence of methicillin-resistant S aureus
and multidrug-resistant gram-negative organisms, such as
P aeruginosa , A baumannii , and the enterobacteriaceae,
need to be considered when deciding on an antimicrobial
regimen. Te only correct pair of choices is S aureus and
P aeruginosa . Enterococci and C albicans are respiratory
pathogens in only select hosts and are very rare causes of
hospital-acquired pneumonia. Although infuenza virus,
S pneumoniae , and H infuenzae can cause hospital-acquired
pneumonia, they generally are involved in a relatively small
proportion of nosocomial cases and their resistance pro-
fles are generally those of the community-acquired strains.
(See American Toracic Society and Infectious Diseases
Society of America in the Suggested Reading list.)
1. Answer e.
Initial assessment of the severity of community-acquired
pneumonia is important for internal medicine physicians to
reduce unnecessary hospitalization and to identify patients
who are at higher risk of death or who need more immediate
intervention. Illness severity scores such as the pneumonia
severity index (PSI) and CURB-65 ( c onfusion, u rea nitro-
gen, r espiratory rate, b lood pressure, and 65 years or older)
have been developed to help in the decision for site of care
for patients with community-acquired pneumonia. In the
outpatient setting, the modifed CRB-65 ( c onfusion, r espi-
ratory rate, b lood pressure, and 65 years or older) score is
useful since it does not require laboratory or radiographic
evidence to determine the severity score. In this question, the
patient is not confused, does not have a respiratory rate of
30 breaths per minute or more, does not have a systolic blood
pressure less than 90 mm Hg or a diastolic blood pressure of
60 mm Hg or less, but is 65 years or older. His CRB-65 score
is 1. Patients with scores of 0 or 1 can generally be treated as
outpatients if they can reliably take oral antimicrobials and
have outpatient support resources. (See Mandell et al and
Capelastegui et al in the Suggested Reading list.)
2. Answer b.
Tis question is related to appropriate initial antimicrobial
treatment of community-acquired pneumonia in a patient
with medical comorbidities who requires hospitalization but
not intensive care unit admission. Recommended treatment
is either a respiratory fuoroquinolone or a -lactam antibiotic
in combination with a macrolide antibiotic. Azithromycin
alone is not a recommended regimen in patients with medical
comorbidities, including diabetes mellitus. Ciprofoxacin does
not have suf cient coverage for Streptococcus pneumoniae and is
thus not considered a respiratory fuoroquinolone. Although
intravenous ampicillin would be an acceptable -lactam anti-
biotic antimicrobial choice, clindamycin is not a macrolide
antibiotic and the patient does not have risk factors to war-
rant empirical coverage of methicillin-resistant Staphylococcus
aureus . Meropenem is a very broad-spectrum antimicrobial
and is not recommended for routine use for hospitalized
patients with community-acquired pneumonia who do not
have risk factors for infection with Pseudomonas aeruginosa .
(See Mandell et al in the Suggested Reading list.)
3. Answer a.
Tis question involves recognition of Legionella as a cause
of community-acquired pneumonia. In healthy hosts,
ANSWERS
5. I NFECTI OUS DI SEASES QUESTI ONS AND ANSWERS

53
11. Answer c.
Tis diabetic man has peripheral vascular disease and a fetid
foot ulcer with surrounding cellulitis, which can be probed
to the bone. He is manifesting systemic toxicity. Te frst
step in management would be initiation of antimicrobial
therapy to cover MRSA and a mixed infection. Te infec-
tion will be polymicrobial, and a culture would have limited
value. A nasal swab for MRSA does not establish the pres-
ence of MRSA in the wound. An MRI ofers no additional
immediate value at this stage, especially since the wound
can be probed to the bone and is likely osteomyelitic.
12. Answer a.
Te QuantiFERON-TB Gold test can be done in all cir-
cumstances in which a PPD test is indicated. Its principal
role may be in sorting out false-positive tuberculin skin test
results in patients who received BCG vaccine since it does
not cross-react with nontuberculous mycobacteria.
13. Answer e.
Pyrazinamide can cause hyperuricemia and gout. Uric acid
levels may need to be monitored in patients at risk who
are receiving pyrazinamide. Key side efects of rifampin
are rash, drug interactions due to induction of hepatic
microsomal enzymes, hepatotoxicity, and orange secre-
tions. Key side efects of isoniazid are hepatotoxicity,
peripheral neuropathy Lupus-like syndrome, and mono-
amine (histamine-tyramine) poisoning. Generally, pyri-
doxine has no side efects except for remote peripheral
neuropathy. Side efects of ethambutol include retrobulbar
neuritis and decreased red-green color discrimination.
14. Answer c.
Tere are many drug interactions with rifampin that have
potentially serious consequences. Rifampin is an impor-
tant inducer of hepatic cytochrome P450 enzymes, which
decrease the efects of drugs such as glyburide, verapamil, war-
farin, phenytoin, and many others. Tese interactions should
always be considered when the use of rifampin is begun or
stopped since doses may need to be adjusted or medications
may need to be changed. For example, verapamil may need to
be changed to an alternative antihypertensive medication.
15. Answer e.
Tese cases focus on the defnitions of a positive PPD skin
test according to the guidelines of the Centers for Disease
Control and Prevention. A TST result of more than 5 mm
is positive in a person who meets any of the following cri-
teria: is positive for HIV, had recent contact with a person
who had pulmonary tuberculosis (ie, a new conversion),
has a chest radiograph consistent with old untreated tuber-
culosis, or has received an organ transplant or tumor necro-
sis factor inhibitor treatment.
16. Answer a.
Tis patients treatment was self-administered for only
1 week, and he is still symptomatic; thus, he should be
6. Answer a.
Tis question is based on an understanding of the microbiol-
ogy and subsequent antimicrobial prophylaxis ramifcations
of cat bites. Major pathogens isolated from cat bites include
numerous anaerobes, streptococci, and staphylococci but
most commonly Pasteurella multocida . Prophylactic anti-
microbials are recommended unless the bite wound is very
superfcial, and P multocida is generally resistant to diclox-
acillin, cephalexin, clindamycin, and erythromycin but is
generally susceptible to amoxicillin-clavulanic acid. (See
Oehler et al in the Suggested Reading list.)
7. Answer d.
Tis question relates to a syndrome of a potential bioter-
rorism agent. Te patient was previously healthy, and a
life-threatening illness developed quickly, suggesting a highly
aggressive pathogen. Te widened mediastinum is suggestive
of inhalational anthrax but could also be seen with diseases
such as tularemia, histoplasmosis, and tuberculosis. Te quick
positivity of the blood cultures with gram-positive bacilli in
this clinical scenario strongly suggests anthrax, which should
not be excluded as a contaminant. (See Quintiliani and
Quintiliani in the Suggested Reading list.)
8. Answer c.
Te patient has type 1 necrotizing fasciitis, which is most
ofen a mixed infection involving aerobic and anaerobic
organisms. Risk factors for this type of infection are gas-
trointestinal tract procedures, diabetes mellitus, or vascular
disease.
9. Answer e.
Tis patient has staphylococcal toxic shock syndrome, which
may be associated with the retained packing afer the nasal
procedure. Ofen patients receive clindamycin until the pack-
ing is removed. Te features of staphylococcal toxic shock
syndrome are due to toxin-mediated cytokine activation
and can occur 1) afer surgical and postpartum procedures;
2) with mastitis, sinusitis, burns, and skin and sof tissue infec-
tions (especially of the extremities, perianal area, and axillae);
and 3) with respiratory infections afer infuenza.
10. Answer d.
Tere are OPAT guidelines for most infectious diseases.
Ofen OPAT is used for infections (eg, bone and joint
infections) that require prolonged parenteral therapy. Key
tenets of OPAT include the following: 1) the patient or
caregiver (or both) is willing to participate and can safely,
efectively, and reliably deliver OPAT; 2) a physician or
home care agency is available and accessible for communi-
cations about problems and for monitoring; 3) the active
infectious diseases are stable and the patient has no active,
new problems, is not at increased risk of complications,
and has received more than 6 days of therapy for bacterial
meningitis; and 4) the patient can pay for treatment, has a
safe and adequate home or outpatient environment to sup-
port care, and is not actively using illicit drugs.
54

MAYO CLINI C INTERNAL MEDI CINE BOARD REVIEW: QUESTI ONS AND ANSWERS
recognized as the most common presentation, but other
symptoms, including prominent gastrointestinal tract
symptoms, frequently occur. Unfortunately, acute HIV is
ofen overlooked as a possibility, even when patients seek
medical care. Symptomatic acute HIV does spontaneously
resolve, and, if acute HIV is being considered, observation
is not appropriate. HIV antibody testing with the acute
retroviral illness should be negative or indeterminate. HIV
antibody testing will usually seroconvert by 3 months but
may take up to 6 months afer exposure to become positive.
Patients should have genotyping done at diagnosis since
resistance is becoming more common. HIV ofen reverts
back to wild-type virus, although the resistant virus is still
present in the patient. Genotyping is therefore recom-
mended as soon as possible afer diagnosis. Many patients
who engage in behaviors that place them at risk of acquir-
ing HIV deny being at risk because of the stigma attached
to these behaviors and because others do not know that
they are at risk since they do not realize that their partner
is engaging in risky behavior or is HIV positive.
21. Answer b.
NRTIs have been associated with asymptomatic, mild ele-
vations in lactate levels (which can generally be ignored)
and with more severe symptomatic lactic acidosis, which
can lead to profound illness and death. Te most common
symptoms of lactic acidosis in HIV-infected patients are
the subacute development of nausea, vomiting, abdominal
pain, fatigue, weakness, and weight loss. Tachypnea, dys-
pnea with exertion, arrhythmias, and neurologic fndings
have also been reported in the absence of gastrointestinal
tract symptoms. Liver test results are generally abnormal.
Te combination of stavudine and didanosine, although
popular in the past, is no longer recommended because it
carries an increased risk of lactic acidosis and neuropathy.
Patients with certain risk factors (preexisting liver disease,
lower CD4 counts, decreased glomerular fltration rate,
and being female or pregnant) seem to be at increased risk
of symptomatic lactic acidosis. Stavudine is thought to be
the NRTI most strongly associated with lactic acidosis, but
all NRTIs can probably cause lactic acidosis and the more
NRTIs used at a time, the more likely lactic acidosis is to
occur.
22. Answer b.
A case control study has shown a 79% decreased risk
of acquiring HIV from use of zidovudine for 4 weeks
promptly afer a percutaneous exposure. Zidovudine in
combination with lamivudine should be equally or more
efective. Tenofovir in combination with emtricitabine
is an alternative that needs to be taken only once daily.
Prophylaxis is recommended for 4 weeks, and there is no
information that a longer period will result in fewer infec-
tions afer exposure. Te addition of a third drug (gener-
ally a protease inhibitor) should be considered when the
patient has known drug resistance, or the risk of transmis-
sion is felt to be high because of the type of exposure (ie,
deep puncture wound, hollow needle with visible blood, or
considered infectious. Te other patients are no longer
considered infectious.
17. Answer a.
Te patient has large joint monoarticular arthritis and infam-
matory synovial fuid. Te leukocyte count of 65 10
9
/L with
65% polymorphonuclear cells suggests infammation more
than purulent infection. All the choices except acute sickle
cell crisis could give this clinical picture. Te diferential diag-
nosis includes crystalline and infectious arthritis. Sickle cell
crisis usually causes a hemorrhagic fuid and is not associated
with this type of cell count unless coinfected with Salmonella .
Gonococcal arthritis may appear infammatory and in a
patient with another sexually transmitted disease (HIV), this
should be considered. Te leukocyte count is usually greater
than 50 10
9
/L (typically >90% polymorphonuclear cells). In
gonococcal arthritis, intracellular gram-negative diplococci
are found in less than 25% of synovial fuid aspirates. Te syn-
ovial fuid should be cultured on prewarmed chocolate agar
for highest yield (positive fndings in only 50% of patients
with gonococcal arthritis and 25% to 30% of patients with
disseminated gonococcal infection). Genital and oral sites
should be cultured for Neisseria gonorrhoeae .
18. Answer c.
Te patient is clinically stable, so empirical fuconazole or
ertapenem is not indicated. Since he does not have a mass
lesion, empirical treatment for toxoplasmosis is inappro-
priate. His symptoms are consistent with possible central
nervous system cryptococcosis. His CD4 count is less than
100 cells/ L, placing him at risk. A CSF examination with
a cryptococcal antigen test is most appropriate, and a lum-
bar puncture should be safe in the absence of papilledema.
If the CSF is normal, an MRI can be reconsidered to look
for small lesions or other abnormalities that may be missed
on the CT scan.
19. Answer c.
Medications and herbs are not known causes of false-positive
HIV antibody tests. Laboratory error can occur but is unusual,
and positive results should be confrmed. Acute infection
can lead to cross-reacting antibodies and, occasionally, to a
false-positive ELISA result; however, the Western blot is very
specifc, and it should be negative or indeterminate if a patient
is not infected with HIV and has not participated in a vac-
cine trial. Patients are ofen reluctant to speak openly about
their risk factors for HIV because of the stigma attached to
many of these behaviors and a desire to be respected by their
caregivers. Te Centers for Disease Control and Prevention
recommends that all patients undergo screening at least once,
regardless of risk, between ages 15 and 65. Pneumonia (par-
ticularly pneumococcal pneumonia) is more common in
patients with HIV than in uninfected adults even when the
CD4 count is relatively preserved.
20. Answer c.
Symptomatic acute HIV infection occurs in up to 85%
of persons with HIV. A mononucleosis-like syndrome is
5. I NFECTI OUS DI SEASES QUESTI ONS AND ANSWERS

55
negative in the absence of antimicrobial therapy, a bacte-
remic seeding of the liver is less likely. Common bacterial
causes of liver abscess include enteric organisms from the
biliary tree, viridans group streptococci from the digestive
tract, and Staphylococcus aureus secondary to bacteremia. In
India, Entamoeba histolytica infection is common and may
manifest as a febrile illness with a focal liver mass. Te best
test is blood serology for E histolytica . Ultrasound-guided
needle aspiration is useful for many liver lesions but should
be delayed pending the serology results. A stool test for ova
and parasites is not specifc, and ameba may not be seen
at the stage of liver abscess. Empirical antibiotic therapy
should be avoided until the diagnosis is established. A PPD
skin test is likely to be positive and not helpful in establish-
ing the cause of the liver lesion.
27. Answer c.
Tis patient is pregnant and has asymptomatic bacteriuria
with a gram-negative rod. She appears to be in the second
trimester. Asymptomatic bacteriuria in pregnancy is defned
as more than 100,000 colony-forming units per milliliter
and is associated with worse pregnancy outcomes. Tus, it
is one of the few situations in which asymptomatic bacteri-
uria should be treated. Ciprofoxacin and other fuoroqui-
nolones are contraindicated in pregnancy and should be
avoided. Two weeks of trimethoprim-sulfamethoxazole is
excessive for asymptomatic bacteriuria. Cefriaxone is use-
ful for treatment of urinary tract infections in pregnancy
but is not needed for asymptomatic bacteriuria with a sus-
ceptible organism. Nitrofurantoin can be safely used in
pregnancy and a 3- to 7-day course is adequate for asymp-
tomatic bacteriuria. A urine culture should be repeated
2 weeks afer completion of the treatment.
28. Answer c.
Tis woman has AML with prolonged febrile neutropenia
and a wedge-shaped peripheral pulmonary infltrate that
has not resolved with broad-spectrum antibiotics. Te
leading infectious cause is invasive fungal infection, partic-
ularly aspergillosis or, less frequently, mucormycosis. Te
appropriate therapy in the absence of voriconazole prophy-
laxis is the addition of empirical voriconazole to treat pre-
sumed invasive aspergillosis while attempting to establish
the diagnosis. Caspofungin is ofen used as second-line or
add-on therapy. Amphotericin B is more toxic and may be
less efective. Fluconazole has no activity against Aspergillus ,
and itraconazole is less efective than voriconazole.
29. Answer d.
Clostridium dif cile is the most common cause of infec-
tious diarrhea in hospitalized patients. Tose who have
1 episode are predisposed to recurrences and those with 2
or more episodes are at highest risk for multiple relapses.
Oral vancomycin is more efective than metronidazole
for treatment of severe disease. Most relapses result from
reinfection or germination of spores, not from antibiotic
resistance. VRE colonization ofen occurs in similar pop-
ulations of patients who have received broad-spectrum
a very high viral load). Te estimated risk of acquiring HIV
from a needlestick is 3 in 1,000, but, as noted above, some
exposures are more risky than others. In animal studies,
prompt initiation of treatment (within hours) was associ-
ated with decreased transmission; initiation afer 72 hours
is unlikely to be of any beneft.
23. Answer d.
Trush is an indication for PCP prophylaxis. TMP-SMX
prophylaxis needs to be restarted, and the patient should
restart antiretroviral medication as soon as possible. A viral
load with genotype should frst be determined to guide
therapy. It is likely that his seborrheic dermatitis, fatigue, and
diarrhea will improve with immune reconstitution. His past
history of cryptococcal meningitis puts him at high risk of
a recurrence when his CD4 count is less than 200 cells/ L,
and fuconazole therapy should be restarted despite his lack
of symptoms if his CD4 count is less than 200 cells/ L.
Azithromycin is not indicated unless his CD4 count is less
than 50 cells/ L and symptomatic MAI infection has been
excluded. Treatment of his seborrheic dermatitis with keto-
conazole cream and shampoo would be reasonable therapy.
24. Answer b.
If used correctly, latex condoms decrease the risk of acquir-
ing or transmitting HIV by 87% among heterosexuals who
consistently use a condom. Condom failure may be more
common with anal intercourse. Nonoxynol-9 spermicide
does not decrease the risk of acquiring HIV and can increase
the risk in women by causing mucosal irritation. Using a
condom with nonoxynol-9 is safer than not using a condom.
A randomized study of men who have sex with men showed
that continuous treatment with tenofovir in combination
with emtricitabine decreased the relative risk of acquiring
HIV by 44% overall and by 73% among those who reported
over 90% adherence. Circumcision signifcantly decreases
the risk of HIV acquisition by men in heterosexual relation-
ships by 60%. Insertive anal or vaginal sex is less risky for
acquiring HIV than receptive anal or vaginal sex.
25. Answer c.
Tis elderly man has had recurrent episodes of urinary
tract infection. In the absence of a neurogenic bladder
or other secondary cause, this almost always refects an
infected prostate. Te diagnosis of chronic prostatitis is
a clinical one, based on symptoms and signs of recurrent
urinary tract infection as seen in this patient. Antibiotics
that penetrate well into the prostate include fuoroquino-
lones, trimethoprim-sulfamethoxazole, and doxycycline.
Amoxicillin and nitrofurantoin penetrate poorly into the
prostate. Te duration of therapy should be prolonged to
reduce the rates of recurrence. Tus, a 28-day course of cip-
rofoxacin would be the best choice for management of this
patients chronic prostatitis due to Escherichia coli .
26. Answer b.
Tis diabetic woman from India has a large abscess in
the lef lobe of her liver. Since blood cultures have been
56

MAYO CLINI C INTERNAL MEDI CINE BOARD REVIEW: QUESTI ONS AND ANSWERS
2-coated catheters and chlorhexidine patches, have also
been efective. Coated catheters are used when other mea-
sures do not reduce the rate of central venous CRBSIs.
Routine guidewire exchange of catheters does not reduce
central venous CRBSI rates. (See OGrady et al in the
Suggested Reading list.)
35. Answer e.
Tis patient has a brain abscess from a contiguous focus
originating from the sinuses. Te microorganisms to cover
empirically are those ofen associated with sinus infec-
tions, which are usually polymicrobial infections involv-
ing streptococci, Haemophilus infuenzae , staphylococci,
and anaerobes. Because of increasing methicillin-resistant
Staphylococcus aureus (MRSA) in the community, initial
therapy should cover MRSA; thus, cefazolin would not be
the best choice. Empirical antifungal therapy is not needed,
nor is anti- Pseudomonas coverage. (See Carpenter et al in
the Suggested Reading list.)
36. Answer d.
In 2007, the American Heart Association issued revised
guidelines for antimicrobial prophylaxis for endocarditis:
Te only patients who should receive antibiotic prophy-
laxis are those who are scheduled to undergo invasive den-
tal procedures and who have a prosthetic heart valve, a prior
history of endocarditis, surgically uncorrected congenital
heart disease, or cardiac valvulopathy afer undergoing
cardiac transplant. Invasive dental procedures are defned
as those that involve manipulation of gingival tissue or the
periapical region of teeth or perforation of oral mucosa.
Administration of antibiotic prophylaxis solely to prevent
endocarditis is not recommended for patients undergoing
gastrointestinal tract or genitourinary tract procedures.
Te presence of a permanent pacemaker with transvenous
cardiac leads is not an indication for antibiotic prophylaxis
before invasive dental procedures. (See Wilson et al in the
Suggested Reading list.)
37. Answer d.
A patient with MRSA prosthetic valve endocarditis should
receive antibiotic therapy for at least 6 weeks. With bio-
flm formation by staphylococci on prosthetic valves, the
addition of rifampin signifcantly improves cure rates of
S aureus prosthetic valve endocarditis. Gentamicin is rec-
ommended for the frst 2 weeks of therapy, not for the
entire 6-week course. Daptomycin is not superior to vanco-
mycin and is reserved for situations in which vancomycin
cannot be used. (See Baddour et al, 2005, in the Suggested
Reading list.)
38. Answer e.
Tis diabetic man has pneumococcal pneumonia and men-
ingitis. Te most appropriate empirical therapy includes
the initial use of dexamethasone in combination with the
2 antimicrobials to ensure coverage for -lactamresistant
pneumococci. Dexamethasone (0.15 mg/kg every 6 hours
for 24 days) should be started before or with the frst dose
antibiotics; however, there is no diference in VRE colo-
nization whether patients receive vancomycin or metron-
idazole for treatment of C dif cile infection. Because stool
toxin assays may remain positive during and afer success-
ful treatment, follow-up stool toxin assays for test of cure
should be avoided.
30. Answer d.
Tis 47-year-old man was recently treated for gonorrhea
and has secondary syphilis with multiple systemic fea-
tures, including headache. He has a high positive RPR
titer (>1:32), which should indicate the need to consider a
lumbar puncture to exclude neurosyphilis before initiating
therapy.
31. Answer c.
Tis woman has acute uncomplicated pyelonephritis.
She has been recently exposed to antimicrobials, so she
is at increased risk for TMP-SMX and ampicillin resis-
tance. Tus, an oral fuoroquinolone is the drug of choice.
Intravenous therapy is not necessary. Nitrofurantoin,
although efective for cystitis, is not appropriate for
pyelonephritis.
32. Answer c.
Tis patient has tunnel infection of her indwelling catheter.
She is not bacteremic. Essentially, this is a sof tissue infec-
tion with a device in place (similar to a pocket infection for
a pacemaker). Te most appropriate management is removal
of the catheter, along with antimicrobial therapy directed at
the likely cause of the sof tissue infection: coagulase-negative
staphylococci, streptococci, or Staphylococcus aureus . Two
weeks of therapy without removal of the catheter is not sat-
isfactory, nor is lock therapy, which is useful for intralumi-
nal infections but not tunnel infections. Topical therapy is
efective for only localized exit site infections. (See Mermel
et al in the Suggested Reading list.)
33. Answer b.
Complete device removal is necessary in all cases of
pacemaker infections regardless of clinical presentation
(ie, pocket infection or endocarditis). Delaying device
explantation until a blood culture is negative is not recom-
mended. Because staphylococci ( Staphylococcus aureus and
coagulase-negative staphylococci) are the most common
pathogens, vancomycin is the preferred drug for empirical
therapy. Cefazolin is not appropriate for empirical treat-
ment because of the high rate of methicillin resistance in
staphylococci (up to 90% in coagulase-negative staphylo-
cocci). Combination therapy with rifampin is not recom-
mended in pacemaker infections. (See Baddour et al, 2010,
and Sohail et al in the Suggested Reading list.)
34. Answer b.
Prevention of central venous CRBSIs has become a major
national issue. Studies have shown that use of a bundle of
evidence-based practices is efective at reducing central
venous CRBSIs. Several diferent technologies, including
5. I NFECTI OUS DI SEASES QUESTI ONS AND ANSWERS

57
Mandell LA, Wunderink RG, Anzueto A, Bartlett JG, Campbell GD,
Dean NC, et al; Infectious Diseases Society of America; American
Toracic Society. Infectious Diseases Society of America/American
Toracic Society consensus guidelines on the management of
community-acquired pneumonia in adults. Clin Infect Dis. 2007 Mar
1;44 Suppl 2:S2772.
Mermel LA, Allon M, Bouza E, Craven DE, Flynn P, OGrady NP,
et al. Clinical practice guidelines for the diagnosis and manage-
ment of intravascular catheter-related infection: 2009 Update by
the Infectious Diseases Society of America. Clin Infect Dis. 2009 Jul
1;49(1):145. Erratum in: Clin Infect Dis. 2010 Feb 1;50(3):457.
Clin Infect Dis. 2010 Apr 1;50(7):1079. Dosage error in article
text.
Oehler RL, Velez AP, Mizrachi M, Lamarche J, Gompf S. Bite-
related and septic syndromes caused by cats and dogs. Lancet Infect
Dis. 2009 Jul;9(7):43947. Erratum in: Lancet Infect Dis. 2009
Sep;9(9):536.
OGrady NP, Alexander M, Burns LA, Dellinger EP, Garland J, Heard SO,
et al; Healthcare Infection Control Practices Advisory Committee
(HICPAC). Guidelines for the prevention of intravascular
catheter-related infections. Clin Infect Dis. 2011 May;52(9):e16293.
Epub 2011 Apr 1.
Quintiliani R Jr, Quintiliani R. Inhalational anthrax and bioterrorism.
Curr Opin Pulm Med. 2003 May;9(3):2216.
Sohail MR, Uslan DZ, Khan AH, Friedman PA, Hayes DL, Wilson
WR, et al. Management and outcome of permanent pacemaker and
implantable cardioverter-defbrillator infections. J Am Coll Cardiol.
2007 May 8;49(18):18519. Epub 2007 Apr 23.
Tunkel AR, Hartman BJ, Kaplan SL, Kaufman BA, Roos KL, Scheld
WM, et al. Practice guidelines for the management of bacterial men-
ingitis. Clin Infect Dis. 2004 Nov 1;39(9):126784. Epub 2004
Oct 6.
Wilson W, Taubert KA, Gewitz M, Lockhart PB, Baddour LM, Levison
M, et al; American Heart Association Rheumatic Fever, Endocarditis,
and Kawasaki Disease Committee; American Heart Association
Council on Cardiovascular Disease in the Young; American Heart
Association Council on Clinical Cardiology; American Heart
Association Council on Cardiovascular Surgery and Anesthesia;
Quality of Care and Outcomes Research Interdisciplinary Working
Group. Prevention of infective endocarditis: guidelines from the
American Heart Association: a guideline from the American Heart
Association Rheumatic Fever, Endocarditis, and Kawasaki Disease
Committee, Council on Cardiovascular Disease in the Young, and
the Council on Clinical Cardiology, Council on Cardiovascular
Surgery and Anesthesia, and the Quality of Care and Outcomes
Research Interdisciplinary Working Group. Circulation. 2007 Oct
9;116(15):173654. Epub 2007 Apr 19. Erratum in: Circulation.
2007 Oct 9;116(15):e3767.

of antibiotics. Te greatest beneft is achieved in the most
severely ill patients. Tere is no beneft when corticoster-
oids are given afer antibiotic therapy has already begun.
Tere is no need to use acyclovir because the clinical pre-
sentation and the spinal fuid fndings are not consistent
with herpes simplex encephalitis. (See Tunkel et al in the
Suggested Reading list.)
SUGGESTED READING
American Toracic Society; Infectious Diseases Society of America.
Guidelines for the management of adults with hospital-acquired,
ventilator-associated, and healthcare-associated pneumonia. Am J
Respir Crit Care Med. 2005 Feb 15;171(4):388416.
Baddour LM, Epstein AE, Erickson CC, Knight BP, Levison ME, Lockhart
PB, et al; American Heart Association Rheumatic Fever, Endocarditis,
and Kawasaki Disease Committee; Council on Cardiovascular
Disease in Young; Council on Cardiovascular Surgery and Anesthesia;
Council on Cardiovascular Nursing; Council on Clinical Cardiology;
Interdisciplinary Council on Quality of Care; et al. Update on cardio-
vascular implantable electronic device infections and their manage-
ment: a scientifc statement from the American Heart Association.
Circulation. 2010 Jan 26;121(3):45877. Epub 2010 Jan 4.
Baddour LM, Wilson WR, Bayer AS, Fowler VG Jr, Bolger AF, Levison
ME, et al; Committee on Rheumatic Fever, Endocarditis, and Kawasaki
Disease; Council on Cardiovascular Disease in the Young; Councils
on Clinical Cardiology, Stroke, and Cardiovascular Surgery and
Anesthesia; American Heart Association; Infectious Diseases Society of
America. Infective endocarditis: diagnosis, antimicrobial therapy, and
management of complications: a statement for healthcare professionals
from the Committee on Rheumatic Fever, Endocarditis, and Kawasaki
Disease, Council on Cardiovascular Disease in the Young, and the
Councils on Clinical Cardiology, Stroke, and Cardiovascular Surgery
and Anesthesia, American Heart Association: endorsed by the Infectious
Diseases Society of America. Circulation. 2005 Jun 14;111(23):
e394434. Erratum in: Circulation. 2007 Apr 17;115(15):e408.
Circulation. 2008 Sep 16;118(12):e497. Circulation. 2007 Nov
20;116(21):e547. Circulation. 2005 Oct 11;112(15):2373.
Capelastegui A, Espana PP, Quintana JM, Areitio I, Gorordo I,
Egurrola M, et al. Validation of a predictive rule for the manage-
ment of community-acquired pneumonia. Eur Respir J. 2006
Jan;27(1):1517.
Carpenter J, Stapleton S, Holliman R. Retrospective analysis of 49 cases of
brain abscess and review of the literature. Eur J Clin Microbiol Infect
Dis. 2007 Jan;26(1):111.
This page intentionally left blank
59
becomes fatigued if she tries to comb her hair. She is
a nonsmoker and denies having any respiratory com-
plaints. On examination, her temperature is 37.5 C.
Her right radial pulse is decreased compared with the
lef, and the blood pressure in her right arm is decreased
compared with the lef. Laboratory studies show mild
normochromic anemia with a hemoglobin of 11.2 g/dL
(reference range >12.0 g/dL), mildly elevated erythro-
cyte sedimentation rate (ESR) at 36 mm/h (reference
range <29 mm/h), negative antineutrophil cytoplasmic
autoantibody (ANCA) test, and normal blood chemis-
try panel results. Urinalysis and chest radiograph fnd-
ings are normal. Which of the following is the most
likely diagnosis?
a. Polymyalgia rheumatica (PMR)
b. Giant cell arteritis (GCA)
c. Buerger disease
d. Wegener granulomatosis
e. Takayasu arteritis
4. A 59-year-old man has a 1-week history of low back dis-
comfort. He says that he has been moving furniture over
the past several weeks, but he does not recall a specifc
injury. Te pain is worse if he is active; it improves if he
is at rest. He denies having pain radiating to the legs.
Te patient has been taking ibuprofen 400 mg twice
daily with food, and this seems to help. He has no prior
history of lower back pain. Neurologic examination
fndings are normal, with a downgoing Babinski sign,
equal and symmetrical knee jerks, and normal strength
in the lower extremities. He has somewhat difuse ten-
derness over the lumbar spine. Laboratory study results
are normal for the complete blood cell count, erythro-
cyte sedimentation rate, and blood chemistry panel.
What would be the most appropriate recommendation
at this point?
a. Bed rest for 2 weeks
b. Radiograph of the lumbar spine
c. Electromyographic (EMG) study
d. Neurologic consultation
e. HLA-B27 testing
5. A 26-year-old woman presents to your of ce because
she aches all over. She tells you that this condition has
been present for several years but has gotten worse over
the past 6 months. She has problems getting to sleep and
does not feel rested when she wakes up. She denies hav-
ing depression. She is stif for 5 minutes in the morning
and has not noticed any joint swelling. On examination,
QUESTI ONS
Multiple Choice (choose the best answer)
NONARTI CULAR RHEUMATI SM AND
VASCULITI S
1. A 72-year-old woman presents with complaints of
new-onset headache. She is fatigued and has lost about
2.3 kg over the past month. Te patient describes daily
bitemporal headaches. On further questioning, she
describes morning stifness lasting about 1 hour and
aching in the shoulders and hips. She reports transient
loss of vision in her right eye for about 30 minutes yes-
terday before her vision returned. Physical examination
fndings are unremarkable for her age. Initial labora-
tory studies show a mild normochromic anemia and
elevated erythrocyte sedimentation rate (ESR) (101
mm/h). Which would be the most appropriate next
step?
a. Request a temporal artery biopsy.
b. Prescribe prednisone 1 mg/kg orally daily.
c. Prescribe prednisone 5 mg orally 3 times daily.
d. Request computed tomography (CT) of the head.
e. Refer the patient to a neurologist.
2. A 55-year-old man presents with a 3-week history of lef
knee pain. He does not recall an injury. He says that the
front of his leg is tender. On examination, he is afebrile
and overweight. Tere is no lef knee efusion, but the
patient is markedly tender over the anterior medial tibia
just distal to the knee joint. Tis area seems somewhat
pufy but is not particularly erythematous. Te patient
has good range of motion in both hips, but the area over
the lef trochanteric bursa is tender. What would be the
most appropriate step for pain relief ?
a. Inject the lef knee joint with corticosteroid and local
anesthetic.
b. Inject the lef trochanteric bursa with corticosteroid and
local anesthetic.
c. Inject the lef pes anserine bursa with corticosteroid and
local anesthetic.
d. Prescribe propoxyphene 65 mg orally every 6 hours.
e. Prescribe prednisone 60 mg orally daily for 7 days.
3. A 26-year-old woman presents for evaluation of
low-grade fevers that have been present about
6 months. She also has lost 3.6 kg and has noted some
arthralgias and myalgias. She says that her right arm
6.
RHEUMATOLOGY QUESTIONS AND ANSWERS
60

MAYO CLINI C INTERNAL MEDI CINE BOARD REVIEW: QUESTI ONS AND ANSWERS
catching, or giving way of the knees. She takes glu-
cosamine chondroitin sulfate (1,200 mg daily) and
acetaminophen (up to 1,000 mg 3 times daily as needed
for pain). She has hypertension and mild renal insuf-
fciency (serum creatinine 1.6 mg/dL, reference range
1.2 mg/dL). She also has a history of coronary artery
disease. On examination, she has a small amount of
efusion in her right knee, mild genu varus deformity,
and mild tenderness along the medial joint line of the
right knee. Te knees appear stable on examination.
Radiographs show medial joint space narrowing of the
right knee. Which would be the best next step in her
care?
a. Obtain a magnetic resonance imaging (MRI) scan of the
right knee.
b. Administer an intra-articular corticosteroid injection.
c. Administer a series of injections with hylan G-F 20.
d. Prescribe naproxen 500 mg twice daily.
e. Obtain an orthopedic consultation for possible arthroscopic
surgery.
9. A 65-year-old man with rheumatoid arthritis (RA) has
noticed an increase in small rheumatoid nodules on
his hands over the past 3 months. Otherwise, he feels
well. His RA has been well controlled with methotrex-
ate 17.5 mg weekly, etanercept 50 mg subcutaneously
weekly, folic acid 1 mg daily, and nabumetone 750 mg
twice daily. On examination, he is afebrile. Tere are
numerous small nodules on the extensor surface of the
fngers, but the patients RA appears quiescent, and he
does not have active synovitis in the hands or wrists.
Which of the following would be the most appropriate
recommendation?
a. Stop taking nabumetone.
b. Add trimethoprim-sulfamethoxazole 1 double-strength tab-
let twice daily.
c. Add prednisone 60 mg daily.
d. Decrease the methotrexate dosage to 7.5 mg weekly.
e. Switch etanercept to infiximab.
10. A 43-year-old woman with rheumatoid arthritis pres-
ents to your of ce for optimization of her therapy. She
is taking methotrexate 15 mg weekly, prednisone 5 mg
daily, and nabumetone 750 mg daily. She saw a televi-
sion advertisement for infiximab and wants to try it.
She has a past medical history signifcant for multiple
sclerosis (MS) at age 25, which has not been active in
recent years. On examination, she does have synovi-
tis in several joints. Which of the following would be
most appropriate to tell her about her request to add
infiximab?
a. She will need a purifed protein derivative (PPD) skin test
before starting infiximab therapy.
b. Etanercept would be a better choice for her.
c. Infiximab can be given either orally or subcuteneously.
d. Infiximab cannot be used with prednisone.
e. Infiximab is relatively contraindicated in patients with a
history of MS.
her body mass index (BMI) is 20. Her muscle strength
is normal, and there is no synovitis. Tere is no rash.
She has multiple tender points. Results of the follow-
ing laboratory studies are normal: complete blood cell
count, erythrocyte sedimentation rate, blood chemis-
try panel, and sensitive thyrotropin. What would be the
most appropriate next step?
a. Obtain an electromyogram (EMG).
b. Prescribe prednisone 15 mg orally daily.
c. Prescribe duloxetine 30 mg orally daily.
d. Obtain overnight oximetry results.
e. Prescribe oxycodone 5 mg orally every 6 hours, as needed for
pain control.
6. A 50-year-old man has sinus drainage, cough, and
hemoptysis. He also says that he has had joint pain for
several weeks and swelling in the feet. On examination,
he has synovitis of several proximal interphalangeal
joints of the hands and bilateral lower extremity edema.
Results of laboratory studies show a normochromic
anemia (hemoglobin 9.6 mg/dL), an elevated erythro-
cyte sedimentation rate (67 mm/h), and an elevated
creatinine level (1.6 mg/dL). Te urinalysis shows
proteinuria (2+) and red blood cell casts. Chest radi-
ography shows multiple nodular lesions in both lungs.
What would be the most appropriate next test to help
establish a diagnosis?
a. Renal biopsy
b. Open lung biopsy
c. Antineutrophil cytoplasmic autoantibody (ANCA) panel
for vasculitis
d. Rheumatoid factor test
e. Cyclic citrullinated peptide (CCP) antibody test
7. A 41-year-old woman presents with a several-week his-
tory of severe headaches and episodes that resemble a
transient ischemic attack (TIA). She has no history of
smoking or of having asthma. On examination, she is
afebrile but dysarthric. Her laboratory test results are
normal for complete blood cell count with diferential
count, erythrocyte sedimentation rate, creatinine, and
urinalysis. Te antineutrophil cytoplasmic autoanti-
body (ANCA) test is negative. Chest radiography is
unrevealing, but magnetic resonance imaging of the
head shows multiple areas suggestive of infarctions in
several arterial distributions. What is the most likely
diagnosis?
a. Wegener granulomatosis (WG)
b. Giant cell arteritis (GCA)
c. Buerger disease
d. Isolated central nervous system (CNS) vasculitis
e. Churg-Strauss vasculitis
OSTEOARTHRITI S, RHEUMATOI D ARTHRITI S,
AND ANTI RHEUMATI C DRUGS
8. A 56-year-old woman has bothersome, but not dis-
abling, osteoarthritis of her right knee. She has pain
if she walks more than 2 blocks but denies locking,
6. RHEUMATOLOGY QUESTI ONS AND ANSWERS

61
and 30 tender joints. Laboratory test results include
the following: mild normochromic anemia (hemoglo-
bin 12.1 g/dL), elevated erythrocyte sedimentation
rate (39 mm/h), negative rheumatoid factor test (14
international units/mL, reference range <15 interna-
tional units/mL), weakly positive antinuclear antibody
(ANA) test (1.2 units, reference range <1.0 units), and
an increased level of antibodies to cyclic citrullinated
peptide (anti-CCP) (>100 units, reference range <5
units). What can you tell him from these results?
a. You are referring him to a hematologist because of anemia.
b. Patients with RA do not have positive ANA tests.
c. Te anti-CCP results suggest that he is at higher risk of
radiographic progression than RA patients who have nega-
tive anti-CCP results.
d. Since his rheumatoid factor test is negative, it is unlikely that
he has RA.
e. Anti-CCP antibodies occur only in RA.
SPONDYLOARTHROPATHI ES
15. What is the goal with allopurinol therapy in the treat-
ment of hyperuricemia in gout patients?
a. Decrease the uric acid level to 8.0 mg/dL or less (reference
range 8.0 mg/dL).
b. Prevent further gout attacks without regard to uric acid
level.
c. Decrease the uric acid level to less than 6.0 mg/dL (reference
range 8.0 mg/dL).
d. Decrease urinary uric acid excretion to normal.
e. Prevent uric acidinduced renal disease.
16. In a patient with ankylosing spondylitis, which of the
following would be most helpful for treating back pain
and preventing episodes of uveitis?
a. Nonsteroidal anti-infammatory drugs
b. Methotrexate
c. Sulfasalazine
d. Etanercept
e. Adalimumab
17. A 25-year-old man presents with a 2-week history of
pain and swelling of the right Achilles tendon and lef
ankle. He has a previous history of uveitis on 2 occa-
sions. He describes stifness in his lower back, which is
worse in the morning. Which disease best accounts for
his symptoms?
a. Ankylosing spondylitis
b. Rheumatoid arthritis
c. Lupus
d. Gout
e. Pseudogout
18. A 62-year-old woman experiences her frst attack of
gout in her great toe. She takes nifedipine for hyperten-
sion, simvastatin for hyperlipidemia, and naproxen for
osteoarthritis of her hips. Laboratory study results (and
reference ranges) include the following: uric acid 9.1
mg/dL (<8 mg/dL), creatinine 1.9 mg/dL (<1.1 mg/dL),
11. A 57-year-old woman comes to your of ce for mild knee
discomfort. She says that if she walks more than 2 miles,
both knees hurt. With daily activity, she is not symp-
tomatic. She currently is not taking any medications for
her occasional knee pain. She is most interested in what
she can do to prevent progression of her condition. On
examination, you note mild varus deformity of both
knees and tenderness over the medial joint line. What
would be the best advice for her?
a. Take naproxen 500 mg twice daily.
b. Take acetaminophen 1,000 mg 3 times daily.
c. Take glucosamine chondroitin sulfate 1,200 mg daily.
d. Undergo magnetic resonance imaging (MRI) scanning of
both knees.
e. Undergo ultrasound-guided aspiration of 1 knee.
12. A 79-year-old woman has had seropositive rheumatoid
arthritis for 3 years. She takes methotrexate 15 mg on
1 day weekly, hydroxychloroquine 400 mg daily, folic
acid 1 mg daily, and diclofenac-misoprostol 75 mg up
to twice daily as needed for joint pain. She says that
these medications have helped her. She weighs 70 kg
and is normotensive. On examination, you fnd only
2 tender joints and no swollen joints. Radiographs of
the hands and feet do not show any erosions. She was
told by an ophthalmologist that she has early cataracts
in both eyes. Of the following, what would be the best
recommendation for her?
a. Stop taking hydroxychloroquine.
b. Increase the methotrexate dosage to 20 mg weekly, and stop
taking hydroxychloroquine.
c. Increase the hydroxychloroquine dosage to 600 mg daily.
d. Have an ophthalmologic examination at least annually while
taking hydroxychloroquine.
e. Stop taking the medications since therapy for rheumatoid
arthritis is no longer needed.
13. A 26-year-old woman presents to your of ce for treat-
ment of newly diagnosed rheumatoid arthritis (RA).
She has had symptoms for about 3 months. She is not
functionally limited. So far, she has been taking ibupro-
fen 600 mg 3 times daily and has received some relief.
On examination, she has 2 swollen joints and 4 tender
joints. She does not have any rheumatoid nodules or
other extra-articular features of RA. Te rheumatoid
factor test is mildly positive, but the cyclic citrullinated
peptide antibody test is negative. Her radiographs do
not show any erosions. She would like to start therapy
with a tumor necrosis factor (TNF) inhibitor. What
would be the best advice for this patient?
a. Start infiximab 3 mg/kg intravenously.
b. Start etanercept 50 mg/wk subcutaneously.
c. Start methotrexate 10 mg orally on 1 day weekly.
d. Additional therapy is not required now.
e. Start glucosamine chondroitin sulfate 1,200 mg daily.
14. A 29-year-old man presents to you with a 10-week his-
tory of joint pain. His mother has severe rheumatoid
arthritis (RA). On examination, he has 20 swollen joints
62

MAYO CLINI C INTERNAL MEDI CINE BOARD REVIEW: QUESTI ONS AND ANSWERS
MI SCELLANEOUS RHEUMATOLOGI C
CONDITI ONS
22. A 42-year-old man from Rhode Island has bilateral
intermittent painful knee efusions. Synovial fuid anal-
ysis is negative with Gram staining and culture, with
a leukocyte count of 6.5 10
9
/L. No crystals are seen.
Tere is no history of skin rash or low back pain. Tere
is no other joint involvement. Afer a fulike illness
2 years previously, the patient did have Bell palsy,
which resolved. His father has gout. Which of the
following diseases is most likely to account for his
symptoms?
a. Rheumatoid arthritis
b. Lyme disease
c. Systemic lupus erythematosus (SLE)
d. Gout
e. Spondyloarthropathy
23. A 63-year-old woman with a 20-year history of
CREST syndrome presents with a 6-month history
of dyspnea with walking. A technetium Tc 99m ses-
tamibi study with exercise does not show any evi-
dence of ischemia. A chest radiograph is normal.
Pulmonary function test results are normal except
for an isolated decrease in the diffusing capacity of
lung for carbon monoxide (D lco ) to 35% of the pre-
dicted value. Which of the following would be the
most likely diagnosis?
a. Deconditioning
b. Atypical angina
c. Recurrent pulmonary embolism
d. Interstitial lung disease
e. Pulmonary hypertension
24. A 32-year-old male intravenous drug user has arthral-
gias and biopsy-proven cutaneous leukocytoclastic
vasculitis. Laboratory study results are shown in Table
6.Q24.
and hemogloblin 11.3 g/dL (>12 g/dL). Besides treating
her with tapering doses of methylprednisolone, which
of the following should be done next?
a. Stop naproxen.
b. Stop simvastatin.
c. Begin colchicine.
d. Begin allopurinol.
e. Begin probenecid.
19. Acute monoarthritis of the lef knee developed in a
62-year-old man receiving long-term hemodialysis.
Joint aspiration analysis is negative with Gram staining,
and polarization microscopy shows birefringent bipy-
ramidal crystals that stain with alizarin red, indicative
of calcium. What is the most likely diagnosis?
a. Gout
b. Infection
c. Basic calcium phosphate disease (hydroxyapatite deposition
disease)
d. Pseudogout
e. Calcium oxalate arthropathy
20. A 43-year-old man has had a 3-year history of pro-
gressive psoriatic arthritis unresponsive to nonsteroi-
dal anti-infammatory drugs. On examination, he has
infammatory arthritis, and radiographs show erosive
changes in his hands and feet. He currently is being
treated for hepatitis C infection, which he acquired
from a blood transfusion 10 years ago. Which of the
following would be the best treatment option at this
time?
a. Methotrexate
b. Prednisone 40 mg daily
c. Prednisone 10 mg daily
d. Etanercept
e. Rituximab
21. An 82-year-old woman had sudden onset of pain and
swelling in her lef knee. Tere was no trauma. She had
no previous episodes or history of fever. She was taking
hydrochlorothiazide for hypertension. Joint aspiration
analysis is negative with Gram staining, the synovial
fuid leukocyte count is 12.5 10
9
/L, and polarization
microscopy shows weakly positive rhomboid bire-
fringent crystals. Laboratory study results include the
following: hemoglobin 11.7 g/dL, leukocyte count
7.2 10
9
/L, erythrocyte sedimentation rate 45 mm/h,
creatinine 1.6 mg/dL, and uric acid 8.6 mg/dL (refer-
ence range <7 mg/dL). Which of the following is the
cause of her acute monoarthritis?
a. Gout
b. Pseudogout
c. Basic calcium phosphate disease
d. Calcium oxalate arthropathy
e. Septic joint
Table 6.Q24
COMPONENT RESULT
Hemoglobin, g/dL 10.2
Leukocyte count, 10
9
/L 8.2
Erythrocyte sedimentation rate, mm/h 59
Rheumatoid factor 1:640
C4 Low
Aspartate aminotransferase, U/L (AST) 3 times upper limit of
reference range
Cryoglobulins Positive
6. RHEUMATOLOGY QUESTI ONS AND ANSWERS

63
a. Deep vein thrombosis
b. Miscarriage secondary to placental infarction
c. Lupus renal disease
d. Pregnancy complications with fetal heart block
e. Hemolytic anemia
28. A 65-year-old man noted slowly progressive muscle
weakness in his arms and legs over the past 2 years. He
considers himself healthy otherwise. He has had no
pain. He takes simvastatin and hydrochlorothiazide. He
is weak both proximally and distally. An electromyo-
gram (EMG) shows both myopathic and neuropathic
fndings. Laboratory test results include the following:
hemoglobin 14.4 g/dL, erythrocyte sedimentation rate
3 mm/h, creatinine 1.0 mg/dL, antinuclear antibodies
1:40 (negative <1:40), and creatine kinase (CK) 639
U/L (reference range <250 U/L). What is the most
likely diagnosis?
a. Polymyositis
b. Amyotrophic lateral sclerosis (ALS)
c. Simvastatin toxicity
d. McArdle disease
e. Inclusion body myositis (IBM)
Which of the following tests would be most likely to
establish the diagnosis?
a. Anticyclic citrullinated peptide
b. Antinuclear antibody
c. Anti-dsDNA antibody
d. Human immunodefciency virus (HIV)
e. Hepatitis C serology
25. A 48-year-old woman with polymyositis began taking
prednisone 60 mg daily 6 months ago. Her initial cre-
atine kinase (CK) level was 7,028 U/L (reference range
<176 U/L). Now her CK is in the reference range and her
prednisone dose has been decreased to 20 mg. She still
has signifcant proximal muscle weakness. What would
be the best course of action at this time to improve her
muscle strength?
a. Increase the dose of prednisone.
b. Continue to decrease the prednisone dose and have her
begin physical therapy.
c. Begin methotrexate.
d. Begin azathioprine.
e. Begin intravenous immune globulin.
26. A 53-year-old woman has a 15-year history of systemic
lupus erythematosus. Multiple fares of her disease dur-
ing the past several years have necessitated high-dose
prednisone therapy. She now presents with a 3-week
history of pain in the lef groin with ambulation. She
denies having fevers, chills, or other symptoms that she
had equated with a fare in the past. Findings on routine
radiography of her pelvis, including hips, are normal.
Which of the following would be most helpful in deter-
mining the cause of her symptoms?
a. Electromyography
b. Erythrocyte sedimentation rate and anti-nDNA antibody
determination
c. Magnetic resonance imaging (MRI) of the hips
d. Empirical trial of a corticosteroid injection into the hip
under fuoroscopy
e. MRI of the lumbar spine
27. A 31-year-old woman with systemic lupus erythemato-
sus is considering having a child. She is asymptomatic.
She has no history of a previous pregnancy or of renal
disease. She takes hydroxychloroquine 200 mg daily. On
the basis of her laboratory study results (Table 6.Q27),
which of the following would she be at increased risk
for during a pregnancy?
Table 6.Q27
COMPONENT RESULT
Hemoglobin, g/dL 13.1
Leukocytes, 10
9
/L 3.2
Platelets, 10
9
/L 162
Erythrocyte sedimentation rate, mm/h 3
Creatinine, mg/dL 0.6
Anti-dsDNA antibody Negative
C3 Normal
Anti-Sm antibody Negative
Anti-RNP antibody Negative
AntiSS-A antibody Positive
AntiSS-B antibody Negative
Antiphospholipid antibody Negative
Antibodies IgG, IgM
64
5. Answer c.
Tis patient presents with fbromyalgia-like symptoms. Te
patient does not have muscle weakness, so an EMG would
not be indicated. Her symptoms, physical examination
fndings, and laboratory test results are not suggestive of
infammation, so prednisone is not indicated. Duloxetine
has been shown to be ef cacious treatment for fbromyalgia
in women. Although the patients sleep is nonrestorative,
she is young and has a low BMI, so sleep apnea is not likely.
She has a chronic pain syndrome, and it is usually best to
avoid using narcotics to treat fbromyalgia pain.
6. Answer c.
Tis patient has features of Wegener granulomatosis
(WG) with upper and lower respiratory tract involvement
and renal involvement. An ANCA panel would likely be
positive in a c-ANCA pattern with a positive proteinase 3
enzyme-linked immunosorbent assay (PR3 ELISA). A renal
biopsy would be expected to show glomerulonephritis,
which would not be specifc for WG. An open lung biopsy
likely would show histologic features of WG (granuloma-
tous vasculitis), but the procedure is invasive and may not
be necessary if the ANCA tests are positive. Patients with
WG can have synovitis. Te rheumatoid factor and CCP
tests are for rheumatoid arthritis. Patients with rheumatoid
arthritis would not typically have hemoptysis, although
they could have pulmonary involvement, and they would
not usually have glomerulonephritis.
7. Answer d.
Patients with WG usually have upper and lower respiratory
involvement and ofen renal disease, which this patient
does not have. Also, most patients with WG have a posi-
tive ANCA test. GCA would not occur in this age group
and usually does not manifest with TIAs. Buerger disease
afects the peripheral circulation and occurs in smokers.
Churg-Strauss vasculitis occurs in patients with a history
of asthma and is usually associated with eosinophilia.
Isolated CNS vasculitis can manifest with TIAs and head-
aches. Ofen patients do not have other features of systemic
vasculitis, so the term isolated CNS vasculitis is used. Most
patients with isolated CNS vasculitis are ANCA negative.
8. Answer b.
Tis patient has osteoarthritis. Her examination, history,
and radiographs do not suggest another process, so MRI
is likely to yield little more information. Because she has
1. Answer b.
In a patient of this age with a new onset of headache, anemia,
and highly elevated ESR, giant cell arteritis (GCA) would be
the number one consideration. Temporal artery biopsy would
be appropriate, but with the high degree of suspicion for GCA
(because of the loss of vision), prednisone therapy should
begin without waiting for the biopsy results. Te appropriate
dose of prednisone for GCA is 1 mg/kg daily. Lower doses
are appropriate for polymyalgia rheumatica. A CT scan of the
head would not be helpful for a patient with suspected GCA.
Likewise, referral to a neurologist would not be helpful.
2. Answer c.
Tis patient has pes anserine bursitis. Injection of the bursa
is ofen helpful. Tere is not an indication to inject the lef
knee joint. Although the area over the lef trochanteric bursa
is tender, the patient is not symptomatic in this area, so there
is no indication for an injection there. Propoxyphene has
been taken of the market because of safety concerns, includ-
ing accidental overdose, suicide, and arrhythmias. High-dose
prednisone is typically not used for bursitis.
3. Answer e.
Of the choices listed, this patient would most likely have
Takayasu arteritis. Tis would be most common in women
younger than 40. Weight loss, arthralgias, myalgias, and
low-grade fevers are all common features. In addition,
patients ofen present with upper extremity claudication.
Te decreased radial pulse in the right arm compared with
the lef and the decreased blood pressure in the right arm
suggest upper aortic arch involvement. Typically, mild
anemia is present. Some patients have active disease but a
normal ESR. Many patients with Takayasu arteritis have a
negative ANCA test. Both PMR and GCA occur in an older
age group. Buerger disease occurs in smokers. Patients with
Wegener granulomatosis usually have respiratory symptoms
and a positive ANCA test; they ofen have upper and lower
respiratory involvement and kidney involvement.
4. Answer b.
New onset of lower back pain afer age 50 would be an indi-
cation for radiograph. Bed rest for more than 3 days is not
helpful. Tere is no suggestion of radiculopathy, so EMG
and neurologic consultation are not indicated. Te patient
is not describing infammatory back pain and is older than
50, so HLA-B27 testing for spondyloarthropathy would
not be helpful.
ANSWERS
6. RHEUMATOLOGY QUESTI ONS AND ANSWERS

65
therapy. It would be appropriate to start a disease-modifying
antirheumatic drug, so methotrexate would be a reasonable
choice. Glucosamine chondroitin sulfate has been shown
to be potentially helpful in osteoarthritis of the knee, but
there is no support from evidence-based medicine that it
helps RA.
14. Answer c.
Te patient likely has an anemia of chronic disease related
to infammatory arthritis. High levels of anti-CCP anti-
bodies are specifc for RA; occasionally, lower values are
present with other conditions, including systemic lupus
erythematosus. A positive ANA test, especially a weakly
positive result such as this patients, is common in RA. Te
anti-CCP test is more specifc than the rheumatoid factor
test for RA but, depending on the assay used, may be less
sensitive than the rheumatoid factor test. Anti-CCP anti-
bodies sometimes occur in patients who are rheumatoid
factornegative; occasionally, anti-CCP antibodies occur
before rheumatoid factor occurs in patients with RA. Te
presence of anti-CCP antibodies has been associated with
more radiographic progression in patients with RA.
15. Answer c.
Te goal of allopurinol therapy in the treatment of gout is
to decrease the uric acid level to less than 6.0 mg/dL. Te
solubility quotient of uric acid is 6.8 mg/dL. Terefore,
decreasing the uric acid level to less than 6.0 mg/dL pro-
motes shrinkage of tophi and results in better clinical
ef cacy.
16. Answer e.
Although both etanercept and adalimumab would be help-
ful in treating the back pain, only adalimumab has been
shown to decrease the frequency of uveitis.
17. Answer a.
Achilles tendinitis (enthesopathy), low back pain with
morning stifness, and uveitis are characteristic of spondy-
loarthropathies, such as ankylosing spondylitis. Te other
diseases do not predispose characteristically to back pain,
uveitis, or enthesopathy.
18. Answer a.
Naproxen should be stopped because of the elevated crea-
tinine. Stopping naproxen may also result in decreased
creatinine and uric acid levels. Only if the patient contin-
ued to have recurrent attacks of gout would allopurinol be
used to lower the uric acid. With an elevated creatinine,
probenecid would not be efective.
19. Answer e.
Calcium oxalate arthropathy usually occurs in patients
receiving long-term hemodialysis, and the bipyramidal
crystals stain positive for calcium. Dialysis patients are also
at increased risk for basic calcium phosphate disease, gout,
and a septic joint. Negative Gram staining rules against a
renal insuf ciency, hypertension, and a history of coronary
artery disease, nonsteroidal anti-infammatory drugs are
relatively contraindicated. A corticosteroid injection would
be a consideration, especially since she has pain in only
1 joint. If the patient had no response to the corticosteroid
injection, viscosupplementation would be a consideration.
Te patient does not have mechanical symptoms, so there
is no clear indication for an arthroscopic procedure.
9. Answer d.
Methotrexate can cause increased nodulosis in some RA
patients. Depending on how problematic it is, one may need
to consider decreasing the dose or even stopping the medi-
cation. Trimethoprim-sulfamethoxazole can interact with
methotrexate and potentially cause toxicity, so it should be
avoided, especially if there is not a specifc reason to add it.
Te RA appears relatively quiescent, so there would not be
a need to add high-dose prednisone. Likewise, there would
not be a reason to switch to a diferent tumor necrosis fac-
tor inhibitor.
10. Answer e.
All the tumor necrosis factor (TNF) inhibitors have been
reported to potentially cause demyelinating lesions in
patients. Trials of infiximab in patients with MS lead to
worsening of the disease. Terefore, TNF inhibitors are
relatively contraindicated for patients with a history of MS.
Infiximab can be used in patients who are taking nonsteroi-
dal anti-infammatory drugs, methotrexate, and prednisone.
A PPD skin test should be checked before starting any of the
TNF inhibitors because tuberculosis has reactivated in some
patients. Infiximab is given by intravenous infusion only.
11. Answer c.
Nonsteroidal anti-infammatory drugs and acetaminophen
are analgesics, but they do not prevent progression of osteoar-
thritis. Tere is some support from evidence-based medicine
for the use of glucosamine chondroitin sulfate to retard pro-
gression of osteoarthritis of the knee. Tere is no indication
for MRI or ultrasound-guided arthrocentesis of the knee.
12. Answer d.
Hydroxychloroquine is typically used at a dosage of up
to 6.5 mg/kg daily. Higher doses may be associated with
increased ocular toxicity and so are generally avoided. Te
toxicity is retinal toxicity, and hydroxychloroquine use is
not associated with cataract formation. Patients taking
hydroxychloroquine should have an ophthalmologic exam-
ination at least annually. Patients with rheumatoid arthritis
usually require long-term therapy with a disease-modifying
antirheumatic drug. Tis patients disease is well controlled
with the current therapy, so there is no indication to change
her regimen.
13. Answer c.
According to present treatment guidelines, initiation of a
TNF would not be indicated for this patient as the frst
66

MAYO CLINI C INTERNAL MEDI CINE BOARD REVIEW: QUESTI ONS AND ANSWERS
in the D lco suggests pulmonary hypertension, and the
patient should undergo right heart catheterization.
24. Answer e.
Hepatitis C infection can cause cryoglobulinemia, which
is responsible for the positive rheumatoid factor and cryo-
globulinemic vasculitis. Te low C4 is also associated
with the cryoglobulinemic vasculitis. Te elevated AST is
indicative of ongoing hepatitis liver disease. Rheumatoid
arthritis, systemic lupus erythematosus, and HIV do not
characteristically cause cryoglobulinemia.
25. Answer b.
Tis patient has steroid myopathy. With the CK value being
normal, the polymyositis is under good control. Tere is no
reason to add a steroid-sparing agent at this time. A slow,
continued corticosteroid taper in conjunction with physi-
cal therapy is indicated.
26. Answer c.
Te most helpful test would be MRI of the hips. Tis
patient has 2 risk factors for avascular necrosis of the hip:
1) systemic lupus erythematosus and 2) prednisone use.
27. Answer d.
Te positive results for antiSS-A antibody give this patient
a 3% risk of a complete heart block developing in the fetus
in the second or third trimester. Tere are no antiphospho-
lipid antibodies, so there is no increased risk of deep vein
thrombosis or miscarriage. Te patient has had no previ-
ous renal disease, and with no anti-dsDNA there is not an
increased risk of active renal disease during pregnancy.
28. Answer e.
IBM occurs in older patients and causes both proximal and
distal muscle weakness. In contrast, polymyositis causes
predominantly proximal weakness. In IBM, the elevation of
CK values is usually relatively small, and characteristically
the EMG has both myopathic and neuropathic fndings.
Polymyositis, simvastatin toxicity, and McArdle disease
would not show neuropathic fndings on EMG. ALS can
be associated with an elevated CK, but the muscle weak-
ness is usually asymmetrical and EMG would show only
neuropathic fndings.

septic joint, and no urate crystals indicative of gout were
seen. In basic calcium phosphate disease, crystals are seen
only with electron microscopy, not with polarization
microscopy.
20. Answer d.
Etanercept is approved for both psoriasis and psoriatic
arthritis and has no deleterious efect on viral titers or
treatment of hepatitis C. Methotrexate is contraindicated
for a patient with hepatitis C. A high dose of prednisone
can increase viral titers, and a low dose of prednisone
should be used only to give temporary beneft until a
disease-modifying antirheumatic agent provides beneft.
Rituximab is not approved for psoriatic arthritis.
21. Answer b.
Weakly positive rhomboid birefringent crystals are char-
acteristic of pseudogout. Tis patient had an elevated
uric acid level, but monosodium urate crystals, which are
needle shaped and strongly negative birefringent, were
not seen. Gram staining was negative, which rules against
a septic joint. Calcium oxalate crystals are bipyramidal. In
basic calcium phosphate disease (calcium hydroxyapatite),
no crystals are seen with polarization microscopy.
22. Answer b.
Not all patients with Lyme disease recall a tick bite or
have erythema chronicum migrans. Tis patient had
fulike symptoms and Bell palsy several years before
the onset of his Lyme arthritis, which characteristically
involves the knees several years afer the initial infec-
tion. Bell palsy is a characteristic neurologic manifesta-
tion of Lyme disease. Rheumatoid arthritis would be less
likely without polyarticular involvement and would not
be associated with Bell palsy. No crystals were seen with
joint aspiration, which would rule against gout. Tere is
no history of low back pain or uveitis, which would make
a spondyloarthropathy unlikely. Tere is no clinical his-
tory to suggest SLE.
23. Answer e.
Pulmonary hypertension is more likely to develop in
patients with CREST syndrome than in patients with dif-
fuse systemic sclerosis (scleroderma). Te isolated decrease
67
3. A 72-year-old man has new-onset atrial fbrillation and
abnormal results on thyroid tests (thyrotropin <0.01
mIU/L, free thyroxine 2.3 ng/dL). He begins antiar-
rhythmic therapy and is referred to you. He reports palpi-
tations and weight loss for 3 months along with redness,
swelling, and pain over his eyes. His past medical history
is signifcant for hypertension, congestive heart failure,
and ongoing nicotine dependence. On examination, his
heart rate is irregular at 92 beats per minute, his blood
pressure is 148/86 mm Hg, and his lungs are clear. Te
thyroid is nontender, about twice the normal size, and
increased in consistency. He has bilateral exophthalmos
with red and signifcantly swollen eyelids and injected
conjunctiva. Tere is pitting edema (trace) of the lower
extremities and an area of brownish thickening of the
skin over the pretibial areas. What is the best manage-
ment for his thyroid dysfunction at this point?
a. Observation, antiarrhythmic treatment, and reevaluation in
6 weeks for possible hypothyroidism
b. Propylthiouracil (PTU)
c. Methimazole treatment
d. Total or near-total thyroidectomy
e. Radioactive iodine treatment
DI ABETES MELLITUS, HYPOGLYCEMI A, AND
HYPERLI PI DEMI A
4. A 54-year-old woman comes to your of ce for her rou-
tine 6-month follow-up for the management of type
2 diabetes mellitus. She has no immediate concerns.
Over the past year, she has consulted a dietitian and has
been trying to adhere to her diabetic diet. She has also
started a walking program. Her medications include
metformin 1,000 mg twice daily, simvastatin 20 mg
daily, lisinopril 20 mg daily, a multivitamin daily, and
a calcium supplement daily. She monitors her blood
glucose levels at home in the morning 2 or 3 times per
week, and the metered glucose values (in milligrams per
deciliter) are in the mid 100s. She denies having hypo-
glycemia. She works outside the home but eats meals
at regular times. On physical examination, her blood
pressure is 135/80 mm Hg, her heart rate is 75 beats
per minute, her height is 160 cm, her weight is 85 kg,
and her body mass index is 33. Other examination fnd-
ings are unremarkable. Laboratory test results include
hemoglobin A
1c
(HbA
1c
) 8.5% and normal serum crea-
tinine and liver transaminase values. What is your best
management option to improve this patients glycemic
control?
QUESTI ONS
Multiple Choice (choose the best answer)
THYROI D DI SEASE
1. A 62-year-old farmer is evaluated as a new patient. He
has a 20-year history of hypothyroidism and hypogo-
nadism. He takes levothyroxine (88 mcg daily) and
testosterone gel 1% (2.5 g daily) and feels well. He
weighs 75 kg and has a small thyroid. Physical exami-
nation fndings are unremarkable. Laboratory test
results include the following: thyrotropin 0.2 mIU/L
(reference range, 0.35.0 mIU/L) and free thyroxine
1.3 ng/dL (reference range, 0.81.8 ng/dL). You decide
to decrease the levothyroxine dosage to 75 mcg daily.
Afer 2 months, thyrotropin was 0.1 mIU/L, free thy-
roxine was 0.9 ng/dL, and he feels tired and cold. Which
of the following should you do now?
a. Decrease the levothyroxine dosage to 50 mcg daily and
recheck in 6 weeks.
b. Stop levothyroxine altogether and evaluate thyroid uptake
for hyperthyroidism.
c. Discontinue the testosterone gel since it increases the
thyroxine-binding globulins.
d. Check his thyroglobulin to distinguish between primary
and secondary hypothyroidism.
e. Continue levothyroxine at 125 mcg daily, get his old records,
and consider cortisol testing.
2. A 29-year-old woman has a 7-year history of primary
hypothyroidism due to Hashimoto thyroiditis. She has
been treated with 125 mcg of levothyroxine with a good
clinical and biochemical response so far. A few months
ago, she decided to start taking oral contraceptives
because she had persistently irregular periods related
to polycystic ovary syndrome. Her periods are regular
now, but she feels tired and has cold intolerance. Her
thyrotropin is 10.8 mIU/L; free thyroxine is 1.1 ng/dL.
Which of the following should you do next?
a. Check the titer of thyroperoxidase antibodies to assess the
severity of Hashimoto thyroiditis.
b. Advise the patient to improve her adherence to the medical
therapy and recheck in 2 to 3 months.
c. Advise the patient to take the oral contraceptive 12 hours
afer taking levothyroxine, and recheck in 2 to 3 months.
d. Add liothyronine to the levothyroxine for a better clinical
response.
e. Increase the levothyroxine dose to 137 mcg, and recheck
thyrotropin in 2 to 3 months.
7.
ENDOCRINOLOGY QUESTIONS AND ANSWERS
68

MAYO CLINI C INTERNAL MEDI CINE BOARD REVIEW: QUESTI ONS AND ANSWERS
d. Insulin sensitizers such as metformin and rosiglitazone have
been shown to slow the progression to type 2 diabetes mel-
litus with minimal risk.
6. A 63-year-old woman with a history of type 2 diabe-
tes mellitus for the past 8 years comes to your of ce
with complaints of worsening paresthesias involving
her hands. She has peripheral neuropathy involving
her lower extremities; the neuropathy is unchanged
and has been attributed to diabetes. She feels more
fatigued, which she attributes to poor sleep as a result
of her neuropathic symptoms. Additional medical
diagnoses are hypertension and hyperlipidemia. She is
up-to-date for all her medical preventive services. Her
current medications are metformin 1,000 mg twice
daily, lisinopril 20 mg daily, and atorvastatin 20 mg
daily. Physical examination fndings include the fol-
lowing: height 155 cm, weight 74.4 kg, body mass index
31, pulse 75 beats per minute, blood pressure 135/82
mm Hg, dry skin on the lower extremities, no areas
of skin breakdown, decreased sensation to monofla-
ment in the lower extremities to the ankles, and absent
ankle refexes bilaterally. Te upper extremities have
normal hand grip and normal refexes, but the patient
describes paresthesias. Laboratory test results include
the following: hemoglobin 11.5 g/dL, hemoglobin A
1c

6.5%, and normal values for serum creatinine, liver
transaminases, and serum thyrotropin. What should
be your next step in the evaluation and management of
this patient?
a. Obtain an electromyogram.
b. Determine the vitamin B
12
level.
c. Start therapy with a tricyclic antidepressant.
d. Intensify her glucose-lowering therapy by adding a second
glucose-lowering agent.
7. A 59-year-old man with type 2 diabetes mellitus
comes to your of ce for a routine visit. He has a his-
tory of coronary artery disease and had a myocardial
infarction 3 years ago. He is currently free of any car-
diac symptoms. He is an ex-smoker. He monitors his
blood glucose levels daily before breakfast, and the
majority of values are less than 150 mg/dL. He denies
having hypoglycemia. Current medications are met-
formin 1,000 mg twice daily, glimepiride 4 mg twice
daily, simvastatin 40 mg daily, metoprolol 50 mg
daily, quinapril 40 mg daily, and aspirin 81 mg daily.
Physical examination fndings include the following:
height 175 cm, weight 113 kg, pulse 65 beats per min-
ute, blood pressure 145/85 mm Hg, body mass index
37, overweight with central obesity, systolic ejection
murmur 2/6 at the lef sternal border, and decreased
sensation to pinprick in the feet up to the ankles with
absent ankle refexes bilaterally. Laboratory test results
are shown in Table 7.Q7.
a. Make no changes in her current glucose-lowering regimen,
and congratulate her.
b. Increase the metformin dose to 2,500 mg daily, with a goal
HbA
1c
of less than 7%.
c. Add a sulfonylurea to her regimen, with a goal HbA
1c
of less
than 7%, and discuss the risk of hypoglycemia.
d. Add rosiglitazone to her regimen, with a goal HbA
1c
of less
than 7%, and discuss the risk of heart failure and cardiovas-
cular events.
e. Add a single injection of insulin glargine to her regimen,
with a goal HbA
1c
of less than 7%, and discuss the risk of
hypoglycemia.
5. A 48-year-old man comes to your of ce for a routine
physical examination. He feels well but is concerned
about his family medical history. Both his mother and
his father have type 2 diabetes mellitus, and his father
(age 73 and an ex-smoker) had coronary bypass surgery
recently. Te patients medical history is positive for
hypertension. He is a nonsmoker. Current medications
include hydrochlorothiazide-triamterene 25 mg/12.5
mg 1 tablet daily and fsh oil capsules 1,000 mg daily. On
physical examination, his pulse is 85 beats per minute,
his blood pressure is 140/85 mm Hg, his height is 177
cm, his weight is 120 kg, and his body mass index is 38.
He has central obesity. Te other cardiac and physical
examination fndings are unremarkable. Diagnostic
test results are shown in Table 7.Q5.
Table 7.Q5
COMPONENT VALUE
Fasting blood glucose, mg/dL 116
Hemoglobin A
1c
, % 6.2
Serum creatinine, mg/dL 1.0
Total cholesterol, mg/dL 210
Triglycerides, mg/dL 225
High-density lipoprotein cholesterol, mg/dL 34
Calculated low-density lipoprotein cholesterol, mg/dL 131
Nonhigh-density lipoprotein cholesterol, mg/dL 176
Which of the following statements about this patient is
false ?
a. He has prediabetes, and his 5-year risk for diabetes mellitus
can be as high as 50%.
b. Lifestyle changes that lower his intake of saturated fats,
increase his exercise, and cause modest weight loss can lower
his risk for type 2 diabetes mellitus by 58%.
c. His lipid profle puts him at increased risk for a premature
cardiac event.
7. ENDOCRINOLOGY QUESTI ONS AND ANSWERS

69
c. Glucose 54 mg/dL, C peptide 0.6 ng/mL, -hydroxybutyrate
0.2 mmol/L with negative results for the sulfonylurea screen-
ing test
d. Glucose 54 mg/dL, C peptide 3.0 ng/mL, -hydroxybutyrate
0.3 mmol/L
e. Glucose 54 mg/dL, C peptide 3.0 ng/mL, -hydroxybutyrate
0.3 mmol/L with negative results for the sulfonylurea screen-
ing test
9. A 65-year-old man comes to your of ce for follow-up of
his diabetes mellitus. Te diagnosis was made a year ago
with a random blood glucose value above 300 mg/dL.
Te patient has been struggling to adhere to the dietary
recommendations and has been unable to establish an
activity program. He monitors his blood glucose lev-
els a few times a week, generally in the morning before
breakfast. All blood glucose values are over 160 mg/dL.
He is a nonsmoker. He has no history of cardiovascu-
lar disease. His current medications are metformin
1,000 mg twice daily, glimepiride 4 mg twice daily,
atorvastatin 40 mg daily, lisinopril 20 mg twice daily,
hydrochlorothiazide-triamterene 50 mg/25 mg 1 tablet
daily, and fsh oil capsules 1,000 mg twice daily. Physical
examination fndings include the following: height
170 cm, weight 95 kg, body mass index 33, pulse 75
beats per minute, and blood pressure 138/82 mm Hg.
Laboratory test results are shown in Table 7.Q9.
Which intervention is associated with a lower risk for a
cardiovascular event?
a. Increasing his aspirin dosage to 325 mg daily
b. Adding fenofbrate to his lipid-lowering regimen
c. Adding insulin to improve his glycemic control until the
hemoglobin A
1c
is less than 6.5%
d. Increasing his simvastatin dosage to 80 mg daily
e. Continuing metformin therapy
8. A 55-year-old woman is seen in your of ce accompa-
nied by her husband for follow-up afer dismissal from
her local emergency department (ED) for further evalu-
ation of reactive hypoglycemia. For years, she has had
intermittent symptoms of diaphoresis and excessive
hunger that resolve afer eating. She remembers having
a glucose tolerance test, during which her blood glucose
level decreased to about 60 mg/dL. She was advised to
see a dietitian, eat regularly, and always have hard candy
available to abort her symptoms. She has been experi-
encing more frequent symptoms and has gained weight
over the past year. She is healthy otherwise. She has no
history of diabetes. She is currently taking no medi-
cations. Te day of her evaluation in the ED, she was
involved in her usual activities at home when she expe-
rienced her usual symptoms related to hypoglycemia,
diaphoresis, and hunger. She recalled missing breakfast
but does not recall any additional events until she was
in the ED. Her husband recognized that she was not
well and was worried that she was having a stroke, so he
called the paramedics. She was sweaty and would not
respond to his questions. At the ED, several laboratory
tests were performed. She was told that her blood glu-
cose was low. She was given fuids and dextrose intra-
venously. She was dismissed and advised to follow up
as an outpatient. Which test results support abnormal
endogenous insulin secretion as a cause for hypogly-
cemia (reference ranges: C peptide, 0.94.3 ng/mL;
-hydroxybutyrate, <0.4 mmol/L)?
a. Glucose 54 mg/dL, C peptide 0.6 ng/mL, -hydroxybutyrate
4.9 mmol/L
b. Glucose 54 mg/dL, C peptide 0.6 ng/mL, -hydroxybutyrate
0.2 mmol/L
Table 7.Q7
COMPONENT VALUE
Hemoglobin A
1c
, % 7.5
Serum creatinine, mg/dL 1.2
Total cholesterol, mg/dL 150
Triglycerides, mg/dL 195
High-density lipoprotein cholesterol, mg/dL 42
Calculated low-density lipoprotein cholesterol, mg/dL 69
Nonhigh-density lipoprotein cholesterol, mg/dL 108
Table 7.Q9
COMPONENT VALUE
Hemoglobin A
1c
, % 9.2
Serum creatinine, mg/dL 1.3
Total cholesterol, mg/dL 198
Triglycerides, mg/dL 335
High-density lipoprotein cholesterol, mg/dL 35
Calculated low-density lipoprotein cholesterol, mg/dL 96
Nonhigh-density lipoprotein cholesterol, mg/dL 162
Which of the following statements about the manage-
ment of this patient is false ?
a. Use of pioglitazone (a thiazolidinedione [TZD]) should be
avoided because of the higher risk of congestive heart failure
and cardiovascular events.
b. Use of glucagon-like peptide 1 analogues (exenatide, lira-
glutide) will improve glycemic control with the potential
for weight loss.
c. Use of dipeptidyl-peptidase-4 inhibitors (sitagliptin, saxa-
gliptin) will improve glycemic control without weight loss
or gain.
d. Insulin therapy will improve glycemic control but with a
higher risk of weight gain and hypoglycemia.
70

MAYO CLINI C INTERNAL MEDI CINE BOARD REVIEW: QUESTI ONS AND ANSWERS
a. She meets the National Institutes of Health consensus crite-
ria for considering bariatric surgery.
b. Laparoscopic adjustable gastric banding and Roux-en-Y gas-
tric bypass are associated with similar outcomes for inducing
remission of type 2 diabetes mellitus.
c. Untreated obstructive sleep apnea is a risk factor for higher
perioperative morbidity and mortality with bariatric
surgery.
d. Te use of nonsteroidal anti-infammatory drugs is contrain-
dicated afer Roux-en-Y gastric bypass surgery.
12. You are called to the emergency department to see one
of your patients. He is a 48-year-old man who under-
went a Roux-en-Y gastric bypass for the management
of his obesity 2 months ago. He has severe nausea and
vomiting and has not been able to keep any food or fu-
ids down for the past 48 hours. He has abdominal pain,
particularly in the epigastric region. He feels dizzy and
complains of tingling and numbness of his hands. He
has lost 16 kg since his surgery. He has a diagnosis of
type 2 diabetes mellitus and takes metformin, which
was discontinued at his hospital dismissal afer surgery.
Other diagnoses are hypertension, hyperlipidemia, and
obstructive sleep apnea, for which he uses continuous
positive airway pressure. Current medications (which
he has not been able to take regularly) are simvastatin
20 mg daily, losartan 100 mg daily, chewable multivi-
tamin 1 tablet twice daily, calcium carbonate 300 mg
3 times daily, and vitamin B
12
1,000 mcg subcutane-
ously monthly. Physical examination fndings include
the following: height 177.5 cm, weight 124 kg, pulse
112 beats per minute, blood pressure 105/68 mm Hg,
and temperature 37.8 C. Te patient appears uncom-
fortable and has frequent dry heaves. Cardiac auscul-
tation is unremarkable except for tachycardia. Lung
sounds are normal. Te abdomen has hyperactive bowel
sounds and is tender to palpation in the epigastric
region. Tere are no acute peritoneal signs. Laboratory
test results are shown in Table 7.Q12.
OBESITY AND NUTRITI ON
10. A 45-year-old woman comes to your of ce for her
routine physical examination. She has no immedi-
ate concerns except that her weight has been increas-
ing in the past few years. She has a clerical job and a
busy home life. Her eating habits have not changed
dramatically, although she admits that she eats at fast
food restaurants several times a week on her way to
extracurricular activities. She currently has no exercise
program, although she owns a treadmill. She is other-
wise healthy. She has regular menses. She has had tubal
ligation. She has a family history of type 2 diabetes
mellitus; her mother received the diagnosis in her 50s.
Te patients medications are vitamin D supplement
400 international units daily and calcium supplement
600 mg daily. Physical examination fndings include
the following: height 162 cm, weight 83 kg, body mass
index 33, pulse 72 beats per minute, and blood pressure
135/80 mm Hg. She is overweight with central obesity.
Examination fndings are otherwise unremarkable.
Which is not an appropriate step in the management of
this patient?
a. Screen for hypothyroidism with a sensitive thyrotropin
(sTSH) test.
b. Screen for type 2 diabetes mellitus by measuring the fasting
blood glucose level.
c. Advise that she start a walking program of 10- to 30-minute
sessions, working toward a goal of 150 minutes of walking
per week.
d. Recommend that she eat a low-carbohydrate diet, which is
superior for promoting and maintaining a weight loss of 5%
to 10%.
11. A 52-year-old woman comes to your of ce to discuss
bariatric surgery. She has struggled to lose weight dur-
ing most of her adult life. She has participated in indi-
vidual and commercial weight loss programs with no
long-term success at managing her weight, and she is
concerned about her health. She has type 2 diabetes
mellitus, hypertension, hyperlipidemia, and joint pain.
Her joint pain and fatigue interfere with her ability to
be physically active. She does not smoke or drink alco-
holic beverages. She has no prior psychiatric history.
Afer she recently spoke to a friend who had bariatric
surgery and is doing very well, she became motivated to
consider this option. Her current medications are met-
formin 1,000 mg twice daily, glipizide 20 mg twice daily,
atorvastatin 20 mg daily, fenofbrate 145 mg daily, fos-
inopril 20 mg daily, hydrochlorothiazide-triamterene
25 mg/12.5 mg 1 tablet daily, aspirin 81 mg daily, and
naproxen 500 mg twice daily as needed. Physical exami-
nation fndings include the following: height 157 cm,
weight 110 kg, body mass index 45, pulse 72 beats per
minute, and blood pressure 138/75 mm Hg. She is
overweight with central obesity. Examination fndings
are otherwise unremarkable. Which statement about
bariatric surgery in the management of this patient is
false ?
Table 7.Q12
COMPONENT VALUE
Sodium, mg/dL 146
Potassium, mg/dL 3.2
Glucose, mg/dL 118
Creatinine, mg/dL 1.5
Aspartate aminotransferase, U/L 52
Alkaline phosphatase, U/L 120
Lipase, U/L 68
Hemoglobin, g/dL 14.1
Leukocyte count, 10
9
/L 10.1
Platelet count, 10
9
/L 158
7. ENDOCRINOLOGY QUESTI ONS AND ANSWERS

71
PITUITARY, GONADAL, AND ADRENAL
DI SORDERS
14. A 31-year-old married white man presents with breast
enlargement and tenderness of 6 months duration.
He reports normal sexual functions and has 2 biologic
children. He is not taking any medications, he does not
smoke or drink, and he has not used any recreational
drugs. Physical examination fndings are unremark-
able except for bilateral, tender, symmetrical gyne-
comastia. His secondary sex characteristics, external
genitalia, and testicular size and consistency are all
normal. Laboratory test results are normal for serum
total and free testosterone, prolactin, thyrotropin, and
dehydroepiandrosterone-sulfate (DHEA-S). Other
results (and reference ranges) include serum estradiol
78 pg/mL (1040 pg/mL) and -human chorionic gonad-
otropin (hCG) 50,000 IU/L (<3.0 IU/L). Which of the
following tests is the most appropriate next step?
a. Ultrasonography of the testicles
b. Computed tomography imaging of the adrenals
c. Magnetic resonance imaging of the pituitary
d. Mammography
e. Liver biopsy
15. A 26-year-old woman presents at 4 weeks post partum
with headaches, profound weakness, nausea, and vomit-
ing. She had been breast-feeding. Physical examination
fndings are unremarkable. Magnetic resonance imaging
(MRI) of the head shows a sellar mass with suprasellar
extension but without chiasmal compression. Laboratory
test results (and reference ranges) include the following:
serum sodium 125 mEq/L (136142 mEq/L), serum pro-
lactin 72 g/L (430 g/L), 8 am serum cortisol 3 g/dL
(525 g/dL), serum adrenocorticotropic hormone
(ACTH) 10 pg/mL (1060 pg/mL), and normal values
for serum free thyroxine and thyrotropin. Which of the
following is the most likely diagnosis?
a. Primary adrenal insuf ciency (Addison disease)
b. Prolactin-producing pituitary tumor
c. Nonfunctioning pituitary tumor
d. Pituitary apoplexy (Sheehan syndrome)
e. Lymphocytic hypophysitis
16. A 32-year-old woman presented with amenorrhea of
9 months duration. Physical examination fndings were
unremarkable except for bilateral expressible galactor-
rhea. Laboratory test results included a negative preg-
nancy test, evidence of a hypogonadotropic state, and a
prolactin level of 250 g/L. Findings on magnetic reso-
nance imaging of the pituitary were consistent with a pitu-
itary microadenoma. Te patient was unable to tolerate
dopaminergic-agonist drugs, and a decision was made to
treat her surgically. On the second day afer transsphenoi-
dal microadenomectomy, postoperative diabetes insipi-
dus developed but resolved by the third postoperative day.
Her medications at dismissal included an analgesic and
replacement doses of hydrocortisone. She was instructed
in hydrocortisone dosage modifcations during an acute
You started intravenous fuid hydration with 0.9%
sodium chloride solution and 20 mEq of potassium
per liter at 100 mL/h. You have consulted the general
surgeon on call. Te patient is scheduled for computed
tomography of the abdomen within the next hour.
What is the most appropriate next step in the manage-
ment of this patient?
a. Give the patient 100 mg of thiamine intravenously.
b. Call a gastroenterologist to perform an emergent esophago-
gastroduodenoscopy for the possibility of an anastomotic
stricture.
c. Give the patient 1,000 mcg of cyanocobalamin (vitamin B
12
)
intramuscularly.
d. Give the patient a multivitamin injection intravenously.
13. A 62-year-old man comes to your of ce for a routine
physical examination. He tries to live a healthy life-
style. He follows a low-fat diet, rich in fruits and vege-
tables, and limits his intake of red meats. He exercises
regularly, 3 to 4 times per week for 30 to 40 minutes
each time. He does not smoke. His medical history
is positive for hypertension, hyperlipidemia, benign
prostatic hypertrophy, and erectile dysfunction. He
does not have diabetes mellitus. He is interested in
disease prevention and wonders about his nutritional
supplements. His family history is signifcant for
coronary artery disease in his father, who had a myo-
cardial infarction at age 65 years, and for type 2 dia-
betes mellitus in his mother. His current medications
are atorvastatin 20 mg daily, metoprolol 50 mg daily,
tamsulosin 0.4 mg daily, tadalafl 10 mg as directed
before intercourse, folic acid 2.5 mg daily, fsh oil
1,000 mg daily, vitamin E 400 international units
daily, and a multivitamin daily. Physical examination
fndings include the following: height 177 cm, weight
85 kg, body mass index 27, pulse 60 beats per minute,
and blood pressure 135/70 mm Hg. He appears well
and physically ft. Tere are no abnormal fndings on
the cardiac examination. His prostate has mild dif-
fuse enlargement without discrete nodularities. Te
remainder of the examination fndings are unremark-
able. Which of the following statements is false about
vitamin and nutritional supplementation to prevent
disease?
a. Routine multivitamin supplementation has not been associ-
ated with health benefts in population-based studies.
b. Antioxidant vitamin supplementation (beta carotene, vita-
min E) has not been associated with a lower risk of cardiovas-
cular events or mortality in randomized, placebo-controlled
clinical trials.
c. Supplementation with fsh oil (1,000 mg daily) is associated
with a greater decrease in cardiovascular mortality compared
with consumption of 2 or 3 meals of fatty fsh (salmon) per
week.
d. Folic acid supplementation with normalization of homo-
cysteine levels has not been associated with a lower risk of
cardiovascular events in randomized, placebo-controlled
clinical trials.
72

MAYO CLINI C INTERNAL MEDI CINE BOARD REVIEW: QUESTI ONS AND ANSWERS
a. Check the parathyroid hormone (PTH) level.
b. Check the 25-hydroxyvitamin D level.
c. Perform a parathyroid scan.
d. Emphasize the use of acetaminophen if fulike symptoms
occur.
e. Use bisphosphonates only if her creatinine clearance is
greater than 35 mL/min.
19. A 70-year-old man was hospitalized because of
anterior chest pain. His vital signs were normal. An
urgent coronary angiogram led to angioplasty, which
was successfully carried out, and the patient was dis-
charged home 2 days later. During the next month,
he had some nervousness, anxiety, and a 10-kg
weight loss while eating his usual diet. His cardiolo-
gist recently determined that the patient had atrial
fbrillation and prescribed a calcium channel blocker.
When you see him a few days later in the emergency
department, he has a ventricular rate of 130 beats per
minute and shortness of breath. On examination, he
is in no acute distress at rest, but he is in atrial fbril-
lation with a rate of 120 to 130 beats per minute. He
has no tremors and no signs of Graves ophthalmopa-
thy or dermopathy. His thyroid is low lying, multi-
nodular, nontender, and slightly enlarged. You learn
of a long-standing history of nontoxic multinodular
goiter. Because of the atrial fbrillation, thyroid tests
were performed, and the results were as follows: thy-
rotropin (TSH) less than 0.01 mIU/L, free thyrox-
ine 3.7 ng/dL, and negative for thyroid peroxidase
(TPO) antibodies. What is the most likely cause of
this patients thyrotoxicosis?
a. Lymphocytic thyroiditis
b. Autonomous thyroid nodule
c. Iodine-induced hyperthyroidism
d. Graves disease
e. TSH-producing pituitary adenoma
20. A 41-year-old woman has an elevated ionized calcium
level of 5.36 mg/dL during an infertility evaluation.
Her only complaints are fatigue and occasional head-
aches. She denies having a history of constipation, nau-
sea, vomiting, kidney stones, hematuria, or fractures.
Her appetite was normal without signifcant weight
changes. She denies having any known family history
of hypercalcemia, metabolic bone disease, or kidney
stones. Her medications include fexofenadine and
multivitamins. Examination fndings are unremark-
able: heart rate 70 beats per minute and regular, weight
72 kg, and blood pressure 132/78 mm Hg. Laboratory
test results are shown in Table 7.Q20.
illness. A week afer her hospital dismissal, she was evalu-
ated in an emergency department for headache, fatigue,
lethargy, confusion, and nausea. She had been taking
her medications regularly. Physical examination fndings
were unremarkable. Which of the following is the most
important laboratory test to perform?
a. Pregnancy test
b. Serum free thyroxine
c. Serum cortisol
d. Serum sodium
e. Serum prolactin
BONE AND PARATHYROI D DI SEASE
17. A 34-year-old woman is referred to you for a preopera-
tive evaluation from an otolaryngology colleague who
diagnosed a medullary thyroid carcinoma. Tat evalu-
ation was prompted by a call from 1 of the patients
sisters who recently received the same diagnosis. Te
patient has no complaints, she has no past medical
problems, and she does not take any medications. Her
vital signs are normal. You can feel a 1.5-cm right thy-
roid lobe nodule without cervical adenopathy. Te rest
of the examination fndings are unremarkable. Which
of the following should you do preoperatively?
a. Measure levels of serum calcium, gastrin, and insulin.
b. Request magnetic resonance imaging of the abdomen.
c. Obtain genetic testing for RET proto-oncogene mutation.
d. Recommend proceeding with surgery if the patients blood
pressure is normal.
e. Measure levels of serum calcium and plasma fractionated
metanephrines.
18. A 79-year-old woman broke her hip 3 months ago by
tripping on a stair. Subsequent evaluation (her frst)
with dual energy x-ray absorptiometry (DEXA) identi-
fed low bone mineral density with T scores less than
2.5 at the contralateral hip and spine. She has come
to you with her daughter to discuss osteoporosis treat-
ment. You are told that she had normal periods until
menopause at age 51, she never took estrogens but has
begun taking 1,200 mg of elemental calcium daily since
she returned home from the hospital, and she takes 400
international units of vitamin D daily. Her calcium level
is normal. She has had dif culty swallowing tablets afer
she had a stroke 5 years ago, and she wants to discuss the
use of intravenous (IV) bisphosphonates. You discuss
their side efects, including the risk of hypocalcemia.
You plan to identify and minimize that risk. Which of
the following should you do next?
7. ENDOCRINOLOGY QUESTI ONS AND ANSWERS

73
21. A 65-year-old woman with recurrent kidney stones
receives a diagnosis of hyperparathyroidism (total cal-
cium 13.3 mg/dL, parathyroid hormone [PTH] 380
pg/mL). An experienced parathyroid surgeon performs
a minimally invasive parathyroidectomy and removes a
large parathyroid adenoma (6 g). Postoperatively, the
patient feels well and begins eating a normal diet. At
24 hours postoperatively, she begins experiencing
perioral tingling and the Chvostek sign is positive. Te
calcium level decreases to 8.1 mg/dL, and calcium car-
bonate (600 mg elemental calcium 3 tablets daily) is
given orally. On postoperative day 2, the tingling contin-
ues and the results of her laboratory tests are as follows:
total calcium 7.4 mg/dL, phosphorus 1.5 mg/dL, albu-
min 3.9 g/dL, and creatinine 1.0 mg/dL. You administer
intravenous calcium, increase her oral calcium dosage,
and begin calcitriol therapy, and her symptoms improve.
What is the best explanation for these fndings?
a. Hungry bone syndrome
b. Severe vitamin D defciency
c. Transient hypoparathyroidism from surgical manipulation
d. Transient hypoparathyroidism from long-term suppression
of normal parathyroids
e. Lack of absorption of oral calcium
What is the best test for diagnosing familial hypocal-
ciuric hypercalcemia (FHH)?
a. Another 24-hour urine collection
b. Parathyroid scan
c. Neck ultrasonography
d. Screening for FHH among family members
e. Calcium-sensing receptor mutational analysis
Table 7.Q20
COMPONENT FINDING
Ionized calcium, mg/dL 5.48
Total calcium, mg/dL 10.4
Albumin, g/dL 4.0
Phosphorus, mg/dL 3.1
Creatinine, mg/dL 1.0
Parathyroid hormone, pg/mL 59
Serum protein electrophoresis Normal
25-hydroxyvitamin D, ng/mL 22
Blood cell counts Normal
Urinary creatinine, mg/d 1,500
Urinary calcium, mg/d 129
Calcium clearance to creatinine clearance ratio 0.0075
74
eye condition, lead to a transient increase in thyroid hor-
mone values, and not achieve euthyroidism for another 2
to 3 months. Surgery would be associated with additional
risks because this patient has several cardiac comorbidities.
Te safest and most efective treatment for him would be
antithyroid drug treatment. Methimazole would be the
frst choice. PTU is a second-line drug because of its asso-
ciation with severe liver failure and death, but PTU is still
preferred in the frst trimester of pregnancy and during
thyroid storm. (See Bahn et al and Bartalena et al in the
Suggested Reading list.)
4. Answer c.
Metformin and sulfonylurea medications have been exten-
sively studied as monotherapy and combination therapy
for type 2 diabetes mellitus. For the overweight or obese
patient, metformin is the recommended frst-line therapy
because of its insulin-sensitizing efects, low risk of hypo-
glycemia, and neutral impact on weight. Sulfonylurea
medications provide additional glucose lowering to the
patient who has inadequate glycemic control with optimal
doses of metformin and a recognized risk of hypoglycemia
and weight gain. No additional glucose-lowering efect
is observed with metformin doses higher than 2,000 mg
daily. Te use of rosiglitazone is restricted because of recog-
nized risks of heart failure and cardiovascular events, and it
is not appropriate for this patient. (See Nathan et al in the
Suggested Reading list.)
5. Answer d.
Tis patient meets the body mass index (BMI) criterion
for obesity (BMI >30) and is at risk for type 2 diabetes
mellitus over the next 5 years. Many patients are unaware
of their risk for progression toward type 2 diabetes mel-
litus. Interventions to delay progression to type 2 diabe-
tes mellitus should be discussed. Lifestyle changes with
dietary modifcation, regular physical activity, and mod-
est weight loss can substantially lower his risks for pro-
gressing to type 2 diabetes mellitus. Several medications
have been studied in type 2 diabetes mellitus prevention,
including metformin, rosiglitazone, acarbose, and orlistat.
However, rosiglitazone was associated with signifcant
risk for weight gain and heart failure despite the younger
population studied (mean age, 55 years) in the Diabetes
Reduction Assessment With Ramipril and Rosiglitazone
Medication (DREAM) trial. Since the DREAM trial, the
use of rosiglitazone has been signifcantly restricted because
1. Answer e.
Te replacement dose of levothyroxine is relatively low for
this patients weight (the usual dose is 1.6 mcg/kg body
weight), and it would be unlikely to cause iatrogenic thyro-
toxicosis. Te free thyroxine level decreased with a decrease
in the levothyroxine dosage without a signifcant change in
the thyrotropin level. Te patient has symptoms of hypo-
thyroidism, and he has hypogonadism. His old records
might explain why he has central hypothyroidism. If those
records are not available, testing his adrenal axis and imag-
ing his pituitary would be the next step. For the same
reason, decreasing the dosage to 50 mcg daily for a 75-kg
person or evaluating him for hyperthyroidism would not
address his symptoms or biochemical response to the frst
change in dosage. Androgens actually decrease the level of
thyroxine-binding globulins. (See Daly et al and Singer et
al in the Suggested Reading list.)
2. Answer e.
Increased estrogen levels, which were likely present in
this patient during oral contraceptive therapy, lead to
an increase in thyroxine-binding globulins, and that
requires an increase in the levothyroxine replacement
dose in hypothyroid patients to maintain the free thy-
roxine levels unchanged. Tyroperoxidase antibodies are
not pathogenic, and their titer does not correlate with
the replacement dose. Te patients hypothyroidism had
been well controlled with a stable dosage of levothyroxine
for several years, so that nonadherence is probably not a
problem. Te estrogen action is sustained, is not related
to the timing of the administration of levothyroxine
and oral contraceptive, and does not interfere with the
absorption of levothyroxine. Te combination of levothy-
roxine and liothyronine has not been proved to be more
efective clinically than levothyroxine monotherapy, but
it has been associated with signifcantly more iatrogenic
thyrotoxicosis in clinical trials; therefore, it is not recom-
mended. (See Ain et al and Sawka et al in the Suggested
Reading list.)
3. Answer c.
Tis patient has thyrotoxicosis with Graves ophthalmopa-
thy and dermopathy, which are pathognomonic for Graves
disease. Terefore, the possibility of thyroiditis is efec-
tively excluded. Te presence of moderate to severe oph-
thalmopathy in a smoker is a relative contraindication for
the use of radioactive iodine, which might exacerbate his
ANSWERS
7. ENDOCRINOLOGY QUESTI ONS AND ANSWERS

75
patient currently meets criteria for obesity, and, with her
family history, screening by measuring the fasting blood
glucose level is indicated. Lifestyle changes with dietary
modifcation and regular physical activity are the corner-
stone of weight management. Randomized studies have
shown that calorie restriction and not macronutrient com-
position (proportions of carbohydrate, protein, and fat)
determine success in weight loss and weight maintenance.
(See Gardner et al in the Suggested Reading list.)
11. Answer b.
Bariatric surgery has become well accepted in the manage-
ment of medically complicated obesity mainly because of
the benefts to the patient with type 2 diabetes mellitus.
Remission rates for diabetes vary according to the operation
performed: Remission rates are highest with biliopancreatic
diversion and duodenal switch (92%) and Roux-en-Y gas-
tric bypass (85%) and lowest with laparoscopic adjustable
gastric banding (47%). A patient considering bariatric sur-
gery should be well informed about the currently accepted
criteria, risk factors that afect perioperative morbidity and
mortality, and long-term management. (See Greenway
et al in the Suggested Reading list.)
12. Answer a.
Acute thiamine defciency can occur in a patient with severe
persistent vomiting. It has been reported in patients who
have had bariatric surgery that involved a restrictive mecha-
nism for weight loss, such as vertical banded gastroplasty,
laparoscopic adjustable gastric banding, and Roux-en-Y
gastric bypass. Early recognition and treatment are key to
avoid neurologic complications and Wernicke encephal-
opathy. (See Serra et al in the Suggested Reading list.)
13. Answer c.
Health benefts of fsh oil supplementation vary according
to the dosage of active docosahexaenoic acid (DHA) and
eicosapentaenoic acid (EPA) consumed. At lower dosages
(1,000 mg daily), omega-3 fatty acids are associated with a
lower risk of cardiovascular mortality. Te same beneft is
observed when individuals consume 2 servings weekly of a
fatty fsh such as salmon. For individuals already consum-
ing this amount of omega-3 fatty acids in their diet, there
is no additional beneft to additional omega-3 fatty acid
supplementation. Higher doses of DHA and EPA (24 g
daily) are needed to lower triglyceride levels. (See Jacobson
in the Suggested Reading list.)
14. Answer a.
Gynecomastia is the most common disorder of the male
breast and accounts for more than 80% of all male breast
masses. It may result from a trivial cause, or it may be an early
sign of a sinister disorder. Te basic mechanism of gyne-
comastia is relative estradiol excess, which can result from
decreased androgen production or efect, or an increase in
estrogen production or efect. In adults, the 2 most com-
mon causes of gynecomastia are drugs and alcohol-related
it is associated with a higher risk of cardiovascular events.
(See American Diabetes Association, Gerstein et al, and
Knowler et al in the Suggested Reading list.)
6. Answer b.
In patients with diabetes mellitus, neuropathic symptoms
are ofen attributed to the underlying disease. However, in
a large percentage of patients (30%), long-term metformin
therapy can lead to vitamin B
12
malabsorption and subse-
quent defciency. If not recognized, a treatable cause for
irreversible neurologic symptoms would be overlooked.
(See Bell in the Suggested Reading list.)
7. Answer e.
Among the interventions listed, metformin therapy is the
only one that has been clearly associated with a lower-
ing of cardiovascular events and mortality in the United
Kingdom Prospective Diabetes Study. Te Action to
Control Cardiovascular Risk in Diabetes (ACCORD)
trial showed a trend toward lower cardiovascular events
only when fenofbrate was used with a statin and triglycer-
ide values were greater than 200 mg/dL. For a patient with
a low-density lipoprotein cholesterol level less than 70 mg/
dL, there would be no additional beneft to increasing the
statin dosage. (See Stratton et al and Toth in the Suggested
Reading list.)
8. Answer e.
Te normal physiologic response to hypoglycemia is low
insulin secretion with gradual development of ketosis.
Endogenous insulin secretion is best determined not only
by measuring insulin levels but by measuring C peptide,
the peptide cleaved from the insulin molecule afer secre-
tion renders it active. To assure that true excessive endog-
enous insulin secretion is involved, the presence of insulin
secretagogues must be ruled out as a potential explanation
for hypoglycemia with elevated or inappropriately nor-
mal insulin and C peptide levels. (See Cryer et al in the
Suggested Reading list.)
9. Answer a.
Tis patient is receiving maximal dosages of the recom-
mended frst- and second-line therapies for the manage-
ment of type 2 diabetes mellitus. Te choice of a third agent
should take several factors into consideration. Te use of
rosiglitazone, a TZD, is signifcantly restricted because
of the recognized risk of cardiovascular events. Te same
risks may not apply to pioglitazone. (See Loke et al in the
Suggested Reading list.)
10. Answer d.
For a patient with recent weight gain, it is reasonable to
screen for thyroid function abnormalities with an sTSH
test. Other endocrinopathies such as Cushing syndrome
are rare, and screening should likely not be pursued in the
absence of other suspicious physical fndings such as facial
plethora, purple striae, hirsutism, and hypertension. Te
76

MAYO CLINI C INTERNAL MEDI CINE BOARD REVIEW: QUESTI ONS AND ANSWERS
and lasts a few days. Permanent diabetes insipidus is rare.
Transient excess AVP leading to syndrome of inappropriate
secretion of antidiuretic hormone (SIADH) and hypona-
tremia occurs in 10% of patients within 5 to 14 days postop-
eratively. Pregnancy in the frst week afer trans sphenoidal
surgery is unrealistic and a pregnancy test is not warranted.
If symptomatic postoperative hypothyroidism occurs, it
usually occurs several weeks postoperatively. Te patient
has been taking her hydrocortisone regularly; therefore,
she would not have cortisol defciency, so measuring serum
cortisol would be unnecessary. Although the patient may
have persistent hyperprolactinemia afer surgery for micro-
prolactinoma, she would not be symptomatic in the frst
week postoperatively. Te most important consideration
in this patient is SIADH; therefore, measuring the serum
sodium level is the most important laboratory test. (See
Lien and Shapiro in the Suggested Reading list.)
17. Answer e.
Multiple endocrine neoplasia (MEN) type 2 must be con-
sidered because of the familial association described. Te
conditions that may also be present in this patient, there-
fore, are pheochromocytoma (in 40%-50% of patients with
MEN type 2) and hyperparathyroidism (in 10%-20%).
Both are important from an operative perspective:
Hemodynamic fuctuations can occur with pheochromocy-
toma, and hyperparathyroidism can be addressed with the
thyroid surgery. Terefore, screening for both with highly
sensitive biomarkers is important. Te testing for calcium,
gastrin, and insulin is a starting point for abnormalities
in MEN type 1. Imaging the abdomen for diagnosing
pheochromocytoma has a very poor specifcity (because
of adrenal incidentalomas) without any signifcant gain
in sensitivity compared with measuring levels of plasma
metanephrines. Genetic testing takes time (26 weeks),
and a positive test does not indicate the presence of 1 or
both of these associated endocrinopathies. Normal blood
pressure is present in about 10% of patients with pheochro-
mocytoma (the percentage is higher among patients with
familial syndromes). (See Kloos et al and Raue et al in the
Suggested Reading list.)
18. Answer b.
Hypocalcemia is more likely to occur in patients who have
vitamin D defciency; therefore, the risk of hypocalcemia
can be minimized by vitamin D and calcium supplemen-
tation. Te serum calcidiol (25-hydroxyvitamin D) con-
centration should be assessed before the patient receives
IV bisphosphonate infusions. Patients with vitamin D
defciency (25-hydroxyvitamin D <15 ng/mL) should be
treated before the infusion. Results from checking the PTH
level and performing the parathyroid scan would be inad-
equate for diagnosing vitamin D defciency. In addition,
having a normal serum calcium level excludes the diagnosis
of hypoparathyroidism. Acetaminophen will not mini-
mize the risk of hypocalcemia since it only afects the sys-
temic infammation that occurs with IV bisphos phonates.
Te risk of hypocalcemia is independent of the creatinine
liver disease. Less common causes include recovery from
malnutrition or other serious chronic illness. Rarely
encountered are hCG-producing tumors, adrenal tumors,
and testicular tumors. In this patient, drugs and alcohol,
gonadal failure, hyperthyroidism, and prolactin-producing
pituitary tumor have been excluded as causes. Tis patient
has an hCG-producing tumor and increased estrogen
production. Although the most common manifestation
of testicular tumors is a testicular enlargement or mass,
small testicular tumors may ofen be occult. Of primary
testicular tumors, 97% arise from seminiferous tubules
(germ cell tumors) and about 3% arise from interstitial tis-
sue. Germ cell tumors are the most common solid tumors
in men between the ages of 15 and 34 years; these tumors
occur primarily in white men and are the main tumors that
secrete hCG. Stimulation of Leydig cells by hCG results
in an increased secretion of both estrogen and andro-
gen. Testicular ultrasonography is the most appropriate
step. A normal DHEA-S value excludes a diagnosis of an
estrogen-producing adrenal tumor, a normal prolactin
value excludes the diagnosis of a prolactin-producing pitu-
itary tumor, and imaging studies of these glands are not
warranted. Mammograms are not required for evaluation
of symmetrical concentric gynecomastia; they are indi-
cated in the evaluation of unilateral breast masses or when
breast cancer is considered. Nothing in the presentation
of this patient suggests an occult liver disease that would
necessitate a liver biopsy. (See Braunstein in the Suggested
Reading List.)
15. Answer e.
Headaches, pituitary insuf ciency, and MRI fndings con-
sistent with a pituitary tumor in relation to pregnancy
should always lead to consideration of lymphocytic hypo-
physitis. Absence of a high ACTH level excludes primary
adrenal insuf ciency. Pituitary tumors, whether prolactin
producing or nonfunctioning, are predominantly benign
and slow growing. A pituitary tumor would have been pres-
ent for a while and would probably have made the patient
hypogonadotropic and infertile, so that she would not
have presented initially in the postpartum state. Patients
with Sheehan syndrome have a history of postpartum
hemorrhage-induced hypotension or shock that requires
blood transfusion; the signs and symptoms of hypopitu-
itarism are usually recognized early afer delivery, and the
patient loses the ability to breast-feed. Lymphocytic hypo-
physitis is therefore the most likely diagnosis. (See Rivera
in the Suggested Reading list.)
16. Answer d.
Major complications of pituitary surgery include postop-
erative hemorrhage, cerebrospinal fuid leak, meningitis,
and visual disturbances; these occur in less than 5% of
patients and occur most frequently in patients who have
large tumors and not microadenomas. Diabetes insipidus,
the most common disorder of arginine vasopressin (AVP)
secretion in the postoperative state, occurs in less than 15%
of patients and usually occurs early afer pituitary surgery
7. ENDOCRINOLOGY QUESTI ONS AND ANSWERS

77
disease (refected by the very large parathyroid gland
and the signifcant calcium and PTH elevations), and it
occurs early afer surgery. Continuing calcium therapy
beyond symptom control will likely improve her overall
bone mass. With the high calcium value preoperatively, it
is quite unlikely that she had signifcant vitamin D def-
ciency to explain these fndings. Te minimal access of
parathyroidectomy avoids bilateral neck exploration, thus
eliminating the possibility of surgical manipulation of all
the parathyroid glands and its consequences. Te relative
insuf ciency of the remaining parathyroids from pro-
longed suppression is not consistent with a low phospho-
rus level since hypoparathyroidism is associated with low
renal phosphate clearance. Oral calcium supplementation
might be decreased initially, but it should improve in the
second day, and the phosphorus level should be normal.
(See Brasier and Nussbaum, and Tohme and Bilezikian in
the Suggested Reading list.)
SUGGESTED READING
Ain KB, Mori Y, Refetof S. Reduced clearance rate of thyroxine-binding
globulin (TBG) with increased sialylation: a mechanism for
estrogen-induced elevation of serum TBG concentration. J Clin
Endocrinol Metab. 1987 Oct;65(4):68996.
American Diabetes Association. Standards of medical care in diabetes:
2011. Diabetes Care. 2011 Jan;34(Suppl 1):S1161.
Bahn RS, Burch HS, Cooper DS, Garber JR, Greenlee CM, Klein IL,
et al. Te role of propylthiouracil in the management of Graves dis-
ease in adults: report of a meeting jointly sponsored by the American
Tyroid Association and the Food and Drug Administration. Tyroid.
2009 Jul;19(7):6734.
Bahn Chair RS, Burch HB, Cooper DS, Garber JR, Greenlee MC, Klein I,
et al; American Tyroid Association; American Association of Clinical
Endocrinologists. Hyperthyroidism and other causes of thyrotoxico-
sis: management guidelines of the American Tyroid Association and
American Association of Clinical Endocrinologists. Tyroid. 2011
Jun;21(6):593646. Epub 2011 Apr 21. Erratum in: Tyroid. 2011
Oct;21(10):1169.
Bartalena L, Marcocci C, Bogazzi F, Manetti L, Tanda ML, DellUnto E,
et al. Relation between therapy for hyperthyroidism and the course of
Graves ophthalmopathy. N Engl J Med. 1998 Jan 8;338(2):738.
Bell DS. Metformin-induced vitamin B12 defciency presenting as a
peripheral neuropathy. South Med J. 2010 Mar;103(3):2657.
Black DM, Delmas PD, Eastell R, Reid IR, Boonen S, Cauley JA, et al;
HORIZON Pivotal Fracture Trial. Once-yearly zoledronic acid for
treatment of postmenopausal osteoporosis. N Engl J Med. 2007 May
3;356(18):180922.
Brasier AR, Nussbaum SR. Hungry bone syndrome: clinical and bio-
chemical predictors of its occurrence afer parathyroid surgery. Am J
Med. 1988 Apr;84(4):65460.
Braunstein GD. Clinical practice: gynecomastia. N Engl J Med. 2007 Sep
20;357(12):122937.
Cryer PE, Axelrod L, Grossman AB, Heller SR, Montori VM, Seaquist
ER, et al; Endocrine Society. Evaluation and management of adult
hypoglycemic disorders: an Endocrine Society Clinical Practice
Guideline. J Clin Endocrinol Metab. 2009 Mar;94(3):70928.
Daly RC, Su TP, Schmidt PJ, Pagliaro M, Pickar D, Rubinow DR.
Neuroendocrine and behavioral efects of high-dose anabolic steroid
administration in male normal volunteers. Psychoneuroendocrinology.
2003 Apr;28(3):31731.
Gardner CD, Kiazand A, Alhassan S, Kim S, Staford RS, Balise RR,
et al. Comparison of the Atkins, Zone, Ornish, and LEARN diets
for change in weight and related risk factors among overweight
clearance even though IV bisphosphonates should not be
used in patients with a creatinine clearance less than 30 to
35 mL/min because of the risk of renal osteodystrophy.
(See Black et al and Rosen and Brown in the Suggested
Reading list.)
19. Answer c.
Iodine-induced hyperthyroidism can be easily overlooked
and is likely more common than is realized. It usually mani-
fests as apathetic hyperthyroidism, which can be masked
even further by the use of -blockers. Te time frame for
exposure to iodinated contrast agents a month earlier is typ-
ical for the development of thyrotoxicosis in patients with
preexisting multinodular goiter. Treatment involves anti-
thyroid drugs and supportive measures. Te other choices
can be excluded on clinical grounds. Lymphocytic thy-
roiditis is relatively rare in the elderly and is almost always
associated with anti-TPO antibodies, which were absent
in this patient. Tis disorder can occur in patients who
have Graves disease and preexisting iodine insuf ciency.
However, iodine insuf ciency is rare in North America,
and the multinodular goiter also argues against Graves
disease. An autonomous toxic nodule evolves slowly over
many years rather than arising abruptly. A TSH-producing
adenoma can be excluded by the suppressed TSH value.
(See Bahn Chair et al and Martin et al in the Suggested
Reading list.)
20. Answer d.
FHH is an autosomal dominant disorder characterized
by asymptomatic hypercalcemia, normal parathyroid
hormone levels, and low urinary calcium levels. Te cause
is usually an inactivating mutation in the calcium-sensing
receptor, which makes the parathyroid glands less sensi-
tive to calcium and increases the tubular reabsorption of
calcium. Screening for FHH among family members is
the most reliable method of making the diagnosis, and
genetic testing should be used only in atypical cases if
family members are not available. Tis patients sister had
asymptomatic hypercalcemia with a calcium clearance
to creatinine clearance ratio of 0.005. For approximately
80% of patients, the ratio is less than 0.01; in contrast, for
primary hyperparathyroidism patients, the ratio is usually
greater than 0.02. Parathyroid scanning and ultrasonog-
raphy will not reliably distinguish these 2 entities and
should be used only for localization purposes if the diag-
nosis of primary hyperparathyroidism has been estab-
lished. Given that the urinary creatinine value from the
24-hour collection appears complete, there is no need to
repeat it. (See Heath, and Tfelt-Hansen and Brown in the
Suggested Reading list.)
21. Answer a.
When PTH values are not available, the most useful evi-
dence of hungry bone syndrome is low calcium and phos-
phorus levels. Both calcium and phosphorus are moving
rapidly into the unmineralized osteoid of this patient.
Hungry bone syndrome occurs in patients with severe
78

MAYO CLINI C INTERNAL MEDI CINE BOARD REVIEW: QUESTI ONS AND ANSWERS
Association for Study of Diabetes. Medical management of hyper-
glycemia in type 2 diabetes: a consensus algorithm for the initiation
and adjustment of therapy: a consensus statement of the American
Diabetes Association and the European Association for the Study
of Diabetes. Diabetes Care. 2009 Jan;32(1):193203. Epub 2008
Oct 22.
Raue F, Frank-Raue K, Grauer A. Multiple endocrine neoplasia type 2:
clinical features and screening. Endocrinol Metab Clin North Am.
1994 Mar;23(1):13756.
Rivera JA. Lymphocytic hypophysitis: disease spectrum and approach to
diagnosis and therapy. Pituitary. 2006;9(1):3545.
Rosen CJ, Brown S. Severe hypocalcemia afer intravenous bisphospho-
nate therapy in occult vitamin D defciency. N Engl J Med. 2003 Apr
10;348(15):15034.
Sawka AM, Gerstein HC, Marriott MJ, MacQueen GM, Jofe RT. Does
a combination regimen of thyroxine (T4) and 3,5,3 -triiodothyronine
improve depressive symptoms better than T4 alone in patients with
hypothyroidism? Results of a double-blind, randomized, controlled
trial. J Clin Endocrinol Metab. 2003 Oct;88(10):45515.
Serra A, Sechi G, Singh S, Kumar A. Wernicke encephalopathy
afer obesity surgery: a systematic review. Neurology. 2007 Aug
7;69(6):615.
Singer PA, Cooper DS, Levy EG, Ladenson PW, Braverman LE, Daniels
G, et al; Standards of Care Committee, American Tyroid Association.
Treatment guidelines for patients with hyperthyroidism and hypothy-
roidism. JAMA. 1995 Mar 8;273(10):80812.
Stratton IM, Adler AI, Neil HA, Matthews DR, Manley SE, Cull CA,
et al. Association of glycaemia with macrovascular and microvascular
complications of type 2 diabetes (UKPDS 35): prospective observa-
tional study. BMJ. 2000 Aug 12;321(7258):40512.
Tfelt-Hansen J, Brown EM. Te calcium-sensing receptor in normal
physiology and pathophysiology: a review. Crit Rev Clin Lab Sci.
2005;42(1):3570.
Tohme JF, Bilezikian JP. Diagnosis and treatment of hypocalcemic emer-
gencies. Endocrinologist. 1996 Jan;6(1):1018.
Toth PP. Fibrate therapy in the management of diabetic dyslipidemia:
there is no ACCORD to be found. Curr Atheroscler Rep. 2010
Sep;12(5):3315.

premenopausal women: the A TO Z Weight Loss Study: a random-
ized trial. JAMA. 2007 Mar 7;297(9):96977. Erratum in: JAMA.
2007 Jul 11;298(2):178.
Gerstein HC, Yusuf S, Bosch J, Pogue J, Sheridan P, Dinccag N, et al;
DREAM (Diabetes REduction Assessment with ramipril and rosigli-
tazone Medication) Trial Investigators. Efect of rosiglitazone on the
frequency of diabetes in patients with impaired glucose tolerance
or impaired fasting glucose: a randomised controlled trial. Lancet.
2006 Sep 23;368(9541):1096105. Erratum in: Lancet. 2006 Nov
18;368(9549):1770.
Greenway SE, Greenway FL 3rd, Klein S. Efects of obesity surgery
on non-insulin-dependent diabetes mellitus. Arch Surg. 2002
Oct;137(10):110917.
Heath H 3rd. Familial benign (hypocalciuric) hypercalcemia: a trouble-
some mimic of mild primary hyperparathyroidism. Endocrinol Metab
Clin North Am. 1989 Sep;18(3):72340.
Jacobson TA. Beyond lipids: the role of omega-3 fatty acids from fsh oil in
the prevention of coronary heart disease. Curr Atheroscler Rep. 2007
Aug;9(2):14553.
Kloos RT, Eng C, Evans DB, Francis GL, Gagel RF, Gharib H, et al;
American Tyroid Association Guidelines Task Force. Medullary
thyroid cancer: management guidelines of the American Tyroid
Association. Tyroid. 2009 Jun;19(6):565612. Erratum in: Tyroid.
2009 Nov;19(11):1295.
Knowler WC, Barrett-Connor E, Fowler SE, Hamman RF, Lachin JM,
Walker EA, et al; Diabetes Prevention Program Research Group.
Reduction in the incidence of type 2 diabetes with lifestyle interven-
tion or metformin. N Engl J Med. 2002 Feb 7;346(6):393403.
Lien YH, Shapiro JI. Hyponatremia: clinical diagnosis and management.
Am J Med. 2007 Aug;120(8):6538.
Loke YK, Kwok CS, Singh S. Comparative cardiovascular efects of thi-
azolidinediones: systematic review and meta-analysis of observational
studies. BMJ. 2011 Mar 17;342:d1309.
Martin FI, Tress BW, Colman PG, Deam DR. Iodine-induced hyper-
thyroidism due to nonionic contrast radiography in the elderly. Am J
Med. 1993 Jul;95(1):7882.
Nathan DM, Buse JB, Davidson MB, Ferrannini E, Holman RR,
Sherwin R, et al; American Diabetes Association; European
79
not tender. Findings from mammography, breast ultra-
sonography, and computed tomography of the chest are
negative except for the presence of right axillary ade-
nopathy. Biopsy of a lymph node shows a moderately
diferentiated adenocarcinoma of unknown primary
origin. What should be the next step in evaluation of
this patient?
a. Perform a mediastinoscopy.
b. Assume that nonsmall cell carcinoma is present, and treat
with cisplatin-based chemotherapy.
c. Clarify the histogenetic origin of the tumor by testing for
tumor markers: carcinoembryonic antigen, cancer antigen
153, and neuron-specifc enolase.
d. Perform breast magnetic resonance imaging (MRI).
e. Recommend bilateral mastectomies.
4. A 38-year-old woman presents for intermittent abdom-
inal pain and bloating that has been getting worse for
the past several months. She has been reading medical
information on the Internet and is very concerned about
ovarian cancer. She has no family history of malig-
nancy and has been otherwise healthy. She is not tak-
ing any medications. Physical examination fndings are
remarkable only for some tenderness to movement of
the uterus. No pelvic masses are detected. She requests
a serum cancer antigen (CA) 125 test; the result is 86
U/mL (reference range <35 U/mL). How should you
advise her at this time?
a. Recommend combination chemotherapy with cisplatin and
paclitaxel.
b. Tell her that this degree of CA 125 elevation occurs only in
ovarian cancer.
c. Tell her that although the CA 125 elevation is concerning,
multiple conditions can cause such an elevation, and further
investigation is warranted.
d. Recommend exploratory laparotomy with total abdomi-
nal hysterectomy, bilateral salpingo-oophorectomy, pelvic
lymphadenectomy, omentectomy, and aggressive surgical
debulking of all disease.
e. Recommend only observation now, and recheck the CA 125
level in 3 months.
5. A 38-year-old woman with recently diagnosed
node-positive breast cancer presents to the emergency
department with a temperature of 38.3 C. She reports
having mild chills and fever but denies having nausea,
vomiting, diarrhea, cough, or dysuria. Seven days ago
she received her third cycle of doxorubicin and cyclo-
phosphamide chemotherapy; thus far, she has tolerated
QUESTI ONS
Multiple Choice (choose the best answer)
1. An otherwise healthy 32-year-old man asks you about
screening tests for colon cancer. He reports that his
2 older sisters had colon cancer at age 40 and 42, his
mother had colon cancer at age 40 and endometrial
cancer at age 45, and a maternal aunt had breast cancer
at a young age. He notes that there was not a history of
colon polyps in any family member. What should you
recommend for this patient?
a. Annual fecal occult blood testing
b. Colonoscopy now and every 1 to 2 years thereafer
c. Colonoscopy with random biopsies to look for infamma-
tory bowel diseaseif not present, follow routine screening
recommendations for average-risk Americans
d. Prophylactic colectomy
e. Reassurance only, since no polyps were found in family
members
2. A 72-year-old male smoker with a 42-pack-year history
presents with anorexia, cough, and altered mental sta-
tus. A chest radiograph shows a right-sided mass that, on
bronchoscopy, is identifed as squamous cell carcinoma.
On physical examination, the patient is thin, cachec-
tic, dehydrated, and disoriented with no focal neuro-
logic defcits. Te calcium level is elevated (15 mg/dL),
creatinine is 2.5 mg/dL, and albumin is 2.2 g/dL.
A bone scan shows only some degenerative changes.
What is the most appropriate next step in the manage-
ment of this patient?
a. Cisplatin-based chemotherapy
b. Radiotherapy to the brain
c. Intravenous fuids and bisphosphonates
d. Dexamethasone 100 mg given as an intravenous push
e. Emergent magnetic resonance imaging of the head
3. A 55-year-old woman with a history of hypertension, a
30-pack-year history of smoking, mild chronic obstruc-
tive pulmonary disease, and moderate obesity presents
with a right axillary mass. She has a family history of
coronary artery disease and strokes. Current medica-
tions include a statin, a diuretic, and a -blocker. On
examination, she is moderately obese and in no dis-
tress. Her lungs have increased sound in the expiratory
phase difusely but no frank wheezing or other sounds.
Findings from examination of the heart, abdomen, and
breasts are unremarkable. On lymph node examina-
tion, a palpable right axillary mass is frm, mobile, and
8.
ONCOLOGY QUESTIONS AND ANSWERS
80

MAYO CLINI C INTERNAL MEDI CINE BOARD REVIEW: QUESTI ONS AND ANSWERS
a. He is at increased risk for secondary cancers of the bladder,
prostate, and rectum, but the risk is only about 1 in 70 at 10
years.
b. Tere is no need to worry since he has no increased risk for
secondary malignancies.
c. He is at high risk for lung cancer in the future, so he should
have routine screening chest radiographs.
d. He is right to worry since secondary cancers are very com-
mon, and screening should be done at regular intervals.
e. Tere is no longer a reason to screen for prostate cancer since
he has had radiotherapy to this area.
7. An 82-year-old man comes to your of ce for rou-
tine follow-up care. He has a prior history of chronic
obstructive pulmonary disease with a forced expiratory
volume in 1 second of 25% of the predicted value, coro-
nary artery disease with mild congestive heart failure,
hypertension, and type 2 diabetes mellitus. He denies
having any urinary symptoms. He takes the following
medication: enalapril 5 mg twice daily, hydrochloro-
thiazide 25 mg twice daily, lovastatin 20 mg once daily,
albuterol inhaler as needed, futicasone propionate 250
mcg and salmeterol 50 mcg inhalation powder daily,
aspirin 325 mg daily, glipizide 10 mg twice daily, and a
multivitamin daily. On examination, he has poor breath
sounds in all areas, distant heart tones, a normal abdo-
men, and edema (2+) of the lower extremities bilater-
ally. On rectal examination, he has an enlarged prostate
with a frm nodule. Te prostate-specifc antigen level is
8.5 ng/mL, and transrectal needle biopsy shows adeno-
carcinoma (Gleason grade 6). A bone scan shows only
some changes consistent with degenerative disease.
What can you tell him at this time?
a. Radical prostatectomy is likely to improve his overall sur-
vival and decrease his chance of death from prostate cancer.
b. Given his lack of symptoms from prostate cancer, combined
with his age and comorbid conditions, a watchful waiting
approach is reasonable.
c. External beam radiotherapy is not efective against prostate
cancer.
d. Chemotherapy can be used to decrease his risk of recurrence
of prostate cancer.
e. Orchiectomy is the standard of care.
the cycles well. On physical examination, she is pleasant
and appears fatigued but in no distress, with the follow-
ing fndings: blood pressure 122/78 mm Hg, pulse 82
beats per minute and regular, respiration rate 14 breaths
per minute, and temperature 38.2 C. Te remainder of
the examination is remarkable only for alopecia. A chest
radiograph is clear of abnormalities. Urinalysis shows
no leukocytes. Laboratory data include the following:
hemoglobin 11.4 g/dL, leukocyte count 0.8 10
9
/L,
absolute neutrophil count 0.25 10
9
/L, and platelet
count 90 10
9
/L. She lives in town with her husband
and 2 children (aged 10 and 14). No one else is ill at
home. At this time, what should you do?
a. Admit her to the hospital to receive broad-spectrum
antibiotics.
b. Administer granulocyte colony-stimulating factor now.
c. Send her home, and ask her to follow up with her oncologist
in the morning.
d. Collect blood and urine samples for cultures, begin therapy
with amoxicillinclavulanate potassium and ciprofoxacin
orally, discharge to home, and ask her to follow up with her
oncologist by telephone within 24 hours.
e. Obtain a throat swab for infuenza virus.
6. A 67-year-old man with a history of stage II rectal can-
cer was treated with resection and combined chemo-
therapy and radiotherapy 6 years ago. He has recovered
well from the operations and treatments; he still has
some rectal and bladder urgency but no incontinence.
He recently retired from his job as an of ce manager
and is physically active. His hypertension is well con-
trolled with a -blocker, and his cholesterol levels are
controlled with diet. Tere is no signifcant family his-
tory. His most recent follow-up colonoscopy was done
10 months ago and showed no evidence of recurrence
or other disease. He is a lifetime nonsmoker. On physi-
cal examination, he is thin and pleasant, and he appears
ft. General examination fndings are unremarkable.
On rectal examination, his prostate feels normal and
smooth without palpable masses. A stool sample is neg-
ative for heme. Liver function test results are normal,
the level of carcinoembryonic antigen (CEA) is within
the reference range, and the level of prostate-specifc
antigen is 1.4 ng/mL. He is concerned about late side
efects of his prior therapy, specifcally about the devel-
opment of new cancers. What should you tell him?
81
MRI for routine screening should be limited to high-risk
women since it has not been shown to be benefcial for
average-risk patients. Te detection of hormone receptors
in the pathologic specimen has diagnostic and therapeu-
tic implications. Serum tumor markers are rarely useful
diagnostic tools (with few exceptions). (See Chen et al and
Saslow et al in the Suggested Reading list.)
4. Answer c.
Further investigation is warranted to determine the cause
of her discomfort and the reason for her elevated CA
125 level. Tis nonspecifc serum marker can be elevated
in many benign conditions, such as endometriosis, preg-
nancy, menstruation, and peritonitis. Te positive pre-
dictive value of CA 125 for screening is only about 2% to
3%. Although very high levels (several hundred to several
thousand units per milliliter) typically occur only in ovar-
ian cancer, patients with endometriosis can have levels
around 200 U/mL with stage IV disease. Chemotherapy is
never indicated until there is tissue confrmation of disease.
Extensive resection would be indicated if ovarian carci-
noma were diagnosed, and this would typically be followed
by systemic chemotherapy. Since the patient is having sig-
nifcant symptoms, observation only is not warranted. (See
Partridge et al in the Suggested Reading list.)
5. Answer d.
Febrile neutropenia is common with many forms of che-
motherapy. Te vast majority of patients have negative
cultures. Patients who are medically stable, are able to
maintain oral intake, are reliable for close follow up, and
live near a medical facility can be safely treated with an
outpatient regimen. Afer neutropenia develops, there is
no role for administration of growth factors. Observation
alone is insuf cient since the low absolute neutrophil count
puts her at signifcant risk for sepsis. (See Behl et al and
Halfdanarson et al in the Suggested Reading list.)
6. Answer a.
Patients treated with pelvic radiotherapy for rectal or pros-
tate cancer are at increased risk for secondary malignancies
in the area, but this risk is low (estimated to be 1 in 125 at 5
years and 1 in 70 at 10 years). Patients should still undergo
screening for cancers that they are at risk for, as long as
their general health and other medical conditions warrant
screening. Tis patient does not have an increased risk for
lung cancer per se, but lung metastases are a common site
1. Answer b.
With his family history, this patient is at very high risk for
colon cancer. It is unlikely that he would have a hereditary
polyposis syndrome since no family member had polyps.
His family history is highly concerning for hereditary
nonpolyposis colorectal cancer (Lynch syndrome) because
multiple frst-degree relatives were afected at an early age
and because there is a family history of breast and endome-
trial cancer. Tis syndrome is associated with a defect in
mismatch repair enzymes and leads to microsatellite insta-
bility. Screening with fecal occult blood testing is not ade-
quately sensitive for patients at high riskor even normal
risk. Infammatory bowel disease does signifcantly increase
the risk of colon cancer, but nothing in the patients history
suggests that it is present. Prophylactic colectomy would
be a consideration only if testing is positive for the defec-
tive gene. (See Rex et al and Umar et al in the Suggested
Reading list.)
2. Answer c.
Te patient needs intravenous fuids and restoration of
intravascular volume along with bisphosphonates to cor-
rect the hypercalcemia. In the absence of any focal neuro-
logic defcits, it is unlikely that his disorientation is due to
metastatic disease; altered mental status is very common
with hypercalcemia. If correction of the hypercalcemia
reverses the altered mental status, central nervous system
imaging is not required. Cranial radiotherapy should be
given only afer metastatic disease is identifed in a patient
with nonsmall cell carcinoma. Dexamethasone is used to
decrease peritumoral edema from intracranial metastases
and is not indicated for this patient. Dexamethasone can
help to signifcantly correct hypercalcemia due to multiple
myeloma or lymphoma, but it is unlikely to correct hyper-
calcemia due to squamous cell carcinoma. (See Behl et al
and Halfdanarson et al in the Suggested Reading list.)
3. Answer d.
Women presenting with axillary lymph node metastases
of adenocarcinoma of unknown primary origin should
undergo thorough evaluation for breast cancer. Breast
MRI has greater sensitivity than mammography or ultra-
sonography, and among women who have occult adeno-
carcinoma in the axillary lymph nodes, MRI can detect a
primary lesion in up to 75% of patients. Although breast
MRI is helpful for evaluation of women with adenocar-
cinoma metastatic to axillary lymph nodes, use of breast
ANSWERS
82

MAYO CLINI C INTERNAL MEDI CINE BOARD REVIEW: QUESTI ONS AND ANSWERS
are typically reserved for patients with metastatic or symp-
tomatic disease. (See National Comprehensive Cancer
Network [NCCN] in the Suggested Reading list.)
SUGGESTED READING
Behl D, Hendrickson AW, Moynihan TJ. Oncologic emergencies. Crit
Care Clin. 2010 Jan;26(1):181205.
Chen C, Orel SG, Harris E, Schnall MD, Czerniecki BJ, Solin LJ.
Outcome afer treatment of patients with mammographically
occult, magnetic resonance imaging-detected breast cancer pre-
senting with axillary lymphadenopathy. Clin Breast Cancer. 2004
Apr;5(1):727.
Halfdanarson TR, Hogan WJ, Moynihan TJ. Oncologic emergencies:
diagnosis and treatment. Mayo Clin Proc. 2006 Jun;81(6):83548.
Erratum in: Mayo Clin Proc. 2006 Nov;81(11):1509.
National Comprehensive Cancer Network (NCCN) guidelines [Internet].
Fort Washington (PA): National Comprehensive Cancer Network;
c2012 [cited 2011 Mar 20]. Available from: www.nccn.org.
Partridge E, Kreimer AR, Greenlee RT, Williams C, Xu JL, Church
TR, et al; PLCO Project Team. Results from four rounds of ovar-
ian cancer screening in a randomized trial. Obstet Gynecol. 2009
Apr;113(4):77582.
Rex DK, Johnson DA, Anderson JC, Schoenfeld PS, Burke CA, Inadomi
JM; American College of Gastroenterology. American College of
Gastroenterology guidelines for colorectal cancer screening 2009 [cor-
rected]. Am J Gastroenterol. 2009 Mar;104(3):73950. Epub 2009
Feb 24. Erratum in: Am J Gastroenterol. 2009 Jun;104(6):1613.
Saslow D, Boetes C, Burke W, Harms S, Leach MO, Lehman CD, et al;
American Cancer Society Breast Cancer Advisory Group. American
Cancer Society guidelines for breast screening with MRI as an adjunct
to mammography. CA Cancer J Clin. 2007 Mar-Apr;57(2):7589.
Erratum in: CA Cancer J Clin. 2007 May-Jun;57(3):185.
Umar A, Boland CR, Terdiman JP, Syngal S, de la Chapelle A, Ruschof
J, et al. Revised Bethesda Guidelines for hereditary nonpolyposis col-
orectal cancer (Lynch syndrome) and microsatellite instability. J Natl
Cancer Inst. 2004 Feb 18;96(4):2618.

for recurrence of rectal cancer. Te pattern of recurrence
is diferent from that of colon cancer, which much more
commonly metastasizes to the liver before traveling to the
lung. Te venous drainage of the rectum is into the inferior
vena cava, while most of the colons venous drainage is to
the portal system. Routine follow-up for otherwise healthy
patients who have colorectal cancer includes the follow-
ing: 1) Evaluate with a history and physical examination
every 3 months for 2 years and then every 6 months for
a total of 5 years. 2) Perform a colonoscopy in 1 year. If
results are abnormal, perform another colonoscopy in 1
year; if results are normal, perform a colonoscopy as clini-
cally indicated. 3) Determine CEA levels every 3 months
for 2 years and then every 6 months for years 3 to 5. 4)
Consider computed tomography of the chest, abdomen,
and pelvis annually for 3 years if the patient is at high risk.
(See National Comprehensive Cancer Network [NCCN]
in the Suggested Reading list.)
7. Answer b.
For patients older than 65, especially those with signif-
cant comorbid conditions, it is unclear whether radical
prostatectomy improves the patients overall survival and
it certainly has adverse efects on quality of life. Patients
in good condition who are younger than 65 do seem to
have a survival advantage if treated with radical prosta-
tectomy instead of watchful waiting; however, this elderly
patient with multiple other illnesses is unlikely to beneft
from aggressive treatment. External beam radiotherapy or
brachytherapy would be a reasonable treatment option, but
each carries a risk of impotence, rectal injury, and inconti-
nence. Chemotherapy has no role except for patients with
metastatic disease. Orchiectomy or hormonal therapies
83
2. A 45-year-old woman is admitted to the surgical service
with severe arterial insuf ciency of the right second toe.
She has no prior medical history and takes no medica-
tions. Physical examination fndings are normal except
for mild splenomegaly and signs of early gangrene in the
right second toe. All pulses are full and equal through-
out. Diagnostic testing results are shown in Table 9.Q2.
QUESTI ONS
Multiple Choice (choose the best answer)
ANEMI AS AND MYELOI D MALI GNANCI ES
1. A 67-year-old man is evaluated for exertional dyspnea.
He recalls that 3 years ago he was told that he had ane-
mia. In reviewing his records, you note that at that time
his hemoglobin level was 9.5 g/dL and his hematocrit
was 33% with an increased mean corpuscular volume
(MCV); the remainder of his complete blood cell count
was normal. On physical examination, he had conjunc-
tival pallor, normal heart and lung fndings, no lymph-
adenopathy, no hepatomegaly or splenomegaly, and no
petechiae or ecchymoses. Diagnostic testing results are
shown in Table 9.Q1.
9.
HEMATOLOGY QUESTIONS AND ANSWERS
Table 9.Q1
COMPONENT FINDING
Hemoglobin, g/dL 7.5
Hematocrit, % 23
Mean corpuscular volume, f L 110 (reference range, 8698)
Leukocyte count, 10
9
/L
Neutrophils, %
Lymphocytes, %
Monocytes, %
Basophils, %
Eosinophils, %
2.1
20
70
6
3
1
Platelet count, 10
9
/L 64
Reticulocyte count, % of
erythrocytes
0.3 (reference range, 0.51.5)
Absolute reticulocyte
count, 10
9
/L
10.0 (reference range, 29.587.3)
Peripheral blood flm Dimorphic erythrocyte population
with pronounced macrocytes
Lactate dehydrogenase, U/L 150 (reference range, 140280)
Which of the following is the most likely explanation
for these fndings?
a. Acute myeloid leukemia (AML)
b. Vitamin B
12
defciency
c. Hemolytic anemia
d. Myelodysplastic syndrome (MDS)
e. Primary myelofbrosis
Table 9.Q2
COMPONENT FINDING
Hemoglobin, g/dL 13.2
Hematocrit, % 39
Leukocyte count, 10
9
/L
Segmented neutrophils, %
Band cells, %
Lymphocytes, %
Monocytes, %
Basophils, %
Eosinophils, %
15.5
78
4
20
5
2
1
Platelet count, 10
9
/L 1,300
Mean corpuscular volume, f L 88
Erythrocyte sedimentation
rate, mm/h
28
Leukocyte alkaline
phosphatase score
110 (reference range, 13130)
Serum ferritin Within reference range
Serum iron Within reference range
Serum total iron-binding
capacity
Within reference range
Peripheral blood flm Increased large platelets with
some clustering; leukocytes and
erythrocytes are unremarkable
Bone marrow aspiration and
biopsy
Increased cellularity with increased
and atypical megakaryocytes in
clusters; reticulin staining is normal
Chromosomal analysis Normal female karyotype (46XY)
Which of the following is the most likely diagnosis?
a. Essential thrombocythemia
b. Vasculitis
c. Philadelphia chromosomenegative chronic myeloid leuke-
mia (CML)
d. Primary myelofbrosis (PMF)
84

MAYO CLINI C INTERNAL MEDI CINE BOARD REVIEW: QUESTI ONS AND ANSWERS
A chest radiograph shows a right middle and upper
lobe air space infltrate. Te patient is given supplemen-
tal oxygen, adequate pain control, and intravenous anti-
biotics. Which of the following should you now order?
a. Hydroxyurea
b. Erythrocyte exchange transfusion
c. Plasma exchange
d. Anticoagulation with unfractionated heparin
e. Aggressive intravenous fuid hydration
5. A 70-year-old man presents with weakness of his right
arm and leg. His symptoms began yesterday and are
now resolved. He also reports a 6-month history of
recurrent headaches and fatigue. He is a nonsmoker.
His medical history is signifcant for high blood pres-
sure. His blood pressure is 167/88 mm Hg, his oxygen
saturation is 93% on room air, his face is plethoric, and
a right carotid bruit is heard. Other fndings on physi-
cal examination are normal. Diagnostic testing results
are shown in Table 9.Q5.
3. A 22-year-old man is admitted to the hospital for an
elective cholecystectomy. You are asked to see him
because he had anemia on preoperative testing. He tells
you that he has always been told by his physicians that
he has mild anemia; his medical history is otherwise
unremarkable. His vital signs are normal. His conjunc-
tivae are mildly icteric, and the spleen is palpable in the
lef upper quadrant. Findings on the remainder of the
physical examination are normal. Diagnostic testing
results are shown in Table 9.Q3.
Table 9.Q4
COMPONENT FINDING
Hemoglobin, g/dL 7.9
Hematocrit, % 25
Mean corpuscular volume, f L 80
Leukocyte count, 10
9
/L
Segmented neutrophils, %
Band cells, %
Lymphocytes, %
Monocytes, %
Basophils, %
Eosinophils, %
16.0
70
7
15
5
2
1
Platelet count, 10
9
/L 490
Creatinine, mg/dL 1.4
Peripheral blood flm Anisopoikilocytosis with multiple
sickle cells
Table 9.Q5
COMPONENT FINDING
Hemoglobin, g/dL 20.5
Hematocrit, % 58
Mean corpuscular volume, f L 88
Leukocyte count, 10
9
/L
Neutrophils, %
Lymphocytes, %
Monocytes, %
Basophils, %
12.5
83
12
3
2
Platelet count, 10
9
/L 600
Erythropoietin, mIU/mL <2 (reference range, 019)
Table 9.Q3
COMPONENT FINDING
Hemoglobin, g/dL 11.2
Hematocrit, % 34
Leukocyte count, 10
9
/L
Diferential count
9.0
Within reference ranges
Platelet count, 10
9
/L 295
Mean corpuscular volume, f L 89
Reticulocyte count, % 4
Absolute reticulocyte count, 10
9
/L 200 (reference range,
29.587.3)
Peripheral blood flm Polychromasia with numerous
microspherocytes
Which of the following tests would most likely help
confrm the diagnosis?
a. Hemoglobin electrophoresis
b. Osmotic fragility test
c. Direct and indirect antiglobulin (Coombs) tests
d. Bone marrow aspiration and biopsy
4. A 28-year-old black man with sickle cell disease pres-
ents to the emergency department with abdominal
pain, chest pain, and shortness of breath. His dyspnea
evolved over 36 hours afer a visit with his niece and
nephew. His history is signifcant for approximately
2 emergency department visits or hospital admissions
per year for painful crises. Tree years ago, he spent 4
weeks in the hospital afer an episode of acute chest
syndrome. He has been taking hydroxyurea but only
intermittently because of fnancial concerns. His pulse
is 116 beats per minute and regular, his blood pressure
is 138/76 mm Hg, his respiratory rate is 18 breaths per
minute, and his temperature is 38.3 C. Pulse oximetry
shows 91% oxygen saturation with room air and 93%
with 4 L of oxygen by nasal cannula. His lungs have
scattered inspiratory crackles in the right midlung feld.
His spleen is not palpable. Te remainder of the physi-
cal examination fndings are normal. Diagnostic testing
results are shown in Table 9.Q4.
9. HEMATOLOGY QUESTI ONS AND ANSWERS

85
Which of the following is the best interpretation of
these data?
a. Te hemolysis is predominantly intravascular.
b. Te bone marrow is not responding to the anemia.
c. Direct Coombs testing results should be positive.
d. Urine hemoglobin testing results should be positive.
7. A 58-year-old woman with active rheumatoid arthritis
presents with fatigue and joint pain. She received the
diagnosis of rheumatoid arthritis 5 years earlier and has
been taking prednisone 10 mg daily and methotrexate
with folate weekly. She has had chronic fatigue and ane-
mia. Her vital signs are normal. Her conjunctivae are
pale, and she has active synovitis afecting both knees,
her wrists, and elbows, with rheumatoid nodules on
the extensor surface of her right forearm. Te remain-
der of the physical examination fndings are normal.
Diagnostic testing results are shown in Table 9.Q7.
Carotid ultrasonography shows a 30% stenotic lesion in
the right carotid. Te patient is hospitalized and begins
antiplatelet therapy. Which of the following should you
order next?
a. JAK2 V617F mutation testing
b. Fluorescence in situ hybridization (FISH) for BCR-ABL
testing
c. Arterial blood gas analysis
d. Bone marrow aspiration and biopsy
6. A 42-year-old woman with a history of systemic lupus
erythematosus (SLE) presents with fatigue. She has been
receiving antitumor necrosis factor therapy and has
been managing the SLE well. However, she has recently
experienced worsening fatigue. Her vital signs are nor-
mal. Her face and conjunctivae are jaundiced, and she
has a fading butterfy rash on her face. Te spleen is pal-
pable on deep inspiration. Diagnostic testing results are
shown in Table 9.Q6, and the peripheral blood flm is
shown in Figure 9.Q6.
Table 9.Q6
COMPONENT FINDING
Hemoglobin, g/dL 7.8
Hematocrit, % 27
Mean corpuscular volume, f L 95
Leukocyte count, 10
9
/L
Diferential count
4.5
Within reference ranges
Platelet count, 10
9
/L 450
Reticulocyte count, % 4
Absolute reticulocyte count, 10
9
/L 170.0 (reference range,
29.587.3)
Erythrocyte sedimentation rate, mm/h 25
Lactate dehydrogenase, U/L 400 (reference range, 140280)
Total bilirubin, mg/dL 3.5 (reference range, 0.11.0)
Indirect bilirubin, mg/dL 3.0
Figure 9.Q6
Table 9.Q7
COMPONENT FINDING
Hemoglobin, g/dL 9.0
Hematocrit, % 30
Mean corpuscular volume, f L 80
Leukocyte count, 10
9
/L
Neutrophils, %
Lymphocytes, %
Monocytes, %
11.5
90
8
2
Platelet count, 10
9
/L 500
Erythrocyte sedimentation rate, mm/h 50
Erythropoietin, mIU/mL 15 (reference range, 019)
Which of the following laboratory fndings are consis-
tent with this condition?
a. Elevated hepcidin, elevated ferritin, elevated total
iron-binding capacity (TIBC), elevated serum iron
b. Elevated hepcidin, elevated ferritin, decreased TIBC, ele-
vated serum iron
c. Decreased hepcidin, elevated ferritin, decreased TIBC, ele-
vated serum iron
d. Elevated hepcidin, elevated ferritin, decreased TIBC, nor-
mal serum iron
e. Decreased hepcidin, elevated ferritin, elevated TIBC, nor-
mal serum iron
COAGULATI ON
8. A 62-year-old man underwent right total knee replace-
ment 8 days ago. Swelling has developed in his right
lower extremity, and Doppler ultrasonography confrms
the presence of a right superfcial femoral vein throm-
bosis. His current medications include oxycodone and
subcutaneous unfractionated heparin. Results of pre-
operative tests, including a complete blood cell count
86

MAYO CLINI C INTERNAL MEDI CINE BOARD REVIEW: QUESTI ONS AND ANSWERS
Food and Drug Administration (FDA) approval of dab-
igatran, which requires no monitoring, and he would
like a prescription for this new drug. Which of the fol-
lowing statements is true about the use of dabigatran in
atrial fbrillation compared with the well-managed use
of warfarin?
a. Switching to dabigatran would result in superior outcomes.
b. Switching to dabigatran would result in inferior outcomes.
c. Switching to dabigatran would provide no signifcant
beneft.
d. Dabigatran is FDA approved for postoperative thrombo-
prophylaxis for knee and hip replacement surgery.
e. Dabigatran is FDA approved as an anticoagulant for patients
who have received a mechanical heart valve.
12. A 22-year-old woman is brought to the emergency
department afer having 1 witnessed tonic-clonic sei-
zure. She had appeared confused for the preceding
few hours. On examination, she is febrile and appears
slightly confused; otherwise, neurologic and physical
examination fndings are normal. Laboratory testing
results are shown in Table 9.Q12, and the peripheral
blood smear is shown in Figure 9.Q12.
and liver and kidney function, were normal. Other lab-
oratory data include the following: hemoglobin 12.2
g/dL, leukocyte count 8.5 10
9
/L, and platelet count
60 10
9
/L. In addition to stopping the use of subcuta-
neous heparin, what is the next most appropriate step
in management of this patient?
a. Start low-molecular-weight heparin therapy.
b. Start intravenous therapeutic doses of heparin.
c. Start direct thrombin inhibitor therapy.
d. Start aspirin therapy.
9. A 45-year-old man presents with deep vein thrombosis
of the right femoral vein. Tree months ago, he received
a diagnosis of systemic lupus erythematosus (SLE). In
addition to confrming SLE, laboratory testing also
documented the presence of a lupus anticoagulant
(LAC). Tere is no family history of venous thrombo-
sis. Current medications include hydroxychloroquine.
Laboratory testing shows normal results for a complete
blood cell count and for tests of liver and kidney func-
tion. Special coagulation testing confrms the persis-
tence of an LAC. What is the most reasonable duration
of warfarin anticoagulation for this patient?
a. 3 months
b. 1 year
c. 6 months
d. Long-term
e. 6 weeks
10. A 20-year-old white woman has been admitted to the
hospital with pulmonary embolism. She has no chronic
illnesses and is receiving no medications except for com-
bination estrogen-progesterone birth control pills that
she started using approximately 1 year earlier. Results
were normal for a complete blood cell count, baseline
prothrombin time, activated partial thromboplastin
time (aPTT), and tests of kidney and liver function.
Te patient is currently receiving therapeutic doses of
intravenous unfractionated heparin, and her aPTT is
therapeutic at 72 seconds. A panel of thrombophilia
tests has been performed. Which of the following state-
ments about her thrombophilia test results is correct?
a. DNA-based testing for factor V Leiden and prothrombin
G20210A mutations are reliable.
b. Low antithrombin confrms a hereditary defciency state.
c. A positive result on lupus anticoagulant (LAC) testing con-
frms antiphospholipid antibody syndrome.
d. Low protein S confrms the presence of a hereditary def-
ciency state.
11. A 62-year-old man with chronic atrial fbrillation has
been treated with warfarin. He has no other chronic ill-
nesses and is receiving no other medications long-term
except for lipid-lowering agents. Results of his complete
blood cell count and tests of renal and kidney function
are normal. He checks his prothrombin time monthly
and has kept the international normalized ratio (INR)
within the therapeutic range (23) for the duration of
his therapy with warfarin. He has heard about recent US
Table 9.Q12
COMPONENT FINDING REFERENCE RANGE
Hemoglobin, g/dL 8 1215
Platelet count, 10
9
/L 50 150450
Leukocyte count, 10
9
/L 8 3.510.0
Creatinine, mg/dL 2.5 0.81.3
Figure 9.Q12
What is the most appropriate next step in
management?
a. Red blood cell transfusion
b. Platelet transfusion
c. Gamma globulin administration
d. Plasma exchange
9. HEMATOLOGY QUESTI ONS AND ANSWERS

87
16. An 80-year-old man is admitted to the hospital after
falling on an icy sidewalk and fracturing his hip.
He undergoes open reduction and internal fixation
of the fracture. At surgery, there does not appear to
be any bone disease at the fracture site. The patient
was previously asymptomatic. Physical examination
findings are otherwise unremarkable. Serum protein
electrophoresis and immunofixation show an IgM
monoclonal protein (0.3 g/dL). The complete blood
cell count and serum creatinine levels are normal.
Skeletal survey shows no additional bone defects.
Which of the following statements is true for this
patient?
a. He has multiple myeloma and requires treatment.
b. He has a lower risk of a clinically signifcant lymphocytic or
plasma cell malignancy than patients with an IgG monoclo-
nal protein.
c. He requires a radioisotope bone scan to evaluate his bone
integrity.
d. He requires regular follow-up and serial measurements of
his monoclonal protein level.
e. He has a 10% annual risk of multiple myeloma.
17. A 75-year-old African American man was seen last week
by his primary care physician for mild dyspnea. He has
also noted intermittent peripheral edema. During the
evaluation, an electrocardiogram showed low-voltage
QRS complexes in the limb leads. Te troponin T level
was elevated (0.07 ng/mL). Tis fnding suggested the
need for a coronary angiogram, which showed no sig-
nifcant coronary artery disease. An echocardiogram
showed difuse lef ventricular thickening with a granu-
lar texture to the myocardium and a septal thickness
of 2.5 cm (normal <1.1 cm). Te complete blood cell
count results were normal. Serum and urine protein
electrophoresis and immunofxation were unremark-
able. Serum free light chain levels were not increased.
What is the most likely diagnosis?
a. AA amyloidosis
b. Light chainrelated amyloidosis
c. Hypertrophic obstructive cardiomyopathy
d. Amyloidosis due to transthyretin deposition
e. Amyloidosis due to
2
-microglobulin deposition
18. A 55-year-old man presented to his primary care phy-
sician for evaluation of fatigue. He was previously
healthy with the exception of chronic musculoskeletal
low back pain, for which he occasionally takes non-
steroidal anti-infammatory drugs. On examination,
he is pale. Complete blood cell count results are as
follows: hemoglobin 8.3 g/dL, mean corpuscular vol-
ume 73 f L, leukocyte count 6.9 10
9
/L, and platelet
count 398 10
9
/L. Results of the fecal occult blood
test are positive. During upper and lower endoscopy, a
1.2 2.5-cm ulcerative lesion is noted in the lesser cur-
vature of the stomach. Te lesion is biopsied and identi-
fed as a MALT lymphoma. Which of the following is
characteristic of MALT lymphoma?
13. A 72-year-old man with chronic atrial fbrillation has
been receiving dabigatran 75 mg twice daily for the past
6 months. He has not had any thrombotic or hemor-
rhagic complications. He has a history of colon pol-
yps, for which he needs to undergo a colonoscopy with
possible polypectomy. Apart from an irregular pulse,
his physical examination fndings are normal. Results
were normal for a complete blood cell count and tests
of renal and liver function. Te calculated creatinine
clearance is 28 mL/min. For how long should dabiga-
tran use be discontinued before the colonoscopy?
a. No need to discontinue
b. 24 hours
c. 48 hours
d. 3 days
e. 7 days
LYMPHOI D MALI GNANCI ES
14. At her annual physical examination, an asymptomatic
68-year-old woman has lymphocytosis (32 10
9
/L)
with a normal hemoglobin level and platelet count.
On examination, she has 1-cm lymphadenopathy in the
cervical region and no palpable liver or spleen enlarge-
ment. A peripheral blood smear shows identically
appearing mature lymphocytes with smudge cells. Flow
cytometry of the peripheral blood lymphocytes shows
a monoclonal B population with dim expression of
light chain and CD20 that is positive for expression of
CD5, CD19, and CD23. Which of the following is the
best next step in her management?
a. Combination chemoimmunotherapy
b. Chlorambucil therapy
c. Allogeneic peripheral blood stem cell transplant
d. Combination monoclonal antibody therapy
e. Active monitoring for disease progression and complications
15. Ten years ago, a previously healthy 20-year-old woman
presented to her physician with a 2-month history of
pruritis, drenching night sweats, unintentional weight
loss, and nonproductive cough. On examination, she
had 2-cm cervical lymphadenopathy. A computed
tomographic scan showed a 12-cm-diameter anterior
mediastinal mass. An excisional biopsy of a cervical
lymph node showed nodular sclerosing Hodgkin lym-
phoma. Afer she was treated with ABVD (doxorubicin
[ A driamycin], b leomycin, v inblastine, and d acarbazine)
combination chemotherapy followed by involved feld
radiotherapy, the disease was in complete remission.
Now you see her for the frst time for an annual physical
examination. Te disease remains in complete remis-
sion. Compared to her peers, this patient is at increased
risk of which of the following conditions?
a. Breast cancer
b. Coronary artery disease
c. Hypothyroidism
d. Skin cancer
e. All of the above
88

MAYO CLINI C INTERNAL MEDI CINE BOARD REVIEW: QUESTI ONS AND ANSWERS
20. A 48-year-old man presents to the emergency depart-
ment with a 6-week history of progressively worsening
abdominal pain and night sweats. Physical examination
fndings were signifcant for palpable bilateral 2-cm
axillary lymph nodes and difuse abdominal tenderness
with no rebound or guarding. Computed tomography
of the abdomen and pelvis showed retroperitoneal and
mesenteric lymphadenopathy. Excisional biopsy of
an axillary node was positive for difuse, large B-cell
lymphoma. Positron emission tomography showed
fuorodeoxyglucose-avidity in the axillary, mesenteric,
and retroperitoneal lymph nodes. Results of the bone
marrow examination were normal. Which of the fol-
lowing is the best next step?
a. Combination therapy with rituximab, cyclophosphamide,
doxorubicin, vincristine, and prednisone (R-CHOP)
b. Observation
c. Combination therapy with cyclophosphamide, doxorubi-
cin, vincristine, and prednisone (CHOP)
d. Autologous stem cell transplant
e. Involved feld radiotherapy
a. Most cases are treated with anthracycline-based
chemotherapy.
b. It is caused by chronic stimulation with Chlamydophila
psittaci .
c. Radiotherapy is necessary in most cases.
d. It frequently undergoes transformation to a large-cell
lymphoma.
e. Te combination of amoxicillin, omeprazole, and clarithro-
mycin is the most appropriate frst-line treatment.
19. A 73-year-old woman presented to the emergency
department with new-onset back pain, confusion, and
constipation over the past week. Her past medical his-
tory is signifcant only for hypertension. On examina-
tion, she is slightly pale with slow cognition and point
tenderness over the lumbar spine. Plain flms of the
lumbar spine show osteolytic lesions in L2, L3, and L5.
Laboratory values are as follows: hemoglobin 9.3 g/dL,
leukocyte count 4.6 10
9
/L with a normal diferential
count, platelet count 230 10
9
/L, creatinine 1.6 mg/dL,
total calcium 13.1 mg/dL, albumin 3.6 g/dL, and total
protein 9.1 g/dL. What is the most likely diagnosis?
a. Metastatic breast cancer
b. Hydrochlorothiazide use
c. Multiple myeloma
d. Primary hyperparathyroidism
e. Milk alkali syndrome
89
hemolytic anemia, which produces positive Coombs test
results, can cause spherocytes as well; however, the history of
lifelong anemia makes this diagnosis unlikely. A hemoglobin
electrophoresis would help in diagnosing thalassemia or a
hemoglobinopathy; however, these conditions do not mani-
fest with microspherocytes on the peripheral blood flm.
Tere is no indication for a bone marrow biopsy since the
reticulocyte response is appropriate and no other cytopenias
are apparent. (See Gallagher in the Suggested Reading list.)
4. Answer b.
Te patient has acute chest syndrome, a sickle cell anemia
complication that is an indication for urgent red cell (not
plasma) exchange transfusion to decrease the hemoglobin
S level to less than 30% to 35%. Gentle fuid resuscitation
is appropriate (along with oxygen support and antibiot-
ics, since about one-third of acute chest syndrome events
are initiated by or associated with bacterial pneumonia).
Aggressive fuid resuscitation, leading to overhydration,
might cause pulmonary edema and worsen the oxygen-
ation. Pulmonary embolism is possible, but full anticoagu-
lation is not warranted until embolism is documented. Use
of hydroxyurea might have prevented this crisis, but it is of
no value for the acute condition. (See Vij and Machado in
the Suggested Reading list.)
5. Answer a.
Polycythemia may be secondary, as with erythropoietin-
mediated causes such as chronic hypoxemia, living at high
altitude, and high oxygen af nity hemoglobinopathies.
Polycythemia vera is a myeloproliferative neoplasm that
can manifest with arterial thrombosis secondary to hyper-
viscosity from the increased concentration of erythrocytes.
Te low erythropoietin rules out erythropoietin-mediated
causes, leaving the presumptive diagnosis of polycythemia
vera. With JAK2 V617F mutation testing of peripheral
blood, results are positive for approximately 90% of patients
who have polycythemia vera. FISH for BCR-ABL testing
would screen for chronic myeloid leukemia, which does not
manifest with polycythemia. Although bone marrow aspira-
tion and biopsy would be helpful, it is not immediately nec-
essary and could be considered later. (See Patnaik and Teferi
in the Suggested Reading list.)
6. Answer c.
Hematologic complications of SLE include anemia of
chronic disease, pure red cell aplasia, and warm autoimmune
1. Answer d.
MDS most commonly manifests as isolated macrocytic
anemia. MDS can evolve to include pancytopenia over
several years; the typical peripheral smear fndings include
a dimorphic erythrocyte population (microcytes and oval
macrocytes) with an overall prominent macrocytosis and
an MCV around 110 fL. Te chronicity of MDSin par-
ticular, anemia preceding the diagnosis of pancytopenia by
several yearsis in contrast to the typically acute manifes-
tation of AML, which is therefore an unlikely possibility
in this patient. Primary myelofbrosis, a myeloproliferative
neoplasm, causes fbrosis in the bone marrow, resulting
in extramedullary hematopoiesis and signifcant spleno-
megaly, and typically does not cause a macrocytic anemia.
Vitamin B
12
defciency can cause a megaloblastic anemia
and manifest with slowly evolving macrocytic anemia and
eventually pancytopenia, but the peripheral smear would
not show a dimorphic erythrocyte population. (See Teferi
and Vardiman in the Suggested Reading list.)
2. Answer a.
Extreme thrombocytosis may be reactive and occur with
severe iron defciency or infammatory states (with elevated
erythrocyte sedimentation rates) or afer splenectomy;
patients are typically asymptomatic. Clonal thrombocy-
tosis is related to a myeloproliferative neoplasm, which
usually causes splenomegaly. Typical bone marrow fnd-
ings include a hypercellular bone marrow with increased
atypical megakaryocytes in clusters. Essential throm-
bocythemia may cause extreme thrombocytosis (plate-
let count >1,000 10
9
/L); however, it can also occur less
commonly with polycythemia rubra vera (typically with
erythrocytosis), the cellular phase of PMF, or rarely CML.
Te normal karyotype makes CML much less likely since
it typically manifests with the Philadelphia chromosome
t(9;22). Increased reticulin fbrosis would have been seen
on the bone marrow biopsy if the patient had PMF. (See
Teferi in the Suggested Reading list.)
3. Answer b.
When a patient presents with premature gallstones, one
should consider whether they may be due to pigment gall-
stones from chronic hemolysis causing indirect hyperbili-
rubinemia. Te presence of microspherocytes is consistent
with hereditary spherocytosis, and the diagnostic test is
an osmotic fragility test, which identifes a congenital
membrane defect. Typically, acquired warm autoimmune
ANSWERS
90

MAYO CLINI C INTERNAL MEDI CINE BOARD REVIEW: QUESTI ONS AND ANSWERS
systemic embolism in patients with nonvalvular atrial
fbrillation.
12. Answer d.
Plasma exchange is the treatment of choice for thrombotic
thrombocytopenic purpura (TTP). Although red blood
cell transfusion may be indicated, it does not address the
underlying pathogenesis of TTP. Platelets are thought to
be contraindicated in TTP because of the theoretical pos-
sibility of worsening the TTP. Gamma globulin is inefec-
tive in increasing the platelet count in TTP.
13. Answer d.
Dabigatran is cleared through the kidneys. It has a pro-
longed half-life in patients who have a creatinine clearance
less than 30 mL/min compared with patients who have a
creatinine clearance greater than 30 mL/min.
14. Answer e.
Chronic lymphocytic leukemia (CLL) is a clonal lym-
phoproliferative disorder of mature lymphocytes. Te clin-
ical diagnosis requires a B-lymphocyte count of more than
5 10
9
/L. Peripheral blood smears typically show smudge
cells, which are lymphocytes that have broken during pro-
cessing of the slide. Te clinical course of CLL is chronic in
most patients. For those with early-stage disease, standard
practice is to withhold treatment until the disease is active
or progressive. However, patients need to be monitored
for disease progression, autoimmune complications, infec-
tions, and second cancers.
15. Answer e.
Hodgkin lymphoma therapy is curative in about 80% of
cases. However, there are late complications of therapy,
particularly in those treated before modern chemotherapy
and radiotherapy. At 15 years, the risk of death from other
causes surpasses that of risk of death from Hodgkin lym-
phoma. Patients are at higher risk of secondary malignan-
cies, cardiovascular disease, thyroid disorders, and infertility
than the general population. Many of these conditions can
be attributed to chemotherapy and radiotherapy.
16. Answer d.
Tis patient has monoclonal gammopathy of undetermined
signifcance (MGUS), the most common dysproteinemia.
In MGUS, the M protein level is typically less than 3 g/dL,
the bone marrow has less than 10% plasma cells, and the
hemoglobin, creatinine, calcium, and bone radiographs are
normal. Te risk of progression to a lymphocytic or plasma
cell malignancy is about 1% per year. Patients with an IgM
or IgA monoclonal protein are at higher risk of progression
than those with an IgG protein. Patients with MGUS need
to be observed.
17. Answer d.
Te patient has senile cardiac amyloidosis. Tis syndrome
is usually isolated to the heart with few clinically signifcant
hemolytic anemia (WAIHA). Te presentation and labo-
ratory data suggest hemolysis, and the blood smear shows
spherocytes. Tese fndings are consistent with WAIHA,
which causes extravascular hemolysis. Te reticulocytosis
suggests that the bone marrow response is adequate. In
intravascular hemolysis, the urine is positive for hemoglo-
bin. (See Packman in the Suggested Reading list.)
7. Answer d.
Rheumatoid arthritis is a chronic infammatory disorder
that may lead to anemia of chronic disease. Anemia of
chronic disease results from the efect of elevated cyto-
kines on hematopoiesis, including upregulation of hepci-
din, leading to increased ferritin from iron malutilization
and downregulation of ferroportin, the main iron export-
ing system. Transferrin is also downregulated, leading to
decreased TIBC and normal to decreased serum iron lev-
els. (See Weiss and Goodnough in the Suggested Reading
list.)
8. Answer c.
Te timing and degree of thrombocytopenia are consistent
with immune-mediated heparin-induced thrombocytope-
nia type II. Unfractioned heparin and low-molecular-weight
heparin are contraindicated. Aspirin would not be the sole
management agent for established thrombosis. Te most
appropriate step is to start a direct thrombin inhibitor.
9. Answer d.
Presentation with a vascular thrombosis and persistence
of a LAC for 12 weeks or more satisfes the criteria for an
antiphospholipid syndrome. Tis patient has a high risk
for recurrent venous thrombosis on discontinuing antico-
agulation; thus, long-term warfarin is recommended with
periodic reassessment for safety.
10. Answer a.
DNA-based testing is reliable for patients receiving hepa-
rin or warfarin and for patients who have acute thrombosis.
However, acute thrombosis and heparin can cause lower
antithrombin activity results, which should be verifed
at another time, when heparin and acute thrombosis are
not factors. A single positive test result for LAC does not
confrm antiphospholipid syndrome; follow-up testing at
12-week intervals is required to demonstrate persistence of
LAC. Acute thrombosis and estrogen use can lower pro-
tein S levels; thus, abnormally low results require follow-up
confrmation.
11. Answer c.
Among patients randomly assigned to receive dabigatran,
overall outcomes were noninferior when compared with
the well-managed use of warfarin, thus providing no sig-
nifcant advantages. Te group of patients that derived the
most beneft from dabigatran was the group with INRs
outside the recommended therapeutic range. Dabigatran
is FDA approved only to reduce the risk of stroke and
9. HEMATOLOGY QUESTI ONS AND ANSWERS

91
care. Rituximab is an anti-CD20 monoclonal antibody
that improves overall survival when added to CHOP che-
motherapy for aggressive B-cell lymphomas. For patients
whose disease relapses or is refractory, autologous stem
cell transplant is the standard therapy. Radiotherapy can
be used in combination with chemotherapy in early-stage
(I-IIA) nonbulky disease but is not standard therapy for
advanced disease.
SUGGESTED READING
Gallagher PG. Red cell membrane disorders. Hematology Am Soc
Hematol Educ Program. 2005:138.
Packman CH. Hemolytic anemia due to warm autoantibodies. Blood Rev.
2008 Jan;22(1):1731. Epub 2007 Sep 27.
Patnaik MM, Teferi A. Te complete evaluation of erythrocytosis: con-
genital and acquired. Leukemia. 2009 May;23(5):83444. Epub 2009
Mar 19.
Teferi A. Annual clinical updates in hematological malignancies: a con-
tinuing medical education series: polycythemia vera and essential
thrombocythemia: 2011 update on diagnosis, risk-stratifcation, and
management. Am J Hematol. 2011 Mar;86(3):292301.
Teferi A, Vardiman JW. Myelodysplastic syndromes. N Engl J Med. 2009
Nov 5;361(19):187285.
Vij R, Machado RF. Pulmonary complications of hemoglobinopathies.
Chest. 2010 Oct;138(4):97383.
Weiss G, Goodnough LT. Anemia of chronic disease. N Engl J Med. 2005
Mar 10;352(10):101123.
deposits elsewhere, and the echocardiographic fndings
are ofen out of proportion to the degree of symptoms.
Transthyretin is the protein causing the amyloid deposits;
most patients have wild-type transthyretin.
18. Answer e.
With combination antibiotic therapy, 70% of gastric
MALT lymphomas are cured. In cases refractory to anti-
biotics, tumors may carry the t(11;18) translocation, and
involved feld radiotherapy is efective. Combination
chemotherapy is reserved for advanced disease. Te
majority of cases are associated with Helicobacter pylori
infection.
19. Answer c.
Tis patient has multiple myeloma with evidence of
end-organ damage from the plasma cell proliferative dis-
order (hypercalcemia, renal failure, anemia, and osteolytic
bone lesions). Te other answer choices are possible causes
of hypercalcemia, but only multiple myeloma accounts for
all the presenting symptoms, including the elevated level of
total protein.
20. Answer a.
Tis patient has advanced-stage, difuse, large B-cell lym-
phoma, and R-CHOP chemotherapy is the standard of
This page intentionally left blank
93
Which of the following contributed to his
hyponatremia?
a. HCTZ
b. Decreased intravascular volume
c. Beer drinking
d. Chronic kidney disease
e. Syndrome of inappropriate secretion of antidiuretic
hormone
3. An 80-year-old woman who is a nursing home resident
with hypertension, multi-infarct dementia, dysphasia,
and atrial fbrillation was admitted because her men-
tal status changed. Physical examination fndings and
laboratory test results are shown in Table 10.Q3.
QUESTI ONS
Multiple Choice (choose the best answer)
ELECTROLYTE DI SORDERS
1. A 68-year-old man, a formerly heavy smoker, presents
with a blood-tinged cough and weakness. He uses an
ipratropium bromide inhaler. On physical examination,
his vital signs are normal, his jugular venous pressure is
normal, and he has no edema. His laboratory values are
as follows: sodium 125 mEq/L, potassium 3.6 mEq/L,
bicarbonate 25 mEq/L, serum urea nitrogen 7 mg/dL,
and serum creatinine 0.9 mg/dL. What should you
obtain next?
a. Serum osmolality value
b. Urine osmolality value
c. Tyrotropin and morning cortisol values
d. Computed tomographic (CT) scan of the chest
e. Magnetic resonance imaging of the brain
2. A 56-year-old man with a history of hypertension and
drinking alcoholic beverages (mostly beer) presents
with episodic confusion, weakness, and imbalance. For
the past 3 weeks, he has been taking hydrocholorothi-
azide (HCTZ) 25 mg once daily. On physical examina-
tion, his blood pressure is 103/50 mm Hg, his pulse is
110 beats per minute, his respiratory rate is 20 breaths
per minute, his jugular venous pressure is normal, the
liver edge is palpable, and he has no edema. Laboratory
test results are shown in Table 10.Q2.
10.
NEPHROLOGY QUESTIONS AND ANSWERS
Table 10.Q2
COMPONENT FINDING
Sodium, mEq/L 105
Potassium, mEq/L 3.2
Chloride, mEq/L 70
Bicarbonate, mEq/L 30
Serum creatinine, mg/dL 1.3
Serum osmolality, mOsm/kg 224
Urine osmolality, mOsm/kg 410
Tyrotropin Within reference range
Morning cortisol Within reference range
Table 10.Q3
COMPONENT FINDING
Blood pressure, mm Hg 106/58
Pulse, beats per minute 90 (irregular)
Respiratory rate, breaths per minute 24
Oxygen saturation with room air, % 97
Weight, kg 60
Mucosal surfaces Dry
Mental status Sleepy, oriented to name only
Lung felds Clear, no edema
Sodium, mEq/L 176
Potassium, mEq/L 3.5
Chloride, mEq/L 129
Bicarbonate, mEq/L 35
Serum creatinine, mg/dL 1.2
Urine osmolality, mOsm/kg 655
What is her estimated water defcit?
a. 6.5 L
b. 7.3 L
c. 9.3 L
d. 10.5 L
e. 2.4 L
94

MAYO CLINI C INTERNAL MEDI CINE BOARD REVIEW: QUESTI ONS AND ANSWERS
only medication she uses is artifcial tears. Physical
examination fndings and laboratory test results are
shown in Table 10.Q7. A computed tomographic scan
of the abdomen is shown in Figure 10.Q7.
4. For the patient in the preceding question, what is the
maximum rate of correction for her hypernatremia in
the next 24 hours?
a. 10 mEq/L daily
b. 12 mEq/L daily
c. 14 mEq/L daily
d. 16 mEq/L daily
e. 18 mEq/L daily
5. A 66-year-old man presents with a history of general-
ized weakness for 1 week. Otherwise, he has hyperten-
sion, diabetes mellitus, and obstructive sleep apnea (he
uses nocturnal continuous positive airway pressure).
He has been taking furosemide 20 mg twice daily for
3 years and metolazone 2.5 mg twice daily for 2 weeks.
Physical examination fndings and laboratory test
results are shown in Table 10.Q5.
Table 10.Q5
COMPONENT FINDING
Body mass index 36.5
Blood pressure, mm Hg 108/78
Pulse, beats per minute 84
Edema Absent
Sodium, mEq/L 134
Potassium, mEq/L 2.4
Bicarbonate, mEq/L 32
Chloride, mEq/L 92
Serum creatinine, mg/dL 1.5
Calcium, mg/dL 8.1
Glucose, mg/dL 600
Table 10.Q7
COMPONENT FINDING
Body mass index 19
Blood pressure, mm Hg 98/60
Pulse, beats per minute 98
Respiratory rate, breaths per minute 20
Edema Absent
Sodium, mEq/L 134
Potassium, mEq/L 2.3
Chloride, mEq/L 114
Bicarbonate, mEq/L 12
Serum creatinine, mg/dL 0.8
Anion gap, mEq/L 8
Arterial blood gas
pH
P co
2
, mm Hg
7.27
27
Urinalysis
pH
Protein
Glucose
Red blood cells per high-power feld
6.7
Trace
Negative
310
Which of the following did not contribute to this
patients hypokalemia?
a. Metolazone
b. Furosemide
c. Transcellular shif
d. High aldosterone caused by a reduced intravascular volume
e. Chronic kidney disease
6. To the patient in the preceding question, what should
be given next?
a. Insulin intravenously
b. Insulin subcutaneously
c. Potassium intravenously
d. Calcium intravenously
e. Potassium orally
ACI DBASE DI SORDERS
7. A 38-year-old woman who has had Sj gren syndrome
for 2 years presents with difuse muscle weakness. Te Figure 10.Q7
10. NEPHROLOGY QUESTI ONS AND ANSWERS

95
What is the most appropriate next step?
a. Calculate the osmolal gap.
b. Examine a urine specimen microscopically.
c. Initiate 4-methylpyrazole (fomepizole) therapy.
d. Initiate hemodialysis.
e. Do all of the above.
10. A 47-year-old woman with a history of substance abuse
was found unresponsive. Physical examination fndings
and laboratory test results are shown in Table 10.Q10.
What is the diagnosis?
a. Proximal renal tubular acidosis (RTA)
b. Distal RTA
c. Idiopathic nephrolithiasis
d. Hyporeninemic hypoaldosteronism
e. Gout
8. A 68-year-old woman presents with new-onset back
pain, weakness, episodic light-headedness, and a recent
spontaneous lef rib fracture. Physical examination
fndings and laboratory test results are shown in Table
10.Q8.
Table 10.Q8
COMPONENT FINDING
Blood pressure, mm Hg 98/50
Pulse, beats per minute 106
Respiratory rate, breaths per minute 20
Heart examination Unremarkable
Lung examination Unremarkable
Edema Extremities: pitting edema
(trace)
Hemoglobin, g/dL 10.5
Sodium, mEq/L 134
Potassium, mEq/L 3.4
Chloride, mEq/L 116
Bicarbonate, mEq/L 17
Serum creatinine, mg/dL 1.6
Phosphorus, mg/dL 2.0
Uric acid, mg/dL 1.9
Glucose, mg/dL 99
Arterial blood gas
pH
P co
2
, mm Hg
7.32
32
Urinalysis
pH
Protein
Glucose
Red blood cells per high-power feld
5.1
1+
2+
13
What is the diagnosis?
a. Proximal renal tubular acidosis (RTA)
b. Distal RTA
c. Drug-induced diarrhea
d. Hypoaldosteronism
e. Multiple myeloma
9. A 39-year-old man with a history of polysubstance abuse
was found confused. Physical examination fndings and
laboratory test results are shown in Table 10.Q9.
Table 10.Q9
COMPONENT FINDING
Mental status Incoherent
Blood pressure, mm Hg 106/70
Heart rate, beats per minute 110
Respiratory rate, breaths per minute 18
Edema Absent
Sodium, mEq/L 146
Potassium, mEq/L 5.0
Chloride, mEq/L 100
Bicarbonate, mEq/L 8
Glucose, mg/dL 110
Serum urea nitrogen, mg/dL 28
Serum creatinine, mg/dL 1.2
Arterial blood gas
pH
P co
2
, mm Hg
7.20
20
Serum osmolality, mOsm/kg 378
Toxicity screen Pending
Table 10.Q10
COMPONENT FINDING
Blood pressure, mm Hg 136/92
Heart rate, beats per minute 106
Respiratory rate, breaths per minute 18
Edema Absent
Sodium, mEq/L 144
Potassium, mEq/L 5.1
Chloride, mEq/L 110
Bicarbonate, mEq/L 6
Glucose, mg/dL 89
Serum urea nitrogen, mg/dL 17
(continued)
96

MAYO CLINI C INTERNAL MEDI CINE BOARD REVIEW: QUESTI ONS AND ANSWERS
12. A 70-year-old man who receives nocturnal oxygen
for severe chronic obstructive pulmonary disease was
admitted for acute pneumonia. He has some nausea but
denied vomiting or having diarrhea. Physical examina-
tion fndings and laboratory test results are shown in
Table 10.Q12.
What is the most appropriate next step?
a. Calculate the osmolal gap.
b. Examine a urine specimen microscopically.
c. Initiate 4-methylpyrazole (fomepizole) therapy.
d. Initiate hemodialysis.
e. Do all of the above.
11. A 47-year-old man who has a history of polysubstance
abuse presents with delirium. Reportedly, he ingested
an unknown substance. Physical examination fndings
and laboratory test results are shown in Table 10.Q11.
Table 10.Q11
COMPONENT FINDING
Mental status Disoriented
Blood pressure, mm Hg 110/70
Pulse, beats per minute 110 (normal heart rhythm)
Respiratory rate, breaths per minute 34
Lung felds Clear
Edema Absent
Sodium, mEq/L 138
Potassium, mEq/L 3.8
Bicarbonate, mEq/L 14
Chloride, mEq/L 100
Serum urea nitrogen, mg/dL 20
Serum creatinine, mg/dL 1.5
Arterial blood gas
pH
P co
2
, mm Hg
P o
2
, mm Hg
7.43
20
85
Table 10.Q12
COMPONENT FINDING
Mental status Somnolent
Blood pressure, mm Hg 110/70
Pulse, beats per minute 90
Respiratory rate, breaths per minute 14
Edema Dependent (1+)
Bicarbonate, mEq/L 32
Arterial blood gas
pH
P co
2
, mm Hg
P o
2
, mm Hg
7.29
70
40
What is the patients acid-base status?
a. Acute respiratory acidosis
b. Chronic respiratory acidosis
c. Acute and chronic respiratory acidosis
d. Acute respiratory acidosis and metabolic alkalosis
e. Chronic respiratory acidosis and metabolic acidosis
13. An 18-year-old woman with no past medical history
reports having nausea and occasional vomiting in the
morning for about 2 months. She denies having diar-
rhea. Physical examination fndings and laboratory test
results are shown in Table 10.Q13.
Table 10.Q13
COMPONENT FINDING
Blood pressure, mm Hg 98/60
Heart rate, beats per minute 92
Respiratory rate, breaths per minute 16
Sodium, mEq/L 134
Potassium, mEq/L 4.0
Chloride, mEq/L 108
Bicarbonate, mEq/L 20
Serum creatinine, mg/dL 0.4
Glucose, mg/dL 88
Arterial blood gas
pH
P co
2
, mm Hg
P o
2
, mm Hg
7.45
30
99
COMPONENT FINDING
Serum creatinine, mg/dL 1.2
Arterial blood gas
pH
P co
2
, mm Hg
6.80
32
Serum osmolality, mOsm/kg 461
Anion gap, mEq/L 28
Table 10.Q10 (CONTINUED)
What is the patients acid-base status?
a. Metabolic acidosis with respiratory compensation
b. Metabolic acidosis and respiratory acidosis
c. Metabolic acidosis and respiratory alkalosis
d. Acute respiratory alkalosis with appropriate metabolic
compensation
e. Chronic respiratory alkalosis and metabolic compensation
10. NEPHROLOGY QUESTI ONS AND ANSWERS

97
15. A 54-year-old woman who had been previously healthy
is evaluated for a rash on her lower extremities that has
been present for 1 week. She has noticed tea-colored
urine for several weeks. She takes no medications. Her
temperature is 37.3 C, her pulse is 88 beats per minute,
and her blood pressure is 154/90 mm Hg. Palpable pur-
pura is present on both legs and feet. Te remainder of
the examination fndings are unremarkable. Results of
laboratory studies are notable for creatinine 1.8 mg/
dL. Te erythrocyte sedimentation rate is 80 mm/h.
Antinuclear antibody, antibodies to double-stranded
DNA, myeloperoxidase, and proteinase 3 assays are
negative. Te C4 complement level is low, and the
results of cryoglobulin testing are positive. Urinalysis
shows proteinuria (2+) and hematuria (3+). Urine
microscopy shows 31 to 40 erythrocytes per high-power
feld (HPF) and 3 to 10 leukocytes per HPF. Which of
the following viruses is most likely to be associated with
this disorder?
a. Epstein-Barr virus
b. Cytomegalovirus
c. Human immunodefciency virus
d. Parvovirus B19
e. Hepatitis C virus
What is the patients acid-base status?
a. Acute respiratory alkalosis
b. Chronic respiratory alkalosis
c. Acute and chronic respiratory alkalosis
d. Metabolic acidosis and acute respiratory alkalosis
e. Metabolic acidosis and chronic respiratory alkalosis
ACUTE RENAL FAI LURE
14. A 73-year-old man who has a history of hypertension
and metastatic prostate cancer presents with dizziness
and weakness. He has no known history of kidney dis-
ease; his creatinine was 1.2 mg/dL 2 months ago. His
medications include lisinopril-hydrochlorothiazide (20
mg/12.5 mg daily), multivitamin daily, tramadol (50 mg
23 times daily as needed for pain), and leuprolide (30
mg intramuscularly every 4 months). His blood pressure
is 130/86 mm Hg, his pulse is 90 beats per minute, and
his temperature is 36.9 C. On examination, he appears
fatigued, heart sounds are normal, lungs are clear, and
there is pretibial edema (trace). Laboratory test results
are shown in Table 10.Q14. Renal ultrasonography
shows no evidence of hydronephrosis.
Table 10.Q14
COMPONENT FINDING
Hemoglobin, g/dL 11.1
Leukocyte count, 10
9
/L 7.5
Platelet count, 10
9
/L 187
Sodium, mEq/L 142
Potassium, mEq/L 5.2
Bicarbonate, mEq/L 22
Chloride, mEq/L 105
Serum creatinine, mg/dL 3.2
Serum urea nitrogen, mg/dL 50
Calcium, mg/dL 13.1
Albumin, g/dL 3.9
Phosphorus, mg/dL 3.3
Urinalysis
Protein
Microscopic examination
Sodium, mEq/L
Creatinine, mg/dL
1+
Occasional granular casts
68
31
What is the most likely cause of this patients acute renal
failure?
a. Membranous nephropathy
b. Acute tubular necrosis
c. Dehydration
d. Obstruction
e. Tumor lysis syndrome
Table 10.Q16
COMPONENT FINDING
Blood pressure, mm Hg 164/94
Pulse, beats per minute 76
Weight, kg 92
Height, cm 185
(continued)
GLOMERULAR DI SEASE
16. A 51-year-old truck driver is referred for evaluation of
persistent asymptomatic microhematuria. He has not
seen a physician since he was frst told about blood
in his urine during a Department of Transportation
physical examination 9 years ago. At that time, he had
a computed tomographic scan of the abdomen and pel-
vis, cystoscopy with retrograde pyelograms, and urine
cytology. He was told last year that his blood glucose
level was elevated. He has never smoked and takes no
medications other than ibuprofen 400 mg approxi-
mately twice monthly for headaches. Physical examina-
tion fndings and laboratory test results are shown in
Table 10.Q16. Results of erythrocyte sedimentation
rate, antinuclear antibody testing, testing for antibod-
ies to myeloperoxidase and proteinase 3, serum pro-
tein electrophoresis, and hepatitis B and C and human
immunodefciency serologies all are negative or normal.
Renal biopsy is performed.
98

MAYO CLINI C INTERNAL MEDI CINE BOARD REVIEW: QUESTI ONS AND ANSWERS
What is the most likely cause of her renal failure?
a. Rhabdomyolysis
b. Acute interstitial nephritis
c. Trombotic thrombocytopenic purpura (TTP)
d. Membranoproliferative glomerulonephritis (MPGN)
e. Antiglomerular basement membrane nephritis
18. Oliguric acute renal failure developed in a 72-year-old
man afer lef femoral-popliteal bypass surgery. He
had a computed tomographic angiogram of the lower
extremity arteries 5 days ago and underwent lef
femoral-popliteal bypass surgery for critical ischemia
of the lef foot 3 days ago. During the procedure, his
systolic blood pressure decreased to 80 to 89 mm Hg
several times. Over the past 24 hours, his urine output
through the indwelling bladder catheter was 200 mL
despite intravenous fuid administration; he has a posi-
tive fuid balance of 4 L postoperatively. His current
medications include insulin, metoprolol, aspirin, sim-
vastatin, and fentanyl patient-controlled analgesia. In
addition to peripheral vascular disease, his past medi-
cal history is signifcant for type 2 diabetes mellitus for
8 years, hypertension, hyperlipidemia, and coronary
artery disease. Baseline creatinine was 1.1 mg/dL last
month, and he had a urine microalbumin to creatinine
ratio of 28 mg/g (reference range <17 mg/g) at that
time. His blood pressure is 107/60 mm Hg, his pulse
is 68 beats per minute, and his temperature is 37.1 C.
On physical examination, he has a regular cardiac rate
and rhythm and a grade 3/6 systolic ejection murmur
at the right upper sternal border without a rub or gal-
lop, there are difuse pulmonary crackles bilaterally,
and the abdomen is sof and nontender. Both lower
extremities have pitting edema (2+). Pedal pulses are
diminished bilaterally. His creatinine was 1.2 mg/dL
on postoperative day 1 and 3.2 mg/dL on postoperative
day 3. Urine Gram staining is negative, and the results
of the urinalysis with microscopy are pending. Which
of the following sets of fndings on urinalysis and urine
microscopy would be most characteristic of this acute
presentation?
What is the most likely diagnosis?
a. Minimal change nephropathy
b. Membranous nephropathy
c. Chronic interstitial nephritis
d. Diabetic nephropathy
e. IgA nephropathy
17. A 34-year-old woman presents to the emergency depart-
ment with difuse myalgias. Her past medical history is
signifcant for arthroscopic knee surgery 3 years ago.
She has no history of recent trauma. She admits to using
cocaine regularly. She takes an oral contraceptive tablet
daily and ibuprofen 400 mg 2 to 3 times daily as needed
for pain, most recently this morning. On auscultation,
the heart rhythm is regular with no murmur, rub, or
gallop, and the lungs are clear. Findings on abdominal
examination are normal. Tere is difuse tenderness in
both upper and lower extremities without any ecchy-
moses or rashes. Ankle edema (trace) is present bilat-
erally. Additional physical examination fndings and
laboratory test results are shown in Table 10.Q17.
COMPONENT FINDING
Heart, lungs, and abdomen Normal
Jugular venous distention Absent
Pitting edema Both lower extremities (trace)
Rashes Absent
Serum creatinine, mg/dL 2.1
Fasting blood glucose, mg/dL 130
Spot urine microalbumin, mg/g 1,586
Urinalysis
Blood
Protein
24-h total protein, g
3+
3+
2.1
Table 10.Q16 (CONTINUED) Table 10.Q17 (CONTINUED)
Table 10.Q17
COMPONENT FINDING
Blood pressure, mm Hg 158/90
Pulse, beats per minute 100
Temperature, C 37.5
Hemoglobin, g/dL 12.1
Leukocyte count, 10
9
/L 8.3
Platelet count, 10
9
/L 189
Sodium, mEq/L 137
Potassium, mEq/L 5.8
Bicarbonate, mEq/L 17
COMPONENT FINDING
Chloride, mEq/L 108
Serum creatinine, mg/dL 2.8
Serum urea nitrogen, mg/dL 39
Creatine kinase, U/L 12,870
Urinalysis
Color
Blood
Protein
Leukocytes
Microscopic examination
Brown
3+
2+
13
Granular casts, epithelial casts,
renal epithelial cells (1120)
(continued)
10. NEPHROLOGY QUESTI ONS AND ANSWERS

99
is 90 beats per minute, his respiratory rate is 16 breaths
per minute, his temperature is 37.4 C, and he weighs 87
kg (body mass index 25.4). He appears fatigued and is
in no acute distress. His skin is pale with scattered red-
dish lesions on the lower legs. Te heart rate is regular
with a grade 2/6 systolic ejection murmur. Scattered
pulmonary crackles are present. Tere is no hepatosple-
nomegaly, and the abdomen is nontender with normal
bowel sounds. Bilateral pretibial edema (1+) is present.
Laboratory test results are shown in Table 10.Q20. Chest
radiography shows multiple small bilateral pulmonary
nodules that appear slightly smaller than 3 months ago.
a. Normal urinalysis; occasional hyaline casts
b. Protein (4+); occasional free fat
c. Protein (2+); blood (3+); 41 to 50 red blood cells; occa-
sional red blood cell casts
d. Protein (trace); blood (trace); positive nitrite and positive
leukocyte esterase tests; 1 to 3 red blood cells; 51 to 100 leu-
kocytes; occasional leukocyte casts
e. Protein (1+); less than 3 red blood cells; 4 to 10 renal epithe-
lial cells
19. A 70-year-old man is found to have a serum creatinine
level of 3.2 mg/dL while undergoing an evaluation for
bilateral lower extremity edema that had been present
for approximately 2 weeks. He has a history of hyperten-
sion, hyperlipidemia, mitral regurgitation, and Barrett
esophagus. Neither the patient nor any family members
are known to have kidney disease. Other than the edema,
he has had no symptoms and denies having hematuria,
decreased urine output, increased urinary frequency,
hesitancy, or urgency. His medications are metoprolol,
simvastatin, omeprazole, and aspirin. His blood pressure
is 136/80 mm Hg, his pulse is 62 beats per minute, his
temperature is 37.2 C, his respiratory rate is 14 breaths
per minute, and his weight is 78.9 kg. He does not appear
ill. His skin turgor is normal. His heart rate and rhythm
are regular. A 2/6 holosystolic murmur is present at
the apex, and there is no rub or gallop. His lungs are
clear, and abdominal examination fndings are normal.
Tere is pitting pretibial edema (1+ to 2+) bilaterally.
Laboratory test results are as follows: serum creatinine
3.2 mg/dL, serum urea nitrogen 54 mg/dL, hemoglo-
bin 12.8 g/dL, leukocyte count 9.8 10
9
/L, and platelet
count 204 10
9
/L. Urinalysis shows protein (1+), posi-
tive results for leukocyte esterase, 31 to 40 leukocytes
per high-power feld, and negative Gram staining. Renal
ultrasonography shows increased echogenicity bilater-
ally and no evidence of hydronephrosis. Venous Doppler
ultrasonography of the lower extremities is negative for
deep vein thrombosis. Which of the following interven-
tions is most likely to improve this patients acute renal
failure by addressing the underlying cause?
a. Initiate subcutaneous low-molecular-weight heparin therapy.
b. Discontinue the use of omeprazole.
c. Insert an indwelling urinary catheter.
d. Decrease the metoprolol dose.
e. Start ciprofoxacin therapy with 250 mg twice daily.
20. A 74-year-old man presents with a 1-week history of
weakness and fatigue. Two years ago, he underwent right
nephrectomy for renal cell carcinoma. Tree months
ago, pulmonary metastases were diagnosed, and therapy
with sunitinib was initiated. Other past medical his-
tory includes hypertension and stage 3 chronic kidney
disease with baseline creatinine 1.6 mg/dL (estimated
glomerular fltration rate 42 mL/min per 1.73 m
2
). His
medications are amlodipine 10 mg daily, furosemide 20
mg daily, and sunitinib 50 mg daily for the past 3 weeks
(following 2 weeks without sunitinib afer the previous
cycle). His blood pressure is 178/96 mm Hg, his pulse
Table 10.Q20
COMPONENT FINDING
Hemoglobin, g/dL 6.8
Leukocyte count, 10
9
/L 8.7
Platelet count, 10
9
/L 37
Sodium, mEq/L 141
Potassium, mEq/L 5.6
Chloride, mEq/L 108
Bicarbonate, mEq/L 21
Serum creatinine, mg/dL 4.7
Serum urea nitrogen, mg/dL 62
Calcium, mg/dL 9.7
Phosphorus, mg/dL 4.9
Albumin, g/dL 3.2
Lactate dehydrogenase, U/L 540
Which of the following is the most likely cause of this
patients acute renal failure?
a. Trombotic microangiopathy
b. Acute tubular necrosis
c. Atheroemboli
d. Microscopic polyangiitis
e. Goodpasture syndrome
CHRONI C RENAL FAI LURE
21. A 56-year-old woman who is doing well is seeing you for
her annual checkup. Her medical history is signifcant
for type 2 diabetes mellitus for 20 years, hypertension,
and diabetic neuropathy. On physical examination, she
is obese and has decreased sensation in her feet. Other
physical examination fndings and laboratory test
results are shown in Table 10.Q21. Her current medi-
cations are metformin 1,000 mg twice daily, simvasta-
tin 40 mg daily, gabapentin 300 mg 3 times daily, and
atenolol 50 mg twice daily.
100

MAYO CLINI C INTERNAL MEDI CINE BOARD REVIEW: QUESTI ONS AND ANSWERS
What should you recommend?
a. Exchange atenolol with metoprolol.
b. Increase the dosage of lisinopril to 80 mg daily.
c. Add losartan 100 mg daily.
d. Increase the dosage of allopurinol to 600 mg daily.
e. Add metformin 1,000 mg twice daily.
23. A 58-year-old man has lower extremity swelling,
decreased stamina, and fatigue. He is obese and has a
15-year history of type 2 diabetes mellitus. His other
medical problems include hyperlipidemia and recur-
rent gout attacks. He weighs 147 kg, he is 176 cm tall,
his blood pressure is 156/78 mm Hg, and his heart
rate is 78 beats per minute. Laboratory test values are
as follows: serum creatinine 2.3 mg/dL, hemoglobin
A
1c
8.6%, and urine albumin to creatinine ratio 2,600
mg/g. He takes losartan 25 mg daily, metformin 1,000
mg twice daily, allopurinol 100 mg daily, simvastatin
20 mg daily, and aspirin 81 mg daily. What therapeutic
intervention should you recommend that is efective in
treating patients with diabetic kidney disease and can
prevent its progression?
a. Increase the losartan dosage to 50 mg daily and monitor
serum potassium and creatinine.
b. Add hydrochlorothiazide 12.5 mg daily.
c. Stop the use of metformin and start insulin therapy.
d. Initiate dietary modifcation with protein and salt
restriction.
e. All of the above would be efective.
24. A 72-year-old man presents with acute chest pain. He
has stage 3 chronic kidney disease (CKD) and a 20-year
history of type 2 diabetes mellitus. He is a smoker and
has hyperlipidemia and degenerative joint disease. His
blood pressure is 165/85 mm Hg, and his heart rate
is 110 beats per minute. His troponin level is elevated.
His electrocardiogram shows ST-segment elevation.
Te patient is being transported to undergo coronary
angiography. What should you recommend be done
next?
a. No treatment is indicated; proceed with coronary
angiography.
b. Start therapy with N -acetylcysteine 1,200 mg twice daily
with 1 L of oral hydration.
c. Start therapy with N -acetylcysteine 1,200 mg twice daily
with 1 L of 0.45% saline.
d. Start therapy with N -acetylcysteine 1,200 mg twice daily
with 1 L of 0.9% saline.
e. Start therapy with N -acetylcysteine 1,200 mg twice daily
with 1 L of sodium bicarbonate.
25. A 52-year-old woman presents with anasarca, decreased
stamina, and fatigue. She has had type 2 diabetes mel-
litus for 8 years, hypertension, and hyperlipidemia. She
quit smoking 2 years ago. She has a history of migraine
headaches. She has pale skin and pitting edema (4+) of
the thigh. On auscultation, breath sounds are mildly
decreased in the bases bilaterally. Other physical exam-
ination fndings and laboratory test results are shown
What additional tests should you order for this patient?
a. Renal ultrasonography
b. A 24-hour urine protein collection
c. A 24-hour urine albumin collection
d. Random urine albumin to creatinine ratio
e. Creatinine clearance
22. A 72-year-old woman presents with fatigue, decreased
stamina, and loss of appetite. She has a 25-year history of
type 2 diabetes mellitus; she has coronary artery disease
and has had coronary artery bypass graf surgery; she
received a diagnosis of breast cancer 10 years ago and had
a mastectomy and radiotherapy; she has hypertension,
hyperlipidemia, and a history of gout. Her heart rate is 48
beats per minute, her blood pressure is 132/78 mm Hg,
her height is 174 cm, and her weight is 88 kg. On physical
examination, she is pale, she is in no acute distress, and
she has lower extremity edema; otherwise, examination
fndings are normal. Laboratory test results are shown in
Table 10.Q22. Results of liver function studies are nor-
mal. Te patient takes atenolol 50 mg daily, lisinopril 40
mg daily, NPH insulin 30 units twice daily, simvastatin
20 mg daily, and allopurinol 150 mg daily.
Table 10.Q21
COMPONENT FINDING
Heart rate, beats per minute 65
Blood pressure, mm Hg 125/65
Height, cm 165
Weight, kg 91
Hemoglobin A
1c
, % 6.5
Serum creatinine, mg/dL 1.2
Total cholesterol, mg/dL 190
Triglycerides, mg/dL 123
High-density lipoprotein cholesterol, mg/dL 34
Low-density lipoprotein cholesterol, mg/dL 89
Table 10.Q22
COMPONENT FINDING
Hemoglobin, g/dL 9.8
Serum creatinine, mg/dL 2.2
Sodium, mEq/L 140
Bicarbonate, mEq/L 17
Chloride, mEq/L 113
Potassium, mEq/L 5.4
Uric acid, mg/dL 11
Hemoglobin A
1c
, % 7.6
Urine albumin to creatinine ratio, mg/g 1,800
10. NEPHROLOGY QUESTI ONS AND ANSWERS

101
per minute. He is mildly obese; examination find-
ings are otherwise unremarkable. Laboratory test
results include the following: serum creatinine 1.8
mg/dL, urine albumin to creatinine ratio 10 mg/g,
low-density lipoprotein cholesterol 89 mg/dL, and
hemoglobin A
1c
6.5%. He takes metformin 1,000
mg twice daily, allopurinol 300 mg daily, losartan
100 mg daily, aspirin 81 mg daily, simvastatin 80 mg
daily, a multivitamin daily, vitamin E daily, fish oil
capsules 2,000 mg twice daily, and sildenafil 50 mg
as needed. Which of the following is true about the
patients current condition?
a. His blood pressure and diabetes are controlled well, so his
risk of death from cardiovascular disease is not increased.
b. His blood pressure and diabetes are controlled well, so he
will not progress to end-stage renal disease (ESRD).
c. He should begin renin-inhibitor therapy to further reduce
his risk of death from cardiovascular disease and his risk of
progression of chronic kidney disease (CKD).
d. He should initiate or intensify lifestyle modifcations,
including diet, exercise, and weight loss.
e. All of the above are true.
27. A 68-year-old man presents with fatigue, bone pain,
and arthralgias. His history is signifcant for chronic
kidney disease (CKD), lef nephrectomy for renal cell
carcinoma, hypertension, and degenerative joint dis-
ease. His height is 173 cm, he weighs 65 kg, his blood
pressure is 133/77 mm Hg, and his heart rate is 62 beats
per minute. He has difuse arthralgias and mild pitting
edema of the lower extremities. Laboratory test results
include the following: serum creatinine 2.3 mg/dL,
serum calcium 8.2 mg/dL, phosphorus 5.8 mg/dL, para-
thyroid hormone (PTH) 298 pg/mL, and hemoglobin
10.2 g/dL. His only medication is the extended-release
formulation of diltiazem 360 mg. What is your next
recommendation?
a. Refer the patient for parathyroidectomy.
b. Schedule a parathyroid sestamibi scan.
c. Schedule an ultrasonographic study of the neck.
d. Start vitamin D therapy.
e. Determine the 1,25-dihydroxyvitamin D level.
in Table 10.Q25. She is taking NPH insulin, simvas-
tatin 40 mg daily, aspirin 81 mg daily, metoprolol 100
mg twice daily, and ibuprofen 200 mg twice daily as
needed. A renal biopsy was performed.
Table 10.Q25
COMPONENT FINDING
Height, cm 170
Weight, kg 82
Blood pressure, mm Hg 98/62
Heart rate, beats per minute 102
Serum creatinine, mg/dL 1.2
Serum albumin, g/dL 2.0
Total cholesterol, mg/dL 260
Low-density lipoprotein cholesterol, mg/dL 180
High-density lipoprotein cholesterol, mg/dL 30
Triglycerides, mg/dL 198
Hemoglobin, g/dL 9.6
Hemoglobin A
1c
, % 9.2
What renal biopsy fnding would be most likely?
a. Fusion of podocyte foot processes on electron microscopy
b. Mesangial matrix expansion and cell proliferation
c. Tickening of the glomerular basement membrane
d. Mesangial nodular sclerosis
e. All of the above
26. A 46-year-old man presents for an annual checkup
with no medical concerns. He has had type 2 dia-
betes mellitus for 9 years, and he has a history of
hypertension, degenerative joint disease with use of
nonsteroidal anti-inflammatory drugs, and gout. His
height is 181 cm, he weighs 101 kg, his blood pres-
sure is 122/72 mm Hg, and his heart rate is 68 beats
102
aldosterone secretion, which creates an abnormal coupling
of high distal sodium delivery and high aldosterone level,
leading to urinary potassium loss. Transcellular shif would
lead to a shif of potassium from the intracellular compart-
ment to extracellular compartments; this shif would not
contribute to the patients hypokalemia.
6. Answer c.
Te best choice is potassium given intravenously at an
appropriate rate. Insulin given intravenously or subcu-
taneously promotes intracellular potassium shifing and
would be expected to further exacerbate hypokalemia.
Te use of insulin should be avoided in patients who have
hypokalemia. Administration of calcium intravenously
would not solve the problem of hypokalemia.
7. Answer b.
Tis is a classic scenario for distal RTA, which is associated
with calcium phosphate nephrolithiasis and nephrocal-
cinosis. Without treatment, the hypokalemia and non
anion gap metabolic acidosis can become severe. Patients
with RTA tend to have slightly low intravascular volume.
Terefore, hyporeninemic hypoaldosteronism would
not be a possibility. Proximal RTA would not be associ-
ated with a severe reduction in serum bicarbonate or with
nephrolithiasis.
8. Answer a.
Tis patient likely has plasma cell dyscrasia (multiple
myeloma or AL amyloidosis, or both). Te unusually low
anion gap indicates the presence of positively charged
paraproteins. Te presence of glucosuria with euglycemia
indicates a proximal tubular dysfunction. Typically, renin
and aldosterone levels are high because the blood pressure
is low.
9. Answer a.
Te history indicates some type of intoxication. Te patient
has a large anion gap, and the most appropriate next step
would be to calculate the osmolal gap to rule out alcohol
(methanol and ethylene glycol) intoxication. Urine micros-
copy would be helpful. Te presence of calcium oxalate
crystals would indicate ethylene glycol intoxication. Te
severe acidosis and large osmolal gap would require the
initiation of hemodialysis. If ethylene glycol or methanol
intoxication is suspected, 4-methylpyrazole therapy should
be initiated.
1. Answer a.
Tis patient presented with a history suspicious for lung
cancer, which is known to be associated with syndrome
of inappropriate secretion of antidiuretic hormone.
Serum osmolality is necessary to confrm the presence
of hypo-osmolar hyponatremia. Urine osmolality is
necessary to confrm the presence of arginine vasopres-
sin (AVP); concentrated urine (absence of maximally
diluted urine) would indicate the presence of AVP.
Hypothyroidism and adrenal insuf ciency could cause
hyponatremia and should be ruled out. Te presence of
AVP in a patient with normal intravascular volume, low
serum osmolality, and lack of other known AVP stimu-
lating factors (pain, nausea, and medications) would
indicate ectopic AVP production. Given the patients
history, high-resolution CT of the chest is indicated to
determine the presence of lung cancer, which could be
the source of AVP.
2. Answer d.
HCTZ blocks absorption of sodium and chloride in the
distal convoluted tubule; the result is a mild degree of
intravascular volume depletion, which stimulates secretion
of arginine vasopressin (AVP). Beer is hypotonic and con-
tains a large amount of free water. When passing through
the distal convoluted tubule, free water is absorbed
under the infuence of AVP; the result is hyponatremia.
Terefore, HCTZ, decreased intravascular volume (which
stimulates AVP), and beer drinking all contributed to the
hyponatremia.
3. Answer c.
Water defcit is calculated as 0.6 body weight (kg)
[(sodium concentration/140) 1]. For this patient, the
water defcit would be 36 [(176/140) 1] = 9.3 L.
4. Answer b.
Te correction of hypernatremia should be limited to no
more than 0.5 mEq/L per hour or 12 mEq/L daily. Te
daily rate is a limit, not a target.
5. Answer c.
Te combination of a loop diuretic (furosemide) and a dis-
tal tubular diuretic (metolazone) is a powerful kaliuretic
regimen. In patients who have reasonably good renal clear-
ance, the combination inevitably results in hypokalemia.
Volume depletion from the diuretic efect increases
ANSWERS
10. NEPHROLOGY QUESTI ONS AND ANSWERS

103
would manifest with pyuria; patients with TTP have ane-
mia and thrombocytopenia; and MPGN and antiglo-
merular basement membrane nephritis would manifest
with hematuria with or without red blood cell casts.
18. Answer e.
Te clinical manifestation of oliguric acute renal failure
afer hypotension during surgery (along with exposure
to intravenous contrast material 2 days preoperatively) is
typical of acute tubular necrosis. Urinalysis would show
low-grade proteinuria, and the renal epithelial cells seen on
microscopy would indicate renal tubular injury.
19. Answer b.
Sterile pyuria with acute renal failure suggests acute
interstitial nephritis. Discontinuing the use of omepra-
zole would be the most appropriate treatment since pro-
ton pump inhibitors are associated with this condition.
Inserting a urinary catheter in the absence of hydronephro-
sis or a history of voiding dysfunction would not be help-
ful. Low-molecular-weight heparin should be avoided in
patients with this degree of renal functional impairment
and is not indicated in the absence of deep vein thrombo-
sis. Decreasing the metoprolol dose in the absence of sig-
nifcant bradycardia would not be benefcial. (See Myers et
al in the Suggested Reading list.)
20. Answer a.
Renal thrombotic microangiopathy is an important side
efect of vascular endothelial growth factor inhibitors such
as sunitinib and would manifest with hemolytic anemia,
thrombocytopenia, and renal failure. Microscopic poly-
angiitis can also manifest with anemia and acute renal
failure, but severe thrombocytopenia is not a feature.
Hemolytic anemia and severe thrombocytopenia are not
features of Goodpasture syndrome, atheroembolic renal
disease, or acute tubular necrosis. (See Eremina et al in the
Suggested Reading list.)
21. Answer d.
In the United States, only 20% to 30% of patients with
type 2 diabetes mellitus are evaluated for diabetic kidney
disease (DKD) with testing for proteinuria. Patients with
DKD and proteinuria have a higher risk of end-stage renal
disease and have a high associated cardiovascular mortal-
ity. Initiation of timely screening and appropriate therapy
can decrease the rate of progression of DKD. (See Te
National Kidney Foundation Kidney Disease Outcomes
Quality Initiative [NKF KDOQI] in the Suggested
Reading list.)
22. Answer a.
Exchange atenolol with metoprolol because atenolol is
cleared by the kidneys and may accumulate to adverse
plasma levels in patients with stage 3 or 4 chronic kidney
disease (CKD). Te patient has a Modifcation of Diet
in Renal Disease (MDRD) glomerular fltration rate
10. Answer a.
Te anion gap acidosis and massively elevated osmolal gap
suggest small-molecule (ie, methanol) intoxication. Te
severe acidosis and large osmolal gap require combina-
tion treatment with 4-methylpyrazole (fomepizole) and
hemodialysis.
11. Answer c.
Tis patient has anion gap acidosis and respiratory alkalo-
sis, which is typical for salicylate intoxication.
12. Answer c.
A change in P co
2
of 30 mm Hg with a change in bicar-
bonate of 7 mEq/L would be consistent with a mixture of
chronic and acute respiratory acidosis.
13. Answer b.
Tis patients arterial blood gas and serum electrolyte val-
ues are consistent with chronic respiratory alkalosis. In an
18-year-old woman, chronic respiratory alkalosis is consis-
tent with pregnancy. Progesterone stimulates the respira-
tory center, causing chronic respiratory alkalosis.
14. Answer b.
Granular casts on urine microscopy and a fractional excre-
tion of sodium that is greater than 3% (4.9% in this case)
are consistent with a diagnosis of acute tubular necrosis.
Proteinuria would be higher in membranous nephropa-
thy, which is associated with malignancies. Tumor lysis
syndrome is associated with hyperphosphatemia and, typi-
cally, hypocalcemia. Absence of hydronephrosis on ultra-
sonography is not consistent with obstruction.
15. Answer e.
Cryoglobulinemia can develop in patients with asymp-
tomatic hepatitis C infection, and this can cause a vasculitis
involving small vessels (skin) and membranoproliferative
glomerulonephritis.
16. Answer e.
A common manifestation of IgA nephropathy is persistent
asymptomatic microhematuria with various degrees of pro-
teinuria. Membranous nephropathy and minimal change
nephropathy typically manifest with nephrotic range pro-
teinuria without signifcant hematuria. Patients who have
chronic interstitial nephritis and diabetic nephropathy do
not present with persistent microhematuria.
17. Answer a.
Rhabdomyolysis precipitated by cocaine use can cause
acute renal failure from acute tubular necrosis that results
from injury to renal tubular epithelial cells afer myoglobin
is released by myocytes. Te urine sediment fndings are
characteristic of acute tubular necrosis. Urinalysis typically
is positive for blood by dipstick since the assay detects myo-
globin in addition to hemoglobin, but red blood cells are
not seen on urine microscopy. Acute interstitial nephritis
104

MAYO CLINI C INTERNAL MEDI CINE BOARD REVIEW: QUESTI ONS AND ANSWERS
including intensifcation of lifestyle modifcations for diet,
exercise, and weight loss. (See Keith et al in the Suggested
Reading list.)
27. Answer d.
Defciency of 1,25-dihydroxyvitamin D is common among
CKD patients, and initiation of vitamin D therapy to
treat secondary hyperparathyroidism in CKD patients is
indicated along with achieving the following target PTH
levels:
PTH 3570 pg/mL for patients with an estimated
glomerular fltration rate (GFR) of 3059 mL/min per
1.73 m
2
(stage 3 CKD)
PTH 70110 pg/mL for patients with an estimated
GFR of 1529 mL/min per 1.73 m
2
(stage 4 CKD)
PTH 150300 pg/mL for patients who receive dialysis
or who have an estimated GFR <15 mL/min per 1.73
m
2
(stage 5 CKD)
(See Te National Kidney Foundation Kidney Disease
Outcomes Quality Initiative [NKF KDOQI] in the
Suggested Reading list.)
SUGGESTED READING
Calvin AD, Misra S, Pfueger A. Contrast-induced acute kidney injury
and diabetic nephropathy. Nat Rev Nephrol. 2010 Nov;6(11):67988.
Epub 2010 Sep 28.
Eremina V, Jeferson JA, Kowalewska J, Hochster H, Haas M, Weisstuch J,
et al. VEGF inhibition and renal thrombotic microangiopathy. N Engl
J Med. 2008 Mar 13;358(11):112936.
Keith DS, Nichols GA, Gullion CM, Brown JB, Smith DH. Longitudinal
follow-up and outcomes among a population with chronic kidney dis-
ease in a large managed care organization. Arch Intern Med. 2004 Mar
22;164(6):65963.
Micromedex 2.0 [Internet]. Truven Health Analytics. c2012. Available
from: http://www.micromedex.com/.
Myers RP, McLaughlin K, Hollomby DJ. Acute interstitial nephritis due
to omeprazole. Am J Gastroenterol. 2001 Dec;96(12):342831.
Te National Kidney Foundation Kidney Disease Outcomes Quality
Initiative (NKF KDOQI) [Internet]. National Kidney Foundation.
New York (NY). c2012. Available from: http://www.kidney.org/pro-
fessionals/kdoqi/index.cfm.
Pfueger A, Abramowitz D, Calvin AD. Role of oxidative stress in
contrast-induced acute kidney injury in diabetes mellitus. Med Sci
Monit. 2009 Jun;15(6):RA12536.
Saiki A, Nagayama D, Ohhira M, Endoh K, Ohtsuka M, Koide N,
et al. Efect of weight loss using formula diet on renal function in
obese patients with diabetic nephropathy. Int J Obes (Lond). 2005
Sep;29(9):111520.
(GFR) of 22 mL/min per 1.73 m
2
. Metoprolol is metab-
olized by the liver and therefore has fewer side efects.
Patients with progressive CKD (stages 3 and 4) have
decreased elimination of medications that are excreted
by the kidneys. Tese medications can accumulate and
lead to adverse efects, such as bradycardia from renally
excreted -blockers (eg, atenolol). Switching to medica-
tions metabolized in the liver is more advantageous in
these patients. Furthermore, although inhibitors of the
renin-angiotensin system decrease progression of CKD
and diabetic kidney disease, their beneft is limited in
patients with progressive CKD. Te adverse efects of
these agents from decreasing the GFR and causing hyper-
kalemia might be harmful. (See Micromedix 2.0, and Te
National Kidney Foundation Kidney Disease Outcomes
Quality Initiative [NKF KDOQI] in the Suggested
Reading list.)
23. Answer e.
Te following therapies are efective for treating chronic
kidney disease or diabetic kidney disease: titration of
angiotensin-converting enzyme inhibitor or angiotensin
receptor blocker therapy, blood pressure control, glycemic
control, and weight loss. (See Saiki et al in the Suggested
Reading list.)
24. Answer d.
Patients with CKD have an increased risk of
contrast-induced acute kidney injury (AKI) and high
cardiovascular mortality. Recurrent episodes of AKI can
lead to progression of CKD. Appropriate therapies for
contrast-induced AKI prevention should be initiated. (See
Calvin et al in the Suggested Reading list.)
25. Answer e.
Diabetic nephropathy is characterized by glomerulopa-
thy involving all segments, including the mesangium, glo-
merular basement membrane, and podocytes. Some of the
phenotypes of diabetic nephropathy are characterized by
rapid progression. Even so, patients with diabetes can have
non-diabetic kidney disease (IgA, membranous nephropa-
thy, etc). Nearly 80% of renal biopsies in this patient popu-
lation show diabetic nephropathy. (See Pfueger et al in the
Suggested Reading list.)
26. Answer d.
Patients with CKD have a high cardiovascular mortality
and risk for progression to ESRD. Patient management
should focus on cardiovascular risk reduction therapy,
105
Within minutes, the area becomes red, painful, pru-
ritic, and swollen. During the next several hours, the
redness and swelling spread to the elbow. He denies
having other symptoms; specifcally, he denies having
dyspnea, light-headedness, nausea, vomiting, or diar-
rhea. Tere are no skin manifestations elsewhere. What
is the most appropriate management?
a. Apply ice to the arm, provide symptomatic relief, and review
strategies to avoid stinging insects.
b. Perform skin testing to bee only; if results are positive,
administer immunotherapy.
c. Perform skin testing to apids (honeybee) and vespids (yel-
low jacket, wasp, and hornet); if any of the results are posi-
tive, administer immunotherapy.
d. Perform an in vitro test for IgE specifc for apids (honeybee)
and vespids (yellow jacket, wasp, and hornet); if results are
positive, administer immunotherapy.
e. Administer 0.3 mL epinephrine 1:1,000 intramuscularly
and observe.
5. A 31-year-old woman presents in her ffh month of
pregnancy with increasing asthma symptoms. Her
asthma had been well controlled throughout pregnancy
but worsened over the past 4 days with the onset of an
upper respiratory tract infection. She has daily symp-
toms and nighttime awakenings due to dyspnea. She
uses her albuterol inhaler every 3 hours and has adhered
to her usual inhaler regimen of budesonide 2 pufs daily.
She denies having fever or purulent mucus. On exami-
nation, she has scattered expiratory wheezes through-
out all lung felds, a respiratory rate of 18 breaths per
minute, oxygen saturation of 97%, and forced expira-
tory volume in 1 second (FEV
1
) of 64% of the predicted
value, which improves to 72% afer albuterol. How
should you change her therapy?
a. Add montelukast 10 mg daily.
b. Increase the dosage of budesonide to 2 pufs twice daily.
c. Start the use of amoxicillin 500 mg 3 times daily.
d. Discontinue the use of the budesonide inhaler.
e. Add prednisone 40 mg daily for 5 days.
6. A 43-year-old woman reports that she had pruritic
hives and wheezing afer her second dose of intravenous
penicillin approximately 10 years ago. She has avoided
penicillin since then and has not had any further drug
reactions. Currently, she is being treated for endocardi-
tis, and high-dose penicillin is considered the drug of
choice. What recommendation can be given for penicil-
lin use?
QUESTI ONS
Multiple Choice (choose the best answer)
1. A 28-year-old woman presents with year-round symp-
toms of nasal congestion, rhinorrhea, and sneezing.
Although the symptoms are present year-round, they
worsen in the spring and fall. Her symptoms are bilat-
eral and have responded partially to treatment with
over-the-counter antihistamines. In an efort to control
her symptoms, which medication should not be used as
part of her treatment plan?
a. Nonsedating antihistamine
b. Leukotriene antagonist
c. Topical decongestant
d. Intranasal corticosteroid
e. Saline nasal spray
2. A 26-year-old woman is treated for Neisseria gonor-
rhoeae infection. She has had 2 episodes of Neisseria
meningitidis infection. As a child, she had 1 episode
of pneumonia. She has no history of skin infections or
abscesses. What type of defect is the most likely under-
lying immunologic abnormality?
a. C2
b. C59
c. IgG
d. Neutrophil chemotaxis
e. Lymphocyte count
3. A 22-year-old man presents with a 10-year history of
recurrent angioedema, particularly of the lips and eyes.
Troat swelling occurred on 3 occasions. Te episodes
seem to occur randomly, approximately monthly, and
there is no association with food, medications (includ-
ing over-the-counter products), or environmental
exposures. His father reports having similar symptoms
for years. Te patient is otherwise healthy. Current
physical examination fndings are normal, although he
brings in photographs that show marked angioedema
of his orbits and lips. What type of abnormality does
this patient have?
a. C4
b. C1q
c. Tryptase
d. Lymphocyte count
e. 24-hour urine N -methylhistamine
4. A 32-year-old man is stung on the distal right forearm
by what he describes as a bee. No stinger is visible.
11.
ALLERGY QUESTIONS AND ANSWERS
106

MAYO CLINI C INTERNAL MEDI CINE BOARD REVIEW: QUESTI ONS AND ANSWERS
recurrent infections, allergic rhinitis, or asthma before
the past 3 years. Sputum cultures have shown growth of
Streptococcus pneumoniae and Haemophilus infuenzae .
Currently, he has a chronic productive cough. With
which of the following laboratory tests is this patient
most likely to have abnormal results?
a. Total leukocyte count
b. IgG level
c. Neutrophil chemotaxis assay
d. Total complement level
e. IgE level
a. Penicillin cannot be given; use an alternate medication.
b. Skin test to the major determinant of penicillin; if results are
negative, penicillin can be given.
c. Skin test to the major and minor determinants of penicillin;
if results are negative, penicillin can be given.
d. Patch test to penicillin; if results are negative, penicillin can
be given.
e. Avoid penicillin and use a -lactam antibiotic.
7. A 62-year-old man presents with a 3-year his-
tory of recurrent sinopulmonary infections docu-
mented by computed tomographic scan of the sinuses
and chest radiographs. He does not recall having
107
1. Answer c.
Te patient has perennial and seasonal allergic rhinocon-
junctivitis. Various medications can help control the symp-
toms and are ofen used in combination. Tese include
nonsedating antihistamines, intranasal corticosteroids, and
leukotriene antagonists. Saline nasal spray has also been
shown to improve symptoms. Topical decongestants, such
as oxymetazoline, should be avoided. Use of this medica-
tion for more than 3 consecutive days can result in rhinitis
medicamentosa.
2. Answer b.
Te patient has had recurrent Neisseria infections, which
are most commonly associated with defects in the ter-
minal complement components (C59). Defects in the
other components of the immune system are associated
with other types of infections. Hypogammaglobulinemia
is associated primarily with upper and lower respiratory
infections caused by encapsulated bacteria. Neutrophil
defects are associated with skin and pulmonary abscesses.
3. Answer a.
Te patient has hereditary angioedema resulting from C1
esterase inhibitor defciency. Te usual screening test for
this is C4 testing; C4 levels are decreased in these patients.
Te C1q level is decreased in the acquired form of this
disease but not in the hereditary form. Te acquired form
occurs in older patients who do not have a family history of
the disease, and in the acquired form there is ofen an asso-
ciated underlying malignancy, particularly a hematologic
malignancy. Tryptase and 24-hour N -methylhistamine are
used to evaluate for mast cell disease, which does not mani-
fest solely with intermittent angioedema. Te lymphocyte
count is normal in hereditary and acquired angioedema
and in the majority of patients with mast cell disease.
4. Answer a.
Te patient has a large local reaction to a stinging insect.
A large local reaction to a stinging insect is not considered
a risk factor for a more serious reaction with a subsequent
sting. Terefore, only symptomatic treatment is required
for treatment. Skin testing to the apids and vespids would
be required only if immunotherapy were being contem-
plated. Tis should be considered for all adults who have
systemic reactions to the sting. A large local reaction, in
the absence of other symptoms, does not require the use
of epinephrine.
5. Answer e.
Tis pregnant patient is having a signifcant fare of her
asthma, and aggressive treatment is warranted by the
decreased FEV
1
, increased use of albuterol, and wheezing.
In this situation, systemic corticosteroids are required. Te
main risk to the mother and child is hypoxia. Systemic cor-
ticosteroids, inhaled corticosteroids, long- and short-acting
inhaled -agonists, and leukotriene receptor blockers are
acceptable for use in pregnant patients who have asthma.
For ongoing management, adding montelukast or increas-
ing the budesonide dosage may be helpful, but these
options are not the most efective in treating an acute
exacerbation.
6. Answer c.
Te clinical history is consistent with an IgE-mediated
reaction to penicillin. Over time, the majority of patients
lose their sensitivity to penicillin. Penicillin allergy can be
assessed by skin testing to the major and minor determi-
nants of penicillin. Tese are allergenic metabolites of pen-
icillin. If the test results are negative, the patient can receive
penicillin with the same risk of a signifcant reaction as for
persons in the general population who have never had a
reaction. Skin testing to only the major determinant will
miss approximately 25% of patients who are penicillin
allergic. If the results of skin testing are positive, the patient
is at high risk for an IgE-mediated reaction and penicillin
should be given only under a desensitization protocol.
7. Answer b.
Recurrent sinopulmonary infections with common respi-
ratory microorganisms are the primary manifestations
of common variable immunodefciency disease (CVID).
Te main laboratory abnormality in CVID is a depressed
level of IgG. Hypogammaglobulinemia predisposes to
recurrent sinus and pulmonary infections. Other mani-
festations include autoimmune processes and infectious
diarrhea. Te neutrophil chemotaxis assay is decreased in
chronic granulomatous disease, which is characterized by
recurrent skin and pulmonary abscesses. Defciencies of
the late-acting complement components (C59) typically
manifest as recurrent infections with meningococci and
gonococci.
ANSWERS
This page intentionally left blank
109
her hands are now raw from all the scrubbing. With fur-
ther queries, she says that she feels compelled to count
ceiling tiles in your examination room and she worries
irrationally about her childrens health. She has sleep
dif culties, and her anxiety is so high throughout the
day that she has dif culty working. Which of the fol-
lowing would not be an appropriate recommendation
for this patient?
a. Treat with bupropion titrated to 150 mg twice daily.
b. Treat with clonazepam 0.5 mg twice daily as a bridge until
other treatment is efective.
c. Treat with fuoxetine 20 mg daily.
d. Taper and eliminate her intake of cafeine.
e. Recommend cognitive behavioral psychotherapy (CBT).
4. A 42-year-old woman presents to your of ce again
afer 16 years of intermittent severe lef lower quadrant
abdominal pain. She denies having weight loss, fever, or
chills. Te cause of her symptoms is not apparent from
previous workups, which included a complete blood
cell count, electrolyte evaluations, urinalysis, computed
tomographic scan of the abdomen and pelvis, colonos-
copy, and gynecologic examination. She has previ-
ously been thoroughly evaluated for episodic dizziness,
headaches, fulike syndromes, back pain, and pain with
intercourse. Te results of all these workups were nega-
tive. What is the most likely diagnosis?
a. Conversion disorder
b. Somatization disorder
c. Hypochondriasis
d. Body dysmorphic disorder
e. Factitial disorder
5. A 56-year-old man with a history of schizophrenia has
a history of mild congestive heart failure and osteoar-
thritis. He harbors the paranoid belief that his wife is
poisoning him, and he gets special messages from the
television. Tese symptoms have been partially con-
trolled with a second-generation antipsychotic, olan-
zapine 5 mg orally daily. You recommend increasing the
olanzapine dosage to 10 mg daily. Which of the follow-
ing is false about the use of olanzapine in this patient?
a. He is at increased risk of neuroleptic malignant syndrome.
b. He is at increased risk of parkinsonism.
c. He is at increased risk of tardive dyskinesia.
d. He is at increased risk of diabetes mellitus.
e. He is at increased risk of anorexia.
QUESTI ONS
Multiple Choice (choose the best answer)
1. A 72-year-old man has a history of poorly controlled
type 2 diabetes mellitus, coronary heart disease with QT
prolongation (corrected QT interval 482 ms), and major
depression. He presents to your of ce with a 2-month
history of worsening depressive symptoms, including
severely depressed mood, tearfulness, decreased con-
centration, hopelessness, middle insomnia, and passive
suicidal ideation without a specifc intent or plan. He
has been taking sertraline 150 mg daily (the dosage was
recently increased from 100 mg daily) without much
beneft. He has been experiencing life stresses including
fnancial strain and his wifes declining health. Which
of the following would not be a reasonable next step in
managing his depression?
a. Cross-taper his medication from sertraline to amitriptyline.
b. Cross-taper his medication from sertraline to bupropion.
c. Perform a preanesthetic medical examination and refer the
patient to a psychiatrist for consideration of electroconvul-
sive therapy (ECT).
d. Refer the patient to a psychologist for psychotherapy.
e. Ask the patient about his access to frearms and enlist family
members to remove them from the home.
2. A 33-year-old woman has recently received a diagnosis
of bipolar disorder and is now taking lithium carbonate
600 mg orally twice daily. She wants to know some of
the long-term risks of taking this medication. Which of
the following is false about long-term management of
bipolar disorder with lithium?
a. Nonsteroidal anti-infammatory drugs (NSAIDs) should
be avoided because of nephrotoxicity associated with taking
the combination of an NSAID and lithium.
b. Tere is an increased risk of birth defects in children of
women taking lithium during pregnancy.
c. Tyroid function needs to be monitored because of the
potential for thyrotoxicity.
d. Acetaminophen should be avoided because of hepatotoxic-
ity associated with taking the combination of acetamino-
phen and lithium.
e. Lithium doses may need to be decreased temporarily if an
acute illness leads to dehydration.
3. A 32-year-old woman presents with anxiety about
germs. She washes her hands 30 to 40 times a day, and
12.
PSYCHIATRY QUESTIONS AND ANSWERS
110

MAYO CLINI C INTERNAL MEDI CINE BOARD REVIEW: QUESTI ONS AND ANSWERS
7. You contact the family of the patient in the previous
question and learn that she has a strong family history
of bipolar disorder. Te family asks you questions about
management of this patient. Which of the following
would not be appropriate?
a. Hospitalize on a psychiatric unit.
b. Begin use of amitriptyline 50 mg orally.
c. Begin use of ziprasidone 20 mg orally.
d. Perform a urine drug screen.
e. Begin use of valproate sodium 1,000 mg orally.
6. A 19-year-old woman is brought to the emergency
department by police afer she was discovered alone on
a rural bike trail dancing nude. I am the lizard queen!
she joyfully proclaims. You learn that she has been acting
quite erratically and sleeping poorly for the past week.
Which of the following is the least likely diagnosis?
a. Major depression with psychotic features
b. Bipolar disorder
c. Substance dependence
d. Schizophrenia
e. Methamphetamine intoxication
111
1. Answer a.
Amitriptyline would be a poor antidepressant choice
because the patient has QT prolongation (tricyclic anti-
depressants can prolong the QT interval) and diabetes
(tricyclic antidepressants can cause increased appetite and
weight gain). Bupropion would be a reasonable medication
alternative. ECT could also be considered because of the
severity of his symptoms and the failure of a medication
trial. Psychotherapy may help treat the depression and help
the patient deal with life stresses. Most completed suicides
are from frearms.
2. Answer d.
Lithium is not hepatotoxic and may be used safely with
acetaminophen. Lithium can cause nephrotoxicity, an
efect that can be increased when taken concomitantly with
NSAIDs. Lithium is a pregnancy category D medication
(associated with cardiac and facial structural abnormali-
ties) and should be avoided if possible during pregnancy,
especially in the frst trimester. Lithium can cause thyro-
toxicity. Because it has a positive ion with a valence similar
to that of sodium, lithium will be retained by the kidneys
during periods of dehydration, rapidly reaching toxic lev-
els in some patients. Renal function, thyroid function, and
electrolytes should be checked periodically (at least annu-
ally) for all patients taking lithium.
3. Answer a.
Tis patient has obsessive-compulsive disorder, which is
responsive to selective serotonin reuptake inhibitors and
CBT. Because she is very anxious, taking a scheduled low
dose of a benzodiazepine and eliminating consumption
of cafeine may improve sleep and provide some relief
while waiting for other treatments to become efective.
Bupropion is not active at the serotonin receptor and will
not provide any beneft.
4. Answer b.
Somatization disorder is characterized by physical symp-
toms without an identifable organic cause. Patients believe
that they have a physical problem and are not consciously
generating the symptoms or pretending to have them.
Somatization disorder entails a history of multiple somatic
complaints over many years, including 4 pain symptoms,
2 gastrointestinal tract symptoms, 1 sexual symptom, and
1 neurologic symptom. Conversion disorder is marked by
a neurologic symptom, such as a motor or sensory defcit,
and is unconscious in origin (unlike factitial disorder, in
which the symptoms are consciously produced). Body
dysmorphic disorder is characterized by a perception that
a normally appearing body part is misshapen or otherwise
has an abnormal appearance. Hypochondriasis is charac-
terized by an irrational fear that one has an illness or serious
disease.
5. Answer e.
Second-generation antipsychotics are associated with
hyperglycemia and an increased risk of diabetes mellitus.
Similar to traditional neuroleptics, second-generation
antipsychotics may cause extrapyramidal symptoms, tar-
dive dyskinesia, and neuroleptic malignant syndrome.
Olanzapine is not usually associated with anorexia.
6. Answer a.
Although any of the disorders mentioned can cause psycho-
sis and odd behavior, major depression is unlikely. Patients
who have major depression with psychotic features would
be unlikely to appear animated and joyful; more ofen,
they have ego-dystonic (very unpleasant) delusions, such
as a belief that their family hates them or that people are
conspiring against them.
7. Answer b.
With this patients age, behavior, and family history,
bipolar disorder is high in the diferential diagnosis.
Antidepressants, especially tricyclics, would be contrain-
dicated because they can induce or prolong mania. With
the severity of her symptoms and her poor judgment,
she poses a danger to herself, so hospitalization is neces-
sary. Ziprasidone or valproate sodium is a reasonable ini-
tial option to manage her symptom of mood elevation.
Substance-induced psychosis or mania is high in the dif-
ferential diagnosis; thus, a drug screen would be important
in the evaluation of this patient.
ANSWERS
This page intentionally left blank
113
except for mild symmetrical facial weakness. On motor
testing, distal muscle weakness is greater than proximal
muscle weakness. Vibratory sensation is decreased dis-
tally. Refexes are absent. Te Babinski test is negative.
Coordination is dif cult to test because of weakness.
Which of the following would be expected on further
evaluation?
a. Magnetic resonance imaging (MRI) of the head showing
multiple areas of increased T2 signal in the subcortical white
matter
b. Cerebrospinal fuid (CSF) examination showing an
increased protein level and a normal cell count
c. Ophthalmoscopic examination showing optic disc pallor
bilaterally
d. Elevated blood glucose concentration
e. Electroencephalography showing lef temporal sharp waves
4. An 83-year-old man has an abrupt, painless onset of
right upper limb weakness and garbled speech. On
examination in the emergency department 2.5 hours
afer the onset of symptoms, he has moderate weakness
of the right deltoid, triceps, and intrinsic hand muscles
and a mild right foot drop. His speech is halting, and
he appears frustrated when trying to speak, but he fol-
lows commands without dif culty. His blood pressure
is 163/87 mm Hg, with a heart rate of 85 beats per min-
ute. His electrocardiogram indicates atrial fbrillation.
What is the most appropriate next step in the evalua-
tion and management of this patient?
a. Infusion of intravenous (IV) tissue plasminogen activator
(tPA)
b. Initiation of warfarin therapy
c. Computed tomography (CT) of the head
d. Infusion of IV labetalol
e. Electroencephalography (EEG)
5. A 53-year-old woman has a 6-week history of increasing
clumsiness with both hands and imbalance that has led
to several falls. Her family has also noted that her speech
sounds drunk. On neurologic examination, she has a
wide-based, cautious, unstable gait. Alternating motion
rates of the limbs are of normal frequency and ampli-
tude but are irregular and imprecise. Finger-to-nose
testing is inaccurate, and the patient becomes tearful
during the examination. Her strength, sensation, and
muscle stretch refexes are normal, and magnetic reso-
nance imaging (MRI) of her brain is normal. Which of
the following is the most appropriate next step in her
evaluation?
QUESTI ONS
Multiple Choice (choose the best answer)
1. A 52-year-old woman is evaluated for dizziness. She
has a history of well-controlled diabetes mellitus and
treated hypertension. On neurologic examination, she
has a moderate loss of all sensory modalities distally
and to the knees and wrists symmetrically. Her blood
pressure is 146/84 mm Hg in the supine position (with
a heart rate of 85 beats per minute). Upon standing, her
blood pressure is 100/70 mm Hg (with a heart rate of
88 beats per minute), and she complains of a dizzy sen-
sation. Which of the following interventions should be
instituted frst?
a. Administer fudrocortisone.
b. Perform magnetic resonance angiography.
c. Discontinue use of antihypertensive drugs.
d. Perform a vestibular evaluation.
e. Perform magnetic resonance imaging of the cervical spine.
2. A 37-year-old man has a 1-month history of fuctuating
dif culties with his speech and swallowing. He notes
that with long conversations he has increasing dif culty
speaking; this is apparent during the history when his
speech becomes nearly unintelligible. He describes sev-
eral choking episodes and nasal regurgitation of liquids
when swallowing. On examination, he has normal eye
movements at baseline, but afer 2 minutes of sustained
upgaze, asymmetrical ptosis (worse on the lef) and
right hypertropia occur. Strength is initially normal, but
with sequential strong contractions of the deltoid and
iliopsoas muscles, mild weakness develops. Sensation,
muscle stretch refexes, coordination, and alternating
motion rates are normal. Which of the following would
be the most appropriate next step in his evaluation?
a. Autonomic testing, including tilt-table testing
b. Urgent magnetic resonance imaging of the cervical spine
c. Cerebrospinal fuid examination
d. Muscle biopsy
e. Serum acetylcholine receptor antibody testing
3. A 42-year-old woman presents with a 3-day history of
dif culty walking. She has also noted a tingling sen-
sation in her fngers and toes. On neurologic exami-
nation, she has a normal mental status and normal
neurovascular examination fndings. She cannot rise
from the couch without assistance and cannot ambu-
late without assistance. She appears somewhat short of
breath. Cranial nerve examination fndings are normal
13.
NEUROLOGY QUESTIONS AND ANSWERS
114

MAYO CLINI C INTERNAL MEDI CINE BOARD REVIEW: QUESTI ONS AND ANSWERS
c. Substitute phenytoin for carbamazepine.
d. Perform prolonged video-EEG monitoring for possible
anterior temporal lobectomy.
e. Discontinue use of valproate and consider vagus nerve
stimulation.
7. A 38-year-old obese woman has a history of depression,
chronic obstructive pulmonary disease, constipation,
and recurrent migraine headaches. Up to 3 years ago,
the headaches averaged 1 attack every 2 to 4 months
and responded to sumatriptan. Over the past 3 months,
they have increased in frequency to 1 to 3 attacks
weekly. Results of neurologic and ophthalmoscopic
examinations are normal. Which of the following is
the most appropriate prophylactic treatment for these
headaches?
a. Daily long-acting triptan
b. Valproate
c. Topiramate
d. Propranolol
e. Amitriptyline
a. Serum paraneoplastic antibodies, including Purkinje cell
cytoplasmic autoantibody type 1 (PCA-1) (also known as
anti-Yo)
b. Consultation with a psychiatrist
c. Cervical spine MRI with contrast medium
d. Nerve conduction studies, including needle electromyo g-
raphy
e. Sural nerve biopsy
6. A 20-year-old man is evaluated for spells during which
he appears alert but stares ahead, is unresponsive to
the environment, and exhibits automatic lip-smacking
movements. Results of his neurologic examination are
normal. Magnetic resonance imaging of the head shows
atrophy of the lef hippocampus. Results of awake elec-
troencephalography (EEG) are normal. Te spells,
which occur up to 3 times weekly despite treatment
with maximally tolerated doses of carbamazepine and
valproate over the past 10 months, have prevented him
from driving and working. Which of the following is
the most appropriate approach?
a. Substitute topiramate for valproate.
b. Add gabapentin or levetiracetam.
115
dysfunction), and systemic symptoms, such as nausea and
vomiting and diarrhea, may precede these symptoms.
3. Answer b.
Weakness may be caused by abnormalities in many areas
of the central nervous system and peripheral nervous sys-
tem. Central nervous system abnormalities causing weak-
ness include supratentorial, infratentorial, and spinal cord
lesions, each of which would cause an upper motor neuron
pattern of weakness with increased refexes and extensor
plantar responses. Te weakness is usually associated with
other fndings that localize the lesion somewhere along the
corticospinal tract. Disorders of the nerve roots (ie, radicu-
lopathies) causing weakness are ofen associated with pain,
they may be associated with decreased refexes, and they are
commonly associated with sensory complaints. Disorders
of the muscle are associated with normal refexes early in
the disease but may be diminished later. Weakness is typi-
cally greater proximally than distally. Plantar responses are
fexor, and there is no sensation loss. Neuromuscular dis-
eases such as myasthenia gravis may cause fatigable weak-
ness of both the axial and the bulbar muscles. Refexes are
typically normal, and sensation is intact.
Peripheral nerve disorders typically cause more difuse
weakness that is greater distally than proximally. Tey are
ofen associated with some degree of sensory loss (there
are more pure motor neuropathies that are associated with
minimal sensory symptoms). Refexes are ofen decreased
early in the course and the plantar responses are fexor.
Guillain-Barr syndrome (GBS) is an acute infamma-
tory demyelinating disorder of the peripheral nerves and
nerve roots. Patients present with subacute onset of weak-
ness and sensory symptoms (eg, paresthesias) that slowly
progess proximally from the hands and feet. Te weak-
ness usually begins in the legs but may occur simultane-
ously in the upper and lower extremities. Facial weakness
occurs later in 50% of cases. Te refexes are diminished
early with large fber joint position and vibratory sensa-
tion loss. Antecedent infection with viral or bacterial infec-
tions occur from 1 to several weeks before symptom onset.
Further fndings include abnormal CSF with an elevated
protein level and a normal cell count (ie, albuminocyto-
logic dissociation). In most GBS patients, these abnormali-
ties can be found within 1 week afer onset. MRI is not
needed because the lesion is localized to the peripheral
nerve. Electromyographic fndings may be normal early
but later show conduction block and nerve conduction
1. Answer c.
Orthostatic hypotension is an important cause of dizzi-
ness, particularly in patients at risk of autonomic failure,
such as those with diabetic neuropathy or synucleinopa-
thies (eg, parkinsonism). Te hallmark of orthostatic
hypotension due to autonomic failure is inability to
increase the heart rate when the blood pressure decreases
profoundly upon standing. Te frst step in managing
orthostatic hypotension is to correct potentially reversible
causes, particularly the use of vasodilators, diuretics, and
anticholinergics. Simple maneuvers, such as increasing salt
and water intake, elevating the head of the bed, and per-
forming postural maneuvers, should be tried before phar-
macologic management.
2. Answer e.
Myasthenia gravis is an autoimmune disorder of the neu-
romuscular junction. It causes fatigable weakness of the
axial musculature or bulbar muscles (or both). Diplopia,
ptosis, speech slurring, and dif culty with chewing or swal-
lowing are typical symptoms of the fatigable weakness of
the bulbar muscles. Additional diagnostic studies include
electromyography, which shows a decrement in amplitude
of compound muscle action potentials with repetitive
stimulation. Computed tomography of the chest should
be performed because thymoma is occasionally associated
with myasthenia gravis. Tymectomy has a role in the treat-
ment of myasthenia gravis, particularly in a person with
generalized myasthenia gravis and in younger patients who
are not at signifcant surgical risk. Acetylcholine receptor
antibody testing is also useful for diagnosis. Te Tensilon
(edrophonium chloride) test involves the administra-
tion of edrophonium, a short-acting acetylcholinesterase
inhibitor, which temporarily increases the concentration
of acetylcholine in the synaptic clef, so that the signs of
myasthenia gravis are temporarily improved. Atropine
should be available because of the potential for bradycardia
or other cholinergic symptoms. Autoimmune thyroid dis-
ease can also occur in myasthenia gravis, and appropriate
blood tests should be performed. Fatigable weakness argues
against muscle disease, and muscle biopsy is not useful in
myasthenia gravis. Another neuromuscular junction dis-
ease, botulism, occurs typically several hours afer ingesting
food containing Clostridium botulinum . Te patients pres-
ent with diplopia, dysphagia, dysarthria, ptosis, and weak-
ness of the jaw muscles. Autonomic symptoms can also
occur (eg, constipation, dilation of the pupils, and urinary
ANSWERS
116

MAYO CLINI C INTERNAL MEDI CINE BOARD REVIEW: QUESTI ONS AND ANSWERS
velocity slowing, with fndings more apparent on motor
testing than on sensory testing; fbrillation potentials indi-
cating denervation also occur within 2 weeks of symptom
onset.
Patients with GBS are usually hospitalized and closely
monitored for respiratory compromise. Treatment is usu-
ally initiated with plasmapheresis or intravenous immu-
noglobulin and is efective particularly if it is started early.
Close attention is also paid to deep vein thrombosis pro-
phylaxis, risk of aspiration pneumonia, constipation, back
pain, and autonomic instability (variable heart rate and
blood pressure).
4. Answer c.
Te patients presentation suggests an acute ischemic cere-
bral infarction in the distribution of the anterior division
of the lef middle cerebral artery, but CT of the head is
required to exclude the management-changing possibility
of cerebral hemorrhage. Te commonly accepted window
for IV thrombolysis is 3 hours, but imaging, laboratory
tests, and a careful history cannot be sacrifced to beat the
deadline. Selected patients may be considered for IV tPA
afer 3 hours, and intra-arterial intervention can be con-
sidered up to 6 or more hours afer the onset of symptoms.
Anticoagulation does not typically have a role in the man-
agement of acute ischemic cerebral infarction, particularly
before imaging has excluded hemorrhage, although it will
likely be considered for secondary prevention of stroke in
this patient who has atrial fbrillation. Te treatment of
mild, asymptomatic hypertension in this patient could lead
to neurologic deterioration. An EEG might be helpful if a
postictal paresis is suspected (Todd paralysis), but there is
no history of seizure.
5. Answer a.
Gait ataxia, limb dysmetria, and ataxic dysarthria all sug-
gest a cerebellar disorder, and the subacute onset of the
symptoms raises the likelihood of an autoimmune or
paraneoplastic mechanism. In this patient, the most appro-
priate next step among the choices listed is assessment for
paraneoplastic antibodies, such as PCA-1, that may mediate
a cerebellar syndrome. Tese antibodies and this syndrome
are predictive of gynecologic malignancies such as ovarian
carcinoma or breast carcinoma. Other diagnostic consider-
ations for this patient might include structural cerebellar
lesions such as a tumor (efectively excluded by the normal
imaging), intoxication with anticonvulsant medications or
alcohol, or infectious cerebellitis.
6. Answer d.
Tis young man has temporal lobe epilepsy with complex
partial seizures that is intractable since it is not responding
to 2 medications. Typically, if a patient does not achieve
seizure control with 2 medications, additional medical
therapy will not help. In this case, the next consideration
is for a surgical procedure to remove the epileptogenic por-
tion of his brain. Continuous EEG monitoring is done in
a hospitalized setting, ofen with withdrawal of the seizure
medications, in order to map the seizure onset. If it local-
izes to the temporal lobe, there is about a 90% chance that
the patient will be free of seizures afer temporal lobec-
tomy. Vagus nerve stimulation is done in patients who are
not surgical candidates.
7. Answer c.
Te frequency of the migraine headaches is such that
preventive medication is appropriate. Answer choices b
through e would be reasonable for prevention of migraine,
but triptans are abortive agents and would not be used
daily. A patients comorbidities must be reviewed before an
appropriate preventive agent is chosen. Since this patient
has lung disease, depression, and obesity, topiramate would
be the best choice. Potential adverse efects of topiramate
include paresthesias, cognitive clouding, taste perturba-
tion (particularly with carbonated beverages), anorexia,
and nephrolithiasis.

117
QUESTI ONS
Multiple Choice (choose the best answer)
1. A patient presents with recurrent episodes of the skin
fndings shown in Figure 14.Q1. What is the most likely
infectious association?
a. Herpes simplex virus
b. Pseudomonas
c. Dermatophyte
d. Borrelia burgdorferi
e. Mycobacterium marinum
14.
DERMATOLOGY QUESTIONS AND ANSWERS
2. An 86-year-old man has an itchy, new rash. At the
nursing home where he lives, he has been treated
with diferent antihistamines and topical corticoster-
oids without response. A clinic visit is arranged. On
examination, he has red papules and excoriations on
the hands, groin, and axillae (Figure 14.Q2). He has
nodular areas on his scrotum. What diagnosis should
you consider?
a. Chronic idiopathic urticaria
b. Urticarial vasculitis
c. Bullous pemphigoid
d. Pemphigus vulgaris
e. Scabies

Figure 14.Q2
Figure 14.Q1. (Adapted from Drage LA, Bundrick JB, Litin SC. Clinical
pearls in dermatology. Mayo Clin Proc. 2012 Jul;87[7]:6959. Used with
permission of Mayo Foundation for Medical Education and Research.)
3. A 26-year-old white woman presents with a facial
rash that first appeared last summer and returned
during her recent trip to Arizona. Initially, she says
that she feels well, but then she describes fatigue
and malaise. She has a family history of rheumatoid
arthritis. On examination, she is afebrile and she has
no active synovitis. Both cheeks are erythematous
and have small papules (Figure 14.Q3). There are no
other concerning signs on examination. What is the
diagnosis?
a. Malar rash of lupus
b. Seborrheic dermatitis
c. Allergic contact dermatitis
d. Rosacea
e. Acne vulgaris
118

MAYO CLINI C INTERNAL MEDI CINE BOARD REVIEW: QUESTI ONS AND ANSWERS
4. A 49-year-old woman presents with a persistent, pru-
ritic, red rash on her face, chest, elbows, knees, and
hands. Te rash began last summer and is unrespon-
sive to topical corticosteroids. Her past medical his-
tory is signifcant for asthma and migraines. For the
past month, she has been having more problems with
shortness of breath and headaches. Some of her skin
fndings are shown in Figure 14.Q4. She has mild
wheezing. Neurologic examination fndings are nor-
mal. Laboratory test results are positive for antinuclear
antibody and normal for the complete blood cell count,
liver function tests, and renal function tests. What other
laboratory testing would be most important?
a. Chest radiography and mammography
b. Patch testing
c. Lupus serologies
d. Tissue transglutaminase testing
e. Light testing
Figure 14.Q3
Figure 14.Q4
Figure 14.Q5
5. A 52-year-old man with chronic obstructive pulmonary
disease has a signifcant fare. You prescribe tapered
dosages of corticosteroid and a course of azithromycin.
A new rash develops (Figure 14.Q5). What is the most
likely cause of the rash?
a. Tapered dosages of corticosteroid
b. Drug rash
c. Pityriasis rosea
d. Tinea corporis
e. Mycoplasma pneumonia
14. DERMATOLOGY QUESTI ONS AND ANSWERS

119
7. A 43-year-old man presents with weight loss and mild
anemia. He is also concerned about a new rash that is
intensely pruritic and burning. He has received a
diagnosis of irritable bowel syndrome from his refer-
ring physician and is being treated with fber. He takes
no other medication. He still has intermittent diar-
rhea and bloating but no other symptoms. On physical
examination, you note the skin changes shown in Figure
14.Q7. What testing will help confrm your diagnosis?
6. A 52-year-old woman presents with a nonhealing ulcer
of the lower extremity. Tree weeks earlier, she noted
a purple papule on her shin. It quickly enlarged, ulcer-
ated, and became extremely painful. D bridement was
performed at a local wound care center; aferward, the
ulcer expanded to 3 times its original size. Te woman
is now admitted for intravenous antibiotics and pain
control. On her lef shin, she has a large, tender ulcer
with a purple border (Figure 14.Q6). Her biopsy from
the wound care center was interpreted as abscess, but
the culture results are negative so far. What is the most
likely diagnosis?
a. Brown recluse spider bite
b. Venous insuf ciency ulcer
c. Necrotizing fasciitis
d. Atypical mycobacteria infection
e. Pyoderma gangrenosum
Figure 14.Q6


Figure 14.Q7
a. Tissue transglutaminase antibody serology
b. Creatine kinase measurement
c. Perinuclear antineutrophil cytoplasmic antibody testing
d. Human immunodefciency virus serology
e. A 24-hour urine sample for measuring the level of
5-hydroxyindoleacetic acid
120
4. Answer a.
Dermatomyositis is an infammatory myositis with distinc-
tive skin fndings. Skin signs seen with dermatomyositis
may include heliotrope rash, Gottron papules, photosen-
sitivity, Gottron sign (erythema over the extensor surfaces
of the fnger joints, knees, and elbows), periorbital edema,
scalp erythema and pruritus, violaceous erythema of
sun-exposed or extensor surfaces, periungual erythema and
telangiectasia, cuticle hypertrophy, generalized pruritus,
shawl sign, and holster sign. Patients may present with skin
fndings but without clinical weakness or muscle enzyme
abnormalities (amyopathic dermatomyositis or dermato-
myositis sine myositis). Dermatomyositis may be a skin
sign of internal malignancy with a signifcantly higher risk
of lung, breast, or ovarian cancer. A workup for underlying
malignancy should be pursued.
5. Answer a.
Te man has psoriasis vulgaris. Psoriasis may be precipitated
by many factors, including infection (streptococcal phar-
yngitis, human immunodefciency virus disease), stress,
smoking, alcoholism, physical trauma (koebnerization),
and medications. Common medications that cause fares
or unmask psoriasis include lithium, -blockers, calcium
channel blockers, angiotensin-converting enzyme inhibi-
tors, some nonsteroidal anti-infammatory drugs, anti-
malarials, and tapered dosages of corticosteroid. Psoriasis
manifests with sharply demarcated plaques with silver scale
involving the elbows, knees, scalp, lumbar region of the
back, and the gluteal clef. Nail signs (nail pits, oil spots,
and onycholysis), scalp scale, gluteal clef pinking, and
koebnerization (development of psoriatic skin disease at
sites of trauma) are subtle signs that can help with the diag-
nosis. Psoriatic arthritis occurs in 5% to 8% of patients who
have psoriasis of the skin and commonly manifests as asym-
metric oligoarthritis or enthesitis.
6. Answer e.
Pyoderma gangrenosum is primarily a clinical diagnosis
based on the patients history and physical examination
fndings. Patients typically present with an expanding
nonhealing ulcer on the lower extremities and may relate
a history of trauma that initiated or worsened the lesion
(ie, pathergy). Te ulcer is extremely painful and unre-
sponsive to conservative wound care measures. On physi-
cal examination, the rapidly enlarging, necrotic ulcer has
a purple undermined margin and a surrounding border of
1. Answer a.
Patients who have erythema multiforme present with
target lesions that are ofen located on the palms and
soles but may occur as generalized eruptions. Drugs such
as sulfonamides, barbiturates, and anticonvulsants are
commonly associated with development of erythema
multiforme, but recurrent lesions are most ofen linked
to a herpes simplex infection. Although the skin signs of
an active herpes simplex infection may be apparent, the
outbreak may be subclinical. Discontinuing use of any
culprit drugs and a trial of acyclovir or other appropri-
ate antiviral would be appropriate in cases of recurrent
erythema multiforme.
2. Answer e.
Scabies is caused by infestation with the mite Sarcoptes
scabiei var hominis . Infection occurs as a result of direct
skin-to-skin contact; fomite transmission is uncommon.
It causes epidemics in schools, hospitals, and nursing
homes. Te rash results from a hypersensitivity reaction
to the mite protein. Te main complaint is pruritus, espe-
cially at night. Clinical features include infammatory,
excoriated papules in the web spaces of the hands and
feet, axillae, groin, and wrists and the areolae and sub-
mammary sites of women. Nodules or thickened areas in
the scrotum are also helpful clues. Te pathognomonic
fnding is a burrow, commonly located on fnger webs.
Identifcation of mites on a scabies preparation is diag-
nostic. In immunocompromised patients, a highly conta-
gious form of scabies may appear as a generalized scaling
eruption (ie, crusted scabies, formerly called Norwegian
scabies). Treatment of classic scabies includes the topical
application of permethrin; oral ivermectin may be used to
treat crusted scabies.
3. Answer d.
Not all malar rashes result from lupus. Rosacea is a com-
mon facial rash with red papules and small pustules on a
base of erythema and telangiectasia. It commonly afects
the central facial area. Sun, vasodilators, cafeine, and hot
or spicy foods may trigger fares. It may be associated with
eye fndings, but otherwise it has no systemic associa-
tions. Mild rosacea is treated with topical metronidazole,
use of sunscreens, and avoidance of triggering factors.
Anti-infammatory antibiotics, such as tetracycline, are
used in more severe cases. Rhinophyma can be treated with
laser and other surgical methods.
ANSWERS
14. DERMATOLOGY QUESTI ONS AND ANSWERS

121
the patient. Tus, on physical examination, excoriations,
erosions, and crusts are more commonly noted than intact
vesicles. Most patients with dermatitis herpetiformis have
subclinical celiac disease (gluten-sensitive enteropathy).
Adults may present with episodic or nocturnal diarrhea,
fatulence, or weight loss. Abdominal discomfort and bloat-
ing may lead to a mistaken diagnosis of irritable bowel syn-
drome. Biopsies show classic jejunal villous atrophy in 75%
of the patients; the remainder have minor mucosal changes.
Te standard for diagnosis of dermatitis herpetiformis is a
skin biopsy that shows granular immunoglobulin A depos-
its in the dermal papillae on direct immunofuorescence
examination. Serologic tests for tissue transglutaminase
antibody or endomysial antibody are highly sensitive and
specifc markers for an associated gluten-sensitive enterop-
athy. Treatment of dermatitis herpetiformis includes a
gluten-free diet and dapsone. Dapsone provides rapid
relief from pruritus and the rash but has no efect on the
enteropathy.

erythema. Te histopathologic features are nonspecifc,
and on skin biopsy the neutrophilic infltrate may be inter-
preted as cellulitis or abscess. Pyoderma gangrenosum
may occur in otherwise healthy people or may be a skin
sign of internal disease. Important associations include
infammatory bowel disease, IgA monoclonal gammo-
pathy, rheumatoid arthritis, chronic active hepatitis,
and hematologic malignancies. Treatment of pyoderma
gangrenosum includes intralesional corticosteroids, oral
corticosteroids, potassium iodide, dapsone, minocycline,
azathioprine, and cyclosporine. Te response to cortico-
steroids is dramatic.
7. Answer a.
Dermatitis herpetiformis manifests with intensely pru-
ritic vesicles and papules occurring symmetrically over the
knees, elbows, lower back, neck, and scalp. Te classically
grouped (herpetiform) vesicles are rarely intact because of
the intensity of the pruritus and subsequent scratching by
This page intentionally left blank
123
3. A 76-year-old woman undergoes a right L5 forami-
notomy and an L5-S1 fusion. Past history is signif-
cant for mixed connective tissue disease. Preoperative
medications are methylprednisolone, hydroxychloro-
quine, and warfarin for symptoms of connective tis-
sue disease (arterial occlusive disease was considered
a component of the connective tissue disease). She is
also taking nortriptyline and tramadol for back pain.
Immediately postoperatively, she has mild hypoxemia
(thought to be from narcotics). On the evening afer
surgery, she is agitated and awake all night. She is
drowsy and dif cult to arouse on postoperative day 1
and refuses to eat or participate in therapy. She does
not sleep. On postoperative day 2, she is not oriented
to time or place and is complaining of chest pain.
What is the most appropriate way to manage her
symptoms?
a. Vest restraints, bed rails, and benzodiazepines titrated to a
dose that causes sedation
b. Vest restraints, haloperidol for agitation, and 24-hour
supervision
c. Sleep enhancement protocol, family support, and haloperi-
dol for agitation
d. Benzodiazepines for sleep and removal of urinary catheter
e. Psychiatric consultation
4. An 87-year-old woman presents with a 5-kg weight loss
over the past 9 months. She had a myocardial infarc-
tion 10 years ago but has been asymptomatic ever since.
She has been treated with atenolol and aspirin, and her
mild hypertension is well controlled with the atenolol.
She fractured her hip when she fell on ice about 7
years ago, but she does not have a history of recurrent
falls. She states that her mood is good. She does her
own shopping, but fnds it hard to carry the grocer-
ies into the house because of generalized fatigue. She
lives alone and her only daughter lives 5 hours away.
She does not have any gastrointestinal tract (GI) com-
plaints. Her only real complaint is fatigue. Her body
mass index is 20 (it was 23 at her previous examination
9 months ago). Vital signs are normal. Te remainder
of the examination fndings are normal. Her health
screening is up-to-date and unremarkable. Laboratory
test results are normal for erythrocyte sedimentation
rate, complete blood cell count, and creatinine. What
should be done next?
QUESTI ONS
Multiple Choice (choose the best answer)
GERI ATRI CS
1. An 80-year-old man is brought in by his son. Te
patient has no complaints, but the son is concerned that
his fathers memory is failing. Te patient resides in an
assisted living facility. He receives 2 meals daily and an
aide helps him with housework weekly. His family moved
him there several months ago when it was clear that he
could no longer live alone. Last week, a staf member
found him wandering in the street, dressed in only a bath-
robe and pajamas (the outside temperature was 20 C).
Te patient told the nurses that he was following a man
who had entered his room. He has a history of hyperten-
sion treated with lisinopril 20 mg daily. His son states
that he found an empty pill vial at home, and it does not
appear that the prescription has been reflled recently.
On examination, the patients blood pressure is 160/95
mm Hg. He is unkempt. He has a resting tremor and a
slow, shuf ing gait. His Mini-Mental State Examination
(MMSE) score is 19 points (out of 30). Six months ago,
his MMSE score was 22 points (out of 30). What is the
most appropriate next step in his management?
a. Perform magnetic resonance imaging (MRI) of his brain.
b. Start donepezil therapy.
c. Advise the son to move him into a nursing home.
d. Start low-dose haloperidol therapy for his psychotic
symptoms.
e. Start citalopram therapy.
2. A 66-year-old female nursing home resident complains of
urinary incontinence. She describes the sensation of hav-
ing an urgent need to void. However, with her arthritic
knees, she ofen cannot reach the toilet before she loses
urine. Tese urinary symptoms have been present for sev-
eral years but have gradually worsened. She is now afraid
to go out in public. She has no dysuria or hematuria. She
takes metoprolol, hydrochlorothiazide, and a stool sof-
ener. Which of the following is the most appropriate frst
step in the management of her incontinence?
a. Begin tolterodine therapy.
b. Try a trial of topical estrogen.
c. Refer to a urologist for further evaluation.
d. Try a trial of timed voiding.
e. Discontinue the use of hydrochlorothiazide.
15.
CROSS-CONTENT AREA QUESTIONS AND ANSWERS
124

MAYO CLINI C INTERNAL MEDI CINE BOARD REVIEW: QUESTI ONS AND ANSWERS
PREVENTI VE MEDI CI NE
8. A 54-year-old man presents to an urgent care clinic
afer stepping on a piece of broken glass in his front
yard. Te glass is lodged in the sole of his lef foot. He
says that he has received several tetanus vaccinations
during his life, but he does not remember all the dates.
Immunization records show that he received his lat-
est tetanus-diphtheria (Td) vaccination 6 years ago.
What further therapy is needed for optimal tetanus
prophylaxis?
a. Tetanus immune globulin
b. Tetanus toxoid and tetanus immune globulin now
c. Tetanus toxoid
d. Tetanus toxoid followed by tetanus immune globulin in
7 days
e. No further therapy
9. An asymptomatic, active 77-year-old man presents for
a general medical examination. He does not have any
medical problems, but he has a 10-pack-year smoking
history. He has no family history of colon cancer. He
had his latest colonoscopy 5 years ago; results were nor-
mal without polyps. He received tetanus-diphtheria
(Td) vaccine and pneumococcal vaccine polyvalent
9 years ago and zoster vaccine live 3 years ago. Which
preventive service should you recommend?
a. Abdominal aortic aneurysm (AAA) screening
ultrasonography
b. Pneumococcal vaccine polyvalent
c. Td vaccine
d. Td vaccine and AAA screening ultrasonography
e. Tetanus and diphtheria toxoids and acellular pertussis
(Tdap) vaccine
10. A 35-year-old woman presents for a general medical
examination. She is healthy but is concerned that her
father had colon cancer at age 57. She wants to know
at what age she should undergo her frst colonoscopy.
What age should you recommend?
a. 37
b. 40
c. 42
d. 47
e. 50
11. A 28-year-old woman presents for a routine general
medical examination. She has had long-standing,
well-controlled asthma. She is otherwise healthy. She
does not smoke or drink alcoholic beverages, and she
works as an of ce secretary. Which of the following
should you recommend?
a. Infuenza and pneumococcal vaccination
b. Human papillomavirus (HPV) vaccination
c. Infuenza vaccination only
d. Pneumococcal vaccination only
e. Infuenza and HPV vaccination
a. Perform a complete GI workup, including computed tomog-
raphy of the abdomen and upper and lower GI endoscopy.
b. Discontinue the use of atenolol.
c. Start antidepressant therapy.
d. Help her arrange assistance with shopping and cooking.
e. Recommend that she move in with her daughter.
5. A 68-year-old woman presents for her annual examina-
tion. She has no complaints. She has a history of hyperten-
sion, which is well controlled with hydrochlorothiazide.
She is physically active. She received an infuenza vacci-
nation last year and plans to receive one again this year.
She received her pneumococcal vaccination at age 66. She
received her most recent tetanus toxoid 10 years ago. She
helps her daughter run a family day care for 12 children
aged 3 months to 6 years. In addition to the infuenza vac-
cine, what other vaccines should you recommend?
a. Tetanus and diphtheria toxoids vaccine booster
b. Tetanus and diphtheria toxoids and acellular pertussis
(Tdap) vaccine
c. Polyvalent pneumococcal vaccine
d. Measles-mumps-rubella vaccine
e. Hepatitis A vaccine
6. An 83-year-old woman is seen in the of ce for her
annual examination. She has a history of osteoporosis
and is taking alendronate. Her body mass index is 27.
She is the primary caretaker for her 85-year-old hus-
band, who has Parkinson disease. She is a nonsmoker.
She had a normal colonoscopy at age 77. Which of the
following screening tests is appropriate?
a. Spirometry
b. Colonoscopy
c. Carotid ultrasonography
d. Hypertension screening
e. Exercise stress test
7. A 78-year-old man is brought to your of ce by his wife.
He has a known history of Alzheimer disease and has
had a rapid loss of memory. Terapy with donepezil
was stopped because he had gastrointestinal tract side
efects. He did not want to try memantine. He can no
longer care for himself. His wife, who has diabetes mel-
litus, osteoarthritis, and hypertension, provides all his
care. She is feeling increasingly stressed. He has periods
of agitation several times each day. On 1 or 2 occasions,
he has struck his wife. He ofen resists getting dressed
or eating. His wife fears that she can no longer continue
to care for him at home. However, she is struggling
with this decision because he has told her that he would
rather die than live in a nursing home. What is the most
appropriate way to manage his symptoms?
a. He can no longer be cared for safely at home, so he should
live in a nursing home.
b. He should be enrolled in an adult day care that incorporates
music therapy and aromatherapy.
c. He should take risperidone.
d. Low-dose lorazepam should be given as needed for agitation.
e. He should take citalopram.
15. CROSS- CONTENT AREA QUESTI ONS AND ANSWERS

125
menopause 2 years ago without the use of hormone
therapy. She has been free of vasomotor symptoms
for 6 months but comments that intercourse with
her partner of 3 years is uncomfortable, so she began
using a vaginal lubricant with some improvement in
symptoms. Otherwise, she is doing very well. Her pre-
ventive health screening tests are all up-to-date with
the exception of her Papanicolaou (Pap) test; results
of her latest test, done 3 years ago, were normal. On
examination, she appears healthy and comfortable.
Her pelvic examination shows vaginal atrophy with no
other fndings. You recommend vaginal estradiol. A
Pap test is performed. Two days later, the results show
atypical squamous cells of undetermined signifcance
(ASC-US). You schedule an appointment for her to
return to the of ce to discuss the results and your rec-
ommendations. Which of the following should you
recommend?
a. Human papillomavirus (HPV) DNA testing and, if the
results are positive, loop electrosurgical excision
b. Loop electrosurgical excision
c. Pap tests in 6 and 12 months and, if the results still show
ASC-US, colposcopy
d. Vaginal estradiol for 2 months and then another Pap test
e. Colposcopy
16. A healthy 47-year-old woman wants your recommenda-
tions for breast cancer prevention because her mother
had breast cancer at age 59. Te patient performs
regular breast self-examinations and practices breast
self-awareness, and she has noticed no changes. She
began having mammograms at age 40 and has had no
abnormalities; her latest mammogram 13 months ago
was unremarkable, showing only dense tissue. Her past
medical history is unremarkable. Her latest menstrual
period was 1 month ago, but the period before that one
was 9 months earlier; before that, her menses had been
irregular. Family history is otherwise notable for ovar-
ian cancer in her maternal aunt at age 61, breast cancer
in her maternal grandmother at age 63, prostate cancer
in her maternal uncle at age 52, and pancreatic cancer in
her maternal great grandfather at age 67. Findings on her
physical examination, including breast examination, are
normal. Te mammogram done the same day is normal
and unchanged compared with prior mammograms.
Dense tissue is still observed. You recommend a medi-
cal genetics consultation to further evaluate her family
history because you are concerned about the possibil-
ity of hereditary breast and ovarian cancer syndrome.
Tree weeks later, the patient returns and reports that
her mother accompanied her to the genetics consulta-
tion. Te geneticist estimated that the patients mother
had a 24% likelihood of a mutation; thus, the patient
has a 12% likelihood. Te patients mother was tested
and the patient has learned that her mother carries a
truncation mutation in BRCA2 . Which of the follow-
ing should you recommend that the patient consider as
her next step?
12. A 42-year-old woman presents to your office for a
routine general medical examination. She is healthy
and does not have a family history of breast cancer.
She had menarche at age 14 and her first child at age
30, and she breast-fed her 3 children. She has never
had a mammogram and asks about the need for mam-
mography. What should you recommend for this
patient?
a. Perform screening now and then annually.
b. Perform screening now and then at age 50.
c. Begin screening at age 50.
d. Perform screening now and then biennially.
e. Weigh the individual risks and values with the benefts and
harms of screening.
13. A 32-year-old man presents for a general medical exam-
ination. He is healthy and does not routinely take any
medications. He has had 3 lifetime partners and has
been in a monogamous relationship for 5 years. He is
a nonsmoker, he drinks 1 or 2 beers each week, and he
does not use illicit drugs. His father had a myocardial
infarction at age 69. His blood pressure in the of ce
is 118/68 mm Hg. Which of the following screening
services would be most appropriate to ofer to this
patient?
a. Blood chemistry panel
b. Fasting blood glucose level
c. Lipid panel
d. Chlamydia screening
e. Voluntary human immunodefciency virus (HIV) testing
14. A 51-year-old healthy bus driver is planning to join
a local gym with his wife. He does not exercise on a
regular basis. He does not take daily medications or
use tobacco. His father died of a myocardial infarc-
tion at age 63. His blood pressure in the office is
122/78 mm Hg, his pulse is 76 beats per minute,
and his body mass index is 30. His total cholesterol
level is 180 mg/dL, and his high-density lipoprotein
cholesterol level is 42 mg/dL. His Framingham risk
score is 5%. He asks for heart testing before start-
ing his new exercise program. What further testing
should you recommend?
a. Electrocardiogram (ECG)
b. Computed tomographic (CT) coronary calcium scan
c. No further testing
d. Exercise treadmill testing
e. High-sensitivity C-reactive protein (hs-CRP) testing
WOMEN S HEALTH
15. A 51-year-old woman who you know well comes for
her annual preventive gynecologic examination. She
feels well. She has a history of type 2 diabetes melli-
tus, which she manages with her lifestyle, and rheu-
matoid arthritis. She tells you that her rheumatologist
prescribed adalimumab shortly afer her visit with you
last year and that her symptoms are well controlled;
she is pleased with the results. She went through
126

MAYO CLINI C INTERNAL MEDI CINE BOARD REVIEW: QUESTI ONS AND ANSWERS
and lef subareolar ultrasonography are unremark-
able, except for mild duct ectasia, with no abnormali-
ties identifed. Which of the following should you
recommend?
a. Reassurance and recommendation to discontinue checking
so frequently
b. Ductoscopy
c. Breast magnetic resonance imaging (MRI)
d. Subareolar duct excision
e. Pituitary MRI
20. A healthy 37-year-old woman presents to your of ce
for further evaluation of vaginal symptoms. She reports
that for the past 3 months she has had persistent vagi-
nal irritation, with itching and burning and a somewhat
thick discharge. She has normal menses. She has been
in a monogamous relationship with the same partner
for the past 15 years. She uses oral contraceptives. She
appears mildly uncomfortable but otherwise well. Her
examination is signifcant for vulvar erythema without
any lesions; she has a mildly thick discharge. Te spec-
ulum examination shows normal fndings, but she is
uncomfortable during it and during the bimanual exam-
ination. A Papanicolaou test is performed. Te pH of
the vagina is 4.3. Te whif test is negative. Microscopy
shows only a moderate number of large rods, and Gram
staining shows gram-positive rods. Culture results are
negative for Trichomonas vaginalis . What is the most
likely diagnosis?
a. Vaginal candidiasis
b. Trichomoniasis
c. Contact dermatitis
d. Bacterial vaginosis
e. Lichen planus
21. A 54-year-old healthy woman asks for a recommenda-
tion for hot fushes and vaginal symptoms. She com-
pleted natural menopause about 18 months ago. She
began having hot fushes and sweats during perimeno-
pause, and the symptoms continued afer menopause.
She wakes up multiple times at night because of hot
fushes and has to change her nightclothes at least once
nightly. She is fatigued and irritable. She is embarrassed
at work and in social situations because she sweats
through her clothes. She has followed all recommenda-
tions about clothing, diet, and her immediate environ-
ment, but her symptoms have continued. Soy and red
clover have not helped, either. She has also had worsen-
ing vaginal symptoms with dryness and burning caus-
ing signifcant discomfort during intercourse despite
use of vaginal lubricants. Tis has led to avoidance of
intimacy with her partner. She reports exercising regu-
larly, and she has never smoked. She has no signifcant
medical or family history. On examination, she is a
healthy-appearing woman in no distress. Her breasts
are normal on examination. Pelvic examination shows
signifcant vaginal atrophy. Her mammogram from 3
months ago was unremarkable. She has never had an
abnormal result from a Papanicolaou test, including
a. Annual digital mammogram and clinical breast examination
b. Bilateral mastectomy and oophorectomy
c. Tamoxifen 20 mg daily for 5 years
d. Raloxifene 60 mg daily for 5 years
e. Genetic testing
17. A healthy 21-year-old premedical student presents to
your of ce for further evaluation of a change in her
menstrual periods. She reports that her periods began
at age 13. Until 7 months ago, her menses were regular
for 5 years and occurred every 27 to 28 days. She feels
well but comments that she has been stressed about tak-
ing the Medical College Admission Test and has been
staying up late studying for several weeks. She attri-
butes her headaches and blurry vision to stress and lack
of sleep. She also tells you that her weight has increased
since freshman year, probably because she snacks while
studying and eats fast food too ofen. She says that she
started exercising and dieting last year, but she has not
lost weight. Records she brought from a student health
clinic visit for an ankle sprain freshman year note body
mass index of 25 and normal vital signs. Her blood
pressure is 127/73 mm Hg, her heart rate is 70 beats per
minute and regular, and her body mass index is 28. She
appears to be a healthy, somewhat overweight young
woman who is comfortable. Findings from the physical
examination, including a pelvic examination, are unre-
markable. Which of the following tests would be most
informative?
a. Serum -human chorionic gonadotropin
b. Tyrotropin and free thyroxine
c. Serum estradiol
d. Serum prolactin
e. Serum follicle-stimulating hormone
18. Which of the following vaccinations should not be
administered during pregnancy?
a. Human papillomavirus (HPV)
b. Meningococcus
c. Pneumococcus
d. Hepatitis A
e. Tetanus-diphtheria (Td)
19. A 35-year-old woman presents to your of ce concerned
about milky discharge from the lef breast. She feels
well and has no other concerns. She reports normal
menstrual periods and no prior breast problems. Her
sister had breast cancer at age 43. Te patient has been
very worried and checks regularly to see if the discharge
is still present. Her self-examination fndings are other-
wise unchanged. She has 2 children and fnished nurs-
ing the second child 18 months ago. On examination,
she appears comfortable, she has no adenopathy, and
her breasts are normal bilaterally, with no skin abnor-
malities and no palpable abnormalities. Several drops
of milky fuid are easily expressible from several ducts of
the lef nipple. Te fuid tests negative for blood. Serum
thyrotropin (TSH) and prolactin (PRL) levels are nor-
mal. Results from bilateral diagnostic mammography
15. CROSS- CONTENT AREA QUESTI ONS AND ANSWERS

127
e. Bilateral diagnostic mammography, ultrasonography, and
magnetic resonance imaging
24. Which of the following statements is true?
a. Rubella is the most common cause of congenital viral
infection.
b. A pregnant woman presenting with pulmonary edema and
atrial fbrillation has a high likelihood of having mitral
stenosis.
c. Te lifetime risk of ovarian cancer is greater than 75% in a
carrier of the BRCA1 mutation.
d. Dysfunctional uterine bleeding is benign and requires no
evaluation.
e. Annual clinical breast examinations decrease breast cancer
mortality.
GENERAL I NTERNAL MEDI CI NE
25. Your clinic has access to magnetic resonance imaging
(MRI) technology. You are evaluating a patient who has
a diabetic foot infection. Tere is no exposed bone. You
are concerned that he has not responded well to anti-
biotic treatment, and you determine that his current
pretest likelihood of having underlying osteomyelitis
is 50%. Recently, you read an article that evaluated the
role of MRI in diagnosing foot osteomyelitis. Te sensi-
tivity of this test is 90%, and the specifcity is 85% (com-
pared with bone biopsy as the referenced standard). If
you use MRI for this patient, and it is positive for osteo-
myelitis, what is the likelihood that he indeed will have
osteomyelitis on bone biopsy?
a. 94%
b. 89%
c. 86%
d. 50%
e. 11%
26. Refer to the information in question 25. If the mag-
netic resonance imaging (MRI) fndings were negative,
what is the likelihood that this patient would not have
osteomyelitis?
a. 94%
b. 89%
c. 86%
d. 50%
e. 11%
27. You have recently read about a randomized controlled
trial that compared the ef cacy of warfarin (interna-
tional normalized ratio 2.03.0) with that of aspirin
(325 mg) in stroke prevention in a subset of elderly
patients (75 years or older) who had atrial fbrillation.
Te patients were followed for 3 years. Te fnal results
showed that the chance of major stroke, intracranial
hemorrhage, or systemic embolism was 5% among
patients treated with warfarin and 10% among patients
treated with aspirin. From these results, what is the rela-
tive risk reduction (RRR) of a major event in the group
treated with warfarin compared with the group treated
with aspirin?
the latest one, done 2 years ago. Which of the following
is the most appropriate recommendation?
a. Start use of vaginal estrogen tablet 25 mcg twice weekly, and
reassess within 12 months.
b. Add black cohosh 40 mg daily, and reassess within 12
months.
c. Start use of venlafaxine 37.5 mg daily, titrating up to 75 mg
daily as needed, and reassess within 12 months.
d. Encourage her to wait it out and use a diferent vaginal
lubricant.
e. Begin combination hormone therapy, and reassess within 12
months.
22. A healthy 47-year-old woman presents to your of ce
with a 2-week history of a grape-sized breast lump that
she felt while showering. She reports that the lump is
nontender and she has not noticed any skin changes
or nipple discharge. Te appearance of the breast is
unchanged. Her latest menstrual period occurred last
week and was normal. She has no prior medical prob-
lems, and her family history is negative for malignan-
cies. She has no notable risk factors for malignancies.
On examination, she appears healthy. Breast examina-
tion is unremarkable with the exception of a 2.5-cm
frm mass in the lateral portion of the right breast. A
mammogram shows dense breast tissue that appears
normal, symmetrical, and, when compared with a prior
screening mammogram, stable. What should you rec-
ommend as the next step?
a. Breast ultrasonography
b. Reassurance
c. Surgical consultation
d. Follow-up mammogram midcycle in 1 to 2 months
e. Breast magnetic resonance imaging
23. A 31-year-old nulliparous woman in good health pres-
ents for evaluation of bilateral breast pain. She reports
aching, burning pain (pain level 5 on a scale from 0
to 10) in both breasts; the discomfort has been inter-
fering with sleep and intimacy. Her breasts feel heavy
and uncomfortable, especially with activity that moves
them. She has not noted any other changes in either
breast, but she does not do self-examinations. Her
latest menstrual period was 3 weeks ago, and her peri-
ods have been normal. She has not paid attention to
whether there is any pattern to the pain, but her qual-
ity of life has been afected since she avoids intimacy
and exercise when her breasts hurt. Her mother had
breast cancer at age 62. On examination, the patient is
overweight and has large, symmetrical breasts that are
normal in appearance. She has nodular, lumpy breasts
bilaterally but no focal lumps or discrete masses. Te
nodularity is thought to be relatively symmetrical when
the breasts are compared. What is the most appropri-
ate next step?
a. Reassurance and breast support with a well-ftting bra
b. Bilateral diagnostic mammography and ultrasonography
c. Danazol 100 mg daily in divided doses
d. Fine-needle aspiration biopsy from the area of greatest pain
128

MAYO CLINI C INTERNAL MEDI CINE BOARD REVIEW: QUESTI ONS AND ANSWERS
He is otherwise healthy. His creatinine value is 1.1 mg/
dL, and his platelet count is 220 10
9
/L. Which of the
following statements is correct?
a. Because the clot is confned to his calf veins, anticoagulation
is not required, but ultrasonography should be repeated in 5
to 7 days to rule out proximal extension.
b. A temporary vena cava flter should be placed to decrease
the risk of pulmonary embolism because anticoagulation
would be too risky in this early postoperative situation.
c. For convenience, oral dabigatran could be used to treat the
clot because no laboratory monitoring would be needed.
d. He should receive anticoagulants for 3 months; an appro-
priate low-molecular-weight heparin (LMWH) (eg, enox-
aparin) dosage would be 120 mg subcutaneously once daily
while administering warfarin.
e. He should receive anticoagulants for 6 months; an appro-
priate LMWH (eg, enoxaparin) dosage would be 120 mg
subcutaneously once daily while administering warfarin.
GENERAL I NTERNAL MEDI CI NE AND QUALITY
I MPROVEMENT
32. You serve on the sentinel event review committee at your
hospital. An event occurred in which a patient received
an overdose of heparin. Your committee completes a
root cause analysis and fnds that the error resulted from
a gap in physician knowledge about heparin dosing, the
lack of an institutional consensus on heparin dosing,
and a cumbersome order entry system. From the root
cause analysis, which of the following interventions is
most likely to have a sustained efect?
a. An online education module on heparin dosing
b. Distribution of a heparin dosing pocket card
c. A heparin order set
d. A new institutional policy on heparin dosing
e. A physician education conference on heparin dosing
33. An 83-year-old woman comes to your office with a
2-day history of a red left eye associated with dull,
achy pain and tears. She denies having any trauma
or visual problems. On examination, she has clear
tears and a focal area of redness in the medial sclera.
What would you recommend for control of her
symptoms?
a. Acetaminophen and avoiding contact with others until the
symptoms resolve
b. Aspirin and reassurance
c. Urgent referral to ophthalmology
d. Gentamicin eyedrops for 7 to 10 days
e. Corticosteroid eyedrops for 1 week
34. A disheveled 20-year-old college student comes into
the university student health center. It is Monday and
he has just returned from a spring break trip to Florida.
He complains of a bright red right eye associated with
copious purulent discharge, which began abruptly last
night. On examination, you note a brightly erythema-
tous sclera associated with purulent discharge and a
prominent right preauricular lymph node. In addition
a. 5%
b. 10%
c. 20%
d. 50%
e. 100%
28. Refer to the information in question 27. For the group
treated with warfarin (international normalized ratio
2.03.0) compared with the group treated with aspirin,
what is the absolute risk reduction (ARR) of a major
event at 3 years?
a. 5%
b. 10%
c. 20%
d. 50%
e. 100%
29. Refer to the information in question 27. What is the
number needed to treat (NNT) for 3 years with warfa-
rin anticoagulation to prevent 1 major event?
a. 5
b. 10
c. 20
d. 25
e. 50
30. A 70-year-old woman presents with acute pain in
her left leg. Ultrasonography has confirmed a new
thrombus in the left superficial femoral vein. She
is otherwise healthy and states that she has not had
recent surgery or trauma. Her only medication is con-
jugated estrogen 0.625 mg daily for hot flushes. She
does not have a personal or family history of venous
thromboembolism, and she has no major bleeding
risks. Her creatinine value is 0.9 mg/dL. She has a
supportive husband at home who can help her with
activities of daily living. She weighs 60 kg. What is
the most appropriate next step in the management of
this patient?
a. Elevate the leg, apply an elastic bandage wrap to the leg, and
administer anti-infammatory drugs until she has recovered.
b. Discontinue use of conjugated estrogen, elevate the leg,
apply an elastic bandage wrap to the leg, and administer
anti-infammatory drugs until she has recovered.
c. Hospitalize and treat with either low-molecular-weight hep-
arin (LMWH) or unfractionated heparin while administer-
ing warfarin.
d. Provide outpatient treatment with LMWH while adminis-
tering warfarin.
e. Request computed tomography (CT) of the chest to look
for pulmonary emboli before deciding on hospitalization or
outpatient treatment.
31. A 62-year-old man weighing 80 kg presents for evalua-
tion of an acute increase in swelling and pain of his right
lower extremity. One week earlier, he had a right total
knee arthroplasty and is currently walking with a walker.
Ultrasonography of his right lower extremity confrms
that a thrombus in the right anterior tibial vein extends
into the popliteal vein. Te femoral vein is free of clot.
15. CROSS- CONTENT AREA QUESTI ONS AND ANSWERS

129
38. Recently, several complaints have been fled by patients
in your clinic about excessive wait times in the lobby.
As a member of the quality team at your clinic, you are
charged to study and fx this problem. Which of the
following improvement methodologies would be most
successful at reducing wait times for patients in the
clinic lobby?
a. LEAN
b. Six Sigma
c. Cause-and-efect diagramming
d. Swim lane diagramming
e. Failure mode and efects analysis
39. A 72-year-old woman with long-standing diabetes mel-
litus is admitted to your hospital service for pain control
for a pelvic insuf ciency fracture. She receives hemodi-
alysis. An electrocardiogram shows that her corrected
QT interval is 480 ms. What pain medication should
be ordered for this patient?
a. Fentanyl
b. Methadone
c. Propoxyphene napsylate
d. Codeine
e. Meperidine
MEDI CAL ETHI CS AND PALLI ATI VE CARE
40. For 6 years, you have been caring for a 54-year-old
unmarried man. He has bipolar disease but has been
faithful in taking his medication. He is intelligent and
works as a computer programmer. He has not had an
episode of signifcant mania or depression for the past 4
years. He is a former smoker, but he quit smoking when
his bipolar disease was adequately treated. Recently,
he presented with the complaint of a new cough with-
out other symptoms of upper respiratory tract infec-
tion, and he reported that on a couple of occasions he
coughed up blood in his sputum. Chest radiography,
followed by computed tomography, showed that he had
a solitary lung mass on the right side. Sputum analysis
confrmed the presence of adenocarcinoma. His evalu-
ation suggested that the lesion was stage I, which would
indicate a high probability of cure by resection (5-year
survival about 85%). When you share the diagnosis and
recommendations for operation with him, he declines
the procedure. In fact, he wants no treatment at all. He
acknowledges that he will probably die of this cancer if
he does not pursue treatment, but he says that he accepts
this as his fate. He is oriented and does not appear to be
manic or depressed. What should you do next?
a. Comply with his refusal and schedule regular appointments
to pursue palliative care.
b. Recommend psychiatric consultation to assess capacity
because of his history of bipolar disease.
c. Declare the patient incompetent because of his irrational
choice and seek a court-appointed guardian.
d. Contact his parents and coworkers and enlist them to con-
vince the patient to pursue treatment.
to immediate referral to the local ophthalmologist for
oral and topical antibiotics, what other testing should
be considered next?
a. Allergy testing for pollen and ragweed
b. Screening for human immunodefciency virus/AIDS
c. Urinary polymerase chain reaction for Chlamydia trachoma-
tis and Neisseria gonorrhoeae
d. Rapid streptococcal antigen test for group A streptococci
e. Complete blood cell count with erythrocyte sedimentation
rate
35. A 42-year-old kindergarten teacher presents to your
of ce with a 2-week history of an upper respiratory
illness. She initially noted a low-grade fever, achiness,
sinus congestion with clear discharge, and cough, which
improved over 10 days but then seemed to recur with a
vengeance. Now she has a severe sore throat, enlarged
glands, and fever. She does not have a cough or short-
ness of breath. On examination, she has a fever (39 C),
tonsillar exudate, and cervical adenopathy, and she
appears ill and uncomfortable. Abdominal examina-
tion fndings are unremarkable. Which of the following
is most appropriate?
a. Mononucleosis spot test
b. Troat culture and Gram stain
c. Rapid streptococcal antigen test
d. Empirical penicillin V for 10 days
e. Chest radiograph
36. Which features could potentially distinguish between
acute angle-closure glaucoma and acute anterior
uveitis?
a. Circumcorneal injection
b. Blurry vision
c. Hypopyon
d. Pain
e. Unilateral presentation of symptoms
37. A 60-year-old male patient presents with gradual-on-
set erectile dysfunction (ED). He is in good health,
and he takes no medications. On physical examina-
tion, there is no evidence of penile abnormalities or
testicular atrophy. Visual feld testing and cardiopul-
monary examination fndings are normal. Serum tes-
tosterone levels are normal. He reports that he has a
healthy libido and is involved in a stable and otherwise
satisfactory marital relationship. What should you
recommend?
a. He should undergo coronary angiography to rule out
vascular disease as a cause of the ED before initiating any
treatment.
b. He should undergo further evaluation for hypogonadism as
a likely cause of the ED.
c. He should undergo magnetic resonance imaging (MRI) of
the brain to rule out pituitary disease as a cause of his ED.
d. He should undergo psychotherapeutic intervention for per-
formance anxiety.
e. He should take a phosphodiesterase-5 (PDE-5) inhibitor 1
hour before sexual intercourse.
130

MAYO CLINI C INTERNAL MEDI CINE BOARD REVIEW: QUESTI ONS AND ANSWERS
a. Negotiate a time-limited period of continued aggressive
intervention with the family, and encourage them to reach a
consensus.
b. Stop the use of the ventilator as requested by the patients
rightful surrogate, his wife.
c. Refuse to withdraw any treatment at any point because there
is no advance directive.
d. Request an ethics consultation.
e. Tell the family members that they must obtain a court order
before use of the ventilator can be stopped.
43. A 16-year-old adolescent boy is brought to the emer-
gency department afer a motorcycle accident. Severe,
continuous hemorrhage has decreased the patients
hemoglobin value to 4.5 g/dL. His systolic blood pres-
sure is 70 mm Hg. He is unconscious. His parents arrive
soon afer the ambulance and refuse for their son to
receive any blood products, in keeping with their reli-
gious beliefs. Tey want to take him home, where they
and their neighbors can pray for the boy. What should
you do?
a. Contact the hospitals legal department to get a court order
for transfusions and operation.
b. Avoid transfusion and manage with crystalloid.
c. Dismiss the patient to the family against medical advice.
d. Start transfusions immediately and prepare the patient for
the operation to stop the bleeding.
e. Ask the hospital chaplain to reason with the parents.
PERI OPERATI VE MEDI CI NE
44. A 72-year-old woman will be undergoing a lef total
knee arthroplasty because of degenerative joint dis-
ease. You are asked by her orthopedist to evaluate
her preoperatively. She has a history of hypertension,
which is currently well controlled with medication.
She also has chronic kidney disease, with a baseline
creatinine value of 2.3 mg/dL, which is thought to be
the cause of the hypertension. She does not have dia-
betes mellitus. She had a stroke 3 years ago, with no
residual impairment. She does not have known coro-
nary artery disease. Her functional capacity is limited
by her knee pain. She cannot walk up a fight of stairs
without stopping. Her current medications are lisino-
pril, metoprolol, simvastatin, metformin, and aspirin.
Physical examination fndings are remarkable for a
blood pressure of 146/75 mm Hg and a heart rate of
55 beats per minute. Her electrocardiogram shows a
frst-degree atrioventricular block and normal sinus
rhythm. Te only laboratory test abnormality is the
creatinine value. What should you recommend for
further preoperative testing?
a. No further testing
b. Dobutamine stress echocardiogram
c. Dipyridamole thallium testing
d. Exercise stress test
e. Transthoracic echocardiogram
e. Schedule another appointment in 1 week to discuss the
issues again afer he has had a chance to think about his
situation.
41. A 63-year-old man had a permanent pacemaker placed a
year ago for symptomatic complete heart block. He has
now had a massive myocardial infarction and is respon-
sive to only deep pain. His ejection fraction is estimated
to be 12%, and recovery of additional myocardial func-
tion is not expected. Te patient has a living will stating
that he would not want life-sustaining treatments initi-
ated or continued if his chances of recovery from a seri-
ous illness were small. Aggressive treatments are being
withheld according to the familys request in com-
pliance with the living will. Te family now comes to
you requesting that the pacemaker be deprogrammed.
What should you do?
a. Comply with the request and deprogram the pacemaker.
b. Give the family the deprogrammer and ask them to depro-
gram the pacemaker.
c. Request an ethics consultation.
d. Refuse to comply with the request because it would con-
stitute actively taking the patients life, and euthanasia and
assisted suicide are illegal in your state.
e. Refuse to comply with the request because the living will is
invalid in this circumstance.
42. A 72-year-old man is admitted to the hospital with
an acute onset of severe bilateral pneumonia afer his
third round of chemotherapy for difuse large cell
non-Hodgkin lymphoma. He is in respiratory fail-
ure and consents to intubation. Afer intubation, he
requires chemical paralysis to tolerate mechanical ven-
tilation. Cultures and bronchoalveolar lavage do not
identify an infectious cause, and corticosteroids are
added to broad-spectrum antibiotic therapy in case the
patient has cyclophosphamide-induced pneumonitis.
Afer 1 week of treatment, a little progress is observed,
although attempts to wean the patient from chemical
paralysis have failed. At this point, the patients wife
and 3 children demand to meet with the treating phy-
sicians. Te wife and oldest son insist that use of the
ventilator be discontinued and the patient be allowed
to die. Tey claim that he would not want this therapy
and that it is torture for him. When queried, they indi-
cate that there is no known advance directive and that
neither of them has ever overtly discussed these sorts of
issues with the patient. At this point, the youngest son
dissents and states that his father is a fghter and would
want the ventilator therapy to continue in hopes that he
could improve. He states that his father was always an
aggressive businessman and overcame 2 previous can-
cers, lung cancer and prostate cancer. Te middle child,
a daughter, is very confused and does not know what
to think or what her family would want. All again reit-
erate that none of them have ever discussed issues such
as life-sustaining treatment or goals of therapy with the
patient. At this point, what should the physician do?
15. CROSS- CONTENT AREA QUESTI ONS AND ANSWERS

131
48. A 78-year-old woman has peripheral vascular disease
and is scheduled for an aortofemoral bypass. Her past
history includes hypertension, diabetes mellitus, and
hyperlipidemia. She has no symptoms of coronary
artery disease, but her activity is limited by claudica-
tion and arthritis in her knee. Current medications
include metformin 1,000 mg twice daily, metoprolol
100 mg twice daily, lovastatin 20 mg daily, aspirin 81
mg daily, and lisinopril 10 mg daily. On examination,
her heart rate is 60 beats per minute, and her blood
pressure is 135/80 mm Hg. Heart and lung examina-
tion fndings are normal. She has no peripheral edema,
but she does have markedly diminished pulses in her
feet. Dipyridamole thallium testing shows a moder-
ately sized, fxed inferior wall defect with peri-infarct
ischemia and an ejection fraction of 50%. Cardiac
catheterization showed a 30% to 50% stenosis in the
lef anterior descending coronary artery and in the
right coronary artery. Which of the following strate-
gies should you recommend?
a. Proceed with surgery, continuing her current medications.
b. Refer her for coronary artery bypass graf (CABG) surgery
before vascular surgery.
c. Refer her for angioplasty and stent placement before vascu-
lar surgery.
d. Cancel the surgery and try to manage her claudication
medically.
e. Prescribe perioperative nitrate therapy.
49. A 68-year-old man is being evaluated for a transurethral
resection of the prostate for prostate cancer. He has a
Starr-Edwards mitral valve and is taking long-term
warfarin for anticoagulation. His international nor-
malized ratio (INR) has been in the therapeutic range.
Te urologist asks for recommendations about the
perioperative management of his anticoagulation.
Other medical history includes hypertension, type 2
diabetes mellitus, obesity, and peripheral arterial dis-
ease. Physical examination fndings are unremarkable.
His blood pressure is 130/85 mm Hg, and his heart
rate is 65 beats per minute. His medications include
warfarin, lisinopril, atenolol, metformin, hydrochlo-
rothiazide, and atorvastatin. Preoperative laboratory
studies show normal values for the complete blood cell
count and electrolyte levels, creatinine 1.1 mg/dL, and
INR 2.5. What is the most efective way to manage his
anticoagulation?
a. Stop use of warfarin 4 days before surgery.
b. Stop use of warfarin 4 days before surgery, and initiate
low-molecular-weight heparin (LMWH) therapy.
c. Continue warfarin therapy through the entire perioperative
period.
d. Stop use of warfarin, and admit 4 days before surgery for
infusion of unfractionated heparin.
e. Stop use of warfarin and begin aspirin therapy 325 mg
daily.
45. A 72-year-old woman is admitted to the hospital with
a hip fracture, which she sustained while shoveling her
driveway. She has been in good health. She denies having
syncope, shortness of breath, chest pain, or palpitations
before the fall. She has no history of diabetes mellitus,
hypertension, coronary artery disease, or hyperlipidemia.
She takes no medications. She does all her own house-
work, yard work, and snow shoveling without symptoms.
On physical examination, her blood pressure is 135/88
mm Hg, and her heart rate is 76 beats per minute and
regular. Her lungs are clear. On cardiac examination, she
has a coarse, ejection-type murmur at the second right
intercostal border, which radiates to the carotids, and
her carotid upstrokes are mildly diminished. Tere is
no peripheral edema. What is the most appropriate next
step in the management of this patient?
a. Delay surgery until an echocardiogram can be obtained.
b. Recommend that the patient begin taking atenolol 25 mg
daily before surgery.
c. Cancel the surgery until a dobutamine stress echocardio-
gram can be obtained.
d. Advise the anesthesiologist of the patients condition.
e. Send the patient to the telemetry unit for further
monitoring.
46. A 75-year-old man is seen preoperatively before a neph-
rectomy for renal cell carcinoma. He has a history of
chronic obstructive pulmonary disease (COPD) due to
smoking, and he has had multiple unsuccessful attempts
at smoking cessation. He currently smokes about 15
cigarettes daily. He has a chronic cough but denies hav-
ing dyspnea, and he walks 2 to 3 miles daily without
symptoms. He has no other medical problems. He has
had 2 exacerbations of COPD in the past 2 years, dur-
ing which he was treated as an outpatient and did not
require systemic corticosteroids. What is the best way
to manage him preoperatively?
a. Perform spirometry and check arterial blood gases.
b. Perform chest radiography, spirometry, and a 6-minute walk
test.
c. Perform spirometry and check lung volumes.
d. No further testing is needed.
e. Delay his surgery for 8 weeks until he has stopped smoking.
47. A 70-year-old man is seen for preoperative medical
evaluation before elective cataract removal. He received
2 drug-eluting stents for symptomatic coronary artery
disease 4 months ago. He is taking both aspirin (81
mg daily) and clopidogrel (75 mg daily). What is your
advice to the patient and his eye surgeon about medica-
tion management in the perioperative period?
a. Stop use of clopidogrel 7 days before surgery, but continue
use of aspirin.
b. Stop use of clopidogrel and aspirin 7 days before surgery.
c. Stop use of aspirin 7 days before surgery, but continue use of
clopidogrel.
d. Defer surgery for an additional 6 months.
e. Continue use of both clopidogrel and aspirin.
132

MAYO CLINI C INTERNAL MEDI CINE BOARD REVIEW: QUESTI ONS AND ANSWERS
a. Proceed with surgery without any intervention.
b. She should receive nonsurgical therapy since her surgical
risk is prohibitive.
c. She should be treated with a transjugular intrahepatic por-
tosystemic shunt (TIPS) before surgery.
d. She should receive a transplant evaluation before surgery.
e. Postpone surgery until her BMI is less than 35.
50. A 66-year-old woman has degenerative joint disease
in her lef knee and wants to have a total knee replace-
ment. She has a history of type 2 diabetes mellitus,
which is controlled with insulin. Her body mass index
(BMI) is 41, and she has cirrhosis, which is thought to
be from nonalcoholic steatohepatitis. Her aspartate
aminotransferase and alanine aminotransferase levels
are twice the upper limit of the reference ranges. Her
Model for End-Stage Liver Disease (MELD) score is 7.
Her Child-Turcotte-Pugh score is 6. What should you
recommend?
133
GI workup is not indicated. She shows no signs of depres-
sion. Her blood pressure is well controlled, and there does
not appear to be a good reason to discontinue the use of
atenolol. Moving in with her daughter may be an option,
but this would involve moving away from a familiar envi-
ronment. Tere are many agencies that could assist this
woman with her shopping and cooking needs.
5. Answer b.
Tis patient has close contact with infants younger than
6 months. She should receive 1 dose of Tdap vaccine. She
was born before 1957, so she is presumed to have immunity
to measles, mumps, and rubella. She has already had 1 dose
of pneumococcal vaccine and will not need another.
6. Answer d.
Te US Preventive Services Task Force recommends hyper-
tension screening for all adults. Screening for carotid artery
stenosis and chronic obstructive pulmonary disease is not
recommended for asymptomatic patients who have no risk
factors. Tis patient has no signs or symptoms of coronary
artery disease, and an exercise stress test is not indicated.
Colonoscopy in patients older than 75 is controversial and
probably not indicated for patients who have had negative
colonoscopy fndings in the past.
7. Answer b.
Tis patient may ultimately need to enter a nursing home,
but that situation would be distressing for both the patient
and his wife. Nonpharmacologic therapies such as music
therapy and aromatherapy have been shown to be success-
ful for patients who have moderate to severe dementia. It
is unclear how long the efect will last, but those therapies
should probably be tried before nursing home placement.
Medications have only limited success in treating behav-
ioral symptoms of dementia, and nonpharmacologic ther-
apy should be tried frst.
8. Answer c.
For a clean minor wound, revaccination with Td is required
if the latest vaccination was more than 10 years ago.
However, for a dirty deep wound, revaccination with Td
is required if the latest vaccination was more than 5 years
ago (Table 15.A8). (See Centers for Disease Control and
Prevention, 2012 in the Suggested Reading list.)
1. Answer b.
Tis patient shows signs of dementia with Lewy bod-
ies. Patients who have dementia with Lewy bodies have
signs of parkinsonism, visual hallucinations, and rapid eye
movement sleep disorder. Tey are extremely sensitive to
neuroleptic agents, which may severely worsen the clinical
course. Donepezil and therapy for Parkinson disease may
be efective. MRI would not contribute anything to his
management.
2. Answer d.
Tis patient has classic symptoms of overactive bladder
with urinary urgency incontinence. Although tolterodine
is likely to give the patient beneft from her symptoms,
there are other, nonpharmacologic treatments that should
be tried before medications. Topical estrogen may beneft
some patients with overactive bladder, but the evidence
supporting beneft is weak. Generally, the best approach
starts with the least aggressive measures and then leads to
more invasive, more expensive, and higher-risk procedures.
Since this patient has overactive bladder associated with
urinary urgency incontinence, bladder suspension surgery
is unlikely to give her any beneft; that procedure is for
stress urinary incontinence. Although a referral to an urol-
ogist is not inappropriate, there are certainly several treat-
ment measures that the primary caregiver could attempt
frst. Tis patient may be a candidate for timed voiding,
which is ofen successful in nursing homes. Te patient is
toileted every 2 hours or at a frequency determined by the
patients voiding diary. Tis is a simple, noninvasive treat-
ment option that efectively decreases the number of daily
incontinence episodes.
3. Answer c.
Vest restraints ofen cause or worsen delirium and should not
be used except in extreme circumstances. Benzodiazepines
may also cause or worsen delirium. Haloperidol may be
appropriate for short-term management of behavioral
symptoms. Orienting strategies, familiar objects and peo-
ple, and normalizing the sleep-wake cycle help to decrease
the severity and shorten the duration of delirium.
4. Answer d.
Many elderly patients exhibit signs of frailty for socioeco-
nomic reasons. Tis patient has no GI symptoms, and a
ANSWERS
134

MAYO CLINI C INTERNAL MEDI CINE BOARD REVIEW: QUESTI ONS AND ANSWERS
9. Answer e.
Te Advisory Committee on Immunization Practices
recently released an updated recommendation for Tdap
vaccination: for adults 65 years or older who have not pre-
viously received Tdap, a single dose of Tdap may be given
instead of Td vaccine. Tdap can be administered regard-
less of the interval since the latest tetanus vaccine. Te US
Preventive Services Task Force recommends screening for
AAA in men aged 65 to 75 who have smoked more than
100 cigarettes. Pneumococcal vaccination is not routinely
repeated for otherwise healthy patients. Zoster vaccine live
is administered once to adults who are 60 or older; they do
not require another vaccination. (See Centers for Disease
Control and Prevention [CDC], 2011 in the Suggested
Reading list.)
10. Answer b.
Te American Cancer Society, the US Multisociety Task
Force on Colorectal Cancer, and the American College of
Radiology developed consensus guidelines for colon cancer
screening. Accordingly, persons who have a frst-degree rel-
ative who had colon cancer or adenomatous polyps before
age 60 should start colonoscopy at age 40, or 10 years
before the youngest case in the immediate family, which-
ever is earlier. Terefore, this patient should start screening
at age 40. (See Levin et al in the Suggested Reading list.)
11. Answer a.
Pneumococcal vaccination is recommended for adults
aged 19 to 64 who have asthma or who are smokers. HPV
vaccination is recommended for women until age 26.
Infuenza vaccination is now recommended for everyone
aged 6 months or older. (See Centers for Disease Control
and Prevention; Advisory Committee on Immunization
Practices and Fiore et al in the Suggested Reading list.)
12. Answer e.
Te US Preventive Services Task Force updated its recom-
mendations on mammography in December 2009: Te
decision to start regular, biennial screening mammog-
raphy before the age of 50 years should be an individual
decision that accounts for patient context, including the
patients values related to specifc benefts and harms. (See

a
Tetanus-diphtheria-acellular pertussis (Tdap) may be substituted for tetanus-diphtheria if the person has not previously received Tdap.

b
Yes, if the latest dose was given more than 10 years ago.

c
Yes, if the latest dose was given more than 5 years ago.
Adapted from Centers for Disease Control and Prevention. Epidemiology and Prevention of Vaccine-Preventable Diseases. 12th ed. Washington DC: Public Health
Foundation; c2012. Chapter 20, Tetanus; p. 291300.
Screening for Breast Cancer, topic page in the Suggested
Reading list.)
13. Answer e.
Te Centers for Disease Control and Prevention recom-
mends that all persons between 13 and 64 years of age be
tested for HIV regardless of recognized risk factors. Te
US Preventive Services Task Force makes no recommenda-
tion for or against routinely screening for HIV in adults
who are not at increased risk of HIV infection (grade C
recommendation). Given that this patient is otherwise
healthy and has few cardiac risk factors, he does not need
fasting blood glucose or lipid screening. Lipid screen-
ing is recommended at age 35 for otherwise healthy men.
Blood glucose screening is recommended for individuals
with hypertension. A blood chemistry panel is not recom-
mended as a routine screening test. (See Branson et al in
the Suggested Reading list.)
14. Answer c.
Te US Preventive Services Task Force (USPSTF) rec-
ommends against routine screening with resting ECG,
CT scanning for coronary calcium, and exercise treadmill
testing. Te USPSTF also concludes that there is insuf-
fcient evidence to balance the risks and benefts of using
nontraditional risk factors such as hs-CRP testing. CT
coronary calcium scanning and hs-CRP testing can be
considered for asymptomatic individuals if their 10-year
risk of a cardiovascular event is between 10% and 20%.
Te patient in this question has a Framingham risk score of
only 5%. (See Screening for Coronary Heart Disease With
Electrocardiography, topic page and Using Nontraditional
Risk Factors in Coronary Heart Disease Risk Assessment,
topic page in the Suggested Reading list.)
15. Answer c.
Te 2001 guidelines for a postmenopausal woman with
vaginal atrophy recommended short-term vaginal estro-
gen treatment and a follow-up Pap test, which generally
would yield normal fndings. Tose guidelines, however,
recommended that immunosuppressed women undergo
colposcopy immediately because they had a higher risk of
neoplasia. More recent guidelines (2006) ofer the same
Table 15. A8
TETANUS PROPHYLAXIS FOR CLEAN MINOR
WOUNDS TETANUS PROPHYLAXIS FOR ALL OTHER WOUNDS
VACCINATION HISTORY Tetanus-Diphtheria
a
Tetanus Immune Globulin Tetanus-Diphtheria
a
Tetanus Immune Globulin
Unknown or <3 doses Yes No Yes Yes
3 doses No
b
No No
c
No
15. CROSS- CONTENT AREA QUESTI ONS AND ANSWERS

135
18. Answer a.
Te HPV vaccine is an inactivated viral vaccine that is likely
safe (pregnancy category B), but it has not been studied
well in this population, and current guidelines recommend
against its use during pregnancy. Women inadvertently
vaccinated during pregnancy should have their exposure to
the vaccine reported to the Vaccine in Pregnancy Registry.
Te other vaccines listed as answer choices are known to
be safe during pregnancy. Hepatitis A, pneumococcal, and
meningococcal vaccines should be administered to preg-
nant women who are considered to be at high risk rather
than to all pregnant women. Td should be administered to
pregnant women whose latest booster was 10 or more years
earlier or who have a deep or dirty wound. (See Pickering
et al in the Suggested Reading list.)
19. Answer a.
Te patients nonbloody nipple discharge is consistent
with galactorrhea. With otherwise normal results for the
examination, for the TSH and PRL levels, and for the
breast imaging and with no objective evidence of blood in
the fuid, there is no need for further breast imaging or any
other testing. Tere is also no need for a surgical evaluation.
Given the normal PRL level, the galactorrhea is not indica-
tive of a prolactinoma and pituitary MRI is unnecessary. If
there is uncertainty as to whether the secretion is milk, a fat
stain could be done, but this is rarely necessary. In addition
to reassurance, the patient should be instructed to discon-
tinue checking for discharge so frequently to avoid stimu-
lating the breast tissue to continue fuid production. (See
Hussain et al in the Suggested Reading list.)
20. Answer c.
Although the sensitivity of the whif test is relatively low,
this patients vaginal pH is normal, efectively excluding
bacterial vaginosis. Large rod-shaped bacilli in the vaginal
secretions represent normal vaginal fora (lactobacilli) and
are not indicative of infection. No motile species are noted
on microscopy, so Trichomonas is unlikely and the nega-
tive culture results confrm the absence of Trichomonas .
Te patient should eliminate potential irritants (laundry
detergent, douching, vulvar sprays, fragranced cleansers,
etc) and be reassessed if her symptoms do not resolve. (See
Laine et al in the Suggested Reading list.)
21. Answer e.
Hot fushes and night sweats (vasomotor symptoms)
are the most frequent menopause-related complaints.
Although symptoms generally decrease over time, they
persist for up to 10 years in up to 50% of women, with 10%
reporting symptoms for even longer. Vaginal atrophy and
dryness due to menopause are also common symptoms.
Tis patients quality of life is signifcantly afected by her
vasomotor symptoms and vaginal symptoms. Since she
has no signifcant risk factors for the use of combination
hormone therapy (breast cancer, smoking, or coronary
artery disease), it is the best choice; combination hormone
recommendations for postmenopausal women, immu-
nosuppressed women, and women in the general popu-
lation because additional data have shown that women
with ASC-US have a low prevalence of invasive cancer.
Loop electrosurgical excision is not appropriate for any
population with ASC-US. HPV DNA testing and colpos-
copy are also reasonable choices for this patient (and the
general public) but choice c is the least invasive and most
cost-efective. Additional management guidelines for dif-
ferent populations and higher-grade cytologic fndings
appear in the article by Wright et al (see the Suggested
Reading list).
16. Answer e.
Although bilateral mastectomy and oophorectomy may
be a reasonable choice, this patient should be ofered
genetic testing because her mother is a known mutation
carrier. If the patient declines, prophylactic surgery or
more intense surveillance would be a reasonable option.
Intense surveillance for a BRCA mutation carrier or for
a frst-degree relative of a carrier should include annual
breast magnetic resonance imaging in addition to mam-
mography, biannual clinical breast examination, annual
gynecologic examination, biannual or annual testing
for cancer antigen 125, and pelvic ultrasonography with
appropriate counseling on the benefts, limitations, and
potential harms of each test. Raloxifene would not be
appropriate for 2 reasons: 1) Tere are no safety data
from premenopausal or perimenopausal women, so it
is indicated only for postmenopausal women, and this
patient is perimenopausal. 2) Tere are no data assess-
ing whether raloxifene decreases breast cancer incidence
among BRCA mutation carriers or women suspected of
being BRCA mutation carriers. Tamoxifen may be an
option, but the data on tamoxifen in BRCA2 mutation
carriers are not extensive and are inconclusive; however,
the data that are available suggest a beneft. (See Nelson
et al in the Suggested Reading list.)
17. Answer d.
Although pregnancy is the most common cause of sec-
ondary amenorrhea and must always be considered and
excluded as a cause of amenorrhea (secondary or primary),
prolactinoma is the cause of secondary amenorrhea in 1 of
7 women. Tis patient does not complain specifcally of
diplopia, but she reports visual changes that she attributes
to stress and lack of sleep and may not realize that she is
experiencing diplopia. Te possibility of a prolactinoma
should not be overlooked. Further, the peak incidence of
prolactinomas is in the third to ffh decades of life and
is most common in women in that age range. Testing for
hypothyroidism should be considered if the clinical presen-
tation suggests that hypothyroidism is possible. Although
this patient has gained weight and is having dif culty losing
it, there are other reasons for her weight gain; weight gain
due to hypothyroidism usually occurs with other symp-
toms and at an older age. (See Mindermann and Wilson in
the Suggested Reading list.)
136

MAYO CLINI C INTERNAL MEDI CINE BOARD REVIEW: QUESTI ONS AND ANSWERS
arm. When women keep a monthly chart, they note that
the pain develops during the 2 weeks before menstrual
bleeding; afer menstruation starts, the pain decreases and
typically resolves completely between cycles. For a woman
who provides this history and has unremarkable clinical
examination fndings, no diagnostic testing is necessary.
If a woman is eligible for a screening mammogram and
if she is overdue for screening, an up-to-date screening
mammogram should be obtained. Otherwise, mammog-
raphy is not necessary for evaluation of this type of breast
pain. Typically, reassurance and education are suf cient.
A well-ftting support bra (eg, a sports bra) is very help-
ful; for some women, decreasing the intake of cafeine and
salt can be helpful, although the evidence is inconsistent.
In addition, there is some inconsistent evidence for the use
of supplements. Despite multiple studies, the underlying
causes are undefned.
Breast pain can be characterized as cyclic, noncyclic,
and extramammary. Noncyclic mastalgia is not associated
with the menstrual cycles and is usually constant or inter-
mittent. Tere are multiple causes of noncyclic mastalgia,
and, although it is uncommon for breast cancer to manifest
with only pain, focal and persistent pain must be evaluated
to exclude an underlying malignancy. Noncyclic breast
pain usually occurs at a postmenopausal age, but it may
occur earlier, and it is usually unilateral with an anatomical
cause that can be identifed. In addition, multiple medica-
tions have been associated with noncyclic breast pain.
Te cause of extramammary pain commonly involves
the underlying chest wall, but other conditions may cause
extramammary pain (eg, ischemia, pulmonary disease, tho-
racic nerve root irritation) and should be considered.
Evaluation must include a careful history (including
quality, location, severity, aggravating and alleviating fac-
tors, and the efect on quality of life). Medications should
be carefully reviewed (including any supplements and
herbs). A thorough clinical examination should include the
chest wall. Focal, persistent pain should be evaluated with
ultrasonography, and mammography should be included
for women 30 or older. Any solid mass should be evaluated
with a biopsy.
Afer exclusion of malignancy and other causes that
require specifc treatment, multiple strategies for man-
agement may be considered. Severe pain may warrant
treatment, but ofen other measures are helpful, and
medications can be avoided in most cases. Reassurance, a
supportive bra, and discontinuation of an ofending medi-
cation or supplement (if possible) are ofen suf cient.
Taking essential fatty acids, vitamin E, and evening prim-
rose oil or decreasing or eliminating the intake of salt and
methylxanthines (eg, cafeine) may be helpful for some
women, although the evidence is conficting, and in mul-
tiple studies there was no beneft compared with placebo.
Multiple medications have been studied and are efective
for the treatment of severe mastalgia, including danazol
(US Food and Drug Administration approved), bro-
mocriptine, nonsteroidal anti-infammatory drugs, and
tamoxifen. However, the side efects ofen cause patients
therapy will address vaginal and vasomotor symptoms.
Since she has an intact uterus, unopposed estrogen can-
not be used because of the risk of uterine malignancy. If
this patients symptoms had been primarily vaginal, vagi-
nal estrogen alone would have been an appropriate choice.
Some data have shown beneft from antidepressants (eg,
venlafaxine) for vasomotor symptoms, but her symptoms
are signifcant and venlafaxine would not help the vaginal
dryness. Tere is insuf cient evidence for the efectiveness
of herbal remedies. Given the severity of her symptoms and
the negative efect on her quality of life, waiting longer to
see if her symptoms improve is not a reasonable choice. Te
lowest efective dose of hormone therapy for the shortest
duration in low-risk women is recommended, and its use
should be reassessed at least annually. Initiation of hor-
mone therapy should be avoided for women who smoke,
have vascular disease, have a history of breast cancer, or are
well past menopause. (See Utian et al in the Suggested
Reading list.)
22. Answer a.
Most palpable breast masses are benign, but evaluation is
necessary to exclude malignancy, even with a normal mam-
mogram and the absence of any breast cancer risk factors.
Of the palpable malignancies, 10% to 15% are mammo-
graphically occult. In a woman older than 30, the likeli-
est cause is a fbroadenoma, a cyst, fbrocystic change, or
hyperplasia. Evaluation should include mammography and
ultrasonography with a strong consideration for biopsy of
a solid mass (fne-needle aspiration, core needle biopsy,
or excision). Ultrasonography reliably distinguishes solid
masses from cystic lesions. A benign biopsy in a woman
with a negative mammogram does not preclude care-
ful follow-up to assess for long-term stability. In women
younger than 30, evaluation should begin with ultrasonog-
raphy; if the results are abnormal, mammography should
be performed. In this situation, it is also acceptable to allow
1 or 2 menstrual cycles to pass before reassessing clinically
and proceeding with imaging if the palpable mass persists.
Difuse lumpiness or nodularity is usually from fbrocystic
change. Discrete cysts occur most ofen in women younger
than 40. Both difuse lumpiness and discrete cysts may be
tender and can fuctuate with the menstrual cycle. Cystic
masses should be aspirated; if the fuid is bloody or the cyst
is recurrent afer prior aspiration, the fuid should be evalu-
ated cytologically. Any palpable mass in a postmenopausal
woman requires immediate evaluation to exclude malig-
nancy without a period of observation. One of the most
common causes of litigation is failure to diagnose breast
cancer. (See Health Care Guideline: Diagnosis of Breast
Disease in the Suggested Reading list.)
23. Answer a.
Tis is a classic history of cyclic breast pain associated with
the luteal phase of the menstrual cycle. Typically, women
describe burning, aching pain and a heavy sensation that
is most ofen located in the upper, outer quadrants and
ofen radiates into the axillae and sometimes the upper
15. CROSS- CONTENT AREA QUESTI ONS AND ANSWERS

137
29. Answer c.
As discussed in the answer to question 28, the absolute risk
reduction (ARR) is 5%. Now you are asked to determine
the NNT in this randomized controlled trial. Te formula
for NNT is
1
ARR
. Terefore, NNT =
1
0 05 .
= 20. Tat is,
20 elderly people with atrial fbrillation would need to be
treated with anticoagulant therapy for 3 years to prevent
1 event.
30. Answer d.
Te superfcial femoral vein is in the deep venous system;
therefore, this patient needs to be treated with heparin
and warfarin, overlapped for 5 days, until the international
normalized ratio is greater than 2.0. Outpatient treatment
with LMWH has been approved by the US Food and Drug
Administration for the treatment of deep vein thrombosis.
Te patient has a good support system at home; therefore,
it would be appropriate for her to be treated as an outpa-
tient. Since she has no symptoms to suggest pulmonary
embolism, CT of the chest is not indicated.
31. Answer d.
Te patient has a clot in the popliteal system. Tis is con-
sidered a deep vein thrombosis (DVT) and does need anti-
coagulation. Te patient is not at high risk of bleeding 1
week postoperatively; therefore, he does not need a vena
caval flter. Dabigatran is not approved by the US Food and
Drug Administration for DVT treatment. Tree months
of treatment is adequate. Te dosage of enoxaparin in the
outpatient setting is 1 mg/kg twice daily or 1.5 mg/kg once
daily; for this 80-kg patient, 120 mg subcutaneously once
daily would be appropriate.
32. Answer c.
Quality improvement interventions have a hierarchy for
sustainability. Education tools, such as online modules,
conferences, pocket cards, and signs, tend to have a posi-
tive initial efect, but this efect is not lasting. Although an
institutional consensus or policy may be a good idea, it is
dif cult to implement and develop consistent adherence.
In this question, the order set is most likely to provide a sus-
tained improvement. If it is integrated into a computerized
order entry system, it will prompt physicians about correct
dosing and thus decrease the risk of human error.
33. Answer b.
Tis patient has episcleritis, which is a self-limited, possi-
bly autoimmune-mediated infammation of the episcleral
vessels. Although vision is unafected and tearing may be
minimal, rapid-onset redness associated with achy pain
and tenderness is evident. In this scenario, one would
also want to rule out the possibility of temporal arteritis,
although it does not manifest with eye symptoms. If symp-
toms persist or recur, the patient should be referred to an
ophthalmologist.
to discontinue use, and medications are rarely necessary
with other measures. (See Smith et al in the Suggested
Reading list.)
24. Answer b.
Cytomegalovirus is the most common cause of con-
genital viral infection. Te lifetime risk of ovarian can-
cer for a BRCA1 mutation carrier is approximately 40%.
Dysfunctional uterine bleeding is a diagnosis of exclusion;
therefore, abnormal uterine bleeding must be evaluated
before this benign diagnosis is made. Annual screening
mammograms have been shown to decrease mortality
related to breast cancer, but there are no such data for clini-
cal breast examinations.
25. Answer c.
A 2 2 table must be constructed to determine the posi-
tive predictive value for MRI among patients who have a
pretest probability of 50% in an evaluation for foot osteo-
myelitis. As determined from Table 15.A25, the correct
answer is
a
a b
or 90/105 = 86%.
Table 15. A25
DISEASE
PRESENT
DISEASE
ABSENT
Diagnostic
Test Result
Positive
90 15 105
a b a+b
Negative
c d c+d
10 85 95
a+c b+d a+b+c+d
100 100 200
26. Answer b.
Te negative predictive value for MRI is
d
c d

or 85/95 = 89%.
27. Answer d.
You are asked to determine the RRR in a random-
ized controlled trial that showed that the anticoagu-
lant therapy group had an experimental event rate
(EER) of 5%. Te aspirin group had a control event rate
(CER) of 10%. Te formula for RRR is
CER EER
CER

.
In

this trial, RRR =
10% %
10%
5
= 50%.
28. Answer a.
You are asked to determine the ARR in a randomized con-
trolled trial that showed that the anticoagulant therapy
group had an experimental event rate (EER) of 5%. Te
aspirin group had a control event rate (CER) of 10%. Te
formula for ARR is CER EER. In this trial, ARR = 10%
5% = 5% over 3 years.
138

MAYO CLINI C INTERNAL MEDI CINE BOARD REVIEW: QUESTI ONS AND ANSWERS
is a tool for classifying errors by severity and likelihood of
recurrence for use in prioritizing quality initiatives.
39. Answer a.
Tis patient has renal failure and a prolonged QT inter-
val. Fentanyl does not afect the QT and can be used with
caution and at lower doses in renal failure. Methadone
prolongs the QT interval but can be used in dialysis.
Propoxyphene, codeine, and meperidine should be
avoided because they have metabolites that can accumu-
late in dialysis patients.
40. Answer e.
From all appearances, the patient is decisionally capable.
Answer choice a is not completely inappropriate. Given
that this is the frst time the patient is confronting a horri-
ble diagnosis, he may need to take some time to process the
information and could change his mind. Answer choice b
points to a very important issuethe patients capacity in
light of his signifcant psychiatric history. Ultimately, psy-
chiatric consultation might be an important step, but it is
probably not the best frst response. Since the patient has
just received horrible news, it might not be the best time
to remind him of his psychiatric issues. Rather, it is best to
give him time to refect and continue the conversation in
the near future. Answer choice d is not appropriate because
such action would violate patient confdentiality unless
you received specifc consent from the patient to make
such contacts.
41. Answer a.
Granting a request to discontinue a form of life-sustaining
therapy is not the same as euthanasia or assisted suicide. In
both euthanasia and assisted suicide, the cause of death is
the lethal agent administered by the physician. With deac-
tivation of a pacemaker, the cause of death is the underlying
conduction defect or the pathologic disorder. Respect for
patient autonomyin this case, the right to be free from
unwanted medical interventionunderpins the right to
refuse or to request the withdrawal of treatments no lon-
ger serving the patients goals of care. Tere is no ethical
or legal distinction between withholding a treatment and
withdrawing it afer it has been initiated. Both are expres-
sions of the patients desire to be lef alone, free from
unwanted touching.
42. Answer a.
It is not at all clear what the patient would want at this
point. Te last directive from the patient was that he wanted
treatment. At the same time, the patient may indeed be in
an irreversible situation in which eventual withdrawal of
use of the ventilator would be appropriate. Te goals are to
try to do what is best for the patient and to demonstrate a
good-faith commitment to work with the family. A reason-
able time limit on the aggressive treatment could accom-
plish both and allow the family members to resolve their
diferences.
34. Answer c.
Tis patient likely has gonorrhea and gonococcal conjunc-
tivitis. Immediate ophthalmologic referral is advised for
treatment with topical antibiotics (bacitracin or ciprofox-
acin) and systemic antibiotics (cefriaxone 1 g intramuscu-
larly). Sexual symptoms and contacts should be reviewed
and treated appropriately.
35. Answer d.
On the basis of the Centor criteria (fever, tonsillar exu-
dates, tender anterior cervical lymphadenopathy, and lack
of cough), this patient has a high probability of Group
A -hemolytic streptococcal (GABHS) pharyngitis.
Guidelines from the Infectious Diseases Society of America
recommend rapid streptococcal antigen testing for adults
with suspected GABHS infection (ie, 2 Centor criteria)
and no testing or treatment for those at low risk (0 or 1
Centor criterion). Patients at high risk (3 or 4 Centor cri-
teria) should receive empirical antibiotic therapy.
36. Answer c.
Both acute angle-closure glaucoma and acute anterior
uveitis are emergent processes associated with rapid onset,
circumcorneal injection, pain, and unilateral manifesta-
tion. Acute angle-closure glaucoma is associated with a
tense afected eye on palpation in an older patient; acute
anterior uveitis usually manifests in a young or middle-aged
adult and is associated with the presence of infammatory
cells and protein in the anterior chamber of the eye. Tis
collection of pus (hypopyon) may occasionally be observed
in severe cases of acute anterior uveitis.
37. Answer e.
You have already taken a good history and examined the
areas most likely to physically contribute to ED. Te onset
of symptoms was gradual in a patient who has excellent
health and who likely has ED due to age-related changes.
Tere is no need for MRI of the brain to rule out pituitary
disease since his testosterone level and testicular examina-
tion are normal. PDE-5 inhibitors are accepted as frst-line
therapy for patients in this category. If his symptoms per-
sist, consultation with a sex psychotherapist would be
reasonable to evaluate the possibility of anxiety related to
sexual performance or loss of sensate focus (mental distrac-
tion). (See Beckman et al in the Suggested Reading list.)
38. Answer a.
LEAN is a quality improvement methodology developed
by Toyota Motor Corporation to provide value by reducing
waste. In medicine, wait times are the most common form
of waste. Six Sigma is an improvement methodology devel-
oped by Motorola to reduce defects. A cause-and-efect
diagram (also known as a fsh bone diagram because of
its shape) is a tool to complete and organize a root cause
analysis. Swim lane diagramming is a quality improvement
tool used to visually show who is responsible for various
processes within a system. Failure mode and efects analysis
15. CROSS- CONTENT AREA QUESTI ONS AND ANSWERS

139
48. Answer a.
Tis patient has a reduced functional status, and noninva-
sive stress testing preoperatively is appropriate, given that
she will undergo a high-risk surgical procedure. Her car-
diac catheterization shows noncritical disease. Her stress
test results would be considered low-risk positive because
she has only peri-infarct ischemia. Perioperative -blocker
therapy is appropriate for her, and her goal heart rate
should be 55 to 65 beats per minute. Tere is no role for
preoperative CABG surgery since she has noncritical dis-
ease. Perioperative nitrates are not indicated for a patient
who is not having angina.
49. Answer b.
Tis patients Starr-Edwards valve presents a high risk
of thromboembolism. In addition, his surgical proce-
dure confers a high risk of bleeding. Simply stopping the
use of warfarin is inappropriate because of the throm-
boembolic risk. However, because of the bleeding risk,
he should receive an agent that can be easily reversed
either unfractionated heparin, which would require hos-
pitalization and would increase his risk of bleeding, or
LMWH. Te most appropriate choice in this situation,
because of cost and ease of administration, is LMWH.
His creatinine level is in the reference range, so there
is no contraindication to the use of LMWH. Aspirin
would not be suf cient to prevent thromboembolism in
this situation.
50. Answer a.
Tis patient has relatively well-compensated liver disease
and a low MELD score. A MELD score less than 10 is asso-
ciated with a low risk of perioperative death. Tere is no
role for TIPS in this situation.
SUGGESTED READING
Beckman TJ, Abu-Lebdeh HS, Mynderse LA. Evaluation and medi-
cal management of erectile dysfunction. Mayo Clin Proc. 2006
Mar;81(3):38590.
Branson BM, Handsfeld HH, Lampe MA, Janssen RS, Taylor AW, Lyss
SB, et al; Centers for Disease Control and Prevention. Revised rec-
ommendations for HIV testing of adults, adolescents, and pregnant
women in health-care settings. MMWR Recomm Rep. 2006 Sep
22;55(RR-14):117.
Centers for Disease Control and Prevention. Epidemiology and Prevention
of Vaccine-Preventable Diseases. 12th ed. Washington DC: Public
Health Foundation; c2012. Chapter 20, Tetanus; p. 291300.
Centers for Disease Control and Prevention (CDC). Updated recom-
mendations for use of tetanus toxoid, reduced diphtheria toxoid and
acellular pertussis (Tdap) vaccine from the Advisory Committee on
Immunization Practices, 2010. MMWR Morb Mortal Wkly Rep.
2011 Jan 14;60(1):135.
Centers for Disease Control and Prevention; Advisory Committee on
Immunization Practices. Updated recommendations for prevention of
invasive pneumococcal disease among adults using the 23-valent pneu-
mococcal polysaccharide vaccine (PPSV23). MMWR Morb Mortal
Wkly Rep. 2010 Sep 3;59(34):11026.
Fiore AE, Uyeki TM, Broder K, Finelli L, Euler GL, Singleton JA,
et al; Centers for Disease Control and Prevention. Prevention and
43. Answer d.
Te patient is a minor; therefore, under presumed consent,
life-sustaining interventions are to be pursued even in the
face of parental refusal. Although surrogates can refuse
life-sustaining therapy for adults for whatever reason, par-
ents have not been allowed to refuse appropriate medical
treatment, especially life-sustaining treatment, of minors
for religious convictions. In such cases, physicians are obli-
gated to obtain a court order appointing a guardian and
permitting the life-sustaining treatment. In this case, there
is not time to receive a court order before intervening, so
it is most appropriate to pursue the lifesaving treatment
while the legal issues are sorted out.
44. Answer a.
No further testing is required. Although this patient is at
increased risk for coronary artery disease because of her
history of hypertension and chronic kidney disease, she
does not have known coronary artery disease. Her hyper-
tension is well controlled, and her heart rate is 55 beats per
minute. American College of Cardiology and American
Heart Association guidelines suggest that patients who
take -blockers and have good heart rate control do not
need further noninvasive testing.
45. Answer d.
Tis patient has signs of mild to moderate aortic stenosis
and no symptoms. She has an excellent functional status
and shows no symptoms of aortic stenosis. Perioperative
-blocker therapy is not indicated for patients with aortic
stenosis. Tere is no reason to delay surgery to obtain an
echocardiogram or to do a functional stress test, since she
easily achieves 4 metabolic equivalent tasks of work and is
asymptomatic. Spinal anesthesia is relatively contraindi-
cated in patients with aortic stenosis because of the risk of
induced hypotension, so the anesthesiologist must be told
of her condition.
46. Answer d.
Tis patient clearly has COPD, but his functional status is
good. Pulmonary function tests are not indicated for pre-
operative pulmonary assessment unless the patient is hav-
ing new symptoms. Te patients condition appears to be
stable, and he does not need any further testing. Although
he would certainly beneft from smoking cessation, there is
no reason to delay his surgery to accomplish this.
47. Answer e.
Cataract surgery can be done safely if a patient is taking anti-
platelet drugs, although there are few data for clopidogrel.
Stopping use of either aspirin or clopidogrel would put the
patient at high risk of stent thrombosis in the time frame
outlined. Deferring surgery would be an option, but it would
require 8 months of additional time to allow the patient to
receive both antiplatelet drugs for a full 12 months. At that
time, clopidogrel therapy could be safely stopped for sur-
gery, but aspirin therapy would be continued.
140

MAYO CLINI C INTERNAL MEDI CINE BOARD REVIEW: QUESTI ONS AND ANSWERS
Pickering LK, Baker CJ, Freed GL, Gall SA, Grogg SE, Poland GA, et al;
Infectious Diseases Society of America. Immunization programs for
infants, children, adolescents, and adults: clinical practice guide-
lines by the Infectious Diseases Society of America. Clin Infect Dis.
2009 Sep 15;49(6):81740. Erratum in: Clin Infect Dis. 2009 Nov
1;49(9):1465.
Screening for Breast Cancer, Topic Page [Internet]. Rockville (MD): U.S.
Preventive Services Task Force; c2010 [cited 2011 Mar 23]. Available
from: http://www.uspreventiveservicestaskforce.org/uspstf/uspsbrca.
htm.
Screening for Coronary Heart Disease With Electrocardiography, Topic
Page [Internet]. Rockville (MD): U.S. Preventive Services Task Force;
c2012 [cited 2011 Mar 23]. Available from: http://www.uspreventi-
veservicestaskforce.org/uspstf/uspsacad.htm.
Smith RL, Pruthi S, Fitzpatrick LA. Evaluation and management of breast
pain. Mayo Clin Proc. 2004 Mar;79(3):35372.
Using Nontraditional Risk Factors in Coronary Heart Disease Risk
Assessment, Topic Page [Internet]. U.S. Preventive Services Task
Force; c2009 [cited 2011 Mar 21]. Available from: http://www.uspre-
ventiveservicestaskforce.org/uspstf/uspscoronaryhd.htm.
Utian WH, Archer DF, Bachmann GA, Gallagher C, Grodstein F,
Heiman JR, et al; North American Menopause Society. Estrogen and
progestogen use in postmenopausal women: July 2008 position state-
ment of Te North American Menopause Society. Menopause. 2008
Jul-Aug;15(4 Pt 1):584602.
Wright TC Jr, Massad LS, Dunton CJ, Spitzer M, Wilkinson
EJ, Solomon D; 2006 American Society for Colposcopy and
Cervical Pathology-sponsored Consensus Conference. 2006
Consensus guidelines for the management of women with abnor-
mal cervical cancer screening tests. Am J Obstet Gynecol. 2007
Oct;197(4):34655.
control of infuenza with vaccines: recommendations of the Advisory
Committee on Immunization Practices (ACIP), 2010. MMWR
Recomm Rep. 2010 Aug 6;59(RR-8):162. Errata in: MMWR
Recomm Rep. 2010 Aug 13;59(31):993. MMWR Recomm Rep. 2010
Sep 10;59(35):1147.
Health Care Guideline: Diagnosis of Breast Disease [Internet].
Bloomington (MN): Institute for Clinical Systems Improvement;
c2012. Available from: http://www.icsi.org/breast_disease_diagno-
sis/diagnosis_of_breast_disease_2.html.
Hussain AN, Policarpio C, Vincent MT. Evaluating nipple discharge.
Obstet Gynecol Surv. 2006 Apr;61(4):27883.
Laine C, Williams D, Wilson JF. Vaginitis and cervicitis. Ann Intern Med.
2009 Sep;151(5):ITC31.
Levin B, Lieberman DA, McFarland B, Smith RA, Brooks D, Andrews
KS, et al; American Cancer Society Colorectal Cancer Advisory
Group; US Multi-Society Task Force; American College of Radiology
Colon Cancer Committee. Screening and surveillance for the early
detection of colorectal cancer and adenomatous polyps, 2008: a joint
guideline from the American Cancer Society, the US Multi-Society
Task Force on Colorectal Cancer, and the American College of
Radiology. CA Cancer J Clin. 2008 May-Jun;58(3):13060. Epub
2008 Mar 5.
Mindermann T, Wilson CB. Age-related and gender-related occurrence of
pituitary adenomas. Clin Endocrinol (Oxf ). 1994 Sep;41(3):35964.
Erratum in: Clin Endocrinol (Oxf ). 1994 Nov;41(5):700.
Nelson HD, Hufman LH, Fu R, Harris EL; U.S. Preventive Services
Task Force. Genetic risk assessment and BRCA mutation testing
for breast and ovarian cancer susceptibility: systematic evidence
review for the U.S. Preventive Services Task Force. Ann Intern Med.
2005 Sep 6;143(5):36279. Erratum in: Ann Intern Med. 2005 Oct
4;143(7):547.
141
A
abdomen
active bowel sounds, 45
CT scan, 94, 97
distention, 30
hyperactive bowel sounds, 70
tenderness (pain), 27, 29, 45, 70, 84, 109
abdominal aortic aneurysm, 14, 16, 23
abdominal hernia repair, 46
abdominal radiotherapy, 29
abdominal wall cellulitis, 50
ABIM Maintenance of Certifcation
(MOC) Examination, 18, 4t
ABVD combination chemotherapy, 87
acarbose, 74
acetaminophen, 17, 30, 34, 60, 65
achalasia, 32
Achilles tendonitis (enthesopathy), 61, 65
Achilles tendon swelling, 61
acid-based disorders
answers, 102
questions, 9497
acute angle-closure glaucoma, 129, 138
acute anterior uveitis, 129
acute chest syndrome, 89
acute eosinophilic pneumonia, 42
acute interstitial nephritis, 103
acute ischemic cerebral infarction, 116
acute ischemic stroke, 41
acute myelogenous leukemia (AML), 49,
55, 83, 89
acute pancreatitis, 31
acute renal failure
answers, 102103
questions, 97
acute respiratory acidosis, 96
acute respiratory alkalosis, 96
acute respiratory distress syndrome, 42
acute thrombosis, 90
acute tubular necrosis, 103
acyclovir, 49
adalimumab, 65, 125
adenocarcinoma
rectal, 80
stage I lesion, 129
adenocarcinoma of unknown primary
origin, 79
adult pulmonary Langerhans cell
histiocytosis, 42
Advisory Committee on Immunization
Practices, 134
AL amyloidosis, 102
albuterol inhaler, 39, 80, 105, 107
alcoholic cirrhosis, 38
alendronate, 37
aliskiren, 17
alkaline phosphatase elevation, 29
allergic bronchopulmonary aspergillosis, 42
allergic rhinitis, 106
allergic rhinoconjunctivitis, 107
allergies
answers, 107108
questions, 105106
allopurinol therapy, 61, 65, 100, 101
Alzheimer disease, 124
amenorrhea, 71
amenorrhea, secondary, 135
amitriptyline, 111
amlodipine, 99
amoxicillin, 55
amoxicillin-clavulanic acid, 53
amyopathic dermatomyositis, 120
anasarca, 100
anemia of chronic disease, 8990
anemias
answers, 8990
autoimmune hemolytic anemia, 89
hemolytic anemia, 83, 103
iron defciency anemia, 28, 31, 32
macrocytic anemia, 89
megaloblastic anemia, 89
normochromic anemia, 59
questions, 8385
sickle cell anemia, 47, 89
warm autoimmune hemolytic anemia, 90
angina, progressive, 15
angioedema, recurrent, 105
angioplasty, 72
anion gap acidosis, 103
ankle edema, bilateral, 98
ankle refex, absence, 68
ankle swelling, 61
ankylosing spondylitis, 61, 65
anorexia nervosa, 79
anterior myocardial infarction, 1415
anthrax, inhalation, 53
antiarrhythmic therapy, 67
anti-CD20 monoclonal antibody, 91
anticoagulation therapy recommendation, 9
antihistamines, 105
nonsedating, 107
antimicrobial therapy, 45, 46
antipsychotics, second-generation, 109, 111
antirheumatic drugs
answers, 6465
questions, 6061
apathetic hyperthyroidism, 77
apical systolic murmur, 11
apnea. See obstructive sleep apnea
arginine vasopressin (AVP) secretion, 76,
102
arm/leg weakness, 84
arrhythmias and clinical syndromes
answers, 18
questions, 911
arterial insuf ciency, right second toe, 83
arthralgias, 32, 59, 62, 64
arthritis, 17
acute monoarthritis, 62
infammatory arthritis, 65
INDEX
Page numbers followed by f indicate a fgure; page numbers followed by t indicate a table
psoriatic arthritis, 62
asbestosis, 42
ascites, 30
Aspergillus, 55
aspirin, 14, 15, 27, 37, 68, 80, 98, 99, 100,
101, 130
assisted suicide, 138
asthma, 118
during pregnancy, 105, 107
ataxic dysarthria, 116
atenolol, 14, 15, 99, 103
atherosclerosis, 16
atorvastatin, 15, 68, 69, 70, 71
atrial fbrillation, 12, 18, 29, 72, 93, 113
chronic, 86, 87
new-onset, 67
atrial septal defect (ASD), 11, 19
atrial septum shunts, 11, 19
atrioventricular block, frst-degree, 130
atypical squamous cells of undetermined
signifcance (ASC-US), 125
autoimmune hemolytic anemia, 89
autoimmune hepatitis, 34
axillary adenopathy, 49
axillary lymph node metastases of adenoma
of unknown origin, 81
axillary mass, right, 79
azithromycin, 52, 118
azoospermia, 44
B
back pain
chest/neck radiation, 15
chronic musculoskeletal, low back, 87
lumbar spine point tenderness, 88
new-onset, 95
bariatric surgery, 75
B-cell lymphoma, 88, 91
bee sting symptoms, 105
Bell palsy, 66
benign prostatic hypertrophy, 37, 71
benzodiazepines, 111, 133
-agonists, 107
-blockers, 21, 35, 77, 79, 139
-lactam antibiotics, 52
bibasilar coarse rales, 36
bibasilar crackles, 35, 37
bilateral carotid bruits, 10
bilateral expressible galactorrhea, 71
bilateral hilar lymphadenopathy, 16, 41
bilateral pitting edema, 13
bilateral pulmonary emboli, 16
biliopancreatic diversion, 75
bioterrorism, 5253
bipolar disorder, 109, 110, 129
bismuth subsalicylate, 31
bisphosphonates, IV, 72, 77
biventricular pacing, 21
bloating, postprandial, 29
blood glucose monitoring, 67
body mass index (BMI), 15, 17, 40, 60,
6771, 68, 74
bone and parathyroid disease
answers, 7677
questions, 7273
bradycardia, 18
BRCA1 mutation, 137
BRCA2 mutation, 125, 135
breast cancer, 50, 79, 81
node-positive, 79
breast enlargement, male, 71, 7576
breast-feeding, 71
breast masses, 127, 136
breast pain, cyclic, 127, 136
breath sounds, diminished, 37
Bruce protocol stress test, 15
budesonide, 31, 105, 107
Buerger disease, 59, 60, 64
bupropion, 111
C
CA 125 elevation, 81
calcitriol therapy, 73
calcium oxalate arthropathy, 6566
calcium oxalate nephrolithiasis, 17
calcium supplement, 70, 73, 76
Candida albicans, 52
cardiac murmur, 18f
cardiac physical examination
answers, 1820, 19f
questions, 1112
cardiogenic syncope, 11, 18
cardiology
answers, 1824
arrhythmia, clinical syndromes, 18
cardiac physical exam, 1820, 19f
congestive heart failure, 2021
coronary artery disease, 2122
hypertension, 24
myocardial infarction, 2122
vascular medicine, 2224
questions
arrhythmia, clinical syndromes, 911
cardiac physical exam, 1112
congestive heart failure, 1213
coronary artery disease, 1415
hypertension, 17
myocardial infarction, 1415
vascular medicine, 1517
cardiomegaly, 30
carotid stenting, 2324
carotid ultrasonography, 85
carvedilol, 11, 12
caspofungin, 55
cataracts, 61, 65
surgery, 131, 139
cefriaxone, 33, 46, 55
celiac disease, 32
Centers for Disease Control and Prevention,
134
142

INDEX
D
dabigatran, 86, 87, 90, 137
daptomycin, 56
daunorubicin, 49
death, patient acceptance, 129, 138
decongestants, topical, 107
deep vein thrombosis (DVT), 16, 23, 137
delirium, 35, 41, 96, 133
dementia, multi-infarct, 93
dementia with Lewy bodies, 133
depression, 109, 114
dermatitis herpetiformis, 119f, 121
dermatology
answers, 120121
questions, 117119
dermatomyositis, 118f, 120
dermatomyositis sine myositis, 120
dermopathy, 74
descending aortic dissection, 23
deviated nasal septum, 46
dexamethasone, 5657, 81
diabetes insipidus, postoperative, 71, 76
diabetes mellitus, 13, 15, 16, 22, 24, 47
answers, 7475
questions, 6769
remission rates, 75
type 2, 9, 45, 50, 80, 98, 109, 125
diabetic kidney disease (DKD), 100, 103
diabetic nephropathy, 98
diabetic neuropathy, 99
diaphoresis, 69
diaphragmatic calcifcation, 37
diarrhea, 27, 28, 29, 32, 45
diclofenac-misoprostol, 61
dicloxacillin, 53
difuse large cell non-Hodgkin lymphoma,
130
difuse lung disease
answers, 4143
questions, 3638
digital clubbing, 36
digoxin, 9, 11
dilated cardiomyopathy, 11, 2021
dilated inferior vena cava, 38
diltiazem, extended-release, 101
distal renal tubular acidosis (RTA), 95, 102
distal tubular diuretics, 102. See also
metolazone
diuretics, 79
diverticulitis, 27, 31
dizziness, 109, 113
DNA-based testing, 90
donepezil, 133
Doppler ultrasonography, 30, 99
doxazosin, 17
doxorubicin, 79
doxycycline, 55
dronedarone, 11, 18
drug-eluting stents, 131
drug-induced lung disease, 42
D-shaped lef ventricle, 38
dual energy x-ray absorptiometry (DEXA),
72
duodenal switch bypass, 75
Duplex ultrasonography, 16
dysphasia, 93
dysplasia, low-grade, 27
dyspnea, 12, 13, 37, 40, 45, 59, 62, 72, 87
acute onset, 16, 35, 36
exertional, 11, 83
progressive, 39, 84
dysproteinemia, 90
E
early-stage (I-IIA) nonbulky disease, 91
edema
intermittent peripheral, 87
lower extremity
bilateral, 60, 67
bilateral ankle, 98
bilateral pitting, 12
leg edema, 30
pitting, thigh, 100
pitting, pretibial, 99
pretibial, 37
periorbital edema, 120
peritumoral edema, 81
pulmonary edema, 13
trace, peripheral, 14
ejection murmur, harsh, 10
electrocardiogram (ECG), 9f, 10f, 11, 14
electrolyte disorders
answers, 102
questions, 9394
embolism
answers, 43
questions, 3839
enalapril, 12, 50, 80
endocarditis, 105
endocrine tumors, 32
endocrinology
answers
bone, parathyroid disease, 7677
diabetes mellitus, 7475
hyperlipidemia, 7475
hypoglycemia, 7475
obesity, nutrition, 75
pituitary, gonadal, adrenal disorders,
7576
thyroid disease, 74
questions
bone, parathyroid disease, 7273
diabetes mellitus, 6769
hypoglycemia, 6769
obesity, nutrition, 7071
pituitary, gonadal, adrenal disorders,
7172
thyroid disease, 67
endometrial cancer, 79
endotracheal intubation, 37, 46
end-stage renal disease (ESRD), 101
Entamoeba histolytica, 55
enzyme-linked immunosorbent assay
(ELISA), 48
epilepsy, temporal lobe, 116
episcleritis, 31, 137
Epstein-Barr virus, 48, 97
erectile dysfunction, 71
gradual-onset, 129, 138
erythema chronicum migrans, 66
erythema multiforme, 117f, 120
erythematous nodosum, 41, 46
erythematous sclera, 128129
erythromycin, 53
Escherichia coli, 32, 55
esophagogastroduodenoscopy (EGD), 28,
30, 31
esophagus and stomach
answers, 3132
questions, 28
essential thrombocytopenia, 83
etanercept, 60, 65, 66
ethambutol, 47, 53
euthanasia, 138
exertional dyspnea (NYHA functional class
III), 11
exophthalmos, bilateral, 67
F
familial amyloidosis, 20
familial hypocalciuric hypercalcemia
(FHH), 73, 77
fatigue, 30
febrile neutropenia, 81
central hypothyroidism, 74
central nervous system
cryptococcosis, 54
central venous catheter-related bloodstream
infections (CRBSIs), 50
cephalexin, 53
cephalosporin (third-generation), 33
cerebellar disorder, 116
cervical adenopathy, 49
cervical lymphadenopathy, 48, 87
chemical gastritis, 28
chemotherapy
ABVD combination, 87
for breast cancer, 50
febrile neutropenia from, 81
R-CHOP chemotherapy, 91
for rectal cancer, stage II, 80
chest pain
anterior, 72
atypical, 19
exertional, 11, 14, 15, 36
at rest, 11
retrosternal, 14
right-sided, 36
substernal, 35
chest radiography
dense upper lef consolidation, 45
difuse pulmonary infltrates, widened
mediastinum, 46
questions, 3940
choking episodes, 113
cholecystectomy, elective, 84
chronic hepatitis C, 30
chronic interstitial nephritis, 98
chronic kidney disease (CKD), 93, 94,
101, 130
stage 3, 99, 100, 103
stage 4, 103
treatment recommendations, 104
chronic lymphocytic leukemia (CLL), 90
chronic obstructive pulmonary disease
(COPD), 39, 40, 43, 79, 80, 96,
114, 118, 131
chronic renal failure, 47
answers, 103104
questions, 99101
chronic respiratory acidosis, 96
chronic respiratory alkalosis, 96, 103
Churg-Strauss syndrome, 42, 60, 64
Chvostek sign, 73
ciprofoxacin, 33, 50, 52, 55
circumcision, 55
cirrhosis, 30, 33, 38
claudication
intermittent, 15, 16, 22
pseudoclaudication, 16, 22
clindamycin, 53
clonal thrombocytosis, 89
clopidogrel, 14, 15
Clostridium dif cile infection, 4950, 5556
clumsiness, hands, 113
coagulase-negative staphylococci, 56
coagulation
answers, 90
questions, 8587
cocaine use, 98, 103
codeine, 129, 138
colitis, microscopic, 31
colon and pancreas
answers, 31
questions, 27
colon cancer
family history, 134
recurrence pattern, 82
screening, 27, 79
colonoscopy, 27, 28, 82
colon polyps, 27, 87
common variable immunodefciency disease
(CVID), 107
community-acquired pneumonia, 7, 37, 52
computed tomography (CT), 12
colorectal cancer, 82
diverticulosis without abscess, 27
familial pheochromocytoma, 24
myasthenia gravis, 115
retroperitoneal/mesenteric
lymphadenopathy, 88
right axillary adenopathy, 79
sinuses, 106
confusion, 93, 95
congestive heart failure (CHF), 18, 67,
80, 109
answers, 2021
questions, 1213
constrictive pericarditis, 12, 20
continuous positive airway pressure (CPAP)
therapy, 70, 94
contrast-induced acute kidney injury (AKI),
104
contrast nephropathy, 22
coronary angiogram, 12, 14, 15, 72, 87
coronary artery bypass graf surgery, 12, 100
coronary artery disease, 50, 68, 79, 80,
98, 100
answers, 2122
questions, 1415
coronary bypass surgery, 68
coronary heart disease, with QT
prolongation, 109, 111
corticosteroids, 65, 107, 118
cough, 79
dry, 36
mild, 16
nonproductive, 37, 87
productive, 45, 106
recurrent, 129
severe, 16
CPAP (continuous positive airway pressure)
therapy, 70, 94
CRB-65 score, 52
C-reactive protein, 47
CREST syndrome, 62, 66
critical care medicine
answers, 41
questions, 3536
Crohn disease, 29
cross-content areas
answers
general internal medicine, 137138
geriatrics, 133
medical ethics, palliative care, 138139
perioperative medicine, 139
preventive medicine, 133134
womens health, 134137
questions
general internal medicine, 127129
geriatrics, 123124
medical ethics, palliative care, 129130
perioperative medicine, 130132
preventive medicine, 124125
womens health, 125127
cryoglobulinemia, 103
cryoglobulinemic vasculitis, 66
cryptogenic organizing pneumonia, 42
crystalloid, IV, 35
CURB-65 score, 52
Cushing syndrome, 75
cutaneous leukocytoclastic vasculitis, 62
cyclophosphamide chemotherapy, 7980
cystic fbrosis, 44
cystic fundic gland polyps, 28
cystitis, 56
cytarabine, 49
cytomegalovirus, 48, 97, 137
INDEX

143
hunger, excessive, 69
hungry bone syndrome, 77
hydralazine, 21
hydration, IV fuid, 71
hydrochloroquine, 123
hydrochlorothiazide (HCTZ), 15, 17, 63,
80, 93, 102
hydrochlorothiazide-triamterene, 68, 69, 70
hydrocortisone, 7172
hydroxychloroquine, 61, 63, 65
hyperbilirubinemia, indirect, 33
hypergammaglobulinemia, 33
hyperlipidemia, 12, 14, 15, 16, 27, 29, 35,
61, 70, 100
answers, 7475
fsh oil treatment, 48
questions, 6769
hypernatremia, 94, 102
hyperparathyroidism, 73, 76
hyperplastic polyps, 31
hypertension, 12, 13, 14, 15, 16, 27, 35, 37,
40, 60, 61, 67, 70, 80, 100
answers, 24, 43
questions, 17, 3839
with type 2 diabetes, 47
hyperthyroidism
apathetic, 77
iodine-induced, 77
hypertrophic cardiomyopathy (HCM),
21, 41
hyperuricemia, 61
hypocalcemia, 72, 7677
hypochondriasis, 111
hypogammaglobulinemia, 107
hypoglycemia
answers, 7475
questions, 6769
hypogonadism, 67, 74
hypokalemia, 94
hyponatremia, 45, 93, 102
hypo-osmolar hyponatremia, 102
hypotension, 35, 37
orthostatic, 115
postpartum hemorrhage-induced, 76
hypothyroidism, 30, 67, 74, 102
hypoxemia, 37
I
iatrogenic thyrotoxicosis, 74
ibuprofen, 30, 101
idiopathic bronchiolitis obliterans with
organizing pneumonia (BOOP),
42
idiopathic dilated cardiomyopathy, 21
idiopathic pulmonary fbrosis (IPF), 42, 44
IgA nephropathy, 98, 103
IgE-mediated reaction to penicillin, 107
ileal resection, 29, 33
immune-mediated heparin-induced
thrombocytopenia type II, 90
immunosuppressive therapy, 34
implantable cardioverter-defbrillator, 11, 12
infectious diseases
answers
bone, joint infections, 5354
HIV infection, 5455
joint infections, 5354
pneumonia, zoonoses, travel,
bioterrorism, 5253
skin, sof tissue infections, 5354
syndromes
cardiovascular, bloodstream, CNS,
5657
gastrointestinal tract infection,
5556
sexually transmitted diseases, 5556
urinary tract infection, 5556
questions
bone, joint infections, 4647
HIV infection, 4849
joint infections, 4647
mycobacterial infections, 4647
pneumonia, zoonoses, travel,
bioterrorism, 4546
skin, sof tissue infections, 4647
syndromes
cardiovascular, bloodstream, CNS,
5051
gastrointestinal tract infection,
4950
sexually transmitted diseases, 4950
urinary tract infection, 4950
infective endocarditis, 50
inferior vena cava (IVC) dilation, 38
inferior vena cava (IVC) flter replacement,
16, 23
infammatory arthritis, 65
infammatory bowel disease, 31
infammatory synovial fuid, 54
infiximab, 60, 65
infuenza vaccination, 124
infrapopliteal arterial occlusive disease, 24
inguinal adenopathy, 49
inguinal hernia, 29
inhalation anthrax, 53
insomnia, 109
insulin, 15, 98
intercourse pain, 109, 126
intermittent claudication, 16
invasive dental procedures, 56
iodine-induced hyperthyroidism, 77
ionized calcium elevation, 72
ipratropium, 39
ipratropium bromide inhaler, 93
iron defciency anemia, 28, 31, 32
irritable bowel syndrome (IBS), 29, 3233
isolated central nervous system (CNS)
vasculitis, 60, 64
isoniazid, 47, 53
isosorbide dinitrate, 21
J
jaundice, 29
jugular venous distention, 13
jugular venous pressure ( JVP), 12, 20, 38
K
kidney stones, recurrent, 73
knees
acute monoarthritis, 62
arthroscopic surgery, 98
bilateral intermittent efusions, 62
joint swelling, 47
lef knee pain, 59
lef, total arthroplasty, 130
mild discomfort, 61
osteoarthritis, 60
sudden onset pain, swelling, 62
synovitis, 85
total knee replacement, 85
L
lamivudine, 34, 54
large joint monoarticular arthritis, 54
LEAN quality improvement methodology,
138
lef bundle branch block (LBBB), 11, 18
lef ventricular assist device (LVAD), 12
lef ventricular ejection fraction (LVEF), 11,
12, 18, 20
lef ventricular failure, 41
lef ventricular hypertrophy, 13
leg/arm weakness, 84
leg edema, 30
fecal leucocytes, 29
femoral-popliteal bypass surgery, 98
femoral-popliteal deep vein thrombosis
(DVT), 16
femoral, popliteal, posterior pulses, 15, 16
fenofbrate, 70
fentanyl, 138
fentanyl patient-controlled analgesia, 98
fevers, low-grade, 59
fexofenadine, 72
fbromyalgia-like symptoms, 64
fbrotic lung disease, 442
fnger telangiectases, 37, 43
fsh oil capsules, 48, 68, 69, 71, 75, 101
fuconazole, 55
fuoroquinolones, 55, 56
futicasone propionate, 44, 80
folic acid supplement, 60, 61, 71, 85
foraminotomy, 123
fosinopril, 70
4-methylpyrazole therapy, 102, 103
furosemide, 11, 12, 35, 94, 99, 102
G
gabapentin, 99
gait ataxia, 113, 116
galactorrhea, 71, 135
gallstones, 31, 89
gastric banding, laparoscopic adjustable, 75
gastric cardia malignancy, 32
gastric MALT lymphoma, 91
gastroenterology and hepatology
answers
colon and pancreas, 31
esophagus and stomach, 3132
liver, 3334
small bowel and intestine, 3233
questions
colon and pancreas, 27
esophagus and stomach, 28
liver, 2930
small bowel and intestine, 2829
gastroesophageal refux disease (GERD),
3132, 37
gastrointestinal tract (upper) bleeding, 30
general internal medicine
answers, 137138
questions, 127129
gentamicin, 56
geriatrics
answers, 133
questions, 123124
giant cell arteritis (GCA), 59, 60, 64
Gilbert syndrome, 33
glaucoma, acute angle-closure, 129, 138
glimepiride, 68, 69
glipizide, 70, 80
glomerular disease
answers, 103
questions, 9799
glucosamine chondroitin sulfate, 60, 65
gluten-sensitive enteropathy, 121
glyburide, 37
glycoprotein IIb/IIIa inhibitors, 21
goiter, multinodular, 77
gonococcal arthritis, 54
gonococcal conjunctivitis, 138
gonorrhea, 56, 138
Gottron papules, 120
gout, 6162, 65, 101
pseudogout, 66
granulomatous disease, 41
Graves disease, 74, 77
Graves ophthalmopathy, 74
group A -hemolytic streptococcal
(GABHS) pharyngitis, 138
Guillain-Barr syndrome (GBS), 115116
gynecologic examination, 125
gynecomastia, 71, 7576
H
H
2
receptor blocker, 32
Haemophilus infuenzae, 52, 56, 106
haloperidol, 133
hands
hand clumsiness, 35, 113
rheumatoid nodules, 60
synovitis, proximal interphalangeal
joints, 60
Hashimoto thyroiditis, 67
hCG-producing tumor, 7576
headache
migraine, 114, 116, 118
new-onset, 59
post-partum, 71
heart
bibasilar crackles, 35, 37
cardiac murmur, 18f
frst sound, normal, 11
fourth sound, 12, 13
holosystolic murmur, 99
second sound, reverse splitting, 10
systolic ejection murmur, 68, 99
systolic murmur, 35
third sound, intermittent, 12
heart block, third-degree, 50
heartburn, intermittent, 28
heart failure
due to biventricular dysfunction, 12
with preserved ejection fraction, 20
heart sounds
Helicobacter pylori, 28, 31, 91
hematology
answers
anemias, myeloid malignancies, 8990
coagulation, 90
lymphoid malignancies, 9091
questions
anemias, myeloid malignancies, 8385
coagulation, 8587
lymphoid malignancies, 8788
hemoglobinopathies, high-oxygen af nity, 89
hemolytic anemia, 83, 103
heparin, 90. See also low-molecular-weight
heparin
dosing consensus, 128
intravenous, 14
subcutaneous, 86
hepatic cytochrome P450 enzymes, 53
hepatitis, autoimmune, 34
hepatitis A infection, 135
hepatitis B infection, 30, 34
hepatitis C infection, 30, 48, 62, 66, 97
asymptomatic, 103
hepatocellular carcinoma (HCC), 34
hepatology. See gastroenterology and
hepatology
hepatopulmonary syndrome, 44
hereditary hemorrhagic telangiectasia, 44
hereditary nonpolyposis colorectal cancer
(Lynch syndrome), 81
herpes simplex encephalitis, 57
Hickman catheter tunnel, 50
hip arthroplasty (total), 17
hip fracture, 87, 131
histoplasmosis, 53
HIV infection, 47, 4849, 5455, 97
hoarseness, 28
Hodgkin lymphoma therapy, 90
honeycombing (lungs), 37
hopelessness, 109
hospital-acquired pneumonia, 46
hot fushes, 126, 135
HPV vaccine, 135
144

INDEX
acute renal failure, 97
chronic renal failure, 99101
electrolyte disorders, 9394
glomerular disease, 9799
neuroleptic agents, 133
neuroleptic malignant syndrome, 109, 111
neurology
answers, 115116
questions, 113114
new-onset headache, 59
niacin, 29
nicotine dependence, 67
nifedipine, 61
night sweats, 135
nitrofurantoin, 37, 55, 56
nitrofurantoin lung toxicity, 42
nitroglycerin, 14, 35
nocturnal continuous positive airway
pressure, 94
node-positive breast cancer, 79
nodular sclerosing Hodgkin lymphoma, 87
nonalcoholic steatohepatitis, 38
nonarticular rheumatism, vasculitis
answers, 64
questions, 5960
non-Hodgkin lymphoma, 30, 130
nonishemic dilated cardiomyopathy, 11
norfoxacin (oral), 33
normochromic anemia, 59
North American Symptomatic Carotid
Endarterectomy Trial
(NASCET), 23
nortriptyline, 123
NPH insulin, 101
nucleoside, oral/nucleoside analogue, 34
nucleoside reverse transcriptase inhibitors
(NRTIs), 54
nutrition
answers, 75
questions, 7071
nutritional supplementation
calcium, 70, 73, 76
fsh oil capsules, 48, 68, 69, 71, 75, 101
folic acid, 60, 61, 71, 85
multivitamin, 67, 70, 71, 72, 80, 97, 101
vitamin B
12
, 70, 75
vitamin D, 70, 72, 76
vitamin E, 71, 101
O
obesity, 29, 68, 70, 100, 101. See also body
mass index
obesity and nutrition
answers, 75
questions, 7071
obsessive-compulsive disorder, 111
obstructive sleep apnea (OSA), 44, 70, 94
occupational lung disease
answers, 4143
questions, 3638
oculomasticatory myorhythmia, 28
olanzapine, 109
oliguric acute renal failure, 98, 103
omega-3 fatty acids, 75
omeprazole, 37, 99
oncology
answers, 8182
questions, 7980
onycholysis, 37
open wound/foul-smelling pus, 47
oral contraceptives, 74, 98
orchiectomy, 82
orlistat, 74
orthopedic injury repairs, 35
orthopnea, 12
orthostatic hypotension, 115
osteoarthritis, rheumatoid arthritis
answers, 6465
questions, 6061
outpatient parenteral antibiotic therapy
(OPAT), 4647, 53
ovarian cancer, 79
oxygen therapy, 44
oxymetazoline, 107
P
pancreatitis, acute, 31
pansystolic murmur, 13
paradoxical embolism, 44
paranoid beliefs, 109
parathyroid adenoma, 73
parathyroidectomy, 73
parathyroid hormone (PTH), 73, 77
parenteral antihypertensive drug choice, 17
paresthesias of the hands, 68
Parkinson disease, 124, 133
paroxysmal nocturnal dyspnea, 12
parvovirus B 19, 97
Pasteurella multocida, 53
patent foramen ovale (PFO), 11, 19
PCP prophylaxis, 55
PDE-5 inhibitors, 138
pelvic radiotherapy, 81
penicillin, 105
IgE-mediated reaction to, 107
percutaneous coronary intervention (PCI),
1415, 22
pericardium myopathy, 11
perioperative medicine
answers, 139
questions, 130132
periorbital edema, 120
peripheral artery disease (PAD), 22
peripheral neuropathy, 20, 68, 98
peripheral neuropathy Lupus-like syndrome,
53
peripheral vascular disease, 47, 131
peritumoral edema, 81
periumbilical discomfort, 29
pes anserine bursitis, 64
pheochromocytoma, 17, 76
Philadelphia chromosome-negative chronic
myeloid leukemia (CML), 83, 89
physiologic sleep, 36
pigment gallstones, 89
pituitary, gonadal, adrenal disorders
answers, 7576
questions, 7172
pituitary tumors, 71, 76
plantar ulcer, 47
plasma cell dyscrasia, 102. See also AL
amyloidosis; multiple myeloma
plasma cell proliferative disorder, 91
plethoric face, 84
pleural efusion
lef-sided, 37
right-sided, 37
pleural thickening, 37
pneumococcal pneumonia, 54, 56
pneumococcal vaccination, 124, 134
pneumocystis pneumonia (PCP), 49
pneumonia
acute, 96
acute eosinophilic, 42
answers, 5253
community-acquired, 7, 37, 52
cryptogenic organizing, 42
hospital-acquired, 46
lobar pneumonia, 48
organizing, with idiopathic bronchiolitis
obliterans, 42
pneumococcal, 54, 56
pneumocystis pneumonia, 49
questions, 4546
Legionella pneumophila, 37, 52
leukocytosis, 45
leukotriene receptor blockers, 107
leuprolide, 97
levofoxacin, 46
levothyroxine, 67, 74
light-headedness, 95
limb dysmetria, 116
lip telangiectases, 37, 43
lisinopril, 11, 14, 15, 17, 27, 37, 67, 68, 69,
123, 130
lisinopril-hydrochlorothiazide, 97
lithium carbonate, 109, 111
liver
answers, 3334
questions, 2930
liver Doppler ultrasonography, 30
liver lobe abscess, 55
lobar pneumonia, 48
Lfgren syndrome, 4142
loop diuretics, 102. See also furosemide
loose stools, 27
loperamide hydrochloride, 31
losartan, 17, 70, 100, 101
lovastatin, 80
low-molecular-weight heparin (LMWH),
128, 131, 137, 139
low-voltage QRS complexes, in the limb
leads, 87
lumbar spine tenderness, 59
lung cancer, 102
lung contusions, bilateral, 35
lungs
dense upper lef consolidation, 45
difuse bilateral crackles, 45, 46, 98
inspiratory crackles, 45
right midlung feld, 84
lupus anticoagulant (LAC), 86, 90
Lyme disease, 66
lymph node biopsy, 79
lymphocytic hypophysitis, 76
lymphocytic thyroiditis, 77
lymphocytosis, 87
lymphoid malignancies
answers, 9091
questions, 8788
Lynch syndrome (hereditary nonpolyposis
colorectal cancer), 81
M
macrocytic anemia, 89
magnetic resonance imaging (MRI)
arterial distribution infarctions, 60
breast cancer evaluation, 81
diabetic foot infection, 127
head: sellar mass with suprasellar
extension, 71
lef hippocampus atrophy, 114
pituitary microadenoma, 71, 76
MALT lymphoma, 8788, 91
mammography recommendations, 134
medical ethics, palliative care
answers, 138139
questions, 129130
medullary thyroid carcinoma, 72
megaloblastic anemia, 89
membranoproliferative glomerulonephritis,
103
membranous nephropathy, 98
memory problems, 123
meperidine, 138
meropenem, 49
mesalamine, 27
mesenteric ischemia, 33
mesenteric lymphadenopathy, 88
metabolic acidosis, 96
metabolic alkalosis, 96
metformin, 15, 29, 33, 67, 68, 69, 70, 74, 99,
100, 101, 130
methicillin-resistant Staphylococcus aureus
(MRSA), 47, 51, 53, 56
methimazole, 74
methotrexate, 60, 61, 85
methylprednisolone, 62, 123
metolazone, 94, 102
metoprolol, 9, 14, 15, 71, 98, 99, 101, 103,
130
microhematuria, asymptomatic, 97
microscopic colitis, 31
midsystolic click, 11, 19
migrant headache, 114, 116, 118
minimal change nephropathy, 98
Mini-Mental State Examination (MMSE), 123
mitral regurgitant murmur, 19, 19f
mitral valve disease, 29
mitral valve prolapse, 11, 19
mitral valve replacement, 51
Model for End-Stage Liver Disease
(MELD), 132
monoamine (histamine-tyramine)
poisoning, 53
monoarthritis, acute, 62
monoclonal gammopathy of undetermined
signifcance (MGUS), 90
motor vehicle collision, 36
MRSA prosthetic valve endocarditis, 56
mucormycosis, 55
multi-infarct dementia, 93
multinodular goiter, 77
multiple endocrine neoplasia (MEN), type
2, 76
multiple myeloma, 91, 102
multiple sclerosis (MS), 60
multivitamin supplement, 67, 70, 71, 72,
80, 97, 101
muscle weakness, 115
difuse, 9495
slowly progressive, 63
myalgias, 59, 64
difuse, 98
myasthenia gravis, 115
Mycobacterium tuberculosis, 47
myelodysplastic syndrome (MDS), 83, 89
myeloid malignancies
answers, 8990
questions, 8385
myeloproliferative neoplasm, 89
myocardial contusion, post-trauma, 41
myocardial infarction, 123, 125, 130
answers, 2122
questions, 1415
N
nabumetone, 60
N-acetylcysteine, 44
naproxen, 61, 70
nasal passages
congestion, 105
hyperemic, 46
nasal regurgitation of liquids, 113
nasal spray, saline, 107
necrotizing fascitis, type 1, 53
Neisseria gonorrhoeae, 107
Neisseria gonorrhoeae infection, 54, 105
Neisseria meningitidis infection, 105
nephrology
answers
acid-based disorders, 102
acute renal failure, 102103
chronic renal failure, 103104
electrolyte disorders, 102
glomerular disease, 103
questions
acid-based disorders, 9497
INDEX

145
selective serotonin reuptake inhibitors
(SSRIs), 111
senile cardiac amyloidosis, 9091
septoplasty, 46
seropositive rheumatoid arthritis, 61
serum brain natriuretic peptide (BNP)
levels, 12, 20
Sheehan syndrome, 76
shock, severe, 37
shunts in the atrial septum, 11
sickle cell anemia, 47, 89
sickle cell crisis, 54
sickle cell disease, 84
sigmoid polyp, 27
sildenafl, 101
simvastatin, 12, 14, 15, 27, 33, 61, 63, 67, 68,
98, 99, 100, 101, 130
sinopulmonary infection, 106, 107
sinus bradycardia, 11
sinusitis, recurrent, 40, 46
sinus tachycardia, 35
Six Sigma quality improvement method, 138
Sjgren syndrome, 9495
skin dryness, lower extremities, 68
skin hyperpigmentation, 28
sleep/sleepiness
disrupted, fragmented, 41
excessive, 36
insomnia, 109
obstructive sleep apnea, 44, 70, 94
small bowel and intestine
answers, 3233
questions, 2829
small-molecule intoxication, 103
smoking/smoking cessation, 15, 79
snoring, 40
socioeconomic issues, 133
somatization disorder, 111
sotalol, 11, 18
speech dif culties, 113
spironolactone, 11, 12, 21
splenomegaly, borderline, 30
spondyloarthropathies
answers, 65
questions, 6162
sputum, progressively purulent, 45
squamous cell carcinoma, 79
staphylococcal toxic shock syndrome, 53
Staphylococcus aureus, 52, 55, 56
Starr-Edwards mitral valve, 131, 139
statins. See individual statins
stavudine-tenofovir-lamivudine-darunavir
with boosted ritonavir with
raltegravir, 48
steatorrhea, 32
stenotic lesion, right carotid, 85
sterile pyuria, 103
steroid myopathy, 66
Streptococcus pneumoniae, 37, 106
stroke, 11, 14, 41, 79
ST-segment depression, 14, 17, 21, 22
subepithelial collagen band (colon), 27
substance abuse, 95, 96, 98
substance-induced psychosis, 111
suicidal ideation, 109
sulfonylurea medications, 74
sumatriptan, 114
sunitinib, 99
superior mesenteric artery embolus, 33
superior vena cava syndrome, 12, 20
swallowing, fuctuating dif culties, 113
symptomatic carotid disease, 16
syncope, 910, 13, 35
post-exertion, 12
syndrome of antidiuretic hormone
(SIADH), 76, 102
synovitis, 60, 85
syphilis, IgM and IgG, 50
syphilis, secondary, 56
systemic lupus erythematosus (SLE), 63, 85,
86, 8990
systolic click, 11, 12, 19
systolic click murmur, 20
systolic ejection murmur, 13
T
tachycardia, 70
tadalafl, 71
Takayasu arteritis, 59, 64
tamsulosin, 71
tardive dyskinesia, 111
tea-colored urine, 97
temporal arteritis, 137
temporal lobe epilepsy, 116
tenofovir, 54
testosterone gel, 67
tetanus-diphtheria (Td) vaccination, 46,
124, 133, 134
thiamine defciency, acute, 75
3-vessel disease, 22
thrombocytopenia, 90
thrombocytosis, extreme, 89
thrombotic thrombocytopenic purpura
(TTP), 90
thrombus
lef superfcial femoral vein, 128
right anterior tibial vein, 128
thrush, 49, 55
thyroid disease
answers, 74
questions, 67
thyroid lobe nodule, 72
thyrotoxicosis, 74, 77
tiotropium, 39, 44
tissue plasminogen activator (tPA), 35
TMP-SMX prophylaxis, 55, 56
tobacco use, 16
Todd paralysis, 116
toe (right second), arterial insuf ciency, 83
tonic-clonic seizure, 86
topiramate, 116
tramadol, 97, 123
transfusion-related acute lung injury
(TRALI), 36, 41
transient ischemic attack (TIA), 16
transient ischemic attack (TIA)-like attacks,
60
transsphenoidal microadenomectomy, 71
transthoracic echocardiogram, 9
travel, 5253
treadmill ECG, 11
trimethoprim-sulfamethoxazole, 55, 65
trochanteric bursa tenderness, 59
Tropheryma whipplei, 32
tubal ligation, 70
tuberculin skin test (TST), 47
tuberculosis, 47
tubular adenomas, 27
tubulovillous adenoma, 27
tularemia, 53
Tumor lysis syndrome, 103
tumor necrosis factor (TNF) therapy, 61, 65
2-block claudication, 16
2-pillow orthopnea, 11
type B aortic dissection, 1516
type 2 diabetes mellitus, 9, 45, 50, 80, 98,
99, 109, 125
U
ulcerative colitis, 27
upper respiratory tract infection, 105
urinary tract infection, 55
urinary urgency incontinence, 133
urine, tea-colored, 97
recurrent, 37
severe, bilateral, 130
ventilator-associated, 45
polyarthritis, 41
polycystic ovary syndrome, 67
polycythemia, 89
polymyalgia rheumatica (PMR), 59
polymyositis, 63
polysomnography, overnight, 44
polysubstance abuse, 95, 96
popliteal system clot, 128, 137
portal hypertension, 33
post-exertion syncope, 12
postpartum hemorrhage-induced
hypotension, 76
postprandial bloating, 29
potassium, IV, 102
prasugrel, 2122
pravastatin, 37, 48
prazosin, 37
prednisone, 60, 63, 64, 85
premature coronary artery disease, 22
premature gallstones, 89
premature ventricular complexes (PVCs), 13
presumed consent, for minors, 139
pretibial edema, 37
preventive medicine
answers, 133134
questions, 124125
primary biliary cirrhosis (PBC), 33
primary hypothyroidism, 67
primary myelofbrosis (PMF), 83
procainamide, 9
prolonged QT interval, 138
propoxyphene, 138
propylthiouracil (PTU), 67, 74
prostate cancer, 81
metastatic, 97
prostatectomy, radical, 35, 82
prostate enlargement, 80
prosthetic hip joint, 47
proteinuria, 60, 103
proton pump inhibitor (PPI) therapy, 28,
3132, 37
proximal renal tubular acidosis (RTA), 102
pruritic hives, 105
pruritic rash, 14, 87
pseudoachalasia, 32
pseudogout, 66
Pseudomonas aeruginosa, 52
psoriasis vulgaris, 118f, 120
psoriatic arthritis, 62
psychiatry
answers, 111
questions, 109110
psychosis, substance-induced, 111
pulmonary artery catheterization, 37
pulmonary artery hypertension, 44
pulmonary diseases
answers
critical care medicine, 41
difuse lung disease, 4143
occupational lung disease, 4143
signs, symptoms, chest radiography,
4344
vascular disease, embolism,
hypertension, 43
questions
critical care medicine, 3536
difuse lung disease, 3638
occupational lung disease, 3638
signs, symptoms, chest radiography,
3940
vascular disease, embolism,
hypertension, 3839
pulmonary edema, 13
pulmonary embolism, 3536, 86
answers, 43
questions, 3839
pulmonary hypertension, 66
pulmonary tuberculosis, 47
purifed protein derivative (PPD) skin test,
47, 53, 55
purpura, legs and feet, 97
pyelonephritis, 56
pyoderma gangrenosum, 119f, 120121
pyrazinamide, 47, 53
pyridoxine, 47
Q
quality improvement interventions, 137, 138
QuantiFERON-TB Gold test, 47, 53
quinapril, 68
R
radial pulse diminishment, 15
rash
heliotrope, 120
lower extremities, 97
pruritic hives, 105
pruritic rash, 14, 87, 119
R-CHOP chemotherapy, 91
rectal cancer, stage II, 80, 81
renal biopsy, 97, 101
renal cell carcinoma, 99, 101, 131
renal thrombotic microangiopathy, 103
renal ultrasonography, 99
respiratory illness, upper, 129
retrobulbar neuritis, 53
retroperitoneal lymphadenopathy, 88
rhabdomyolysis, 103
rheumatoid arthritis, 85
answers, 6465
questions, 6061
rheumatology
answers, 6466
antirheumatic drugs, 6465
miscellaneous conditions, 66
nonarticular rheumatism, vasculitis, 64
osteoarthritis, rheumatoid arthritis,
6465
spondyloarthropathies, 65
questions, 5963
antirheumatic drugs, 6061
miscellaneous conditions, 6263
nonarticular rheumatism, vasculitis,
5960
osteoarthritis, rheumatoid arthritis,
6061
spondyloarthropathies, 6162
rhinoconjunctivitis, allergic, 107
rhinorrhea, 105
rhomboid birefringent crystals, 66
rib fractures, 35, 95
rifampin, 47, 53, 56
rifaximin, 33
right (dormant) hand clumsiness, 35
right ventricular heave, 38
rituximab, 91
rosacea, 117, 118f, 120
rosiglitazone, 74, 75
Roux-en-Y gastric bypass, 70, 75
S
saddle pulmonary embolism, 35
salmeterol, 44, 80
sarcoidosis, 41
Sarcoptes scabiei var hominis, 117f, 120
schizophrenia, 109
seborrheic dermatitis, 49
secondary amenorrhea, 135
second heart sound splitting, 10
sedentary lifestyle, 9
146

INDEX
supplementation, 70, 72, 76
vitamin E supplement, 71, 101
voriconazole, 55
W
walking dif culties, 113
warfarin, 9, 11, 23, 86, 90, 123
ef cacy trial, 127128
treatment number, 128
warm autoimmune hemolytic anemia
(WAIHA), 90
water defcit estimation, 93, 102
weakness, generalized, 94
Wegener granulomatosis, 59, 60, 64
weight loss, 123
Wernicke encephalopathy, 75
Western blot test, 48
Whipple disease, 28, 32
Wolf-Parkinson-White syndrome, 18
womens health
answers, 134137
questions, 125127
X
xanthelasmas, 29, 33
Z
zidovudine, 54
ziprasidone, 111
zoonoses
answers, 5253
questions, 4546
US Preventive Services Task Force, screening
recommendations, 134
uveitis, 31, 61, 65
V
vaccinations, 124, 126, 134, 135
vaginal irritation, 126, 135136
valproate sodium, 111
Valsalva maneuver, 11, 12, 19
vancomycin, 49
vascular disease, embolism, hypertension
answers, 43
questions, 3839
vascular medicine
answers, 2224
questions, 1517
vasculitis, 83, 103
answers, 64
questions, 5960
venous Doppler ultrasonography, 99
ventilator-associated pneumonia, 45
ventilator dyssynchrony, 35
ventricular thickening, 87
verapamil, 53
visceral arteriovenous
malformation, 44
viscosupplementation, 65
vitamin B
12
defciency, 83, 89
supplementation, 70, 75
vitamin D
defciency, 76

Você também pode gostar